Chapters 13-18 Ricci 2nd edition

¡Supera tus tareas y exámenes ahora con Quizwiz!

To teach patients about the process of labor adequately, the nurse knows that which event is the best indicator of true labor?

Cervical dilation and effacement

What is given to the baby to prevent eye infection?

Erythromycin ointment, within the first two hours

How often should you assess in the first hour?

Every 15 minutes

Which position would the nurse suggest for second-stage labor if the pelvic outlet needs to be increased?

Kneeling or squatting moves the uterus forward and aligns the fetus with the pelvic inlet; this can facilitate the second stage of labor by increasing the pelvic outlet.

A client reports pain in the lower back, hips, and joints 10 days after the birth of her baby. What instruction should the nurse give the client after birth to prevent low back pain and injury to the joints?

Maintain correct posture and positioning. The nurse should recommend that clients maintain correct position and good body mechanics to prevent pain in the lower back, hips, and joints. Avoiding carrying her baby and soaking several times per day is unrealistic. Application of ice is suggested to help relieve breast engorgement in nonbreastfeeding clients.

When applying a skin temperature probe to a newborn who is lying on his side, which location would be most appropriate? a) Over the opposite hip b) Between the scapulae c) In the mediastinal area d) Over the liver

Over the liver

D/C ed?

PCP Car seat test Hep B Tdap for caregiver Diff between baby blues + depression Back to sleep Safety @ home for baby Cord/circ care What to expect w/ stools Birth control Lactation consultant

Newborn skin: Can you identify a skin variation from a photo: for example stork bit, milia?

Page 591 in textbook

Episiotomy comfort?

Pain relieving spray Heat packs Toilet cleanse Water pump

A nurse is assessing the temperature of a newborn using a skin temperature probe. Which point should the nurse keep in mind while taking the newborn's temperature? a) Ensure that the newborn is lying on its abdomen b) Place the temperature probe over the liver c) Use the skin temperature probe only in open bassinets d) Tape the temperature probe on the forehead

Place the temperature probe over the liver

As you are examining the newborn female, you notice a small pinkish discharge from the vaginal area. What should you suspect? a) Pseudomenstruation, a normal finding b) Impending hemorrhage from a congenital defect c) Infection d) Evidence of birth trauma

Pseudomenstruation, a normal finding Correct Explanation: Pseudomenstruation is seen when a newborn female has a small amount of pinkish discharge. It comes from the withdrawal of maternal hormones and is a normal finding.

A nurse is discussing breastfeeding with a new mother and demonstrates that when she strokes the baby's cheek, the baby turns his head in that direction. This reflex is known as which of the following? a) Extrusion reflex b) Rooting reflex c) Moro reflex d) Babinski reflex

Rooting reflex

A nurse caring for a pregnant client in labor observes that the fetal heart rate (FHR) is below 110 beats per minute. Which interventions should the nurse perform? Select all that apply.

Turn the client on her left side. Administer oxygen by mask. Assess client for underlying causes. The nurse should turn the client on her left side to increase placental perfusion, administer oxygen by mask to increase fetal oxygenation, and assess the client for any underlying contributing causes. The client's questions should not be ignored; instead, the client should be reassured that interventions are to effect FHR pattern change. A reduced IV rate would decrease intravascular volume, affecting the FHR further.

The nurse is making a home visit to a woman who is 4 days postpartum. Which finding would indicate to the nurse that the woman is experiencing a problem? a) Diaphoresis b) Lochia serosa c) Edematous vagina d) Uterus 1 cm below umbilicus

Uterus 1 cm below umbilicus Explanation: By the fourth postpartum day, the uterus should be approximately 4 cm below the umbilicus. Being only at 1 cm indicates that the uterus is not contracting as it should. Lochia serosa is normal from days 3 to 10 postpartum. After birth the vagina is edematous and thin with few rugae. It eventually thickens and rugae return in approximately 3 weeks. Diaphoresis is common during the early postpartum period, especially in the first week. It is a mechanism to reduce fluids retained during pregnancy and restore prepregnant body fluid levels.

Newborn Isaac has been taken to the nursery after delivery. He has been cleaned in the labor and delivery suite and swaddled in a blanket. The nurse is going to check his pulse. What must the nurse do? a) Wear gloves. b) Use infection transmission precautions. c) Clean his or her hands with a betadine scrub. d) Perform a 3-minute surgical type scrub before touching him.

Wear gloves

What should the assessement of the eyes reveal?

ability to track objects to midline, transient strabismus, searching nystagmus, and red reflex

The expected fetal heart rate response in an active fetus is:

acceleration of at least 15 bpm for 15 seconds. A reassuring active fetal heart rate is a change in baseline by increase of 15 bpm for 15 seconds. This is a positive and reassuring periodic change in fetal heart rates as a response to fetal movement.

The nurse is reviewing the uterine contraction pattern and identifies the peak intensity, documenting this as which phase of the contraction?

acme

The coach of a client in labor is holding the client's hand and appears to be intentionally applying pressure to the space between the first finger and thumb on the back of the hand. The nurse recognizes this as which form of therapy?

acupressure

The nurse caring for a client in preterm labor observes abnormal fetal heart rate (FHR) patterns. Which nursing intervention should the nurse perform next?

administration of oxygen by mask The client should be administered oxygen by mask because the abnormal FHR pattern could be due to inadequate oxygen reserves in the fetus. Because the client is in preterm labor, it is not advisable to apply vibroacoustic stimulation, tactile stimulation, or fetal scalp stimulation.

A client in the latent phase of the first stage of labor is noted to be uncomfortable with intact membranes and mild contractions on assessment. The nurse should encourage the client to pursue which action?

ambulation ad lib

What is the stepping reflex? How do you assess this? What is normal findings?

assess the reflex by holding the newborn upright and incline forward with the soles of the feet touching a flat surface. Expected/normal reflex = newborn will make a stepping motion or walking, alternating flexion and extension with the soles of the feet

The nurse is providing preoperative care for a client who will undergo a cesarean section. The nurse should

confirm that consent has been provided by the client.

How is the rooting reflex tested?

stoking infant cheek, newborn should turn to the side that was stoked

Normal B/P of the newborn?

systolic 50-75 (mean of 62) diastolic 30-45 (mean of 37)

How is the Moro reflex tested?

occur when the newborn is startled, use lifting motion release arms suddenly

The nurse working on a postpartum client must check lochia in terms of amount, color, change with activity and time, and:

odor The nurse when assessing lochia must do so in terms of amount, color, odor, and change with activity and time.

A nurse is monitoring a fetal heart rate (FHR) pattern on her client in labor. The earlier baseline FHR was 140. The FHR now is 168. The nurse knows that which factors can affect changes in the FHR? Select all that apply.

fetal movement fetal distress maternal fever An increase in the FHR from baseline can mean that there is some type of fetal distress. This can happen with a maternal fever also. Narcotics may affect the heart rate variability but not the baseline FHR. Fetal movement can cause an increase in FHR, and utero-placental insufficiency can cause late decelerations in the FHR.

The LVN/LPN will be assessing a postpartum client for danger signs after a vaginal birth. What assessment finding would the nurse assess as a danger sign for this client?

fever more than 100.4° F (38° C) A fever more than 100.4° F (38° C) is a danger sign that the client may be developing a postpartum infection. Lochia rubra is a normal finding as is a firm uterine fundus. A uterine fundus above the umbilicus may indicate that the client has a full bladder but does not indicate a postpartum infection.

When is the baby most alert?

first two hours

Why is vitamin K given to the baby?, when is it given?

for clotting factors first two hours

Where is blood taken from in baby?

heel

In providing culturally competent care to a laboring woman, which is a priority?

identify how the client expresses labor pain.

The labor and delivery nurse knows that internal monitoring during labor is an invasive technique. She correctly identifies an increase in the risk of which of the following for both mother and fetus if this is done?

infection Because internal monitoring techniques are invasive, both the woman and the fetus can become ill with infection. Internal methods should be used only when the benefit clearly outweighs the risk.

The nurse is monitoring a client's uterine contractions. Which factors should the nurse assess to monitor uterine contraction? Select all that apply.

intensity of contractions frequency of contractions uterine resting tone The nurse should assess the frequency of contractions, intensity of contractions, and uterine resting tone to monitor uterine contractions. Monitoring changes in temperature and blood pressure is part of the general physical examination and does not help to monitor uterine contraction.

When teaching the pregnant patient about self-medicating for pain during labor, why did the nurse instruct the patient to avoid taking acetylsalicylic acid?

interference with blood coagulation with increased risk of bleeding in mother or infant.

During labor, a pregnant patient's doula uses therapeutic touch and massage. Which outcome indicates that these approaches have been effective?

the patient is not requesting pain medication

A mother points out to you that following three meconium stools, her newborn has had a bright green stool. You would explain to her that a) this is a normal finding. b) her child will be isolated until the stool can be cultured. c) her child may be developing an allergy to breast milk. d) this is most likely a symptom of diarrhea.

this is a normal finding. Correct Explanation: Newborn stools typically pass through a pattern of meconium, green transitional, and then yellow.

A nurse is preparing a patient for rhythm strip testing. She places the woman into a semi-Fowler's position. What is the appropriate rationale for this measure?

to prevent supine hypotension syndrome

A nursing student is studying labor and delivery and has learned that the first stage of labor consists of which of the following phases? (Select all that apply.)

transition active latent The first stage of labor includes three phases: latent, active, and transition.

The nurse is assessing the abdomen of the neonate. When inspecting the umbilical cord area of a newborn, the nurse would expect which finding?

two arteries and one vein The normal umbilical cord contains three vessels: two arteries and one vein.

How should infants with hyperbilirubinemia be hydrated?

with formula not water

Which of the following interventions would a nurse implement to best prevent heat loss in a 1 day of age newborn? a) Bathe and wash the newborn when temperature is 97.5°F (36.4°C) b) Warm all surfaces and objects that come in contact with the newborn. c) Keep the newborn under the radiant heater when not with mom. d) Cover the newborn with several blankets while under the warmer.

Warm all surfaces and objects that come in contact with the newborn.

When assessing the uterus of a 2-day postpartum client, which finding would the nurse evaluate as normal?

a moderate amount of lochia rubra The client should have lochia rubra for 3 to 4 days postpartum. The client would then progress to lochia serosa being expelled from day 3 to 10. Last the client would have lochia alba from day 10 to 14 until 3 to 6 weeks.

The nurse is caring for a client who has been in labor for the past 8 hours. The nurse determines that the client has transitioned into the second stage of labor based on which sign?

the urge to push occurs

Review of a woman's labor and birth record reveals a laceration that extends through the anal sphincter muscle. The nurse identifies this laceration as which type?

third-degree laceration A third-degree laceration extends through the anal sphincter muscle. A first-degree laceration involves only skin and superficial structures above the muscle. A second-degree laceration extends through the perineal muscles. A fourth-degree laceration continues through the anterior rectal wall.

A mother points out to you that following three meconium stools, her newborn has had a bright green stool. THe nurse would explain to her that a) her child will need to be isolated until the stool can be cultured. b) her child may be developing an allergy to breast milk. c) this is most likely a symptom of impending diarrhea. d) this is a normal finding.

this is a normal finding.

"3. What is the use of fiber optic pads in treat- ment of physiologic jaundice?"

"3. Fiber optic pads (Biliblanket or Bilivest) are used for treatment of physiologic jaundice and can be wrapped around newborns or newborns can lie upon them. These pads consist of a light that is delivered from a tungsten-halogen bulb through a fiber optic cable and is emitted from the sides and ends of the fibers inside a plastic pad. They work on the premise that phototherapy can be improved by delivering higher-intensity therapeu- tic light to decrease bilirubin levels. The pads do not produce appreciable heat like banks of lights or spotlights do, so insensible water loss is not increased. Eye patches are also not needed; thus, parents can feed and hold their newborns continu- ously to promote bonding."

"3. What education should the nurse impart to Karen to discourage bed-sharing?"

"3. The nurse should educate Karen about potential risks of bed-sharing. Bringing a newborn into bed to nurse or quiet her down and then falling asleep with the newborn is not a safe practice. Infants who sleep in adult beds are up to 40 times more likely to suffocate than those who sleep in cribs. Suffocation also can occur when the infant gets entangled in bedding or caught under pillows, or slips between the bed and the wall or the headboard and mattress. It can also happen when someone accidentally rolls against or on top of them. Therefore, the safest sleeping location for all newborns is in their crib, without any movable objects close."

What percentage of birth wt do baby's lose by 3-4 days?

10%

New parents are upset their newborn has lost weight since birth. The nurse explains that newborns typically lose how much of their birth weight by 3 to 4 days of age?

10% Newborns typically lose approximately 10% of their initial birth weight by 3 to 4 days of age secondary to the loss of meconium, extracellular fluid, and limited food intake. This weight loss is usually regained by the 10th day of life.

To indicate that the infant is making a successful transition immediately after birth, the nurse checks the heart rate for 6 seconds. What should the count minimally be? a) 9 b) 10 c) 11 d) 12

11

What is the normally accepted fetal heart rate range?

110-160 bpm The standard acceptable fetal heart rate baseline is the range of 110-160 beats per minute. Sustained heart rates above or below the norm are cause for concern.

Normal HR in newborn? Normal heart rate in newborn due to crying?

120/160 bpm crying up to 180

A nurse is performing Apgar scoring on a newborn. The newborn demonstrates the following: a heart rate of 110; a good, strong cry; muscles of the extremities well flexed; a grimace in response to a slap to the sole of the foot; and normal pigment in most of the body, with blue at the extremities. Which of the following would be the total Apgar score for this newborn? a) 7 b) 6 c) 8 d) 9

8 Explanation: The heart rate of 110, the strong cry, and the muscles of the extremities being well flexed each indicate a score of 2 in the heart rate, respiratory effort, and muscle tone areas, respectively. The grimace in response to a slap to the sole of the foot and the blue at the extremities each indicate a score of 1 for the reflex irritability and color areas, respectively. Thus, the total Apgar score for this infant is 8.

How often should an infant with hyperbilirubinemia feed?

8-12 times per day for the first several days

What is the Normal temperature for newborn

97.7-99.5 (36.5-37.5)

Normal newborn temperature?

97.9-99.5 F

According to Ballard assessment scale, what size determines and large for gestational age baby and a small for gestational age baby? What measurements are used to get to this score?

>90% growth = LGA <10% growth = SGA 1. Head Circumference 2. Crown to Rump Distance 3. Femur Length

A woman's husband expresses concern about risk of paralysis from an epidural block being given to his wife. Which would be the most appropriate response by the nurse? A) "The injection is given in the space outside the spinal cord." B) "An injury is unlikely because of expert professional care given." C) "I have never read or heard of this happening." D) "The injection is given at the third or fourth thoracic vertebrae so paralysis is not a problem."

A) "The injection is given in the space outside the spinal cord." An epidural block, as the name implies, does not enter the spinal cord but only the epidural space outside the cord.

Inspection of a woman's perineal pad reveals a 5-inch stain. How should the nurse document this amount? A) moderate B) heavy C) light D) scant

A) moderate Moderate lochia would describe a 4- to 6-inch stain, scant lochia a 1- to 2-inch stain, and light or small an approximately 4-inch stain. Heavy or large lochia would describe a pad that is saturated within 1 hour.

Which factors influence cervical dilation (Select all that apply) ? a. Strong uterine contractions b. The force of the presenting fetal part against the cervix c. The size of the female pelvis d. The pressure applied by the amniotic sac e. Scarring of the cervix

A, B, D, E Dilation of the cervix occurs by the drawing upward of the musculofibrous components of the cervix, which is caused by strong uterine contractions. Pressure exerted by the amniotic fluid while the membranes are intact or by the force applied by the presenting part also can promote cervical dilation. Scarring of the cervix as a result of a previous infection or surgery may slow cervical dilation. Pelvic size does not affect cervical dilation.

When caring for a client with preeclampsia, which action is a priority? 1. Monitoring the client's labor carefully and preparing for a fast delivery 2. Continually assessing the fetal tracing for signs of fetal distress 3. Checking vital signs every 15 minutes to watch for increasing blood pressure 4. Reducing visual and auditory stimulation

Answer: 4 RATIONALES: A client with preeclampsia is at risk for seizure activity because her neurologic system is overstimulated. Therefore, in addition to administering pharmacologic interventions to reduce the possibility of seizures, the nurse should lessen auditory and visual stimulation. Although the other actions are important, they're of a lesser priority.

A preterm infant is experiencing cold stress after delivery. For which symptom should the nurse assess to best validate the problem? a) Hyperglycemia b) Metabolic alkalosis c) Shivering d) Apnea

Apnea

A nurse is caring for a client who gave birth about 10 hours earlier. The nurse observes perineal edema in the client. What intervention should the nurse perform to decrease the swelling caused due to perineal edema? a) Use a warm sitz bath or tub bath b) Use ointments locally c) Apply ice d) Apply moist heat

Apply ice Explanation: Ice is applied to perineal edema within 24 hours after delivery. Use of ointments is not advised for perineal edema. Moist heat and a sitz or tub bath are encouraged if edema continues 24 hours after delivery.

As a woman enters the second stage of labor, her membranes spontaneously rupture. When this occurs, what would the nurse do next?

Assess fetal heart rate for fetal safety.

As a woman enters the second stage of labor, her membranes spontaneously rupture. When this occurs, what would the nurse do next?

Assess fetal heart rate for fetal safety. Rupture of the membranes may lead to a prolapsed cord. Assessment of FHR detects this.

The nurse is documenting assessment of infant reflexes. She strokes the side of the infants face and the baby turns toward the stroke. What reflex has the nurse elicited? a) Tonic neck b) Sucking c) Moro d) Rooting

Rooting

The nurse is caring for a client in active labor who has had a fetal blood sampling to check for fetal hypoxia. The nurse determines that the fetus has acidosis when the pH is:

7.15 or less. In the hypoxic fetus, the pH will fall below 7.2, which is indicative of fetal distress.

The nurse has completed the initial assessment and vital signs for an infant born at 12:00 p.m. The assessment and vital signs were completed at 1:30 p.m. What time will the nurse plan to complete the next set of vital signs?

2:00 p.m. The nurse needs to complete vital signs every half hour for the first 2 hours of life. This makes the other options incorrect.

What are the 3 types of cesarean section cuts that can be made. What are their advantages?

1. Kerr (Transverse) : Allows VBAC for 2nd birth 2. Selheim : Low incision 3. Classical : Fastest(Risk or Trauma), Dihisance Risk, No vaginal supsequent births.

How long is the neonatal period for a newborn? _______ days

28

How many vessels should be in the umbilical cord?

3...two arteries and one vein

What is the expected range for respirations in a newborn?

30 to 60 breaths per minute Although episodic breathing is normal and short periods of apnea can occur, the normal respiratory rate for a newborn is 30 to 60 breaths per minute. For adults, it is typically 8 to 20 breaths per minute.

Average chest circumference for baby?

30-36 cm (12-14 inches)...generally 2-3 cm less than head circumference

Baby Eliza is 7 minutes old. Her heart rate is 92, her cry is weak, her muscles are limp and flaccid, she makes a face when she is stimulated, and her body and extremities are pink. What would the nurse assign as her Apgar score? a) 6 b) 3 c) 4 d) 5

5

One minute after delivery, the neonate's heart rate is 98 beats per minute (bpm), respirations are slow and irregular, arms are flexed, hips are extended, the neonate has no grimace, and the hands/feet are acrocyanotic. What Apgar score should the nurse assign to the neonate? a) 5 b) 6 c) 4 d) 7

5

Baby Eliza is 7 minutes old. Her heart rate is 92, her cry is weak, her muscles are limp and flaccid, she makes a face when she is stimulated, and her body and extremities are pink. What would the nurse assign as her Apgar score? a) 3 b) 4 c) 6 d) 5

5 Correct Explanation: The infant is not demonstrating a good transition to extra uterine life; the APGAR score ranges from 0 to 10 with a score of 0, 1, or 2 points for each: respiratory effort, heart rate, tone, grimace, and color. A score of 5 indicates the infant need support. Options A, B, and D are incorrect Apgar scores based on the scenario provided.

Which occurrence is associated with cervical dilation and effacement? a. Bloody show b. Lightening c. False labor d. Bladder distention

A As the cervix begins to soften, dilate, and efface, expulsion of the mucous plug that sealed the cervix during pregnancy occurs. This causes rupture of small cervical capillaries. Cervical dilation and effacement do not occur with false labor. Lightening is the descent of the fetus toward the pelvic inlet before labor. Bladder distention occurs when the bladder is not emptied frequently. It may slow down the descent of the fetus during labor.

The nurse would expect which maternal cardiovascular finding during labor? a. Increased cardiac output b. Decreased pulse rate c. Decreased white blood cell (WBC) count d. Decreased blood pressure

A During each contraction, 400 mL of blood is emptied from the uterus into the maternal vascular system. This increases cardiac output by about 51% above baseline pregnancy values at term. The heart rate increases slightly during labor. The WBC count can increase during labor. During the first stage of labor, uterine contractions cause systolic readings to increase by about 10 mm Hg. During the second stage, contractions may cause systolic pressures to increase by 30 mm Hg and diastolic readings to increase by 25 mm Hg.

Which description of the four stages of labor is correct for both definition and duration? a. First stage: onset of regular uterine contractions to full dilation; less than 1 hour to 20 hours b. Second stage: full effacement to 4 to 5 cm; visible presenting part; 1 to 2 hours c. Third state: active pushing to birth; 20 minutes (multiparous women), 50 minutes (first-timer) d. Fourth stage: delivery of the placenta to recovery; 30 minutes to 1 hour

A Full dilation may occur in less than 1 hour, but in first-time pregnancies it can take up to 20 hours. The second stage extends from full dilation to birth and takes an average of 20 to 50 minutes, although 2 hours is still considered normal. The third stage extends from birth to expulsion of the placenta and usually takes a few minutes. The fourth stage begins after expulsion of the placenta and lasts until homeostasis is reestablished (about 2 hours).

To care for a laboring woman adequately, the nurse understands that the __________ stage of labor varies the most in length? a. First b. Third c. Second d. Fourth

A The first stage of labor is considered to last from the onset of regular uterine contractions to full dilation of the cervix. The first stage is much longer than the second and third stages combined. In a first-time pregnancy the first stage of labor can take up to 20 hours. The second stage of labor lasts from the time the cervix is fully dilated to the birth of the fetus. The average length is 20 minutes for a multiparous woman and 50 minutes for a nulliparous woman. The third stage of labor lasts from the birth of the fetus until the placenta is delivered. This stage may be as short as 3 minutes or as long as 1 hour. The fourth stage of labor, recovery, lasts about 2 hours after delivery of the placenta.

APGAR what does the letters stand for?

A = appearance (color) P = pulse (HR) G = grimace (reflex irritability) A= activity (muscle tone) R = respiratory (respiratory effort)

After teaching a class about the changes in the gastrointestinal system of a newborn, which of the following if stated by the class indicates the need for additional teaching? a) A newborn's stomach capacity is approximately 300 mL. b) The cardiac sphincter is immature. c) The newborn's gut is sterile at birth. d) Oral intake is necessary for vitamin K production.

A newborn's stomach capacity is approximately 300 mL.

On examination, the hands and feet of a 12-hour-old infant are cyanotic without other signs of distress. The nurse should document as: a) Cold stress. b) Acrocyanosis. c) Potential for respiratory distress. d) Poor oxygenation.

Acrocyanosis. Correct Explanation: Acrocyanosis is a blue tint to the hands and feet of newborns during the first few days of life. Acrocyanosis is a normal finding and is not indicative of a potential for respiratory distress, poor oxygenation, nor cold stress.

Nurses can help their clients by keeping them informed about the distinctive stages of labor. What description of the phases of the first stage of labor is accurate?

Active: moderate, regular contractions; 4 to 7 cm dilation; duration of 3 to 6 hours

What can cause DVT?

Stasis of blood Altered coag Localized vascular damage

As relates to fetal positioning during labor, nurses should be aware that: a. Position is a measure of the degree of descent of the presenting part of the fetus through the birth canal. b. Birth is imminent when the presenting part is at +4 to +5 cm below the spine. c. The largest transverse diameter of the presenting part is the suboccipitobregmatic diameter. d. Engagement is the term used to describe the beginning of labor.

B The station of the presenting part should be noted at the beginning of labor so that the rate of descent can be determined. Position is the relation of the presenting part of the fetus to the four quadrants of the mother's pelvis; station is the measure of degree of descent. The largest diameter usually is the biparietal diameter. The suboccipitobregmatic diameter is the smallest, although one of the most critical. Engagement often occurs in the weeks just before labor in nulliparas and before or during labor in multiparas.

You are the oncoming nursery nurse in a normal newborn nursery. You receive report on four infants. Baby A is reported as being 16 hours old, vital signs within normal limits (WNL), bilirubin 3.5 mg/dL rooming in with mother; Baby B is 8 hours old, vital signs WNL, bilirubin 3 mg/dL, returning to nursery for night; Baby C is 19 hours old, vital signs WNL, bilirubin 4 mg/dL, rooming in with mother; Baby D is 4 hours old, vital signs WNL, bilirubin 2 mg/dL, returning to nursery for night. Which baby would you assess first? a) Baby C b) Baby B c) Baby A d) Baby D

Baby C

As relates to fetal positioning during labor, nurses should be aware that:

Birth is imminent when the presenting part is at +4 to +5 cm below the spine

As a part of the newborn assessment, the nurse examines the infant's skin. Which nursing observation would warrant further investigation? a) Small pink or red patches on the baby's eyelids, and back of the neck b) Bright red, raised bumpy area noted above the right eye c) Blue or purplish splotches on buttocks d) Fine red rash noted over the chest and back

Bright red, raised bumpy area noted above the right eye

A new mother has been reluctant to hold her newborn. A nurse can promote this mother's attachment by a) Allowing the mother to pick the best time to hold her newborn b) Showing a video of parents feeding their babies c) Talking about how the nurse held her own newborn while on the delivery table d) Bringing the newborn into the room

Bringing the newborn into the room Correct Explanation: Proximity of the newborn and the mother can promote interest in the newborn and a desire to hold. Exposure to other mothers and their behaviors can only serve to set up unrealistic and fearful situations for a reluctant mother.

What is the primary mechanism for temperature regulation in a newborn infant? a) External with blankets by the nursing staff b) Shivering and increased metabolic rate c) Skin to skin contact with mother d) Brown fat store usage

Brown fat store usage

A nurse is teaching a couple about patterned breathing during their birth education. Which technique should the nurse suggest for slow-paced breathing? A) Inhale and exhale through the mouth at a rate of 4 breaths every 5 seconds. B) Hold breath for 5 seconds after every 3 breaths. C) Inhale slowly through nose and exhale through pursed lips. D) Punctuated breathing by a forceful exhalation through pursed lips every few breaths.

C) Inhale slowly through nose and exhale through pursed lips. For slow-paced breathing, the nurse should instruct the woman to inhale slowly through her nose and exhale through pursed lips. In shallow or modified-pace breathing, the woman should inhale and exhale through her mouth at a rate of 4 breaths every 5 seconds. In pattern-paced breathing, the breathing is punctuated every few breaths by a forceful exhalation through pursed lips. Holding the breath for 5 seconds after every three breaths is not recommended in any of the three levels of patterned breathing.

Which basic type of pelvis includes the correct description and percentage of occurrence in women? a. Gynecoid: classic female; heart shaped; 75% b. Android: resembling the male; wider oval; 15% c. Anthropoid: resembling the ape; narrower; 10% d. Platypelloid: flattened, wide, shallow; 3%

D A platypelloid pelvis is flattened, wide, and shallow; about 3% of women have this shape. The gynecoid shape is the classical female shape, slightly ovoid and rounded; about 50% of women have this shape. An android, or malelike, pelvis is heart shaped; about 23% of women have this shape. An anthropoid, or apelike, pelvis is oval and wider; about 24% of women have this shape.

When milia occur in newborn mouth and gums what are they called?

Epstein's pearls

Attachment

Development of a strong affection between infant and a significant other

Commitment

Enduring nature of the relationship

Which description of the four stages of labor is correct for both definition and duration?

First stage: onset of regular uterine contractions to full dilation; less than 1 hour to 20 hours

While providing care to a patient in active labor, the nurse should instruct the woman that:

Frequent changes in position will help relieve her fatigue and increase her comfort.

What are newborn weights recorded in?

Grams

What is the tx for newborn with hypoglycemia?

IV if acute, rapid acting sugar water mix or early formula

Which of the following supports why a preterm fetus usually is more affected by medication given at birth than a full-term fetus?

Inability of the immature liver to metabolize or inactivate drugs

Signs that precede labor include

Lightening. Bloody show. Rupture of membranes.

What are symptoms of hypoglycemia

Jitteriness, lethargy cyanosis, apnea, seizures, high pitched or weak cry, hypothermia, and poor feeding

The nurse observes tiny white pinpoint papules on a newborn's nose. The nurse documents this finding as: a) Vernix caseosa b) Harlequin sign c) Lanugo d) Milia

Milia

When does moro reflex appear?

Newborn , it disappear 3-6 mo

Can we put a cold pack on a healing perineum?

No b/c you want blood to get to site to heal it

The factors that affect the process of labor and birth, known commonly as the five Ps, include all except:

Pressure.

Baby Tarik has been circumcised, his temperature is stable, his breathing and heart rate are healthy, and he is ready to be discharged from the hospital. What can the nurse tell his parents to be on the lookout for that might indicate that Tarik needs medical attention? a) Redness at the base of the umbilical cord b) Crying for 2 hours or more each day c) Straining when he is passing stools d) A yellowish crusty substance on the circumcision site

Redness at the base of the umbilical cord

Letting-go?

Reestablishes relationships w/ others Adapts to parenthood More confident

Attachment?

Strong affection between infant + mother BABY

With regard to the turns and other adjustments of the fetus during the birth process, known as the mechanism of labor, nurses should be aware that:

The effects of the forces determining descent are modified by the shape of the woman's pelvis and the size of the fetal head.

In order to evaluate the condition of the patient accurately during labor, the nurse should be aware that:

The endogenous endorphins released during labor will raise the woman's pain threshold and produce sedation.

Concerning the third stage of labor, nurses should be aware that

The third stage of labor lasts from birth of the fetus until the placenta is delivered. The duration may be as short as 3 to 5 minutes, although up to 1 hour is considered within normal limits.

Which nursing assessment indicates that a woman who is in second-stage labor is almost ready to give birth?

The vulva bulges and encircles the fetal head.

Which reason explains why women should be encouraged to perform Kegel exercises after birth?

They promote blood flow, enabling healing and muscle strengthening. Exercising the pubococcygeal muscle increases blood flow to the area. The increased blood flow brings oxygen and other nutrients to the perineal area to aid in healing. Additionally, these exercises help strengthen the musculature, thereby decreasing the risk of future complications, such as incontinence and uterine prolapse. Performing Kegel exercises may assist with lochia removal, but that isn't their main purpose. Bowel function is not influenced by Kegel exercises. Kegel exercises do not generate sufficient energy expenditure to burn many calories.

Which statement is false regarding bathing the newborn?

To reduce the risk of heat loss, the bath should be performed by the nurse, not the parents, within 2 to 4 hours of birth. Bathing the newborn is not necessary for thermal stability. It can be postponed until the parents are able to do it

A 23-year-old P1011 has just delivered a term infant who is not crying and has decreased tone. Place the nursing actions in the order they should be carried out. Clear the airway. Check the heart rate. Dry the newborn. Transfer the newborn to a preheated radiant warmer. Stimulate the newborn by rubbing the back.

Transfer the newborn to a preheated radiant warmer. Dry the newborn. Clear the airway. Stimulate the newborn by rubbing the back. Check the heart rate. Correct Explanation: Commonly the first step in a nursing intervention cascade is assessment. However, the nurse already has assessed that the newborn is term, is not crying, and has decreased tone that would require intervention. The first step is to warm the newborn and then to decrease any further loss of heat through evaporation by drying the newborn. The airway should be cleared before the newborn is stimulated to avoid aspiration. The nurse would then check the heart rate to see if further resuscitation efforts are necessary.

What should be avoided until the cord has fallen off?

Tub bath

When assessing the umbilical cord of a newborn, which of the following would the nurse expect to find? a) Three arteries and no veins b) Two arteries and two veins c) Two arteries and one vein d) One artery and two veins

Two arteries and one vein

What are milia?

Unopened sebaceous gland that are frequently found on the newborn's nose. Will disappear in 2-4 weeks.

When does the postpartum assessment begin?

W/in 1 hr of delivery

A nurse is caring for a client in her fourth stage of labor. Which of the following assessments would indicate normal physiologic changes occurring during the fourth stage of labor? Select all that apply. a) Well-contracted uterus in the midline b) Mild uterine cramping and shivering c) Increase in the blood pressure d) Decreased intra-abdominal pressure e) Decrease in the pulse rate

a) Well-contracted uterus in the midline b) Mild uterine cramping and shivering d) Decreased intra-abdominal pressure Rationale: The normal physiologic changes for which a nurse should assess during the fourth stage of labor are a well-contracted uterus in the midline of the abdomen, mild cramping pain and generalized shivering, and decreased intra-abdominal pressure. Hemodynamic changes are due to normal blood loss during delivery, causing moderate tachycardia and a slight fall in the blood pressure during the fourth stage of labor. A fall in the pulse rate and increased blood pressure are not normal findings occurring during the fourth stage of labor.

One of the theories about the onset of labor is the prostaglandin theory. While not being conclusively proven that the action of prostaglandins initiate labor, it is known that prostaglandins do play a role in labor. What is an action of prostaglandins? a) Stimulates uterine muscle to relax b) Softens cervix c) Initiates relaxation of perineum d) Initiates cervical dilation

b) Softens cervix Rationale: The prostaglandin theory is another theory of labor initiation. Prostaglandins influence labor in several ways, which include softening the cervix and stimulating the uterus to contract. However, evidence supporting the theory that prostaglandins are the agents that trigger labor to begin is inconclusive.

When does rooting reflex appear?

birth, disappear 4-6 mo

Cephaalhematoma involes the collection of

blood and does not cross the suture line..serious

In preparing for the actual birth, which fetal presentation would a nurse be least likely to find? a) Transverse lie b) Breech c) Shoulder d) Oblique lie

c) Shoulder Rationale: Shoulder presentations are the least likely to occur in less than 0.3 percent of all births. Approximately 97 percent of fetuses are in a cephalic presentation at the end of pregnancy. A longitudinal lie, in which the long axis of the fetus is parallel to the long axis of the mother, is the most common. When the fetus is in a transverse lie, the long axis of the fetus is perpendicular to the long axis of the woman. An oblique lie is in between the two. (

Which position would the nurse suggest for second-stage labor if the pelvic outlet needs to be increased? a) Semirecumbent b) Sitting c) Squatting d) Side-lying

c) Squatting

The nurse is concerned with the interactions between a mother and her 2-day-old infant. The nurse observes signs of impaired bonding and attachment. Which action should the nurse document as a cause for concern?

calling the baby it or they Many new parents will need assistance with diaper changes; this is not a flag for concern; making eye contact and breastfeeding are positive interaction behaviors; if the mother calls the baby "it" and does not use the child's name, this is a sign that further information needs to be gathered and assessments should be completed.

Forces of contractions, mild asphyxia, increased intracranial pressure, and cold stress all play a role in the newborn transition by releasing which of the following critical components? a) cortisol b) epinephrine c) norepinephrine d) catecholamines

catecholamines

Which intervention has been demonstrated to reduce the release of catecholamines and anxiety, and has resulted in better birth outcomes for women?

continuous labor support Continuous labor support has been demonstrated to result in better labor outcomes in the current research. Continuous labor support is defined as caring for the labor patient by a nurse, doula, or primary support person who does not leave the client. Massage therapy is effective in that it engages gate control. Pharmacological interventions are useful but pose potential side effects to the mother and fetus. Hypnosis is less well understood.

A client is now in the second stage of labor. While doing the assessment, the nurse would gather what data at this time?

contraction pattern every 15 minutes Assess the contraction pattern every 15 minutes. The pattern will be similar to that found in the transition phase (i.e., contractions occur every two to three minutes, last 60 to 90 seconds, and are of strong intensity).

A client is now in the second stage of labor. While doing the assessment, the nurse would gather what data at this time?

contraction pattern every 15min.

When educating clients in a maternal-newborn unit about prevention of infant abduction, what is essential in the effectiveness of prevention of abduction?

cooperation by the parents with the hospital policies The most essential piece to an effective infant abduction prevention plan is the cooperation of the parents. If the parents are not willing to participate in the unit policy, the unit is at risk. Posting security policies, placing monitors on the babies, and educating the staff about infant abduction profiles are not the most essential elements of an effective abduction prevention plan.

What term is used to describe the position of the fetal long axis in relation to the long axis of the mother? a) Fetal position b) Fetal presentation c) Fetal attitude d) Fetal lie

d) Fetal lie Rationale: Fetal lie describes the position of the long axis of the fetus in relation to the long axis of the pregnant woman.

The nurse is assessing the laboring client to determine fetal oxygenation status. What indirect assessment method will the nurse likely use?

external electronic fetal monitoring Analysis of the FHR using external electronic fetal monitoring is one of the primary evaluation tools used to determine fetal oxygen status indirectly. Fetal pulse oximetry measures fetal oxygen saturation directly and in real time. It is used with electronic fetal monitoring as an adjunct method of assessment when the FHR pattern is abnormal or inconclusive. Fetal scalp blood is obtained to measure the pH. The fetal position can be determined through ultrasonography or abdominal palpation but is not indicative of fetal oxygenation.

What should be protected during phototherapy?

eyes

Maternal-infant Bonding

feeding time is a time for mother baby bonding eye contact sucking/rooting reflexes attachment Evidence-based practice shows best time to promote bonding is immediately after birth via placing the baby to mothers chest/breast.

As a woman enters the second stage of labor, which would the nurse expect to assess?

feelings of being frightened by the change in contractions

If a fetus were not receiving enough oxygen during labor because of uteroplacental insufficiency, which pattern would the nurse anticipate seeing on the monitor?

fetal heart rate declining late with contractions and remaining depressed Lack of blood supply to the fetus because of poor placental filling prevents the fetal heart rate from recovering immediately following a contraction.

One of the nurse's responsibilities is to educate new parents on the best method to prevent infections in the newborn environment. Which method would the nurse identify as best to control infection?

handwashing Educate parents about appropriate home measures that will prevent infections, such as practicing good handwashing, keeping the newborn well hydrated, avoiding bringing the infant into crowds, observing for early signs of infection, and keeping pediatrician appointments for routine visits.

While assessing a newborn, the nurse notes that half the body appears red while the other half appears pale. The nurse interprets this finding as:

harlequin sign. Harlequin sign refers to the dilation of blood vessels on only one side of the body. It gives a distinct midline demarcation, which is pale on one side and red on the opposite. Stork bites are superficial vascular areas found on the nape of the neck, eyelids, between the eyes and upper lip. Mongolian spots are blue or purple splotches that appear on the lower back and buttocks. Erythema toxicum is a benign, idiopathic, generalized, transient rash that resembles flea bites.

Screening for this most common birth defect is required by law in most states. Each nurse should know the law for his or her state and the requirements for screening. The nurse would expect a newborn to be screened for which defect as the most common?

hearing Hearing loss is the most common birth defect in the United States: one in 1,000 newborns are profoundly deaf, and 3 in 1,000 have some degree of hearing impairment. Newborn hearing screening is required by law in most states. Vision, genetic-linked, and skeletal malformations are other forms of birth defects that can occur.

While teaching a student, the nurse should include which signs and symptoms to recognize hypoglycemia in the neonate? Select all that apply.

jitteriness poor feeding tachypnea Signs and symptoms of hypoglycemia in newborns include jitteriness or tremors, exaggerated Moro reflex, irritability, lethargy, poor feeding, listlessness, apnea, respiratory distress including tachypnea, and a high-pitched cry.

During an initial newborn assessment, the nurse recognizes certain signs need to be reported to the primary care provider as they indicate potential problems. Which signs might indicate a problem? Select all that apply.

labored breathing generalized cyanosis flaccid body posture During the initial newborn assessment, the nurse should look for signs that might indicate a problem, including nasal flaring, chest retractions, grunting on exhalation, labored breathing, generalized cyanosis, abnormal breath sounds, abnormal respiratory rates (less than 25 or more than 60 breaths per minute), flaccid body posture, abnormal heart rates (less than 100 or more than 160 beats per minute), or abnormal size.

A woman who gave birth to a healthy baby 5 days ago is experiencing fatigue and weepiness, lasting for short periods each day. Which condition does the nurse believe is causing this experience?

postpartum baby blues Postpartum baby blues is common in women after giving birth. It is a mild depression; however, functioning usually is not impaired. Postpartum blues usually peaks at day 4 or 5 after birth. Postpartum anxiety and postpartum depression do not usually start until at least 3 to 4 weeks and up to 1 year following the birth of a baby. Postpartum reaction is a term to include postpartum depression, anxiety, and psychosis.

A nurse is caring for a client on the second day postpartum. The client informs the nurse that she is voiding a large volume of urine frequently. Which factor should the nurse identify as a potential cause for urinary frequency?

postpartum diuresis The nurse should identify postpartum diuresis as the potential cause for urinary frequency. Urinary overflow occurs if the bladder is not completely emptied. Urinary tract infection may be accompanied by fever and a burning sensation. Trauma to pelvic muscles does not affect urinary frequency.

A client has presented in the early phase of labor, experiencing abdominal pain and signs of growing anxiety about the pain. Which pain management technique should the nurse prioritize at this stage?

practicing effleurage on the abdomen

When assessing fetal heart rate patterns, which finding would alert the nurse to a possible problem?

prolonged decelerations

Blinking, gagging, sneezing, coughing reflexes

protective reflexes that are elicited when an object comes close to the eye (blinking) something irritating is swallowed or bulb syringe is used while suctioning (gagging and coughing) or an irritant is brought close to the nose (sneezing).

A nursing student correctly chooses which stage of behavioral adaptation in the infant to reinforce teaching about feeding, positioning for feeding, and diaper-changing techniques? a) none are good times b) first period of reactivity c) second period of reactivity d) period of decreased responsiveness

second period of reactivity

A nursing student observing newborns in the nursery is amazed that a crying infant put her fingers in her mouth, instantly stops crying, and then falls asleep. This behavior can best be explained as: a) motor maturity. b) the sleep state. c) social behavior. d) self-quieting ability.

self-quieting ability.

While caring for a client following a lengthy labor and birth, the nurse notes that the client repeatedly reviews her labor and birth and is very dependent on her family for care. The nurse is correct in identifying the client to be in which phase of maternal role adjustment?

taking-in The taking-in phase occurs during the first 24 to 48 hours following the birth of the newborn and is characterized by the mother taking on a very passive role in caring for herself, as well as recounting her labor experience. The second maternal adjustment phase is the taking-hold phase and usually lasts several weeks after the birth. This phase is characterized by both dependent and independent behavior, with increasing autonomy. During the letting-go phase the mother reestablishes relationships with others and accepts her new role as a parent. Acquaintance/attachment phase is a newer term that refers to the first 2 to 6 weeks following birth when the mother is learning to care for her baby and is physically recuperating from the pregnancy and birth.

How do we monitor mother's interest in the newborn?

Is she feeding/caring for the newborn? Is she interested or disinterested? Does she want the baby in the nursery all the time? Post partum screening tool

The nurse is preparing to apply a thermistor probe to a newborn to monitor the newborn's temperature. At which location would the nurse most likely apply the probe? a) Lower back b) Upper left arm c) Right great toe d) Right upper abdominal quadrant

Right upper abdominal quadrant

What is a Moro reflex and how do you check it, what is normal findings?

"embrace reflex" occurs when neonate startled place newborn on back, support the upper body weight of the supine newborn by the arms using a lifting motion without lifting the newborn off the surface, release the arms suddenly. Expected/normal reflex = newborn will throw arms outward and flex the knees, the arms return to the chest.

New parents are upset their newborn has lost weight since birth. The nurse explains that newborns typically lose how much of their birth weight by 3 to 4 days of age? A) 10% B) 12% C) 14% D) 16%

A) 10% Newborns typically lose approximately 10% of their initial birth weight by 3 to 4 days of age secondary to the loss of meconium, extracellular fluid, and limited food intake. This weight loss is usually regained by the 10th day of life.

At what point should the nurse expect a healthy newborn to pass meconium? a) By 12 to 18 hours of life b) Within 24 hours after birth c) Within 1 to 2 hours of birth d) Before birth

Within 24 hours after birth Correct Explanation: The healthy newborn should pass meconium within 24 hours of life.

The standard of care and recommendation by the Centers for Disease Control is to administer an immunization to all newborns. Which immunization is recommended to be administered prior to discharge?

hep B Hep B is the vaccination against hepatitis B recommended by the CDC. All the other immunizations are recommended to be started at 2 months of age

What is the Normal respirations for newborn

30-60 at rest, will increase when crying

How many ID bracelets does the baby have?

4

How does a woman who feels in control of the situation during labor influence her pain? a) Feelings of control are inversely related to the patient's report of pain. b) There is no association between the two factors. c) Feeling in control shortens the overall length of labor. d) Decreased feeling of control helps during the third stage.

Feelings of control are inversely related to the patient's report of pain. Correct Explanation: Studies reveal that women who feel in control of their situation are apt to report less pain than those who feel they have no control.

Breast care?

Well fitting, supportive bra 24 hrs/day Ice packs to sore breasts Air dry after feeding Lanolin cream once per day No stim/heat if bottle feeding Cabbage leaves may help dry milk

When doing vital sign on baby...

do full minute on respirations and pulse

With jaundice what is seen within 3 days of life?

yellow sclera, mucous membranes, sclera

What is pitocin drug half-life and where should it be administered and why?

• 6 min. 1/2 life • Closest port to IV site • To provide tight control of medication infusion

The nurse tests the pH of fluid found on the vaginal exam and determines that the woman's membranes have ruptured based on which result?

6.5

The nurse tests the pH of fluid found on the vaginal exam and determines that the woman's membranes have ruptured based on which result?

6.5 Amniotic fluid is alkaline, so the membranes are probably ruptured if the pH ranges from 6.5 to 7.5.

A client who gave birth 5 days ago reports profuse sweating during the night. What should the nurse recommend to the client in this regard?

"Be sure to change your pajamas to prevent you from chilling." The nurse should encourage the client to change her pajamas to prevent chilling and reassure the client that it is normal to have postpartal diaphoresis. Drinking cold fluids at night will not prevent postpartum diaphoresis.

To meet the needs for milk production, the woman nutritional needs increase as follows:

Calories: +500 cal a day Protein: +20 g a day Calcium: +400 mg daily Fluid: +2 to 3 quarts of fluids daily

-Fetal Heart Rate Pattern Acid-Base Status -Category 3

-Category 3 Abnormal: Predictive of abnormal fetus acid-base status -Fetal bradycadia (less than 110 bpm) -Recurrent late or variable decelerations -Sinusoidal pattern (smooth, undulating baseline) -Interventions: give maternal O2, change position, d/c labor augmentation medications, tx maternal HTN

To indicate that the infant is making a successful transition immediately after birth, the nurse checks the heart rate for 6 seconds. What should the count minimally be?

11 The normal infant heart rate should be greater than 100 bpm. Twelve is an acceptable heart rate in an infant, but it is not the minimally accepted heart rate.

The nurse tests the pH of fluid found on the vaginal exam and determines that the woman's membranes have ruptured when the result is which of the following? a) 5.5 b) 6.5 c) 5.0 d) 6.0

6.5 Correct Explanation: Amniotic fluid is alkaline, so the membranes are probably ruptured if the pH ranges from 6.5 to 7.5.

"1. How can a mother achieve the football-hold position for breastfeeding?"

"1. The football hold is achieved by holding the infant's back and shoulders in the palm of the mother's hand and tucking the infant under the mother's arm. The infant's ear, shoulder, and hip should be in a straight line. The mother's hand should support the breast and bring it to the infant's lips to latch on until the infant begins to nurse. This position allows the mother to see the infant's mouth as she guides her infant to the nipple. Mothers who have had a cesarean birth can avoid pressure on the inci- sion lines by adopting the football hold position for breastfeeding."

"Karen, a first-time mother, is worried that her baby does not sleep properly and wakes up every 2 hours. Karen informs the nurse that she often brings the baby to her bed to nurse and falls asleep with the baby in her bed. 1. What information should the nurse offer regarding the sleeping habits of newborns?"

"1. The nurse should inform the mother that newborns usually sleep for up to 20 hours daily, for periods of 2 to 4 hours at a time, but not through the night. This is because their stomach capacity is too small to go long periods of time without nour- ishment. All newborns develop their own sleep patterns and cycles."

11. "The mother of a newborn observes a diaper rash on her baby's skin. Which of the fol- lowing should the nurse instruct the parent to prevent diaper rash? a. Expose the newborn's bottom to air several times a day. b. Use plastic pants while bathing the new- born. c. Use products such as powder and items with fragrance. d. Place the newborn's buttocks in warm water often."

"11. Answer: a RATIONALE: The nurse should instruct the par- ent to expose the newborn's bottom to air several times per day to prevent diaper rashes. Use of plastic pants and products such as powder and items with fragrance should be avoided. The parent should be instructed to place the newborn's buttocks in warm water after having had a diaper on all night."

12. "A nurse is caring for a newborn with transient tachypnea. What nursing interventions should the nurse perform while providing supportive care to the newborn? Select all that apply. a. Provide warm water to drink. b. Provide oxygen supplement. c. Massage the newborn's back. d. Ensure the newborn's warmth. e. Observe respiratory status frequently."

"12. Answer: b, d, & e RATIONALE: The nurse should give the new- born oxygen, ensure the newborn's warmth, and observe the newborn's respiratory status frequently. The nurse need not give the newborn warm water to drink or massage the newborn's back."

13. "A nurse is caring for a newborn with hypo- glycemia. What symptoms of hypoglycemia should the nurse monitor the newborn for? Select all that apply. a. Lethargy b. Low-pitched cry c. Cyanosis d. Skin rashes e. Jitteriness"

"13. Answer: a, c, & e RATIONALE: The nurse should monitor the new- born for lethargy, cyanosis, and jitteriness. Low- pitched crying or rashes on the infant's skin are not signs generally associated with hypoglycemia."

14. "A mother who is 4 days postpartum, and is breastfeeding, expresses to the nurse that her breast seems to be tender and engorged. What education should the nurse give to the mother to relieve breast engorgement? Select all that apply. a. Take warm-to-hot showers to encourage milk release. b. Feed the newborn in the sitting position only. c. Express some milk manually before breastfeeding. d. Massage the breasts from the nipple toward the axillary area. e. Apply warm compresses to the breasts prior to nursing."

"14. Answer: a, c, & e RATIONALE: To relieve breast engorgement in the client, the nurse should educate the cli- ent to take warm-to-hot showers to encourage milk release, express some milk manually before breastfeeding, and apply warm compresses to the breasts before nursing. The mother should be asked to feed the newborn in a variety of positions—sitting up and then lying down. The breasts should be massaged from under the axil- lary area, down toward the nipple."

15. "A nurse is performing a detailed newborn assessment of a female baby. Which of the following observations indicate a normal finding? Select all that apply. a. Mongolian spots b. Enlarged fontanelles c. Swollen genitals d. Low-set ears e. Short, creased neck"

"15. Answer: a, c, & e RATIONALE: Mongolian spots, swollen genitals in the female baby, and a short, creased neck are normal findings in a newborn. Mongolian spots are blue or purple splotches that appear on the lower back and buttocks of newborns. Female babies may have swollen genitals as a result of maternal estrogen. The newborn's neck will appear almost nonexistent because it is so short. Creases are usually noted. Enlarged fontanelles are associated with malnutrition; hydrocephaly; congenital hypothyroidism; triso- mies 13, 18, and 21; and various bone disorders such as osteogenesis imperfecta. Low-set ears are characteristic of many syndromes and genetic abnormalities such as trisomies 13 and 18 and internal organ abnormalities involving the renal system."

A nurse is working with the parents of a newborn. The parents have a 2-year-old boy at home. Which statement would the nurse include when teaching these parents?

"Ask your 2-year-old to pick out a special toy for his sister." The parents should encourage the sibling to participate in some of the decisions about the baby, such as names or toys. Typically siblings experience some regression with the birth of a new baby. The parents should talk to the sibling during relaxed family times. The parents should arrange for the sibling to come to the hospital to see the newborn.

2. "A nurse is caring for a 5-hour-old newborn. The physician has asked the nurse to maintain the newborn's temperature between 97.7° and 99.5° F (between 36.5° and 37.5° C). What nursing intervention should the nurse perform to maintain the temperature within the recommended range? a. Avoid measuring the weight of the infant, as scales may be cold. b. Use the stethoscope over the baby's garment. c. Place the newborn close to the outer wall in the room. d. Place the newborn skin-to-skin with the mother."

"2. Answer: d RATIONALE: The nurse should place the newborn skin-to-skin with mother. This would help to maintain baby's temperature as well as promote breastfeeding and bonding between the mother and baby. The nurse can weigh the infant as long as a warmed cover is placed on the scale. The stethoscope should be warmed before it makes contact with the infant's skin, rather than using the stethoscope over the garment because it may obscure the reading. The newborn's crib should not be placed close to the outer walls in the room to prevent heat loss through radiation."

"2. What is colostrum?"

"2. Colostrum is a thick, yellowish substance secreted during the first few days after birth. It is high in protein, minerals, and fat-soluble vitamins. It is rich in immunoglobulins (e.g., IgA), which help protect the newborn's GI tract against infections. It is a natural laxative to help rid the intestinal tract of meconium quickly."

"2. What safety precautions should the mother take when putting the baby to sleep?"

"2. The nurse should ask the mother to place the newborn on her back to sleep; remove all fluffy bedding, quilts, sheepskins, stuffed animals, and pillows from the crib to prevent potential suffocation. Parents should avoid unsafe conditions such as placing the newborn in the prone position, using a crib that does not meet federal safety guidelines, allowing window cords to hang loose and in close proximity to the crib, or having the room temperature too high, causing overheating."

3. "As a part of the newborn assessment, the nurse determines the skin turgor. Which of the following nursing interventions is relevant when observing the turgor of the newborn's skin? a. Pinch skin and note return to original position. b. Examine for stork bites or salmon patches. c. Check for unopened sebaceous glands. d. Inspect for blue or purple splotches on buttocks."

"3. Answer: a RATIONALE: Skin turgor is checked by pinching the skin over chest or abdomen and noting the return to original position; if the skin remains "tented" after pinching, it denotes dehydration. Stork bites or salmon patches, unopened sebaceous glands, and blue or purple splotches on buttocks are common skin variations not related to skin turgor."

"Which of the following information should the nurse give to a client who is breastfeeding her newborn regarding the nutritional requirements of newborns, as per the recommendations of the American Academy of Pediatrics (AAP)? a. Feed the infant at least 10 mL per kg of water daily." "b. Give iron supplements to the newborn daily. c. Give vitamin D supplements daily for the first 2 months. d. Ensure adequate fluoride supplementation."

"4. Answer: c RATIONALE: As per the recommendations of AAP, all newborns should receive a daily supplement of vitamin D during the first 2 months of life to prevent rickets and vitamin D deficiency. There is no need to feed the newborn water, as breast milk contains enough water to meet the newborn's needs. Iron supplements need not be given, as the newborn is being breastfed. Infants over 6 months of age are given fluoride supplementation if they are not receiving fluoridated water."

"4. How can a nurse test Moro reflex?"

"4. The Moro reflex, or the embrace reflex, occurs when the neonate is startled. To elicit this reflex, the newborn is placed on his back. The upper body weight of the supine newborn is supported by the arms with use of a lifting motion, without lifting the newborn off the surface. When the arms are released suddenly, the newborn will throw the arms outward and flex the knees; arms then return to the chest. The fingers also spread to form a C. The newborn initially appears startled and then relaxes to a normal resting position."

"5. A first-time mother informs the nurse that she is unable to breastfeed her baby through the day as she is usually away at work. She adds that she wants to express her breast milk and store it for her baby. What instruction should the nurse offer the woman to ensure the safety of stored expressed breast milk? a. Use sealed and chilled milk within 24 hours b. Use frozen milk within 6 months of obtaining it c. Use microwave ovens to warm chilled milk d. Refreeze any unused milk for later use"

"5. Answer: a RATIONALE: The nurse should instruct the woman to use the sealed and chilled milk within 24 hours. The nurse should not instruct the woman to use frozen milk within 6 months of obtaining it, to use microwave ovens to warm chilled milk, or to refreeze the used milk and reuse it. Instead, the nurse should instruct the woman to use frozen milk within 3 months of obtaining it, to avoid using microwave ovens to warm chilled milk, and to discard any used milk and never refreeze it."

"5. What is caput succedaneum?"

"5. Caput succedaneum is a localized edema on the scalp that occurs from the pressure of the birth process. It is commonly observed after prolonged labor. Clinically, it appears as a poorly demarcated soft tissue swelling that crosses suture lines. Pitting edema and overlying petechiae and ecchymosis are noted. The swelling will gradually dissipate in about 3 days without any treatment. Newborns who were delivered via vacuum extraction usually have a caput in the area where the cup was used."

"6. A nurse is educating the mother of a new- born about feeding and burping. Which of the following strategies should the nurse offer to the mother regarding burping? a. Hold the baby upright with the baby's head on her mother's shoulder. b. Lay the baby on its back on its mother's lap. c. Gently rub the baby's abdomen while the baby is in a sitting position. d. Lay the baby on its mother's lap and give it frequent sips of warm water."

"6. Answer: a RATIONALE: The nurse should ask the mother to hold the baby upright with the baby's head on her mother's shoulder. Alternatively, the nurse can also suggest the mother sit with the newborn on her lap with the newborn lying face down. Gently rubbing the baby's abdomen or giving frequent sips of warm water to the infant will not significantly induce burping; burping is induced by the newborn's position. Placing the baby on her back while trying to elicit burping after feeding may cause choking or aspiration."

"6. What is erythema toxicum?"

"6. Erythema toxicum is a benign, idiopathic, very common, generalized, transient rash occurring in as many as 70% of all newborns during the first week of life. It consists of small papules or pustules on the skin resembling flea bites. The rash is common on the face, chest, and back. One of the chief characteristics of this rash is its lack of pattern. It is caused by the newborn's eosinophils reacting to the environment as the immune system matures. It does not require any treatment, and it disap- pears in a few days."

"7. The mother of a formula-fed newborn asks how she will know if her newborn is receiving enough formula during feedings. Which response by the nurse is correct? a. "Your newborn should finish a bottle in less than 15 minutes." b. "A sign of good nutrition is when your newborn seems satisfied and is gaining sufficient weight." c. "If your newborn is wetting three to four diapers and producing several stools a day, enough formula is likely being consumed." d. "Your newborn should be taking about 2 oz of formula for every pound of body weight during each feeding.""

"7. Answer: b RATIONALE: A sign of adequate formula intake is when the newborn seems satisfied and is gaining weight regularly. The formula fed newborn should take 30 minutes or less to finish a bottle, not less than 15 minutes. The newborn does normally produce several stools per day, but should wet 6 to 10 diapers rather than 3 to 4 per day. The newborn should consume approximately 2 oz of formula per pound of body weight per day, not per feeding."

"8. A nurse, while examining a newborn, observes salmon patches on the nape and on the eyelids. Which of the following is the most likely cause of the salmon patches?" "a. Concentration of pigmented cells b. Eosinophils reacting to environment c. Immature autoregulation of blood flow d. Concentration of immature blood vessels"

"8. Answer: d RATIONALE: A concentration of immature blood vessels causes salmon patches. Mongolian spots are caused by a concentration of pigmented cells and usually disappear within the first 4 years of life. Erythema toxicum is caused by the newborn's eosinophils reacting to the environment as the immune system matures, and Harlequin sign is a result of immature autoregulation of blood flow and is commonly seen in low-birth-weight newborns."

9. "A nurse is required to obtain the temperature of a healthy newborn who is placed in an ordinary crib. Which of the following is the most appropriate method for measuring a newborn's temperature? a. Tape electronic thermistor probe to the abdominal skin. b. Obtain temperature orally. c. Place electronic temperature probe in the midaxillary area. d. Obtain temperature rectally."

"9. Answer: c RATIONALE: The nurse should obtain a new- born's temperature by placing an electronic tem- perature probe in the midaxillary area. The nurse should not tape an electronic thermistor probe to the abdominal skin, as this method is applied only when the newborn is placed under a radiant heat source. Rectal temperatures are no longer taken because of the risk of perforation. Oral temperature readings are not taken for newborns."

The mother of a formula-fed newborn asks how she will know if her newborn is receiving enough formula during feedings. Which response by the nurse is correct? a) "If your newborn is wetting three to four diapers and producing several stools a day, enough formula is likely being consumed." b) "Your newborn should be taking about 2 oz of formula for every pound of body weight during each feeding." c) "A sign of good nutrition is when your newborn seems satisfied and is gaining sufficient weight." d) "Your newborn should finish a bottle in less than 15 minutes."

"A sign of good nutrition is when your newborn seems satisfied and is gaining sufficient weight."

The mother of a formula-fed newborn asks how she will know if her newborn is receiving enough formula during feedings. Which response by the nurse is correct?

"A sign of good nutrition is when your newborn seems satisfied and is gaining sufficient weight." A sign of adequate formula intake is when the newborn seems satisfied and is gaining weight regularly. The formula fed newborn should take 30 minutes or less to finish a bottle, not less than 15 minutes. The newborn does normally produce several stools per day, but should wet 6 to 10 diapers rather than 3 to 4 per day. The newborn should consume approximately 2 oz of formula per pound of body weight per day, not per feeding.

A woman who gave birth to her infant 1 week ago calls the clinic to report pain with urination and increased frequency. What response by the nurse is appropriate?

"After birth it is easier to develop an infection in the urinary system; we need to see you today." The urinary system is more susceptible to infection during the postpartum period. The woman needs to be checked to rule out a urinary infection. The other responses are incorrect because they do not acknowledge her in an appropriate manner.

1. "The nurse caring for a newborn has to perform assessment at various intervals. When should the nurse complete the second assessment for the newborn? a. Immediately after birth, in the birthing area b. Within the first 2 to 4 hours, when the newborn is in the nursery c. Before the newborn is discharged d. The day after the newborn's birth"

"Answer: b RATIONALE: The nurse should complete the sec- ond assessment for the newborn within the first 2 to 4 hours, when the newborn is in the nursery. The nurse should complete the initial newborn assessment in the birthing area and the third assessment before the newborn is discharged."

10. "A nurse observes that a newborn has a 1-minute Apgar score of 5 points. What should the nurse conclude from the observed Apgar score? a. Severe distress in adjusting to extrauter- ine life b. Better condition of the newborn c. Moderate difficulty in adjusting to extra- uterine life d. Abnormal central nervous system status"

"Answer: c RATIONALE: The nurse should conclude that the newborn is facing moderate difficulty in adjusting to extrauterine life. The nurse need not conclude severe distress in adjusting to extra- uterine life, better condition of the newborn, or abnormal central nervous system status. If the Apgar score is 8 points or higher, it indicates that the condition of the newborn is better. An Apgar score of 0 to 3 points represents severe distress in adjusting to extrauterine life."

A client who recently gave birth to her third child expresses a desire to have her older two come to the hospital for a visit. What should the nurse say in response to this request?

"As long as they are well, absolutely. Why don't we give you a dose of pain medication beforehand so that you will enjoy the visit?" Separation from children is often as painful for a mother as it is for her children. A chance to visit the hospital and see the new baby and their mother reduces feelings that their mother cares more about the new baby than about them. It can help to not only relieve some of the impact of separation but also to make the baby a part of the family. Assess to be certain siblings are free of contagious diseases such as upper respiratory tract illnesses or recent exposure to chickenpox before they visit. Then, have them wash their hands and, if they choose, hold or touch the newborn with parental assistance. Allowing the siblings to walk with the baby out in the hall unsupervised would be unsafe.

When counseling a patient about the advantages of circumcision, which should NOT be included in the nurse's teaching? a) "Circumcision decreases rates of urinary tract infection." b) "Circumcision decreases risks of skin dehiscence, adhesions, and urethral fistulas." c) "Circumcision decreases rates of penile cancer." d) "Males who are circumcised have lower rates of sexually transmitted infection."

"Circumcision decreases risks of skin dehiscence, adhesions, and urethral fistulas."

When counseling a patient about the advantages of circumcision, which should NOT be included in the nurse's teaching? a) "Circumcision decreases rates of penile cancer." b) "Circumcision decreases risks of skin dehiscence, adhesions, and urethral fistulas." c) "Males who are circumcised have lower rates of sexually transmitted infection." d) "Circumcision decreases rates of urinary tract infection."

"Circumcision decreases risks of skin dehiscence, adhesions, and urethral fistulas." Correct Explanation: Newborn males who are circumcised have higher rates of skin dehiscence, adhesions, and urethral fistulas.

A client asks her nurse what effleurage means. After instruction is given, the nurse determines learning has taken place when the client states:

"Effleurage is light abdominal massage used to displace pain." Effleurage is a light abdominal massage used to keep the laboring woman's focus on the massage instead of the pain of labor.

After teaching a group of pregnant women about the skin changes that will occur after the birth of their newborn, the nurse understands there is a need for additional teaching when one of the women makes which statement?

"I can't wait for these stretch marks to disappear after I give birth." Stretch marks gradually fade to silvery lines but do not disappear completely. As estrogen and progesterone levels decrease, the darkened pigmentation on the abdomen, face, and nipples gradually fades.

When assessing a new father's adaptation to his new role, which statement would indicate that he is in the reality stage?

"I didn't realize all that went into being a dad. I wasn't prepared for this." The statement about not feeling prepared reflects the realization that the man's expectations were not realistic. Many wish to be more involved but do not feel prepared to do so, and this is characteristic of the second stage, reality. The statement that it will be fun to have a baby around but life will not change too much indicates a preconceived idea about what home life will be like with a newborn; this is characteristic of the first stage, expectations. The statement about things not changing reflects the first stage of expectations, where the partner is unaware of the changes that may occur after the birth of the newborn. The statement about learning new skills and enjoying being involved indicate a conscious decision to be at the center of the newborn's life; this is characteristic of the third stage, transition to mastery.

After teaching a postpartum woman about postpartum blues, which statement indicates effective teaching? a) "I should call this support line only if I hear voices." b) "I might feel like laughing one minute and crying the next." c) "I'll need to take medication to treat the anxiety and sadness." d) "If the symptoms last more than a few days, I need to call my doctor."

"I might feel like laughing one minute and crying the next." Correct Explanation: Emotional lability is typical of postpartum blues. Further evaluation is necessary if symptoms persist for more than 2 weeks. Postpartum blues are usually self-limiting and require no medication. Support lines can be used whenever the woman feels down.

A woman states that she does not want any medication for pain relief during labor. Her doctor has approved this for her. What is your best response to her concerning this choice? a) "I respect your preference whether it is to have medication or not." b) "Your doctor (a man) has never been in labor; he may be underestimating the pain you will have." c) "That's wonderful. Medication during labor is not good for the baby." d) "Let me get you something for relaxation if you don't want anything for pain."

"I respect your preference whether it is to have medication or not." Correct Explanation: Individualizing care to meet women's specific needs is a nursing responsibility.

A patient who gave birth 2 hours ago expresses concern about her baby developing jaundice. How should the nurse respond? Choose the best response. a) "I understand your concern because as many as 50% of babies can develop jaundice." b) "If you are concerned about your baby developing jaundice, don't breastfeed your baby until you get home." c) "We will monitor the baby now, and your baby will not develop jaundice after the first 24 hours of life." d) "You don't need to worry about your baby developing jaundice because you are both A+."

"I understand your concern because as many as 50% of babies can develop jaundice."

A patient who gave birth 2 hours ago expresses concern about her baby developing jaundice. How should the nurse respond? Choose the best response. a) "You don't need to worry about your baby developing jaundice because you are both A+." b) "I understand your concern because as many as 50% of babies can develop jaundice." c) "If you are concerned about your baby developing jaundice, don't breastfeed your baby until you get home." d) "We will monitor the baby now, and your baby will not develop jaundice after the first 24 hours of life."

"I understand your concern because as many as 50% of babies can develop jaundice." Correct Explanation: As many as 50% of term newborns will develop physiologic jaundice. Physiologic jaundice occurs after the first 24 hours of life and is not pathologic. Pathologic jaundice will develop within the first 24 hours of life.

The nurse is educating a client who is breastfeeding her 2-week-old newborn regarding the nutritional requirements of newborns, according to the recommendations of the American Academy of Pediatrics (AAP). Which response by the mother would validate her understanding of the information she received? a) "I will feed him at least 30 cc of water daily." b) "Since we live in a rural area, I must ensure he receives adequate fluoride supplementation." c) "I need to give him iron supplements daily." d) "I will give him vitamin D supplements daily for the first 2 months of life."

"I will give him vitamin D supplements daily for the first 2 months of life."

The nurse has presented a teaching session to graduate nurses on physiologic jaundice. Which student statement indicates that additional teaching is needed? a) "Physiologic jaundice begins before the neonate goes home." b) "Physiologic jaundice happens as a result of a breakdown of RBCs." c) "Physiologic jaundice happens because the RBC count built in utero is being decreased." d) "Physiologic jaundice begins in the first 24 hours of after birth."

"Physiologic jaundice begins in the first 24 hours of after birth."

The nurse is educating a client who is breastfeeding her 2-week-old newborn regarding the nutritional requirements of newborns, according to the recommendations of the American Academy of Pediatrics (AAP). Which response by the mother would validate her understanding of the information she received?

"I will give him vitamin D supplements daily for the first 2 months of life." As per the recommendations of AAP, all newborns should receive a daily supplement of vitamin D during the first 2 months of life to prevent rickets and vitamin D deficiency. There is no need to feed the newborn water, as breast milk contains enough water to meet the newborn's needs. Iron supplements need not be given, as the newborn is being breastfed. Infants over 6 months of age are given fluoride supplementation if they are not receiving fluoridated water.

A nurse recommends to a client in labor to try concentrating intently on a photo of her family as a means of managing pain. The woman looks skeptical and asks, "how would that stop my pain?" Which of the following explanations should the nurse give? a) "It disrupts the nerve signal of pain via mechanical irritation of the nerves." b) "It blocks the transmission of nerve messages of pain at the receptors." c) "It causes the release of endorphins." d) "It distracts your brain from the sensations of pain."

"It distracts your brain from the sensations of pain." Correct Explanation: Concentrating intently on an object is another method of distraction, or another method of keeping sensory input from reaching the cortex of the brain. The other answers refer to other means of pain management.

A mother just delivered 3 hours ago. The nurse enters the room to continue hourly assessments and finds the patient on the phone telling the listener about her fear while driving to the hospital and not making it in time. The mother finishes the call, and the nurse begins her assessment with which phrase? a) "If you plan to breastfeed, you need to calm down." b) "You have a beautiful baby, why worry about that now?" c) "I need to assess your fundus now." d) "It sounded like you had quite a time getting here. Would you like to continue your story?"

"It sounded like you had quite a time getting here. Would you like to continue your story?" Correct Explanation: The mother is going through the taking-in phase of relating events during her pregnancy and delivery. The nurse can facilitate this phase by allowing the mother to express herself. Diverting the conversation, admonishing the mother, or warning of potential problems does not accomplish this facilitation.

A nurse is making an initial call on a new mother who gave birth to her third baby five days ago. The woman says,"I just feel so down this time. Not at all like when I had my other babies. And this one just doesn't sleep. I feel so inadequate." What is the best response to this new mother?

"It sounds like you have the 'baby blues.' They are common after having a baby when you are not getting enough sleep, are busy with your other children, and are still a bit uncomfortable from the birth. They will most likely go away in a day or two." A combination of factors likely contributes to the baby blues. Psychological adjustment along with a physiologic decrease in estrogen and progesterone appear to be the greatest contributors. Additional contributing factors include too much activity, fatigue, disturbed sleep patterns, and discomfort.

You are the home health nurse making an initial call on a new mother who delivered her third baby five days ago. The woman says to you "I just feel so down this time. Not at all like when I had my other babies. And this one just doesn't sleep. I feel so inadequate." What is the best response to this new mother? a) "It sounds like you have the 'baby blues.' They are common after having a baby when you are not getting enough sleep, are busy with your other children, and are still a bit uncomfortable from the delivery. They will most likely go away in a day or two." b) "Tell me, are you seeing things that aren't there, or hearing voices?" c) "Every baby is different with their own temperament. Maybe this one just isn't ready to sleep when you want him to." d) "It sounds like you need to make an appointment with a counselor. You may have postpartum depression."

"It sounds like you have the 'baby blues.' They are common after having a baby when you are not getting enough sleep, are busy with your other children, and are still a bit uncomfortable from the delivery. They will most likely go away in a day or two." Explanation: A combination of factors likely contributes to the baby blues. Psychological adjustment along with a physiologic decrease in estrogen and progesterone appear to be the greatest contributors. Additional contributing factors include too much activity, fatigue, disturbed sleep patterns, and discomfort.

A new mother is concerned because it is 24 hours after birth and her breasts have still not become engorged with breast milk. How should the nurse respond to this concern?

"It takes about 3 days after birth for milk to begin forming." The formation of breast milk (lactation) begins in a postpartal woman regardless of her plans for feeding. For the first 2 days after birth, an average woman notices little change in her breasts from the way they were during pregnancy as, since midway through pregnancy, she has been secreting colostrum, a thin, watery, prelactation secretion. On the third day post birth, her breasts become full and feel tense or tender as milk forms within breast ducts and replaces colostrum. There is no need to recommend formula feeding to the mother. Mastitis is inflammation of the lactiferous (milk-producing) glands of the breast; there is no indication that the client has this condition. Lactational amenorrhea is the absence of menstrual flow that occurs in many women during the lactation period.

A client who gave birth vaginally 16 hours ago states she does not need to void at this time. The nurse reviews the documentation and finds that the client has not voided for 7 hours. Which response by the nurse is indicated?

"It's not uncommon after birth for you to have a full bladder even though you can't sense the fullness." After a vaginal birth, the client should be encouraged to void every 4 to 6 hours. As a result of anesthesia and trauma, the client may be unable to sense the filling bladder. It is premature to catheterize the client without allowing her to attempt to void first. There is no need to contact the primary care provider at this time, because the client is demonstrating common adaptations in the early postpartum period. Allowing the client's bladder to fill for another 2 to 3 hours might cause overdistention.

A nurse is discharge teaching with a group of new parents before they are discharged home with their infants. One couple inquires as to why they need to place their new baby on its back to sleep. What is the nurse's best response?

"Research has shown that placing an infant on its back to sleep reduces the risk for SIDS."

A client who gave birth vaginally 16 hours ago states she does not need to void at this time. The nurse reviews the documentation and finds that the client has not voided for 7 hours. Which response by the nurse is indicated?

"It's not uncommon after birth for you to have a full bladder even though you can't sense the fullness." After a vaginal birth, the client should be encouraged to void every 4 to 6 hours. As a result of anesthesia and trauma, the client may be unable to sense the filling bladder. It is premature to catheterize the client without allowing her to attempt to void first. There is no need to contact the primary care provider at this time, because the client is demonstrating common adaptations in the early postpartum period. Allowing the client's bladder to fill for another 2 to 3 hours might cause overdistention.

A woman comes to the nursery and states "Mrs. Smith is ready for her baby. I will be glad to take the baby to her." The woman is dressed in hospital scrub attire but has no name badge showing. What is the best response by the nurse caring for the baby? a) "You must be Mrs. Smith's sister. She said her sister is a nurse." b) "Leave immediately! I'm calling security." c) "May I see your identification, please?" d) "I don't know you. Are you trying to take a baby?"

"May I see your identification, please?"

A woman comes to the nursery and states "Mrs. Smith is ready for her baby. I will be glad to take the baby to her." The woman is dressed in hospital scrub attire but has no name badge showing. What is the best response by the nurse caring for the baby? a) "Leave immediately! I'm calling security." b) "May I see your identification, please?" c) "I don't know you. Are you trying to take a baby?" d) "You must be Mrs. Smith's sister. She said her sister is a nurse."

"May I see your identification, please?" Correct Explanation: Each member of the hospital staff should have an identification badge clearly displayed. The nurse in the nursery is appropriate in asking to see the identification of the woman who is offering to take Mrs. Smith's baby to her. Education and watchful vigilance are the keys to preventing infant abductions. Each facility that cares for newborns should have specific policies and procedures in place that address this problem. Review these policies and know the protocols for the facility in which you will be working.

A nursing student asks the instructor why are all the babies in the nursery are wrapped up like it is freezing and are wearing little hats. Which would be the best response? a) "Newborns lose body heat and need to be kept warm until their temperature stabilizes." b) "Studies show that newborns like the extra warmth." c) "They look so cute like that." d) "That's how we have always done it and it seems to work."

"Newborns lose body heat and need to be kept warm until their temperature stabilizes." Correct Explanation: Nurses provide an appropriate environment to help newborns maintain thermal stability. Newborns lose body heat easily and need to kept warm until their temperature stabilizes. The other answers are not adequate and do not explain the correct rationale.

A nursing student observes that the babies in the nursery are wrapped up warmly and are wearing knit caps. Which explanation by the nursery staff would be the most correct?? a) "Newborns lose body heat easily and need to be kept warm until their body temperature stabilizes." b) "Studies show that newborns like the extra warmth." c) "That's how we have always done it and it seems to work out well." d) "The caps and blankets simulate the temperature of he mother's womb that they are used to."

"Newborns lose body heat easily and need to be kept warm until their body temperature stabilizes."

The primapara tells the nurse, "My baby jumps every time I pick her up. Is she afraid that I will drop her?" Which response by the nurse would be best?

"No, it is the Moro reflex. This reflex simulates the action of warding off an attacker." The Moro reflex is known as the startle reflex. A startled newborn will extend the arms and legs away from the body and to the side. Then the arms come back toward each other with the fingers spread in a "C" shape. The arms look as if the newborn is trying to embrace something. The Moro reflex should be symmetrical.

A nursing instructor teaching students how to check the patient's uterus postpartum realizes that further instruction is needed when one of the students says: a) "Normally the fundus progresses downward at a rate of 1 fingerbreadth per day after birth." b) "One to two hours after birth the fundus is typically at the level of the umbilicus." c) "Six to twelve hours after birth the fundus is typically at the level of the umbilicus." d) "One to two hours after birth the fundus is typically between the umbilicus and symphysis pubis."

"One to two hours after birth the fundus is typically at the level of the umbilicus." Explanation: One to two hours after birth the fundus is typically between the umbilicus and symphysis pubis. At 6ix to 12 hours after birth the fundus usually is at the level of the umbilicus. Normally the fundus progresses downward at at rate of one fingerbreadth per day after birth.

A client gave birth vaginally 2 days prior and wishes to prevent getting pregnant again. She asks the nurse when she will need to begin birth control measures. How should the nurse respond?

"Ovulation may return as soon as 3 weeks after birth." Ovulation may start at soon as 3 weeks after birth. The client needs to be aware and use a form of birth control. She needs to be cleared by her provider prior to intercourse if she has a vaginal birth, but in the event that she has intercourse, needs to be prepared for the possibility of pregnancy. Ovulation can occur without the return of the menstrual cycle, and ovulation does return sooner than six months after birth.

Opioids are often used in labor for pharmacologic pain management. A patient in the transition phase of labor is requesting fentanyl (Sublimaze) for pain. How should the nurse respond to her request? a) "I will page the provider and ask for your pain medication." b) "You are so close to delivery; don't you want to have natural child birth?" c) "Rather than use fentanyl, we can ask the provider to order another analgesic at this point." d) "Pain medication given now might cause the baby to have slow respirations and is not recommended, let's try to focus and breathe."

"Pain medication given now might cause the baby to have slow respirations and is not recommended, let's try to focus and breathe." Correct Explanation: Once the woman has entered into the transition phase of labor, she is considered to be imminent for delivery. Any opioid medication might pass to the fetus and is not recommended due to the effects of respiratory compromise. The nurse will need to encourage nonpharmacologic methods at this point and should not consult the provider. The nurse should also remain supportive of the mother.

You are doing discharge teaching with a group of new parents before they are discharged home with their infant. One set of parents inquire as to why they need to place their new baby on its back to sleep. What is your best response?

"Research has shown that placing an infant on its' back to sleep reduces the risk for SIDS." Correct Explanation: Newborns should always be placed on their backs to sleep to reduce the risk for SIDS.

Two days after giving birth, a client is to receive Rho(D) immune globulin. The client asks the nurse why this is necessary. The most appropriate response from the nurse is:

"Rho(D) immune globulin suppresses antibody formation in a woman with Rh-negative blood who gave birth to a baby with Rh-positive blood." Rho(D) immune globulin is indicated to suppress antibody formation in women with Rh-negative blood who gave birth to babies with Rh-positive blood. Rho(D) immune globulin is also given to women with Rh-negative blood after miscarriage/pregnancy termination, abdominal trauma, ectopic pregnancy, and amniocentesis.

Two days after giving birth, a client is to receive RhoGAM. The client asks the nurse why this is necessary. The most appropriate response from the nurse is: a) "RhoGAM suppresses antibody formation in a woman with Rh-negative blood who gave birth to a baby with Rh-negative blood." b) "RhoGAM suppresses antibody formation in a woman with Rh-positive blood who gave birth to a baby with Rh-negative blood." c) "RhoGAM suppresses antibody formation in a woman with Rh-negative blood who gave birth to a baby with Rh-positive blood." d) "RhoGAM suppresses antibody formation in a woman with Rh-positive blood who gave birth to a baby with Rh-positive blood."

"RhoGAM suppresses antibody formation in a woman with Rh-negative blood who gave birth to a baby with Rh-positive blood." Correct Explanation: RhoGAM is indicated to suppress antibody formation in women with Rh-negative blood who gave birth to babies with Rh-positive blood. RhoGAM is also given to women with Rh-negative blood after miscarriage/pregnancy termination, abdominal trauma, ectopic pregnancy, and amniocentesis.

A woman who is breast-feeding her newborn says, "He doesn't seem to want to nurse. I must be doing something wrong." Which response by the nurse would be least helpful? a) "Some babies latch on and catch on quickly; others take a little more time." b) "Some women just can't breast-feed. Maybe you're one of these women." c) "Breast-feeding takes time. Let's see what's happening." d) "Let me contact our lactation specialist and together maybe we can work through this."

"Some women just can't breast-feed. Maybe you're one of these women." Correct Explanation: This response ignores the woman's feelings and displays a negative attitude, indicating that the woman is at fault for the current situation. The woman needs reassurance that she can breast-feed and accomplish the task. She needs to understand that although breast-feeding is a natural process, it takes time and practice. By offering to observe her breast-feeding, the nurse offers support and can provide the woman with some practical suggestions as necessary. The statement that some babies need more time would reduce her frustration and uncertainty about her ability to breast-feed. A lactation consultant can provide the woman with additional support and teaching to foster empowerment in this situation

A woman refuses to have an epidural block because she does not want to have a spinal headache after delivery. Which of the following would be your best response? a) "Spinal headache is not a usual complication of epidural blocks." b) "The pain relief offered will compensate for the discomfort afterward." c) "Your doctor knows what is best for you." d) "The anesthesiologist will do her best to avoid this."

"Spinal headache is not a usual complication of epidural blocks." Correct Explanation: Because epidural anesthesia does not enter the cerebral spinal fluid space, it is unlikely to cause a "spinal headache."

Which instruction should the nurse provide to a breastfeeding woman experiencing breast engorgement?

"Take a warm shower just before feeding your infant." Standing in a warm shower or applying warm compresses immediately before feedings will help soften the breasts and nipples to allow the newborn to latch on more easily and will enhance the let-down reflex. Wearing a tight supportive bra all day is appropriate for the woman who is not breastfeeding. Frequent emptying of the breasts helps to resolve engorgement, so the mother should be encouraged to feed the newborn, which would involve touching her breasts and nipples. The breastfeeding woman should apply cold compresses but not ice to her breasts between feedings to reduce swelling.

A woman's husband expresses concern about risk of paralysis from an epidural block being given to his wife. Which of the following would be the most appropriate response? a) "The injection is given at the third or fourth thoracic vertebrae so paralysis is not a problem." b) "I have never read or heard of this happening." c) "The injection is given in the space outside the spinal cord." d) "An injury is unlikely because of expert professional care given."

"The injection is given in the space outside the spinal cord." Correct Explanation: An epidural block, as the name implies, does not enter the spinal cord but only the epidural space outside the cord.

The parents of a newborn baby boy ask you about circumcising their son. They are undecided as to what to do. Which response by the nurse is best? a) "It is best not to circumcise your baby because the procedure is very painful." b) "Circumcision is best in order to protect the baby from diseases like cancer." c) "If you do not circumcise your baby, he will always have difficulty maintaining adequate hygiene." d) "There are pros and cons to circumcision. Let me ask the pediatrician to come and talk to you about the procedure."

"There are pros and cons to circumcision. Let me ask the pediatrician to come and talk to you about the procedure."

The parents of a newborn baby boy ask you about circumcising their son. They are undecided as to what to do. Which response by the nurse is best? a) "If you do not circumcise your baby, he will always have difficulty maintaining adequate hygiene." b) "It is best not to circumcise your baby because the procedure is very painful." c) "There are pros and cons to circumcision. Let me ask the pediatrician to come and talk to you about the procedure." d) "Circumcision is best in order to protect the baby from diseases like cancer."

"There are pros and cons to circumcision. Let me ask the pediatrician to come and talk to you about the procedure." Correct Explanation: If the parents decide to have their male newborn circumcised, informed consent is necessary. It is the physician's responsibility to obtain informed consent, although you may be responsible for witnessing the parents' signatures to a written documentation of that consent. If the parents have unanswered questions, notify the physician before the procedure is done.

A mother is concerned because her daughter has lost 8 ounces 3 days after birth. What response by the nurse is appropriate? a) "Your baby needs to be checked for a viral illness." b) "Your baby is probably just dehydrated." c) "This is a normal and expected finding." d) "You need to give your baby formula since she has lost weight during breastfeeding."

"This is a normal and expected finding." Correct Explanation: The infant has a 5-10% loss of birth weight during the first few days of life as the body looses excess fluid and has limited food intake. You would not tell the new mother that her infant needs to be checked for a viral illness, this is inappropriate because if the infant were ill you would have no way of knowing if it was a viral or a bacterial disease process. Option C is incorrect as weight loss in a newborn is a normal finding. Option D is incorrect as a new breastfeeding mother should not supplement feedings with formula.

When instructing a new mom on providing skin care to her newborn, which of the following should NOT be included in the teaching? a) "Give the newborn sponge baths until the umbilical cord falls off." b) "Change diapers frequently." c) "Use talc powders to prevent diaper rash." d) "Daily tub baths are not necessary."

"Use talc powders to prevent diaper rash."

When instructing a new mom on providing skin care to her newborn, which of the following should NOT be included in the teaching? a) "Change diapers frequently." b) "Give the newborn sponge baths until the umbilical cord falls off." c) "Use talc powders to prevent diaper rash." d) "Daily tub baths are not necessary."

"Use talc powders to prevent diaper rash." Correct Explanation: Talc powders can be a respiratory hazard and should not be used with a newborn.

How should the nurse counsel the postpartum patient about sleep and her newborn? a) "In the first few days at home, you can expect your newborn to sleep 22 hours in a 24-hour period." b) "Using a fan in the baby's room or opening a window may reduce the incidence of sudden infant death syndrome (SIDS)." c) "Always put the baby on his back or side to sleep until he is able to hold his neck up." d) "Introducing solid foods early will help the baby sleep at night."

"Using a fan in the baby's room or opening a window may reduce the incidence of sudden infant death syndrome (SIDS)." Correct Explanation: Introducing solid foods early will not help the baby sleep better at night. A normal newborn sleeps a lot, but less than 22 hours a day in the first week of life. The American Academy of Pediatrics recommends always putting a baby on his back to sleep. Oxygen-rich air from a fan or a window has been shown to decrease SIDS.

How should the nurse counsel the postpartum patient about sleep and her newborn? a) "Introducing solid foods early will help the baby sleep at night." b) "Always put the baby on his back or side to sleep until he is able to hold his neck up." c) "In the first few days at home, you can expect your newborn to sleep 22 hours in a 24-hour period." d) "Using a fan in the baby's room or opening a window may reduce the incidence of sudden infant death syndrome (SIDS)."

"Using a fan in the baby's room or opening a window may reduce the incidence of sudden infant death syndrome (SIDS)." Explanation: Introducing solid foods early will not help the baby sleep better at night. A normal newborn sleeps a lot, but less than 22 hours a day in the first week of life. The American Academy of Pediatrics recommends always putting a baby on his back to sleep. Oxygen-rich air from a fan or a window has been shown to decrease SIDS.

New parents are getting ready to go home and have received information to help them learn how best to care for the new infant. Which statement indicates that they need additional teaching about how to soothe their newborn if he becomes upset?

"We'll hold off on feeding him for a while because he might be too full." The parents need more teaching that feeding or burping can be helpful in relieving air or stomach gas. Turning on a mobile above the newborn's head is helpful in calming the newborn. The movement is distracting, and the music is comforting. The newborn's back should be rubbed lightly while the parents speak softly to him. Swaddling the newborn provides security and comfort.

A nurse teaches new parents about how to soothe their crying newborn. Which statement by the parents indicates that they understand how to soothe their newborn if he becomes upset?

"We'll turn the mobile on that's hanging above his head in his crib." Turning on a mobile above the newborn's head is helpful in calming the newborn. The movement is distracting, and the music is comforting. The newborn's back should be rubbed lightly while the parents speak softly to him. Swaddling the newborn rather than having him lie on a blanket on the floor provides security and comfort. Feeding or burping can be helpful in relieving air or stomach gas.

Which of the following statements by the parents of a newborn indicate that they understand how to soothe their newborn if he becomes upset? a) "We'll place him on his belly on a blanket on the floor." b) "We'll turn the mobile on that's hanging above his head in his crib." c) "We'll vigorously rub his back as we play some music." d) "We'll hold off on feeding him for a while because he might be too full."

"We'll turn the mobile on that's hanging above his head in his crib." Correct Explanation: Turning on a mobile above the newborn's head is helpful in calming the newborn. The movement is distracting and the music is comforting. The newborn's back should be rubbed lightly while the parents speak softly to him. Swaddling the newborn rather than having him lie on a blanket on the floor provides security and comfort. Feeding or burping can be helpful in relieving air or stomach gas.

The nursing instructor is teaching a group of nursing students about the various responsibilities of the labor and delivery medical team. The instructor determines the session is successful when the students correctly choose which function as the primary role of the LPN/LVN members of the team?

Provide care under the supervision of an RN.

A nursing instructor is teaching about changes the newborn must make to survive outside of the uterus. The instructor realizes that further teaching is needed when a student says which of the following? a) "When the baby is ready to leave the uterus, it takes its first breath." b) "When the umbilical cord is clamped the first breath is taken." c) "When the umbilical cord is clamped the lungs begin to function." d) "The first breath is taken when the baby is stimulated by a slight slap."

"When the baby is ready to leave the uterus, it takes its first breath."

A nursing instructor is teaching about changes the newborn must make to survive outside of the uterus. The instructor realizes that further teaching is needed when a student says which of the following? a) "When the umbilical cord is clamped the first breath is taken." b) "When the baby is ready to leave the uterus, it takes its first breath." c) "The first breath is taken when the baby is stimulated by a slight slap." d) "When the umbilical cord is clamped the lungs begin to function."

"When the baby is ready to leave the uterus, it takes its first breath." Correct Explanation: Changes in circulation begin immediately at birth as the fetus separates from the placenta. When the umbilical cord is clamped, the first breath is taken and the lungs begin to function.

A patient expresses concern to the nurse that her baby is dehydrated and is not getting enough milk from breastfeeding. What is the best response from the nurse? a) "We will give him some water through a bottle in the nursery tonight while you rest." b) "Does he pass urine that is a light amber color right after eating?" c) "You can tell that your baby is adequately hydrated because he is making 8 wet diapers a day." d) "You should supplement with formula because your baby is 24 hours old and has not passed meconium yet."

"You can tell that your baby is adequately hydrated because he is making 8 wet diapers a day."

A client expresses concern to the nurse that her baby is dehydrated and is not getting enough milk from breastfeeding. What is the best response from the nurse?

"You can tell that your baby is adequately hydrated because he is making 8 wet diapers a day." The nurse will know that a newborn is adequately hydrated if he has 6 to 12 wet diapers a day. It is still within normal limits if the newborn has not passed meconium by 24 hours of age. Although urinating after feeding is common, it is not essential to ensure adequate hydration.

A patient expresses concern to the nurse that her baby is dehydrated and is not getting enough milk from breastfeeding. What is the best response from the nurse? a) "You can tell that your baby is adequately hydrated because he is making 8 wet diapers a day." b) "We will give him some water through a bottle in the nursery tonight while you rest." c) "Does he pass urine that is a light amber color right after eating?" d) "You should supplement with formula because your baby is 24 hours old and has not passed meconium yet."

"You can tell that your baby is adequately hydrated because he is making 8 wet diapers a day." Correct Explanation: The nurse will know that a newborn is adequately hydrated if he has 6 to 12 wet diapers a day. It is still within normal limits if the newborn has not passed meconium by 24 hours of age. Although urinating after feeding is common, it is not essential to ensure adequate hydration.

A woman asks the nurse if she can eat something during labor. Which response by the nurse would be best?

"You could have some hard candy to suck on." If women are kept NPO during labor, they can be administered anesthesia safely in an emergency. Stomach-emptying time is decreased.

A woman at 39 weeks gestation has been in labor for 8 hours and is asking how far she is dilated, she attended child birth classes and is aware of the stages and phases of labor. She had a vaginal exam 30 minutes prior to her asking again. How should the nurse respond to her question? a) "Checking your cervix will not speed up labor, let's wait." b) "The health care provider will have to check you, we can call them." c) "I can arrange for a cervix check, if you want." d) "You labor signs have not changed; we are looking for changes in your labor pattern before we check you again."

"You labor signs have not changed; we are looking for changes in your labor pattern before we check you again." Correct Explanation: The cervix must be assessed with a vaginal exam. The frequency of vaginal exams is based on the signs of changes in labor. The patient has not demonstrated any changes in her labor pattern; the nurse should provide education on the reason for not checking her.

A woman who gave birth to a healthy newborn 2 months ago comes to the clinic and reports discomfort during sexual intercourse. Which suggestion by the nurse would be most appropriate?

"You might try using a water-soluble lubricant to ease the discomfort." Coital discomfort and localized dryness usually plague most postpartum women until menstruation returns. Water-soluble lubricants can reduce discomfort during intercourse. Although it may take some time for the woman's body to return to its prepregnant state, telling the woman this does not address her concern. Telling her that dyspareunia is normal and that it takes time to resolve also ignores her concern. Kegel exercises are helpful for improving pelvic floor tone but would have no effect on vaginal dryness.

A woman who gave birth to a healthy newborn 2 months ago comes to the clinic and reports discomfort during sexual intercourse. Which suggestion by the nurse would be most appropriate? a) "It takes a while to get your body back to its normal function after having a baby." b) "Try doing Kegel exercises to get your pelvic muscles back in shape." c) "This is entirely normal, and many women go through it. It just takes time." d) "You might try using a water-soluble lubricant to ease the discomfort."

"You might try using a water-soluble lubricant to ease the discomfort." Correct Explanation: Coital discomfort and localized dryness usually plague most postpartum women until menstruation returns. Water-soluble lubricants can reduce discomfort during intercourse. Although it may take some time for the woman's body to return to its prepregnant state, telling the woman this does not address her concern. Telling her that dyspareunia is normal and that it takes time to resolve also ignores her concern. Kegel exercises are helpful for improving pelvic floor tone but would have no effect on vaginal dryness.

A client has been discharged from the hospital after a cesarean birth. Which instruction should the nurse include in the discharge teaching?

"You should be seen by your healthcare provider if you have blurred vision." The client needs to notify the healthcare provider for blurred vision as this can indicate preeclampsia in the postpartum period. The client should also notify the healthcare provider for a temperature great than 100.4° F (38° C) or if a peri-pad is saturated in less than 1 hour. The nurse should ensure that the follow-up appointment is fixed for within 2 weeks after hospital discharge.

A client who gave birth to a baby 36 hours ago informs the nurse that she has been passing unusually large volumes of urine very often. How should the nurse explain this to the client?

"Your body usually retains extra fluids during pregnancy, so this is one way it rids itself of the excess fluid." Postpartum diuresis is due to the buildup and retention of extra fluids during pregnancy. Bruising and swelling of the perineum, swelling of tissues surrounding the urinary meatus, and decreased bladder tone due to anesthesia cause urinary retention

A woman at 39 weeks gestation has been in labor for 8 hours and is asking how far she is dilated. She attended childbirth classes and is aware of the stages and phases of labor. She had a vaginal exam 30 minutes prior to her asking again. How should the nurse respond to her question?

"Your labor signs have not changed; we are looking for changes in your labor pattern before we check you again." "Your labor signs have not changed; we are looking for changes in your labor pattern before we check you again." The cervix must be assessed with a vaginal exam. The frequency of vaginal exams is based on the signs of changes in labor. The client has not demonstrated any changes in her labor pattern; the nurse should provide education on the reason for not checking her.

Postpartum Danger Signs

- Fever >100.4 - Foul smelling lochia - Large blood clots - Severe headache/ blurred vision - Calf pain with dorislfexion of foot - Swelling, redness, discharge at the episiotomy - Dysuria, burning, or incomplete emptying of the bladder - SOB - Depression/mood swings

What uterine contraction intensity signals an initiation of cervical dilation?

- Intensity of contraction Over 30 mm Hg = initiation of cervical dilation. - 50-80 mm Hg = Active labor - Intensity of resting tone -5-10 mm Hg = early labor -12-18 mm Hg = active labor

Factors increasing a woman's risk for postpartum infection

- Operative procedure (forceps, section, vacuum) - Hx of diabetes - Prolonged labor (>24 hrs) - Anemia (hgb <10.5) - Prolonged rupture of membranes - Manual extraction of placenta - Compromised immune system (HIV positive)

Factors increasing a woman's risk for postpartum hemorrhage

- Precipitous labor - Uterine atony - Placenta previa - Labor induction or augmentation - Operative procedures - Retained placental fragments - Prologned third stage of labor - Multiparity - Uterine over distention

Question: During labor, progressive fetal descent occurs. Place the stations listed in their proper sequence. -4 station +2 station +4 station 0 station -2 station

-4 station -2 station 0 station +2 station +4 station Correct Explanation: Progressive fetal descent (-5 to +4) is the expected norm during labor, moving downward from the negative stations to zero station to the positive stations in a timely manner.

-Fetal Heart Rate Pattern Acid-Base Status -Category 2

-Category 2 Intermediate: not predictive of abnormal fetal acid base status - Fetal tachycardia present (above 160 bpm) - Bradycardia not accompanied by absent baseline variability -Absent variability not accompanied recurrent decelerations. -Minimal or marked variability - Recurrent late decelerations with moderate baseline variability -Recurrent variable decelerations accompanied by min/moderate baseline variability, overshoots, or shoulder -Prolong decelerations greater than 2 min but less than 10.

What are the 4 objectives of Healthy People 2020 when addressing maternal health?

-Reducing maternal deaths -Reducing maternal illness and complications due to pregnancy -Increasing women who receive early and adequate prenatal care -Increasing women who attend a series of prepared child birth classes.

What techniques are used to assess maternal's status?

-Used to determine women's response and her progress in labor. -Maternal's vital signs (temp, b/p, pulse, resp, pain) -Prenatal record (for risk factors decrease circulation) - Vaginal Exam (if there are no bleeding on admission) -Cervical dilation -Level of pain and pain management strategies

A nurse practitioner is conducting an in-service education program for a group of nurses working in the labor and birth unit. The program is focusing on interpreting FHR patterns. The nurse practitioner determines that the teaching was successful when the group identifies which patterns as indicating abnormal fetal acid-base status? Select all that apply -fetal bradycardia -fetal tachycardia -sinusoidal pattern -recurrent late decelerations

-fetal bradycardia -sinusoidal pattern -recurrent late decelerations

The nurse is assigned to a client on postpartum day 1. Prior to assessing her uterus, where should the nurse anticipate she will locate the fundus?

1 cm below the umbilicus The fundus of the uterus should be at the umbilicus after birth. Every day after birth it should decrease 1 cm until it is descended below the pubic bone.

APGAR is assessed when?

1 minute and 5 minutes after birth if the 5 minute score was less than 7 the APGAR will be reassessed again in 10 minutes

After pitocin is administed, how often is it increased determining on dilation and effacement?

1-2mu per 15 min 20 mu max. 30 mu max with physician orders

Transition to parenthood

1. Commitment, attachment, and preparation 2. Acquaintance with and increasing attachment to infant 3. Moving toward a new normal routine in the first 4 months after birth 4. Achievement of parenthood role around 4 months

Stages

1. Engagement 2. Descent 3. Flexion 4. Internal rotation 5. Extension 6. External rotation 7. Expulsion

What are the 2 types of Episiotomies? What are there benefits?

1. Midline (Faster Healing, Less Muscle Damage, Risk of tearing anus) 2. Medial Lateral (Slow healing, More muscle repair, No risk of tearing Anus)

What three test are done to confirm a rupture of the amniotic membranes?

1. Nitrazine Paper (Turn blue to change in pH) 2. Ferning Test (Fluid dried shows fern like design) 3. Free Flow (Cervical exam, pt asked to bear down, fluid seen on cervix)

Teaching guidelines for suppressing lactation?

1. Wear a supportive, snug bra 2. 5-7 days 3. Take mild analgesics for discomfort 4. Avoid breast stimulation 5. Drink to quench thirst 6. Reduce salt intake 7. use ice packs/cool compresses

A nurse is caring for a postpartum client who has a temperature. Which temperature protocols would the nurse use to indicate a possible infection?

100.5º F (38.1º C) at 48 hours postbirth and remains the same the third day postpartum A temperature that is greater than 100.4º F (38º C) on 2 postpartum days after the first 24 hours puts the client at risk for a postpartum infection. A fever in the first 24 hours of birth is considered normal and could be caused by dehydration and analgesia.

A client is in the active phase of labor. She is a low-risk client. The nurse evaluates the fetal monitor strip at 10:00 a.m. Moderate variability is present. The FHR is in the 130s with occasional accelerations, no decelerations. At what time does the nurse need to reevaluate the FHR?

10:30 a.m. Assess and document fetal status at least every 30 minutes. Record the baseline FHR every 30 minutes and evaluate the fetal monitor tracing for abnormal patterns. Variability should be present, except for brief periods of fetal sleep or when the mother receives narcotics or other selected medications, and no late decelerations should be present. Accelerations of the FHR are normal.

To indicate that the infant is making a successful transition immediately after birth, the nurse checks the heart rate for 6 seconds. What should the count minimally be? a) 9 b) 10 c) 11 d) 12

11 Explanation: The normal infant heart rate should be greater than 100 bpm. Therefore, options A and B are incorrect. Option D is an acceptable heart rate in an infant but is not the minimal accepted heart rate

What is the Normal HR for newborn

120-160..may be to 180 if crying

The heart rate of the newborn in the first few minutes after birth will be in which of the following ranges? a) 80-120 bpm b) 120-180 bpm c) 180-220 bpm d) 120-130 bpm

120-180 bpm

A new mother asks the nurse when the "soft spot" on her son's head will go away. The nurse's answer is based on the knowledge that the anterior fontanel closes after birth by _____ months.

18

The nurse assesses her client and notes that the fetus is at +1 station. The nurse interprets +1 station as indicating that the fetal presenting part is at

1cm below the ischial spine

The nurse is assigned to a patient on postpartum day 1. Prior to assessing her uterus, where should the nurse anticipate she will locate the fundus? a) At level of umbilicus b) 1cm below the umbilicus c) At the symphysis pubis d) 1cm above the umbilicus

1cm below the umbilicus Explanation: The fundus of the uterus should be at the umbilicus after delivery. Every day after delivery it should decrease 1cm until it is descended below the pubic bone. Therefore options A, B, and D are incorrect.

A client is having contractions every 6-8 minutes apart lasting for 30 seconds and are mild in intensity. Her cervix is 2cm dialated, what stage and phase of labor is she in?

Stage One, Latent(Early) Phase

Taking-hold?

2 wks post partum, lasts several wks Mother more self sufficient, but needs reassurance Preoccupied w/ present

Neonatal screening is done before the infant leaves the hospital. Blood is drawn through a heel stick and tested for several disorders that can cause lifelong disabilities. When is the ideal time to collect this specimen?

24 hours after the newborn's first protein feeding The laws in most states require this initial screening, which is done within 72 hours of birth. The ideal time to collect the specimen is after the newborn is 36 hours old and 24 hours after he has his first protein feeding.

Neonatal screening is done before the infant leaves the hospital. Blood is drawn through a heel stick and tested for several disorders that can cause lifelong disabilities. When is the ideal time to collect this specimen? a) When the infant is 48 hours old. b) 24 hours after the newborn's first protein feeding. c) 36 hours before the infant is discharged home with its parents. d) Just before discharge home..

24 hours after the newborn's first protein feeding.

Neonatal screening is done before the infant leaves the hospital. Blood is drawn through a heel stick and tested for several disorders that can cause lifelong disabilities. When is the ideal time to collect this specimen? a) 24 hours after the newborn's first protein feeding. b) When the infant is 48 hours old. c) Just before discharge home.. d) 36 hours before the infant is discharged home with its parents.

24 hours after the newborn's first protein feeding. Correct Explanation: The laws in most states require this initial screening, which is done within 72 hours of birth. The ideal time to collect the specimen is after the newborn is 36 hours old and 24 hours after he has his first protein feeding.

When can PKU be tested?

24-48 hours after feeding

How long is the neonatal period for a newborn? ____ days

28 Correct Explanation: The neonatal period is the first 28 days of life.

The nurse has completed the initial assessment and vital signs for an infant born at 12 noon. The assessment and vital signs were completed at 1:30pm. What time will the nurse plan to complete the next set of vital signs? a) 1:45pm b) 2:00pm c) 2:30pm d) 3:30pm

2:00pm

The nurse has completed the initial assessment and vital signs for an infant born at 12 noon. The assessment and vital signs were completed at 1:30pm. What time will the nurse plan to complete the next set of vital signs? a) 1:45pm b) 2:00pm c) 2:30pm d) 3:30pm

2:00pm Correct Explanation: The nurse needs to complete vital signs every half hour for the first 2 hours of life. This makes options A, C, and D incorrect.

A mother is upset because her newborn has lost 6 ounces since birth 2 days ago. The nurse informs the mother that it is normal for a newborn to lose which of the following within the first week of life? a) 10% to 15% of their birth weight b) 5% to 10% of their birth weight c) 15% to 18% of their birth weight d) 20% of their birth weight

5% to 10% of their birth weight

When caring for a newborn several hours after birth, you assess his respiratory rate. In a normal newborn, this would be a) 16 to 20 breaths/min. b) 30 to 60 breaths/min. c) 20 to 30 breaths/min. d) 12 to 16 breaths/min.

30 to 60 breaths/min. Correct Explanation: Newborns typically breathe more rapidly than adults or older children, at a rate of 30 to 60 breaths/min. 12 to 16 breaths/min is a normal respiratory rate for an adult; 16 to 20 breaths/min is normal for older children; 20 to 30 breaths/min is normal for preschoolers; and 30 to 60 breaths/min is normal for infants.

What is the expected range for respirations in a newborn? a) 10-30 breaths per minute b) 30-60 breaths per minute c) 20-40 breaths per minute d) 40-80 breaths per minute

30-60 breaths per minute

What is the expected range for respirations in a newborn? a) 20-40 breaths per minute b) 40-80 breaths per minute c) 10-30 breaths per minute d) 30-60 breaths per minute

30-60 breaths per minute Correct Explanation: Although episodic breathing is normal and short periods of apnea can occur, the normal respiratory rate for a newborn is 30-60 breaths per minute. For adults, it is typically 8-20 breaths per minute.

Normal respiratory of newborn (APGAR)

30-60 breaths/min, strong/good cry

Normal respirations in the newborn?

30-60, will increase with crying Always count respirations first because when you do the other assessment skills the newborn may start crying.

What concentrations do Pitocin usually come in?

30u/500L 20u/1000L 10u/1000L

Average head circumference for baby?

32-38 cm (13-15 inches)

What is the average wt of the baby?

3400 g (7.5lbs) can range 2500-4000 gram 5lbs 8oz- 8lbs 14oz

What is the average length of the newborn

50 cm ( 20 inches) 44-55cm range 17-22 inches

A nurse is serving as a doula to a client who is now in labor at an alternative birthing center. The client has opted for a water birth, and the nurse is now drawing the water into a large tub. What temperature should the nurse keep the water at? a) 35°C b) 41°C c) 39°C d) 37°C

37°C Correct Explanation: Standing under a warm shower or soaking in a tub of warm water, jet hydrotherapy tub, or whirlpool is another way to apply heat to help reduce the pain of labor. The temperature of water used should be 37°C to prevent hyperthermia of the woman and also the newborn at birth.

When caring for a client in the first stage of labor, the nurse documents cervical dilation of 9 cm and intense contractions lasting 45 to 60 seconds and occurring about every 2 minutes. Based on these findings, the nurse should recognize that the client is in which phase of labor? 1. Active phase 2. Latent phase 3. Descent phase 4. Transitional phase

4 RATIONALES: In the transitional phase, the cervix dilates from 8 to 10 cm, and intense contractions occur every 1½ to 2 minutes and last for 45 to 90 seconds. In the active phase, the cervix dilates from 5 to 7 cm, and moderate contractions progress to strong contractions that last 60 seconds. In the latent phase, the cervix dilates 3 to 4 cm, and contractions are short, irregular, and mild. No descent phase exists. (Fetal descent may begin several weeks before labor but usually doesn't occur until the second stage of labor.)

Which finding would the nurse describe as "light" or "small" lochia?

4-inch stain or a 1 to 25 ml loss Typically the amount of lochia is described as follows: scant: a 1- to 2-inch lochia stain on the pad or a 10 ml loss; light or small: 4-inch stain or a 10 to 25 ml loss; moderate: 4- to 6-inch stain with an estimated loss of 25 to 50 ml; large or heavy: a pad is saturated within 1 hour after changing it.

Which finding would the nurse describe as "light" or "small" lochia? a) pad is saturated within 1 hour after changing it b) 4-inch stain or a 1 to 25 ml loss c) 4- to 6-inch stain with an estimated loss of 25 to 50 ml d) 1- to 2-inch lochia stain on the perineal pad or a 10 ml loss

4-inch stain or a 1 to 25 ml loss Explanation: Typically the amount of lochia is described as follows: Scant: a 1- to 2-inch lochia stain on the pad or a 10 ml loss; Light or small: 4-inch stain or a 10 to 25 ml loss; Moderate: 4- to 6-inch stain with an estimated loss of 25 to 50 ml; Large or heavy: a pad is saturated within 1 hour after changing it

What is the Normal BP for the newborn?

50-75 systolic 30-45 diastolic

A newborn has a heart rate of 90 beats per minute, a regular respiratory rate of 40 breaths per minute, tight flexion of the extremities, a grimace when stimulated, and acrocyanosis. The nurse assigns an Apgar score of: a) 7 b) 6 c) 5 d) 8

7 Correct Explanation: The newborn would receive an Apgar score of 7: 1 point for heart rate (<100 beats/minute), 2 points for respiratory rate (regular respirations at a rate between 30 and 60 breaths/minute), 2 points for muscle tone (tight flexion), 1 point for reflex irritability (grimace), and 1 point for skin color (acrocyanosis).

You record a newborn's Apgar score at birth. A normal 1-minute Apgar score is a) 5 to 9. b) 7 to 10. c) 1 to 2. d) 12 to 15.

7 to 10.

The nurse records a newborn's Apgar score at birth. A normal 1-minute Apgar score is:

7 to 10. An Apgar score of 7 to 10 implies the infant is breathing well and cardiovascular adaptation is occurring.

You record a newborn's Apgar score at birth. A normal 1-minute Apgar score is a) 5 to 9. b) 1 to 2. c) 12 to 15. d) 7 to 10.

7 to 10. Correct Explanation: An Apgar score of 7 to 10 implies the infant is breathing well and cardiovascular adaptation is occurring.

A nurse is performing Apgar scoring on a newborn. The newborn demonstrates the following: a heart rate of 110 bpm; a good, strong cry; muscles of the extremities well flexed; a grimace in response to a slap to the sole of the foot; and normal pigment in most of the body, with blue at the extremities. Which score would be the total Apgar score for this newborn?

8 The heart rate of 110 bpm, the strong cry, and the muscles of the extremities being well flexed each indicate a score of 2 in the heart rate, respiratory effort, and muscle tone areas, respectively. The grimace in response to a slap to the sole of the foot and the blue at the extremities each indicate a score of 1 for the reflex irritability and color areas, respectively. Thus, the total Apgar score for this infant is 8.

APGAR <8 interpreted as ?

8 is great with 10 being the best 3-7 not good <3 a major concern

Which presentation is described accurately in terms of both presenting part and frequency of occurrence? a. Cephalic: occiput; at least 95% b. Shoulder: scapula; 10% to 15% c. Breech: sacrum; 10% to 15% d. Cephalic: cranial; 80% to 85%

A In cephalic presentations (head first), the presenting part is the occiput; this occurs in 96% of births. In a breech birth, the sacrum emerges first; this occurs in about 3% of births. In shoulder presentations, the scapula emerges first; this occurs in only 1% of births.

In relation to primary and secondary powers, the maternity nurse comprehends that: a. Primary powers are responsible for effacement and dilation of the cervix. b. Effacement generally is well ahead of dilation in women giving birth for the first time; they are closer together in subsequent pregnancies. c. Scarring of the cervix caused by a previous infection or surgery may make the delivery a bit more painful, but it should not slow or inhibit dilation. d. Pushing in the second stage of labor is more effective if the woman can breathe deeply and control some of her involuntary needs to push, as the nurse directs.

A The primary powers are responsible for dilation and effacement; secondary powers are concerned with expulsion of the fetus. Effacement generally is well ahead of dilation in first-timers; they are closer together in subsequent pregnancies. Scarring of the cervix may slow dilation. Pushing is more effective and less fatiguing when the woman begins to push only after she has the urge to do so.

Signs that precede labor include (Select all that apply): a. Lightening. b. Exhaustion. c. Bloody show. d. Rupture of membranes. e. Decreased fetal movement.

A, C, D Signs that precede labor may include lightening, urinary frequency, backache, weight loss, surge of energy, bloody show, and rupture of membranes. Many women experience a burst of energy before labor. A decrease in fetal movement is an ominous sign that does not always correlate with labor.

After teaching a class about the changes in the gastrointestinal system of a newborn, which of the following if stated by the class indicates the need for additional teaching? a) The newborn's stomach is sterile at birth b) Oral intake is required for the production of vitamin K c) A newborn's stomach capacity is approximately 300 mL d) The cardiac sphincter is immature.

A newborn's stomach capacity is approximately 300 mL. Correct Explanation: A newborn's stomach capacity is approximately 30 to 90 mL. The gut is sterile at birth but changes rapidly depending on what feeding is received. Colonization of the gut is dependent on oral intake; oral intake is required for the production of vitamin K. The cardiac sphincter and nervous control of the stomach are immature.

Bonding between a mother and her infant can be defined how? a) Family growing closer together after the birth of a new baby b) A process of developing an attachment and becoming acquainted with each other c) The skin to skin contact that occurs in the delivery room d) An ongoing process in the year after delivery

A process of developing an attachment and becoming acquainted with each other Explanation: Bonding in the maternal-newborn world is the attachment process that occurs between a mother and her newborn infant. This is how the mother and infant become engaged with each other and is the foundation for the relationship. Because bonding is a process and not a single event, option B is incorrect. The process of bonding is not a year-long process, so option C is incorrect. The family growing closer together after the birth of a new baby is not bonding, so option D is incorrect.

Upon assessing the newborn's respirations, when would the nurse need to notify the MD? a) A respiratory rate of 15 breaths per minute with nasal flaring b) A respiratory rate of 45 breaths per minute with acrocyanosis c) Coughing and sneezing in the newborn d) Short periods of apnea that last 10 seconds in a pink newborn

A respiratory rate of 15 breaths per minute with nasal flaring

A nurse is caring for a pregnant client at her 34-week checkup. The client has chosen the Lamaze method for her birthing plan but states that her partner does not agree. The client says she will just change her plan. Which response by the nurse would be appropriate to support the female client? A) "Have you and your partner discussed what his / her role will be in the birth?" B) "Just wait until the birth; your partner's mind could change." C) "Preparing for the birth works for some clients, but not for all." D) "Maybe you should choose a different support person."

A) "Have you and your partner discussed what his / her role will be in the birth?" The nurse should explain to the client the different roles the partner can take in the birth process. The nurse should encourage the client to discuss both her feelings and her partner's feelings to better understand the partner's disapproval of Lamaze. This could help improve family communication. The other three answers are not therapeutic because they ignore the client's concern and do not help to prepare the client.

A primigravida, who is 1 cm dilated, is in early latent labor. She has expressed a desire to avoid epidural anesthesia and asks about nonpharmacologic options for pain relief as her labor progresses. How could the nurse appropriately respond? Select all that apply. A) "The tub usually is not recommended when you are at such an early stage because it can cause your labor to slow down. Let's talk about using the tub as your labor progresses." B) "If you haven't already practiced these techniques, meditation and imagery won't work." C) "I can show you some simple breathing exercises that can help you relax." D) "You may want to go for a walk now. When you come back, I'll show you how to use the birthing ball." E) "If you are already asking about pain relief at this point in labor, you will most likely end up with an epidural anyway."

A) "The tub usually is not recommended when you are at such an early stage because it can cause your labor to slow down. Let's talk about using the tub as your labor progresses." C) "I can show you some simple breathing exercises that can help you relax." D) "You may want to go for a walk now. When you come back, I'll show you how to use the birthing ball." Hydrotherapy may slow labor if used too early. Although it is often more beneficial for the client to have practiced meditation, imagery, or breathing exercises before presenting in labor, the nurse can teach a laboring client simple exercises when she presents in labor. Position changes and ambulation can help ease labor pains. A birthing ball helps the patient continue to change positions while her weight is partially supported.

A nurse is preparing a patient for rhythm strip testing. She places the woman into a semi-Fowler's position. What is the appropriate rationale for this measure? A) To prevent supine hypotension syndrome B) To aid the woman as she pushes during labor C) To prevent the woman from falling out of bed D) To decrease the heart rate of the fetus

A) To prevent supine hypotension syndrome The term "rhythm strip testing" means assessment of the fetal heart rate for whether a good baseline rate and long- and short-term variability are present. For this, help a woman into a semi-Fowler's position (either in a comfortable lounge chair or on an examining table or bed with an elevated backrest) to prevent her uterus from compressing the vena cava and causing supine hypotension syndrome during the test. Placing her in this position does not decrease the heart rate of the fetus. It is not done to aid the woman as she pushes in labor, as she is not in labor yet. It is not done to prevent her from falling out of bed.

A nurse is conducting an in-service program for staff nurses working in the labor and birth unit. The nurse is discussing ways to promote a positive birth outcome for the woman in labor. The nurse determines that additional teaching is necessary when the group identifies which measure? A) allowing the woman time to be alone B) promoting the woman's feelings of control C) providing clear information about procedures D) encouraging the woman to use relaxation techniques

A) allowing the woman time to be alone Positive support, not being alone, promotes a positive birth experience. Being alone can increase anxiety and fear, decreasing the woman's ability to cope. Feelings of control promote self-confidence and self-esteem, which in turn help the woman to cope with the challenges of labor. Information about procedures reduces anxiety about the unknown and fosters cooperation and self-confidence in her abilities to deal with labor. Catecholamines are secreted in response to anxiety and fear and can inhibit uterine blood flow and placental perfusion. Relaxation techniques can help to reduce anxiety and fear, in turn decreasing the secretion of catecholamines and ultimately improving the woman's ability to cope with labor.

The newborn has been placed in skin-to-skin contact with his mother. A blanket covers all of his body except his head. His hair is still wet with amniotic fluid, etc. What is the most likely type of heat loss this baby may experience? A) evaporative B) convective C) conductive D) radiating

A) evaporative Evaporative heat loss occurs with the evaporation of fluid from the infant.

A nurse is caring for a client in her third stage of labor. Which findings would the nurse assess as indicating placental separation? Select all that apply. A) fresh gushing of blood from the vagina B) a relaxed and distended uterus C) umbilical cord descending lower down D) renewed bearing down efforts by client E) falling downward of uterus in the abdomen

A) fresh gushing of blood from the vagina C) umbilical cord descending lower down D) renewed bearing down efforts by client The signs of placental separation include a fresh gush of blood from the vagina, lengthening of the umbilical cord, and renewed bearing-down efforts by the client. When the client is in her third stage of labor, these indicate placental separation. A rising upwards of the uterus and a well-contracted globular uterus are the other signs of placental separation. Falling downward of the uterus in the abdomen and a relaxed uterus are the signs of uterine atony.

A pregnant client has come to the labor and birth suite in labor. The nurse reviews the client's medical record and determines that a vaginal birth is favorable based on which finding related to the client's pelvic shape? A) gynecoid B) platypelloid C) anthropoid D) android

A) gynecoid Vaginal birth is most favorable with a gynecoid pelvis because the inlet is round and the outlet is roomy. This shape offers the optimal diameters in all three planes of the pelvis. This type of pelvis allows early and complete fetal internal rotation during labor. Although vaginal birth is favorable with an anthropoid pelvis, it is less favorable than a gynecoid pelvis. However, vaginal birth is more favorable with an anthropoid pelvic shape compared with the android or platypelloid shape.

A nurse is caring for a pregnant client with rhythmic uterine contractions. Which feature should the nurse identify as associated with true labor? A) increase in frequency of the contractions B) irregularity in the duration of the contractions C) lessening of the contractions with position change D) decrease in the intensity of the contractions

A) increase in frequency of the contractions The nurse should identify that there is an increase in the duration of the contractions associated with true labor. In true labor, the duration, frequency, and intensity of uterine contractions increase. Position change does not reduce the uterine contractions. In false labor, the uterine contractions often disappear with ambulation and sleep. Also, there is no increase in frequency, duration, or intensity of the contractions, and the cervix fails to dilate any further.

A nurse sees a pregnant client at the clinic. The client is close to her due date. During the visit the nurse would emphasize that the client get evaluated quickly should her membranes rupture spontaneously based on the understanding of which possibility? A) increased risk of infection B) increased risk of breech presentation C) potential placenta previa D) potential rapid birth of fetus

A) increased risk of infection After the amniotic sac has ruptured, the barrier to infection is gone, and an ascending infection is possible. In addition, there is a danger of cord prolapse. The spontaneous rupture does not hasten labor, although it might signal the beginning of labor. The client may have placenta previa with the membranes intact.

What are the functions of kangaroo care? Select all that apply. A) is skin-to-skin contact B) keeps the neonate warm C) helps the parents bond with their neonate D) causes hypothermia

A) is skin-to-skin contact B) keeps the neonate warm C) helps the parents bond with their neonate The method of keeping the neonate warm, kangaroo care, is an excellent way to meet the needs of the neonate and provide family-centered care. Kangaroo care does not cause hypothermia; it actually normalizes the neonate's temperature.

A nurse is describing the many changes a newborn will go through during his or her first couple of weeks after birth. The nurse explains how the functions of the placenta are taken over by which organ? A) liver B) intestine C) kidneys D) cardiovascular system

A) liver At birth, the newborn's liver, not the intestine, cardiovascular system, or kidneys, assumes the functions that the placenta handled during fetal life. This includes iron storage, carbohydrate metabolism, blood coagulation, and conjugation of bilirubin.

A nurse working in the neonatal nursery anticipates the primary care provider to prescribe which medication for a premature newborn having difficulty breathing? A) surfactant B) albuteral C) epinephrine D) norepinephrine

A) surfactant Surfactant is a protein that keeps small air sacs in the lungs from collapsing. Its use was introduced in 1990 and continues today, especially for premature babies and those who have respiratory distress syndrome. The other medications are not given to help premature babies breathe.

The nurse is monitoring a client's uterine contractions. Which factors should the nurse assess to monitor uterine contraction? Select all that apply. A) uterine resting tone B) frequency of contractions C) change in blood pressure D) change in temperature E) intensity of contractions

A) uterine resting tone B) frequency of contractions E) intensity of contractions The nurse should assess the frequency of contractions, intensity of contractions, and uterine resting tone to monitor uterine contractions. Monitoring changes in temperature and blood pressure is part of the general physical examination and does not help to monitor uterine contraction.

APGAR stands for?

A- Appearance (color) P- pulse (heart rate G- grimace ( reflex irritability A- Activity (muscle tone) R- Respiratory (respiratory effort)

The expected fetal heart rate response in an active fetus is which of the following: a) Deceleration followed by acceleration of 15 bpm b) Decrease in variability for 15 seconds c) Acceleration of at least 15 bpm for 15 seconds d) Increase in variability by 15 bpm

Acceleration of at least 15 bpm for 15 seconds Explanation: A reassuring active fetal heart rate is a change in baseline by increase of 15 bpm for 15 seconds. This is a positive and reassuring periodic change in fetal heart rates as a response to fetal movement.

The nurse is reviewing the uterine contraction pattern and identifies the peak intensity, documenting this as which of the following? a) Diastole b) Decrement c) Increment d) Acme

Acme Correct Explanation: The acme is the peak intensity of a contraction. The increment refers to the building up of the contraction. The decrement refers to the letting down of the contraction. Diastole refers to the relaxation phase of a contraction

On examination, the hands and feet of a 12-hour-old infant are cyanotic without other signs of distress. The nurse should document as: a) Potential for respiratory distress. b) Cold stress. c) Poor oxygenation. d) Acrocyanosis.

Acrocyanosis

The coach of a client in labor is holding the client's hand and appears to be intentionally applying pressure to the space between the first finger and thumb on the back of the hand. The nurse recognizes this form of therapy as which of the following? a) Biofeedback b) Effleurage c) Acupuncture d) Acupressure

Acupressure Correct Explanation: Acupressure is the application of pressure or massage at designated susceptible body points. A common point used for a woman in labor is Co4, which is located between the first finger and thumb on the back of the hand. Women may report their contractions feel lighter when a support person holds and squeezes their hand because the support person is accidentally triggering this point. Acupuncture involves insertion of needles into the same body points. Effleurage, the technique of gentle abdominal massage often taught with Lamaze in preparation for childbirth classes is a classic example of therapeutic touch. Biofeedback is based on the belief people have control and can regulate internal events such as heart rate and pain responses.

The infant has APGAR scores of 7 at one minute and 9 at five minutes. What is the indication of this assessment finding? a) Adjusting to extrauterine life. b) Predicts fair neurologic future outcomes. c) Moderate difficulty and may need intervention. d) Severe distress and absolute need of resuscitation.

Adjusting to extrauterine life.

The infant has APGAR scores of 7 at one minute and 9 at five minutes. What is the indication of this assessment finding?

Adjusting to extrauterine life. Correct Explanation: The infant is tolerating the adjustment to extrauterine life, the APGAR scores are within normal limits for appropriate transition. Severe distress and absolute need for resuscitation is an APGAR score of 0 to 3; moderate difficulty is indicated by a score of 4 to 7. An APGAR of 8 to 10 at five minutes indicates a fair neurologic future outcome.

Susan is in labor with her second child. She knows that she will want epidural anesthesia and she has already signed her consent form. What must the nurse do before Susan receives the epidural? a) Administer a fluid bolus through the IV line to reduce the risk of hypotension b) Prepare a sterile field with the supplies and medications that will be needed c) Review Susan's medical history and laboratory results, and interview Susan to confirm all information is accurate and up to date d) Place Susan in the fetal position on the table and keep her steady so that she won't move during the procedure

Administer a fluid bolus through the IV line to reduce the risk of hypotension Explanation: Epidurals can cause vasodilatation and result in hypotensive episodes, IV fluid bolus prior to epidural placement can help prevent the hypotensive episode. She has signed the consent form, so it is unnecessary to interview her again. Do not place her in the fetal position or prep the site until the anesthesiologist arrives; then, assist them.

A nurse is caring for a female client in the postpartum phase. The client reports "afterpains." Which intervention should the nurse complete first?

Administer pain medications. "Afterpains" should be expected in postpartum clients. These are commonly treated with pain analgesics. The client should not stop breastfeeding. Assessing vital signs and helping the client to void are not the priority interventions for this client.

A client has opted to receive epidural anesthesia during labor. Which of the following interventions should the nurse implement to reduce the risk of a significant complication associated with this type of pain management? a) Administration of aspirin b) Administration of 500 mL of IV Ringer's lactate c) Administration of 1000 mL of IV glucose solution d) Move the woman into a supine position

Administration of 500 mL of IV Ringer's lactate Correct Explanation: The chief concern with epidural anesthesia is its tendency to cause hypotension because of its blocking effect on the sympathetic nerve fibers in the epidural space. This risk can be reduced by being certain a woman is well hydrated with 500 to 1000 mL of IV fluid, such as Ringer's lactate, before the anesthetic is administered. Ringer's lactate is preferable to a glucose solution, because too much maternal glucose can cause hyperglycemia with rebound hypoglycemia in the newborn. Be certain a woman does not lie supine but remains on her side after an epidural block, to help prevent supine hypotension syndrome. Be sure to caution women not to take acetylsalicylic acid (aspirin) for pain in labor as aspirin interferes with blood coagulation, increasing the risk for bleeding in the newborn or herself.

A client in labor has requested the administration of narcotics to reduce pain. At 2 cm cervical dilatation, she says that she is managing the pain well at this point but does not want it to get ahead of her. Which of the following should the nurse do? a) Advise the client to hold out a bit longer, if possible, before administration of the drug, to prevent slowing labor b) Agree with the client and administer the drug immediately to keep the pain manageable c) Refuse to administer narcotics, because they can develop dependency in the client and the fetus d) Explain to the client that narcotics should only be administered an hour or less before birth

Advise the client to hold out a bit longer, if possible, before administration of the drug, to prevent slowing labor Correct Explanation: The timing of administration of narcotics in labor is especially important as, if given too early (before 3 cm cervical dilatation), they tend to slow labor. If given close to birth, because the fetal liver takes 2 to 3 hours to activate a drug, the effect will not be registered in the fetus for 2 to 3 hours after maternal administration. For this reason, narcotics are preferably given when the mother is more than 3 hours away from birth. This allows the peak action of the drug in the fetus to have passed by the time of birth.

What is the best thing the nurse can do to manage pain in a neonate? a) Teach the infant's caregivers ways to soothe and comfort the child during any episode of pain. b) Adhere carefully to the plan for administration of any analgesics to the child. c) Advocate to the physician to use effective treatment methods that cause no pain or less pain. d) Provide a soothing environment, swaddling, and holding to the newborn experiencing pain.

Advocate to the physician to use effective treatment methods that cause no pain or less pain.

What is the best thing the nurse can do to manage pain in a neonate? a) Teach the infant's caregivers ways to soothe and comfort the child during any episode of pain. b) Adhere carefully to the plan for administration of any analgesics to the child. c) Advocate to the physician to use effective treatment methods that cause no pain or less pain. d) Provide a soothing environment, swaddling, and holding to the newborn experiencing pain.

Advocate to the physician to use effective treatment methods that cause no pain or less pain. Correct Explanation: It is the ethical responsibility of the nurse to prevent and treat pain. The nurse should advocate to the provider to provide pain treatment. All answer options are correct; however, the best answer is to advocate for effective treatment methods that cause no pain, or less pain. Therefore, options A, B, and D are incorrect.

Nurses can advise their patients that which of these signs precede labor? (Select all that apply.)

After lightening a return of the frequent need to urinate occurs as the fetal position causes increased pressure on the bladder. In the run-up to labor, women often experience persistent low backache and sacroiliac distress as a result of relaxation of the pelvic joints. Before the onset of labor, it is common for Braxton Hicks contractions to increase in both frequency and strength. Bloody show may be passed. A surge of energy is a phenomenon that is common in the days preceding labor. In first-time pregnancies, the uterus sinks downward and forward about 2 weeks before term.

Jennifer has just received IV sedation. What must the nurse tell Jennifer to do? a) Ambulate only with assistance from the nurse or caregiver b) Sit on the edge of the bed with her feet dangling before ambulating c) Remain in bed for at least 30 minutes d) Ambulate within 15 minutes to prevent spinal headache

Ambulate only with assistance from the nurse or caregiver Correct Explanation: The patient may have decrease sensory from the medication. She needs assistance to ambulate for safety. She will be largely unable to move, so she should remain in bed unless absolutely necessary.

Jennifer has just received IV sedation. What must the nurse tell Jennifer to do? a) Ambulate within 15 minutes to prevent spinal headache b) Ambulate only with assistance from the nurse or caregiver c) Sit on the edge of the bed with her feet dangling before ambulating d) Remain in bed for at least 30 minutes

Ambulate only with assistance from the nurse or caregiver Correct Explanation: The patient may have decrease sensory from the medication. She needs assistance to ambulate for safety. She will be largely unable to move, so she should remain in bed unless absolutely necessary.

Which of the following findings would lead you to suspect that a woman is developing a postpartum complication? a) Red-colored lochia for the first 24 hours b) Lochia that is the color of menstrual blood c) Lochia appearing pinkish-brown on the fourth day d) An absence of lochia

An absence of lochia Correct Explanation: Women should have a lochia flow following childbirth. Absence of a flow is abnormal; it suggests dehydration from infection and fever.

Which newborn neuromuscular system adaptation would the nurse NOT expect to find? a) An extrusion reflux at 9 months of age b) A positive Babinski reflex at 2 months of age c) A Moro reflex at 3 months of age d) A plantar grasp reflex at 7 months of age

An extrusion reflux at 9 months of age

Which of the following is true regarding analgesia versus anesthesia? a) Decreased FHR variability is a common side effect when regional anesthesia is used. b) Analgesia only reduces pain, but anesthesia partially or totally blocks all pain in a particular area. c) Hypotension is the most common side effect when systemic analgesia is used. d) Regional anesthesia should be given with caution close to the time of delivery because it crosses the placenta and can cause respiratory depression in the newborn.

Analgesia only reduces pain, but anesthesia partially or totally blocks all pain in a particular area. Correct Explanation: Systemic analgesia should be used with caution near the time of delivery because it can cause respiratory depression, in addition to decreased FHR variability. Hypotension is a common side effect of regional anesthesia.

What is the term for a small collection of blood that forms underneath the skull as a result of birth trauma? a) Caput succedaneum b) Vernix caseosa c) Erythema toxicum d) Cephalhematoma

Cephalhematoma Correct Explanation: Vernix caseosa is a thick white substance found on a newborn. Erythema toxicum is a newborn rash. Caput succedaneum is molding or edema.

Which presentation is described accurately in terms of both presenting part and frequency of occurrence?

Cephalic: occiput; at least 95%

A laboring mom is 9 cm dilated and experiencing contractions every 2 minutes for 90 seconds each. Which stage and phase of labor is she in?

Stage One, Transition Phase

A primiparous mother delivered a 8 pound 12 ounce infant daughter yesterday. She is being bottle fed, is Rh positive, has a cephalohematoma, and received her hepatitis A vaccine last evening. Which factor places the newborn at risk for the development of jaundice? a) Rh positive blood type b) Hepatitis A vaccine c) Female gender d) Cephalohematoma e) Formula feeding

Cephalohematoma

On inspecting a newborn's abdomen, which finding would you note as abnormal? a) Liver palpable 2 cm under the right costal margin b) Bowel sounds present at two to three per minute c) Abdomen slightly protuberant (rounded) d) Clear drainage at the base of the umbilical cord

Clear drainage at the base of the umbilical cord

When should the baby get ID band?

Within 2 hours of birth

During the first hour of the fourth stage of labor, the nurse would assess the woman's fundus how often? a) Every 5 minutes b) Every 15 minutes c) Every 10 minutes d) Every 20 minutes

Every 15 minutes Correct Explanation: During the first hour of the fourth stage of labor, the nurse would assess the woman's fundus every 15 minutes, and then every 30 minutes for the next hour.

How often should you assess in the second hour?

Every 30 minutes

How often should you asses in the first 24 hours?

Every 4 hours

A client in labor for the past 10 hours shows no change in cervical dilation and has stayed at 5 to 6 cm for the past 2 hours. Her contractions remain regular at 2-minute intervals, lasting 40 to 45 seconds. Which of the following would be the nurse's initial action? 1. Assess for presence of a full bladder. 2. Suggest the placement of an internal uterine pressure catheter to determine adequacy of contractions. 3. Encourage the mother to relax by assisting her with appropriate breathing techniques. 4. Suggest to the physician that oxytocin augmentation be started to stimulate labor.

Answer: 1 RATIONALES: A full bladder will slow or stop cervical dilation and produce symptoms that could be misdiagnosed as arrest in labor. Other strategies, such as internal uterine monitoring, relaxation, and oxytocin augmentation, would be appropriate later, but assessing the bladder first is key.

The nurse observes a late deceleration. It's characterized by and indicates which of the following? 1. U-shaped deceleration occurring after the first half of the contraction, indicating uteroplacental insufficiency 2. U-shaped deceleration occurring with the contraction, indicating cord compression 3. V-shaped deceleration occurring after the contraction, indicating uteroplacental insufficiency 4. Deep U-shaped deceleration occurring before the contraction, indicating head compression

Answer: 1 RATIONALES: A late deceleration is U-shaped and occurs after the first half of the contraction, indicating uteroplacental insufficiency. It's an ominous pattern and requires immediate action — such as administering oxygen, repositioning the mother, and increasing the I.V. infusion rate — to correct the problem. U- and V-shaped decelerations are variable decelerations occurring at unpredictable times during contractions and are related to umbilical cord compression. Deep U-shaped deceleration occurring before the contraction is early deceleration.

For a client who's moving into the active phase of labor, the nurse should include which of the following as the priority of care? 1. Offer support by reviewing the short-pant form of breathing. 2. Administer narcotic analgesia. 3. Allow the mother to walk around the unit. 4. Watch for rupture of the membranes.

Answer: 1 RATIONALES: By helping the client use the pant form of breathing, the nurse can help the client manage her contractions and reduce the need for opioids and other forms of pain relief, which can have an effect on fetal outcome. In the active phase, the mother most likely is too uncomfortable to walk around the unit. The nurse will observe for rupture of membranes and may administer opioid analgesia but these don't take priority.

A primigravid client is admitted to the labor and delivery area. Assessment reveals that she's in the early part of the first stage of labor. Her pain is likely to be most intense: 1. around the pelvic girdle. 2. around the pelvic girdle and in the upper legs. 3. around the pelvic girdle and at the perineum. 4. at the perineum.

Answer: 1 RATIONALES: During most of the first stage of labor, pain centers around the pelvic girdle. During the late part of this stage and the early part of the second stage, pain spreads to the upper legs and perineum. During the late part of the second stage and during childbirth, intense pain occurs at the perineum.

Assessment of a client in active labor reveals meconium-stained amniotic fluid and fetal heart sounds in the upper right quadrant. Which of the following is the most likely cause of this situation? 1. Breech position 2. Late decelerations 3. Entrance into the second stage of labor 4. Multiple gestation

Answer: 1 RATIONALES: Fetal heart sounds in the upper right quadrant and meconium-stained amniotic fluid indicate a breech presentation. The staining is usually caused by the squeezing actions of the uterus on a fetus in the breech position, although late decelerations, entrance into the second stage of labor, and multiple gestation may contribute to meconium-stained amniotic fluid.

Which of the following describes the term fetal position? 1. Relationship of the fetus's presenting part to the mother's pelvis 2. Fetal posture 3. Fetal head or breech at cervical os 4. Relationship of the fetal long axis to the mother's long axis

Answer: 1 RATIONALES: Fetal position refers to the relationship of the fetus's presenting part to the mother's pelvis. Fetal posture refers to "attitude." Presentation refers to the part of the fetus at the cervical os. Lie refers to the relationship of the fetal long axis to that of the mother's long axis.

During labor, a client asks the nurse why her blood pressure must be measured so often. Which explanation should the nurse provide? 1. Blood pressure reflects changes in cardiovascular function, which may affect the fetus. 2. Increased blood pressure indicates that the client is experiencing pain. 3. Increased blood pressure signals the peak of the contraction. 4. Medications given during labor affect blood pressure.

Answer: 1 RATIONALES: Frequent blood pressure measurement helps determine whether maternal cardiovascular function is adequate. During contractions, blood flow to the intervillous spaces changes, compromising fetal blood supply. Increased blood pressure is expected during pain and contractions. Measuring blood pressure frequently helps determine whether blood pressure has returned to precontraction levels, ensuring adequate blood flow to the fetus. Although medications given during labor can affect blood pressure, the main purpose of measuring blood pressure is to verify adequate fetal status.

When assessing a client who has just delivered a neonate, the nurse finds that the fundus is boggy and deviated to the right. What should the nurse do? 1. Have the client void. 2. Assess the client's vital signs. 3. Evaluate lochia characteristics. 4. Massage the fundus.

Answer: 1 RATIONALES: Having the client void can determine whether the boggy, deviated fundus results from a full bladder — the most common cause of these fundal findings. Vital sign assessment is unnecessary unless the nurse suspects hemorrhage from delayed involution. In a client who doesn't have a full bladder, the nurse should evaluate lochia characteristics to detect possible hemorrhage. If the client has a full bladder, massaging the fundus won't stimulate uterine contractions (which aid uterine involution) or prevent uterine atony — a possible cause of hemorrhage.

The nurse applies an external electronic fetal monitor (EFM) to assess a client's uterine contractions and evaluate the fetal heart rate (FHR). However, the client is uncomfortable and changes positions frequently, making FHR hard to assess. Consequently, the physician decides to switch to an internal EFM. Before internal monitoring can begin, which of the following must occur? 1. The membranes must rupture. 2. The client must receive anesthesia. 3. The cervix must be fully dilated. 4. The fetus must be at 0 station.

Answer: 1 RATIONALES: Internal EFM can be used only after the client's membranes rupture, when the cervix is dilated at least 2 cm and when the presenting part is at least at -1 station. Anesthesia isn't required for internal EFM.

A client, age 19, goes into labor at 40 weeks' gestation. When assessing the fetal monitor strip, the nurse sees that the fetal heart rate (FHR) has decreased to 60 beats/minute and that the waveforms sometimes resemble a V and begin and end abruptly. The nurse should interpret this pattern as: 1. variable decelerations. 2. decreased short-term variability. 3. increased long-term variability. 4. early decelerations.

Answer: 1 RATIONALES: On a fetal monitor strip, variable decelerations are characterized by an FHR that commonly decreases to 60 beats/minute; waveform shapes that vary and may resemble the letter U, V, or W; and deceleration waveforms with an abrupt onset and recovery. Decreased short-term variability manifests as fewer than 2 to 3 beats/amplitude of the baseline FHR. Increased long-term variability manifests as more than 5 to 20 beats/minute of the baseline FHR in rhythmic fluctuation. Early decelerations are seen as the descent, peak, and recovery of the deceleration waveform that mirrors the contraction waveform.

During labor, a client's cervix fails to dilate progressively, despite her uncomfortable uterine contractions. To augment labor, the physician orders oxytocin (Pitocin). When preparing the client for oxytocin administration, the nurse describes the contractions the client is likely to feel when she starts to receive the drug. Which description is accurate? 1. Contractions will be stronger and more uncomfortable and will peak more abruptly. 2. Contractions will be weaker, longer, and more effective. 3. Contractions will be stronger, shorter, and less uncomfortable. 4. Contractions will be stronger and shorter and will peak more slowly.

Answer: 1 RATIONALES: Oxytocin administration causes stronger, more uncomfortable contractions, which peak more abruptly than spontaneous contractions. Oxytocin doesn't affect the duration of contractions.

The nurse is administering oxytocin (Pitocin) to a client in labor. During oxytocin therapy, why must the nurse monitor the client's fluid intake and output closely? 1. Because oxytocin causes fluid retention 2. Because oxytocin causes excessive thirst 3. Because oxytocin has a diuretic effect

Answer: 1 RATIONALES: Oxytocin has an antidiuretic effect; prolonged I.V. infusion may lead to severe fluid retention, resulting in seizures, coma, and even death. Excessive thirst results from the work of labor and lack of oral fluids, not oxytocin administration. Oxytocin isn't toxic to the kidney.

At 28 weeks' gestation, a client is admitted to the labor and delivery area in preterm labor. An I.V. infusion of ritodrine (Yutopar) is started. Which client outcome reflects the nurse's awareness of an adverse effect of ritodrine? 1. "The client remains free from tachycardia." 2. "The client remains free from polyuria." 3. "The client remains free from hypertension." 4. "The client remains free from hyporeflexia."

Answer: 1 RATIONALES: Ritodrine and other beta-adrenergic agonists may cause tachycardia, hypotension, bronchial dilation, increased plasma volume, increased cardiac output, arrhythmias, myocardial ischemia, reduced urine output, restlessness, headache, nausea, and vomiting. These drugs aren't associated with polyuria, hypertension, or hyporeflexia.

A client is admitted to the labor and delivery department in preterm labor. To help manage preterm labor the nurse would expect to administer: 1. ritodrine (Yutopar). 2. bromocriptine (Parlodel). 3. magnesium sulfate. 4. betamethasone (Celestone).

Answer: 1 RATIONALES: Ritodrine reduces frequency and intensity of uterine contractions by stimulating B2 receptors in the uterine smooth muscle. It's the drug of choice when trying to inhibit labor. Bromocriptine, a dopamine receptor agonist and an ovulation stimulant, is used to inhibit lactation in the postpartum period. Magnesium sulfate, an anticonvulsant, is used to treat preeclampsia and eclampsia — a life-threatening form of pregnancy-induced hypertension. Betamethasone, a synthetic corticosteroid, is used to stimulate fetal pulmonary surfactant (administered to the mother).

A diabetic client in labor tells the nurse she has had trouble controlling her blood glucose level recently. She says she didn't take her insulin when the contractions began because she felt nauseated; about an hour later, when she felt better, she ate some soup and crackers but didn't take insulin. Now, she reports increased nausea and a flushed feeling. The nurse notes a fruity odor to her breath. What do these findings suggest? 1. Diabetic ketoacidosis 2. Hypoglycemia 3. Infection 4. Transition to the active phase of labor

Answer: 1 RATIONALES: Signs and symptoms of diabetic ketoacidosis include nausea and vomiting, a fruity or acetone breath odor, signs of dehydration (such as flushed, dry skin), hyperglycemia, ketonuria, hypotension, deep and rapid respirations, and a decreased level of consciousness. In contrast, hypoglycemia causes sweating, tremors, palpitations, and behavioral changes. Infection causes a fever. Transition to the active phase of labor is signaled by cervical dilation of up to 7 cm and contractions every 2 to 5 minutes.

Because cervical effacement and dilation aren't progressing in a client in labor, the physician orders I.V. administration of oxytocin (Pitocin). Why must the nurse monitor the client's fluid intake and output closely during oxytocin administration? 1. Oxytocin causes water intoxication. 2. Oxytocin causes excessive thirst. 3. Oxytocin is toxic to the kidneys. 4. Oxytocin has a diuretic effect.

Answer: 1 RATIONALES: The nurse should monitor fluid intake and output because prolonged oxytocin infusion may cause severe water intoxication, leading to seizures, coma, and death. Excessive thirst results from the work of labor and limited oral fluid intake — not oxytocin. Oxytocin has no nephrotoxic or diuretic effects. In fact, it produces an antidiuretic effect.

When assessing the fetal heart rate tracing, the nurse becomes concerned about the fetal heart rate pattern. In response to the loss of variability, the nurse repositions the client to her left side and administers oxygen. These actions are likely to improve which of the following? 1. Fetal hypoxia 2. The contraction pattern 3. The status of a trapped cord 4. Maternal comfort

Answer: 1 RATIONALES: These actions, which will improve fetal hypoxia, increase the amount of maternal circulating oxygen by taking pressure created by the uterus off the aorta and improving blood flow. These actions won't improve the contraction pattern, free a trapped cord, or improve maternal comfort.

The nurse is caring for a client in labor. The external fetal monitor shows a pattern of variable decelerations in fetal heart rate. What should the nurse do first? 1. Change the client's position. 2. Prepare for emergency cesarean section. 3. Check for placenta previa. 4. Administer oxygen.

Answer: 1 RATIONALES: Variable decelerations in fetal heart rate are an ominous sign, indicating compression of the umbilical cord. Changing the client's position may immediately correct the problem. An emergency cesarean section is necessary only if other measures, such as changing position and amnioinfusion with sterile saline, prove unsuccessful. Administering oxygen may be helpful, but the priority is to change the woman's position and relieve cord compression.

The nurse is evaluating a client who is 34 weeks pregnant for premature rupture of the membranes (PROM). Which findings indicate that PROM has occurred? 1. Fernlike pattern when vaginal fluid is placed on a glass slide and allowed to dry 2. Acidic pH of fluid when tested with nitrazine paper 3. Presence of amniotic fluid in the vagina 4. Cervical dilation of 6 cm 5. Alkaline pH of fluid when tested with nitrazine paper

Answer: 1,3,5 RATIONALES: The fernlike pattern that occurs when vaginal fluid is placed on a glass slide and allowed to dry, presence of amniotic fluid in the vagina, and alkaline pH of fluid are all signs of ruptured membranes. The fernlike pattern seen when the fluid is allowed to dry on a slide is a result of the high sodium and protein content of the amniotic fluid. The presence of amniotic fluid in the vagina results from the expulsion of the fluid from the amniotic sac. Cervical dilation and regular contractions are signs of progressing labor but don't indicate PROM.

The nurse is monitoring a client who is receiving oxytocin (Pitocin) to induce labor. The nurse should be prepared for which maternal adverse reactions? 1. Hypertension 2. Jaundice 3. Dehydration 4. Fluid overload 5. Uterine tetany 6. Bradycardia

Answer: 1,4,5 RATIONALES: Adverse reactions to oxytocin in the mother include hypertension, fluid overload, and uterine tetany. The antidiuretic effect of oxytocin increases renal reabsorption of water, leading to fluid overload — not dehydration. Jaundice and bradycardia are adverse reactions that may occur in the neonate. Tachycardia, not bradycardia, is reported as a maternal adverse reaction.

The nurse notices that a client in the first stage of labor seems agitated. When the nurse asks why she's upset, she begins to cry and says, "I guess I'm a little worried. The last time I gave birth, I was in labor for 32 hours." Based on this information, the nurse should include which nursing diagnosis in the client's care plan? 1. Anxiety related to the facility environment 2. Fear related to a potentially difficult childbirth 3. Compromised family coping related to hospitalization 4. Acute pain related to labor contractions

Answer: 2 RATIONALES: A client's ability to cope during labor and delivery may be hampered by fear of a painful or difficult childbirth, fear of loss of control or self-esteem during childbirth, or fear of fetal death. A previous negative experience may increase these fears. Therefore, Fear related to a potentially difficult childbirth is the most appropriate nursing diagnosis. The client's anxiety stems from her past history of a long labor, not from being in the facility; therefore a diagnosis of Anxiety related to the facility environment isn't warranted. There is no evidence of compromised family coping related to hospitalization. Although acute pain related to labor contractions may be a problem, this isn't mentioned in the question.

When caring for a client who's having her second baby, the nurse can anticipate the client's labor will be which of the following? 1. Shorter than her first labor 2. About half as long as her first labor 3. About the same length of time as her first labor 4. A length of time that can't be determined based on her first labor

Answer: 2 RATIONALES: A woman having her second baby can anticipate a labor about half as long as her first labor. The other options are incorrect.

The physician decides to artificially rupture the membranes. Following this procedure, the nurse checks the fetal heart tones for which reason? 1. To determine fetal well-being 2. To assess for fetal bradycardia 3. To assess fetal position 4. To prepare for an imminent delivery

Answer: 2 RATIONALES: After a client has an amniotomy, the nurse should assure that the cord isn't prolapsed and that the baby tolerated the procedure well. The most effective way to do this is to check the fetal heart rate. Fetal well-being is assessed via a nonstress test. Fetal position is determined by vaginal examination. Artificial rupture of membranes doesn't indicate an imminent delivery.

During the active phase of the first stage of labor, a client undergoes an amniotomy. After this procedure, which nursing diagnosis takes the highest priority? 1. Deficient knowledge (testing procedure) related to amniotomy 2. Ineffective fetal cerebral tissue perfusion related to cord compression 3. Acute pain related to increasing strength of contractions 4. Risk for infection related to rupture of membranes

Answer: 2 RATIONALES: Amniotomy increases the risk of cord prolapse. If the prolapsed cord is compressed by the presenting fetal part, the fetal blood supply may be impaired, jeopardizing the fetal oxygen supply. Because lack of oxygen to the fetus may cause fetal death, the nursing diagnosis of Ineffective fetal cerebral tissue perfusion takes priority over diagnoses of Deficient knowledge, Acute pain, and Risk for infection.

A client with Rh isoimmunization delivers a neonate with an enlarged heart and severe, generalized edema. Which nursing diagnosis is most appropriate for this client? 1. Ineffective denial related to a socially unacceptable infection 2. Impaired parenting related to the neonate's transfer to the intensive care unit 3. Deficient fluid volume related to severe edema 4. Fear related to removal and loss of the neonate by statute

Answer: 2 RATIONALES: Because the neonate is severely ill and needs to be placed in the neonatal intensive care unit, the client may have a nursing diagnosis of Impaired parenting related to the neonate's transfer to the neonatal intensive care unit. (Another pertinent nursing diagnosis may be Compromised family coping related to lack of opportunity for bonding.) Rh isoimmunization isn't a socially unacceptable infection. This condition causes an excess fluid volume (not deficient) related to cardiac problems. Rh isoimmunization doesn't lead to loss of the neonate by statute.

After admission to the labor and delivery area, a client undergoes routine tests, including a complete blood count, urinalysis, Venereal Disease Research Laboratory test, and gonorrhea culture. The gonorrhea culture is positive, although the client lacks signs and symptoms of this disease. What is the significance of this finding? 1. Maternal gonorrhea may cause a neural tube defect in the fetus. 2. Maternal gonorrhea may cause an eye infection in the neonate. 3. Maternal gonorrhea may cause acute liver changes in the fetus. 4. Maternal gonorrhea may cause anemia in the neonate.

Answer: 2 RATIONALES: Gonorrhea in the cervix may cause neonatal eye infection during delivery as well as a serious puerperal infection in the client. Maternal gonorrhea isn't associated with neural tube defects, acute fetal liver changes, or neonatal anemia.

A primigravid client, age 20, has just completed a difficult, forceps-assisted delivery of twins. Her labor was unusually long and required oxytocin (Pitocin) augmentation. The nurse who's caring for her should stay alert for: 1. uterine inversion. 2. uterine atony. 3. uterine involution. 4. uterine discomfort.

Answer: 2 RATIONALES: Multiple fetuses, extended labor stimulation with oxytocin, and traumatic delivery commonly are associated with uterine atony, which may lead to postpartum hemorrhage. Uterine inversion may precede or follow delivery and commonly results from apparent excessive traction on the umbilical cord and attempts to deliver the placenta manually. Uterine involution and some uterine discomfort are normal after delivery.

At 40 weeks' gestation, a client is admitted to the labor and delivery area. She and her husband are worried about the fetus's health because she had problems during her previous childbirth. The nurse reassures them that the fetus will be monitored closely with an electronic fetal monitor (EFM). On the fetal monitor strip, what is the single most reliable indicator of fetal well-being? 1. Normal long-term variability 2. Normal short-term variability 3. Normal baseline fetal heart rate (FHR) 4. Normal contraction sequence

Answer: 2 RATIONALES: Normal short-term variability — 2 to 3 beats per amplitude — is the single most reliable indicator of fetal well-being on an EFM strip. It represents actual beat-to-beat fluctuations in the FHR. Normal long-term variability, although a helpful indicator, takes into account larger periodic and rhythmic deviations above and below the baseline FHR. Baseline FHR serves only as a reference for all subsequent FHR readings taken during labor. Contraction sequence provides no information about fetal well-being, although it does give some indication of maternal well-being and progress.

Several minutes after a vaginal delivery, nursing assessment reveals blood gushing from the client's vagina, umbilical cord lengthening, and a globular-shaped uterus. The nurse should suspect which condition? 1. Cervical or vaginal laceration 2. Placental separation 3. Postpartum hemorrhage 4. Uterine involution

Answer: 2 RATIONALES: Placental separation is characterized by a sudden gush or trickle of blood from the vagina, further protrusion of the umbilical cord from the vagina, a globular-shaped uterus, and an increase in fundal height. With cervical or vaginal laceration, the nurse notes a consistent flow of bright red blood from the vagina. With postpartum hemorrhage, usually caused by uterine atony, the uterus isn't globular. Uterine involution can't begin until the placenta has been delivered.

The nurse notices that a large number of clients who receive oxytocin (Pitocin) to induce labor, vomit as the infusion is started. The nurse assesses the situation further and discovers that these clients received no instruction before arriving on the unit and haven't fasted for 8 hours before induction. How should the nurse intervene? 1. Notify the physicians and explain that they need to teach their clients before inducing labor. 2. Initiate a unit policy involving staff nurses, certified nurse midwives, and physicians in teaching clients before labor induction. 3. Report the physicians for providing inferior care. 4. Initiate a protocol order that allows the nurse to administer promethazine (Phenergan) before administering oxytocin.

Answer: 2 RATIONALES: The best intervention by the nurse is to initiate a unit policy that involves the multidisciplinary team. This approach creates an atmosphere of collegiality and professionalism with the goal of providing the best care for clients in labor. Option 1 blames the physician and doesn't promote multidisciplinary teamwork. Reporting the physicians is unnecessary because nothing indicates that the physicians provided inferior care. The nurse can approach the medical staff about initiating a protocol order that allows the nursing staff to administer promethazine; however, this option doesn't address the problem — the lack of client education.

A pregnant client arrives at the health care facility, stating that her bed linens were wet when she woke up this morning. She says no fluid is leaking but complains of mild abdominal cramps and lower back discomfort. Vaginal examination reveals cervical dilation of 3 cm, 100% effacement, and positive ferning. Based on these findings, the nurse concludes that the client is in which phase of the first stage of labor? 1. Active phase 2. Latent phase 3. Expulsive phase 4. Transitional phase

Answer: 2 RATIONALES: The latent phase of the first stage of labor is associated with irregular, short, mild contractions; cervical dilation of 3 to 4 cm; and abdominal cramps or lower back discomfort. During the active phase, the cervix dilates to 7 cm and moderately intense contractions of 40 to 50 seconds' duration occur every 2 to 5 minutes. Fetal descent continues throughout the active phase and into the transitional phase, when the cervix dilates from 8 to 10 cm and intense contractions of 45 to 60 seconds' duration occur every 1½ to 2 minutes. The first stage of labor doesn't include an expulsive phase.

A client is admitted to the labor and delivery area. How can the nurse most effectively determine the duration of the client's contractions? 1. By timing the period between one contraction and the beginning of the next contraction 2. By timing the period from the onset of uterine tightening to uterine relaxation 3. By timing the period from the increment (building-up) phase to the acme (peak) phase 4. By timing the period from the acme (peak) phase to the decrement (letting-down) phase

Answer: 2 RATIONALES: To determine the duration of contractions, the nurse should time the period from the onset of uterine tightening to uterine relaxation. Timing the period between one contraction and the beginning of the next contraction helps determine the frequency of contractions. Timing the period from the increment to the acme or from the acme to the decrement supplies only partial information about contractions.

An adolescent in the early stages of labor is admitted to the labor and delivery unit. The nurse notes lymphadenopathy and a macular rash on the palmar surfaces of the hands and plantar surfaces of the feet. Admission laboratory testing reveals trace ketones in the urine, white blood cell count 10,000/μl, hemoglobin 14.5 g/dl, hematocrit 40%, and the nontreponemal antibody test is positive. The nurse notifies the physician of the laboratory results. Which action by the nurse takes priority? 1. Notifying the laboratory that a repeat hemoglobin and hematocrit have been ordered. 2. Recommending that the client drink plenty of fluids. 3. Consulting with the infection control nurse. 4. Asking the client if she has been exposed to varicella in the past 3 weeks.

Answer: 3 RATIONALES: A nontreponemal test screens the client for syphilis. The positive test result, along with the lymphadenopathy and rash, indicate that the client has secondary syphilis. Based on these findings, the neonate will most likely have signs and symptoms of congenital syphilis. The hemoglobin and hematocrit results are normal for a pregnant client. The laboratory results don't show signs of dehydration, so having the client drink plenty of fluids isn't necessary. The lesions associated with varicella are vesicular, and don't resemble the rash associated with syphilis.

A client with intrauterine growth retardation is admitted to the labor and delivery unit and started on an I.V. infusion of oxytocin (Pitocin). Which of the following is least likely to be included in her care plan? 1. Carefully titrating the oxytocin based on her pattern of labor 2. Monitoring vital signs, including assessment of fetal well-being, every 15 to 30 minutes 3. Allowing the client to ambulate as tolerated 4. Helping the client use breathing exercises to manage her contractions

Answer: 3 RATIONALES: Because the fetus is at risk for complications, frequent and close monitoring is necessary. Therefore, the client shouldn't be allowed to ambulate. Carefully titrating the oxytocin, monitoring vital signs, including fetal well-being, and assisting with breathing exercises are appropriate actions to include.

The nurse is assessing the fetal heart rate (FHR) of a client, who is at term, admitted to the labor and delivery area. Which of the following should the nurse identify as the normal range of the baseline FHR? 1. 60 to 80 beats/minute 2. 80 to 120 beats/minute 3. 120 to 160 beats/minute 4. 160 to 200 beats/minute

Answer: 3 RATIONALES: In a full-term fetus, the baseline FHR normally ranges from 120 to 160 beats/minute. FHR below 120 beats/minute reflects bradycardia; above 160 beats/minute, tachycardia.

Which of the following would be an inappropriate indication of placental detachment? 1. An abrupt lengthening of the cord 2. An increase in the number of contractions 3. Relaxation of the uterus 4. Increased vaginal bleeding

Answer: 3 RATIONALES: Relaxation isn't an indication for detachment of the placenta. An abrupt lengthening of the cord, an increase in the number of contractions, and an increase in vaginal bleeding are all indications that the placenta has detached from the wall of the uterus.

The third stage of labor ends with which of the following? 1. The birth of the baby 2. When the client is fully dilated 3. After the delivery of the placenta 4. When the client is transferred to her postpartum bed

Answer: 3 RATIONALES: The definition of the third stage of labor is the delivery of the placenta. The first stage of labor ends with complete cervical dilation and effacement. The second stage of labor ends with the birth of the baby. The fourth stage of labor includes the first 4 hours after birth.

During labor, a client tells the nurse that her last baby "came out really fast." The nurse can help control a precipitous delivery by: 1. applying counterpressure to the fetus's head. 2. encouraging the client to push. 3. massaging and supporting the perineum. 4. instructing the client to contract the perineal muscles.

Answer: 3 RATIONALES: The nurse can help control a precipitous delivery by stretching the labia, such as by massaging and bracing the perineum with gentle back pressure. This helps prevent perineal lacerations — the primary maternal complication of precipitous delivery. Applying counterpressure to the fetus's head reduces perineal stress temporarily; however, delivery proceeds when the client pushes with uterine contractions. Pushing puts further stress on the perineum, promoting delivery. When the fetus's head exerts pressure on the perineum, contracting the perineal muscles is virtually impossible.

A nursing assistant escorts a client in the early stages of labor to the bathroom. When the nurse enters the client's room, she detects a strange odor coming from the bathroom and suspects the client has been smoking marijuana. What should the nurse do next? 1. Tell the client that smoking is prohibited in the facility, and that if she smokes again, she'll be discharged. 2. Explain to the client that smoking poses a danger of explosion because oxygen tanks are stored close by. 3. Notify the physician and security immediately. 4. Ask the nursing assistant to dispose of the marijuana that the client can't smoke anymore.

Answer: 3 RATIONALES: The nurse should immediately notify the physician and security. The physician must be informed because illegal drugs can interfere with the labor process and affect the neonate after delivery. Moreover, the client might have consumed other illegal drugs. The nurse should also inform security who are specially trained to handle such situations. Most hospitals prohibit smoking. The nurse needs to alert others about the client's illegal drug use, not simply explain to the client that smoking is prohibited. Smoking is dangerous around oxygen and it's fine for the nurse to explain the hazard to the client; however, the nurse must first notify the physician and security. The nursing assistant shouldn't be asked to dispose of the marijuana.

A client in the fourth stage of labor asks to use the bathroom for the first time since delivery. The client has oxytocin (Pitocin) infusing. Which response by the nurse is best? 1. "You'll have to wait until the vaginal bleeding stops." 2. "You'll have to wait until the oxytocin is infused." 3. "You may use the bathroom with my assistance." 4. "You may get up to the bathroom whenever you need to."

Answer: 3 RATIONALES: The nurse should tell the client that she may use the bathroom with the nurse's assistance. The nurse should assist the client for the client's first trip to the bathroom after delivery. It isn't uncommon for a client to faint after delivery. Telling the client she must wait until her vaginal bleeding stops is inappropriate; vaginal bleeding continues for about 6 weeks after delivery. The nurse shouldn't tell the client she can get up whenever she needs to use the bathroom; doing so places the client at risk for injury.

A nulliparous client has been in the latent phase of the first stage of labor for several hours. Despite continued uterine contractions, her cervix hasn't dilated further since the initial examination. Her latent phase may be considered prolonged after: 1. 6 hours. 2. 10 hours. 3. 14 hours. 4. 20 hours.

Answer: 4 RATIONALES: Based on research, the latent phase may be considered prolonged if it exceeds 20 hours in a nulliparous client or 14 hours in a multiparous client.

The nurse is caring for a woman receiving a lumbar epidural anesthetic block to control labor pain. What should the nurse do to prevent hypotension?

Answer: 4 RATIONALES: Because the woman is in a state of relative hypovolemia, administering fluids I.V. before the epidural anesthetic is given may prevent hypotension. Administration of an epidural anesthetic may lead to hypotension because blocking the sympathetic fibers in the epidural space reduces peripheral resistance. Ephedrine may be administered after an epidural block if a woman becomes hypotensive and shows evidence of cardiovascular decompensation. However, ephedrine isn't administered to prevent hypotension. Oxygen is administered to a woman who becomes hypotensive, but it won't prevent hypotension. Placing a pregnant woman in the supine position can contribute to hypotension because of uterine pressure on the great vessels.

During assessment for admission to the labor and delivery area, a client and her husband ask the nurse whether their sons, ages 8 and 10, can witness the childbirth. Before answering this question, the nurse should consider which guideline? 1. The children and client should share a support person during the childbirth. 2. Children should attend childbirth only if it takes place at home. 3. Children shouldn't attend childbirth because it will frighten them. 4. Each child attending the childbirth should have a separate support person.

Answer: 4 RATIONALES: Each child attending the childbirth should have a support person — one who isn't also serving as the client's support person. The support person explains what is happening, reassures the child, and removes the child from the area if an emergency occurs or if the child becomes frightened. Children can attend childbirth in any setting. The decision to have a child present hinges on the child's developmental level, ability to understand the experience, and amount of preparation.

A client with active genital herpes is admitted to the labor and delivery area during the first stage of labor. Which type of birth should the nurse anticipate for this client? 1. Mid forceps 2. Low forceps 3. Induction 4. Cesarean

Answer: 4 RATIONALES: For a client with active genital herpes, cesarean birth helps avoid infection transmission to the neonate, which would occur during a vaginal birth. Mid forceps and low forceps are types of vaginal births that could transmit the herpes infection to the neonate. Induction is used only during vaginal birth; therefore, it's inappropriate for this client.

For a client who's fully dilated, which of the following actions would be inappropriate during the second stage of labor? 1. Positioning the mother for effective pushing 2. Preparing for delivery of the baby 3. Assessing vital signs every 15 minutes 4. Assessing for rupture of membranes

Answer: 4 RATIONALES: In most cases, the membranes have ruptured (spontaneously or artificially) by this stage of labor. Positioning for effective pushing, preparing for delivery, and assessing vital signs every 15 minutes are appropriate actions at this time.

A client with hemolysis, elevated liver enzymes, and low platelet count (HELLP) syndrome is admitted to the labor and delivery unit. The client's condition rapidly deteriorates and despite efforts by the staff, the client dies. After the client's death, the nursing staff displays many emotions. Who should the nurse manager consult to help the staff cope with this unexpected death? 1. The human resource director, so she can arrange vacation time for the staff 2. The physician, so he can provide education about HELLP syndrome 3. The social worker, so she can contact the family about funeral arrangements and pass along the information to the nursing staff 4. The chaplain, because his educational background includes strategies for handling grief

Answer: 4 RATIONALES: The chaplain should be consulted because his educational background provides strategies for helping others handle grief. Providing the staff with vacation isn't feasible from a staffing standpoint and doesn't help staff cope with their grief. The staff needs grief counseling, not education about HELLP syndrome. Asking the social worker to contact the family about the funeral arrangements isn't appropriate.

The nurse is caring for a client who's in the first stage of labor. What is the shortest but most difficult part of this stage? 1. Active phase 2. Complete phase 3. Latent phase 4. Transitional phase

Answer: 4 RATIONALES: The transitional phase, which lasts 1 to 3 hours, is the shortest but most difficult part of the first stage of labor. This phase is characterized by intense uterine contractions that occur every 1½ to 2 minutes and last 45 to 90 seconds. The active phase lasts 4½ to 6 hours; it's characterized by contractions that start out moderately intense, grow stronger, and last about 60 seconds. The complete phase occurs during the second, not first, stage of labor. The latent phase lasts 5 to 8 hours and is marked by mild, short, irregular contractions.

Which intervention would be helpful to a bottle-feeding client who's experiencing hard or engorged breasts? a) Applying ice b) Administering bromocriptine (Parlodel) c) Restricting fluids d) Applying warm compresses

Applying ice Explanation: Ice promotes comfort by decreasing blood flow (vasoconstriction), numbing the area, and discouraging further letdown of milk. Restricting fluids doesn't reduce engorgement and shouldn't be encouraged. Warm compresses will promote blood flow and hence, milk production, worsening the problem of engorgement. Bromocriptine has been removed from the market for lactation suppression.

When completing the morning postpartum data collection, the nurse notices the client's perineal pad is completely saturated. Which action should be the nurse's first response? a) Have the charge nurse review the assessment. b) Immediately call the primary care provider. c) Vigorously massage the fundus. d) Ask the client when she last changed her perineal pad.

Ask the client when she last changed her perineal pad. Correct Explanation: If the morning assessment is done relatively yearly, it's possible that the client hasn't yet been to the bathroom, in which case her perineal pad may have been in place all night. Secondly, her lochia may have pooled during the night, resulting in a heavy flow in the morning. Vigorous massage of the fundus, which is indicated for a boggy uterus, wouldn't be recommended as a first response until the client had gone to the bathroom, changed her perineal pad, and emptied her bladder. The nurse wouldn't want to call the primary care provider unnecessarily. If the nurse were uncertain, it would be appropriate to have another qualified individual check the client but only after a complete assessment of the client's status

A client gave birth to a healthy boy 2 days ago. Both mother and baby have had a smooth recovery. The nurse enters the room and tells the client that she and her baby will be discharged home today. The client states, "I do not want to go home." What is the nurse's most appropriate response?

Ask the client why she does not want to go home. It is important for the nurse to identify the client's concerns and reasons for wanting to stay in the hospital. Open-ended questioning facilitates both effective and therapeutic communication and allows the nurse to address concerns appropriately. Asking about supports at home implies that the nurse has made assumptions about why the client may not want to go home. Informing the care provider or telling the client that discharge is hospital policy is not appropriate at this time because the nurse has not addressed the underlying reason for the client's comment. The client may have safety-related concerns, undisclosed fears, or a need for increased support before discharge. It is imperative that the nurse not make assumptions but further explore concerns.

During an admission assessment of a client in labor, the nurse observes that there is no vaginal bleeding yet. What nursing intervention is appropriate in the absence of vaginal bleeding when the client is in the early stage of labor?

Assess amount of cervical dilation. If vaginal bleeding is absent during admission assessment, the nurse should perform vaginal examination to assess the amount of cervical dilation. Hydration status is monitored as part of the physical examination. A urine specimen is obtained for urinalysis to obtain a baseline. Vital signs are monitored frequently throughout the maternal assessment.

Seven hours ago, a G5 P4014 woman delivered a 4133-g male infant. She has voided once and calls for a nurse to check because she states that she feels "really wet" now. Upon examination, her perineal pad is saturated. The immediate nursing action is to a) Assess and massage the fundus b) Increase the flow of an IV c) Call the physician or the nurse-midwife d) Inspect the perineum for lacerations

Assess and massage the fundus Correct Explanation: This woman is a multigravida who delivered a large baby and is at risk for hemorrhage. The other actions are to be done after the initial fundal massage.

A postpartal woman has a history of thrombophlebitis. Which action would help the nurse determine if she is developing this postpartally?

Assess for calf redness and edema. Calf redness and edema, especially at the ankle and along the tibia, suggest thrombophlebitis.

A postpartal woman has a history of thrombophlebitis. Which of the following would help you to determine if she is developing this postpartally? a) Take her temperature every 4 hours. b) Assess for calf redness and edema. c) Ask her if she feels any warmth in her legs. d) Palpate her feet for tingling or numbness.

Assess for calf redness and edema. Correct Explanation: Calf redness and edema, especially at the ankle and along the tibia, suggest thrombophlebitis.

A 30-year-old G2P0010 has been in labor for 10 hours and is 6 cm dilated. She has already expressed a desire to use nonpharmacologic pain management techniques. For the past hour, she has been lying in bed with her doula rubbing her back. Now, she has begun to moan loudly, grit her teeth, and bear down with each contraction. She rates her pain as 8/10 with each contraction. What should the nurse do first? a) Assist the patient in ambulating to the bathroom. b) Assess for labor progression. c) Instruct the patient to do slow-paced breathing. d) Prepare the patient for an epidural.

Assess for labor progression. Correct Explanation: Performing breathing exercises, ambulating, changing position, and emptying the bladder all can help the patient experience a reduction in pain. However, the best first step is to assess the patient for labor progress before assisting her otherwise. Bearing down can be a sign that the patient is 10 cm dilated.

Betsy is recovering from a standard delivery. The nurse has just removed Betsy's epidural catheter and applied a sterile pressure dressing. What is it important for the nurse to do now? a) Make sure that Betsy receives plenty of fluids b) Help Betsy to get up and walk around immediately c) Let Betsy rest and recover while keeping her legs slightly elevated d) Assess return of sensory and motor functions to the lower extremities

Assess return of sensory and motor functions to the lower extremities Correct Explanation: After removal of the epidural catheter and medication is terminated, the nurse needs to assess for return of motor function to ambulate the mother. The mother will not be able to walk for some time; at least until the medication wears off. Do not elevate the legs; you want to maintain normal circulation. Fluids are important, but they are not related to the epidural or to the metabolism of the medication.

contraction pattern every 15 minutes

Assess the contraction pattern every 15 minutes. The pattern will be similar to that found in the transition phase (i.e., contractions occur every two to three minutes, last 60 to 90 seconds, and are of strong intensity).

Using the acronym COAT or TOCA, what is the nurse assessing and what are important to ask the client?

Assessing the amniotic fluid after the bag of membranes has been broken. • C : Color (Clear, Blood Tinged, Green Meconium) • O : Odor (Possible Infection) • A : Amount (Trickle (scant), Copius, Moderate) • T : Time (When? To prevent infection labor within 24 hrs.)

When caring for a postpartum woman who is Muslim, which of the following would be a priority? a) Ensuring that the newborn's daily bath is performed by the nurses b) Allowing time for the numerous visitors who come to see the woman and newborn c) Providing time for prayers to be performed at the bedside d) Assigning a female nurse to care for her

Assigning a female nurse to care for her Correct Explanation: Muslims prefer the same-sex health care provider; male-female touching is prohibited except in emergency situations. Nurses give the daily bath for newborns of some Japanese-American women. Numerous visitors can be expected to visit some women of the Filipino-American culture because families are very closely knit. Bedside prayer is common due to the strong religious beliefs of the Filipino-American culture.

A woman is bottle-feeding her baby. When the nurse comes into the room the woman says that her breasts are painful and engorged. Which nursing intervention is appropriate? a) Assist the woman in placing ice packs on her breasts b) Explain to the woman that she should breastfeed because she is producing so much milk c) Assist the woman into the shower and have her run cold water over her breasts d) Ask if she wants a breast pump to empty her breasts

Assist the woman in placing ice packs on her breasts Explanation: If the breasts are engorged and the woman is bottle-feeding her newborn, instruct her to keep a support bra on 24 hours per day. Cool compresses or an ice pack wrapped in a towel will usually be soothing and help to suppress milk production

A woman is bottle-feeding her baby. When the nurse comes into the room the woman says that her breasts are painful and engorged. Which nursing intervention is appropriate?

Assist the woman in placing ice packs on her breasts. If the breasts are engorged and the woman is bottle-feeding her newborn, instruct her to keep a support bra on 24 hours per day. Cool compresses or an ice pack wrapped in a towel will usually be soothing and help to suppress milk production.

Which of the following would alert the nurse to suspect that a newborn is experiencing respiratory distress? a) Asymmetrical chest movement b) Acrocyanosis c) Short periods of apnea (less than 15 seconds) d) Respiratory rate of 50 breaths/minute

Asymmetrical chest movement

A newborn is discharged from the hospital before undergoing metabolic screening. A community health nurse scheduling a follow-up home visit knows that the most appropriate time to perform the heel stick is: a) At least 24 hours after birth. b) Within 24 hours of birth. c) At least 36 hours after birth. d) Before the baby has received 8 feeds of breast milk or formula.

At least 24 hours after birth.

The AGPAR score is based on which 5 parameters? a) Heart rate, muscle tone, reflex irritability, respiratory effort, and color b) Heart rate, breaths per minute, irritability, reflexes, and color c) Hear rate, breaths per minute, irritability, tone, and color d) Heart rate, respiratory effort, temperature, tone, and color

Heart rate, muscle tone, reflex irritability, respiratory effort, and color

A newborn is discharged from the hospital before undergoing metabolic screening. A community health nurse scheduling a follow-up home visit knows that the most appropriate time to perform the heel stick is: a) Before the baby has received 8 feeds of breast milk or formula. b) Within 24 hours of birth. c) At least 36 hours after birth. d) At least 24 hours after birth.

At least 24 hours after birth. Correct Explanation: This screening needs to be done on an infant 24 hours to 7 days after birth. The timing of the screening test is determined by the age of the infant, not the number of feedings.

When assessing a woman in labor, the nurse is aware that the relationship of the fetal body parts to one another is called fetal:

Attitude

What is the best way for the nurse to assess the newborn's heartbeat? a) Auscultating the apical pulse for 30 seconds and multiplying by 2 b) Palpating the femoral pulse for 30 seconds and multiplying by 2 c) Auscultating the apical pulse for 60 seconds d) Palpating the brachial pulse for 60 seconds

Auscultating the apical pulse for 60 seconds

What is the best way for the nurse to assess the newborn's heartbeat? a) Palpating the femoral pulse for 30 seconds and multiplying by 2 b) Palpating the brachial pulse for 60 seconds c) Auscultating the apical pulse for 30 seconds and multiplying by 2 d) Auscultating the apical pulse for 60 seconds

Auscultating the apical pulse for 60 seconds Correct Explanation: The best way for the nurse to assess the newborn's heart rate is to listen at the apical pulse for a full minute.

What are signs/symptoms physically observed in the newborn that shows physical maturity?

Lanugo, plantar creases, breast tissue

Which statement is the best rationale for assessing maternal vital signs between contractions? a. During a contraction, assessing fetal heart rates is the priority. b. Maternal circulating blood volume increases temporarily during contractions. c. Maternal blood flow to the heart is reduced during contractions. d. Vital signs taken during contractions are not accurate.

B During uterine contractions, blood flow to the placenta temporarily stops, causing a relative increase in the mother's blood volume, which in turn temporarily increases blood pressure and slows pulse. It is important to monitor fetal response to contractions; however, this question is concerned with the maternal vital signs. Maternal blood flow is increased during a contraction. Vital signs are altered by contractions but are considered accurate for that period of time.

On inspecting a newborn's abdomen, which finding would you note as abnormal? a) Clear drainage at the base of the umbilical cord b) Bowel sounds present at two to three per minute c) Liver palpable 2 cm under the right costal margin d) Abdomen slightly protuberant (rounded)

Clear drainage at the base of the umbilical cord Correct Explanation: Clear drainage at the base of the umbilical cord suggests the child may have a patent urachus or a fistula to the bladder.

The slight overlapping of cranial bones or shaping of the fetal head during labor is called: a. Lightening. c. Ferguson reflex. b. Molding. d. Valsalva maneuver.

B Fetal head formation is called molding. Molding also permits adaptation to various diameters of the maternal pelvis. Lightening is the mother's sensation of decreased abdominal distention, which usually occurs the week before labor. The Ferguson reflex is the contraction urge of the uterus after stimulation of the cervix. The Valsalva maneuver describes conscious pushing during the second stage of labor.

The primary difference between the labor of a nullipara and that of a multipara is the: a. Amount of cervical dilation. c. Level of pain experienced. b. Total duration of labor. d. Sequence of labor mechanisms.

B In a first-time pregnancy, descent is usually slow but steady; in subsequent pregnancies, descent is more rapid, resulting in a shorter duration of labor. Cervical dilation is the same for all labors. Level of pain is individual to the woman, not to the number of labors she has experienced. The sequence of labor mechanisms is the same with all labors.

To teach patients about the process of labor adequately, the nurse knows that which event is the best indicator of true labor? a. Bloody show c. Fetal descent into the pelvic inlet b. Cervical dilation and effacement d. Uterine contractions every 7 minutes

B The conclusive distinction between true and false labor is that contractions of true labor cause progressive change in the cervix. Bloody show can occur before true labor. Fetal descent can occur before true labor. False labor may have contractions that occur this frequently; however, this is usually inconsistent.

The nurse has received report regarding her patient in labor. The woman's last vaginal examination was recorded as 3 cm, 30%, and ?2-2. The nurse's interpretation of this assessment is that: a. The cervix is effaced 3 cm, it is dilated 30%, and the presenting part is 2 cm above the ischial spines. b. The cervix is 3 cm dilated, it is effaced 30%, and the presenting part is 2 cm above the ischial spines. c. The cervix is effaced 3 cm, it is dilated 30%, and the presenting part is 2 cm below the ischial spines. d. The cervix is dilated 3 cm, it is effaced 30%, and the presenting part is 2 cm below the ischial spines.

B The correct description of the vaginal examination for this woman in labor is the cervix is 3 cm dilated, it is effaced 30%, and the presenting part is 2 cm above the ischial spines. The sterile vaginal examination is recorded as centimeters of cervical dilation, percentage of cervical dilation, and the relationship of the presenting part to the ischial spines (either above or below).

A new mother has been reluctant to hold her newborn. A nurse can promote this mother's attachment to her newborn by a) Allowing the mother to pick the best time to hold her newborn b) Talking about how the nurse held her own newborn while on the delivery table c) Showing a video of parents feeding their babies d) Bringing the newborn into the room

Bringing the newborn into the room Correct Explanation: Proximity of the newborn and the mother can promote interest in the newborn and a desire to hold. Exposure to other mothers and their behaviors can only serve to set up unrealistic and fearful situations for a reluctant mother.

A woman is bottle-feeding her baby. When the nurse comes into the room the woman says that her breasts are painful and engorged. Which nursing intervention is appropriate? A) Ask if she wants a breast pump to empty her breasts. B) Assist the woman in placing ice packs on her breasts. C) Explain to the woman that she should breastfeed because she is producing so much milk. D) Assist the woman into the shower, and have her run cold water over her breasts.

B) Assist the woman in placing ice packs on her breasts. If the breasts are engorged and the woman is bottle-feeding her newborn, instruct her to keep a support bra on 24 hours per day. Cool compresses or an ice pack wrapped in a towel will usually be soothing and help to suppress milk production.

When assessing a woman in labor, the nurse is aware that the relationship of the fetal body parts to one another is called fetal: a. Lie. c. Attitude. b. Presentation. d. Position.

C Attitude is the relation of the fetal body parts to one another. Lie is the relation of the long axis (spine) of the fetus to the long axis (spine) of the mother. Presentation refers to the part of the fetus that enters the pelvic inlet first and leads through the birth canal during labor at term. Position is the relation of the presenting part to the four quadrants of the mother's pelvis.

A nurse is caring for a breastfeeding client who reports engorgement. The nurse identifies that the client's condition is due to not fully emptying her breasts at each feeding. Which suggestion should the nurse make to help her prevent engorgement? A) Apply cold compresses to the breasts. B) Feed the baby at least every two or three hours. C) Provide the infant oral nystatin. D) Dry the nipples following feedings.

B) Feed the baby at least every two or three hours. The nurse should suggest the client feed the baby every two or three hours to help her reduce and prevent further engorgement. Application of cold compresses to the breasts is suggested to reduce engorgement for nonbreastfeeding clients. If the mother has developed a candidal infection on the nipples, the treatment involves application of an antifungal cream to the nipples following feedings and providing the infant with oral nystatin. The nurse can suggest drying the nipples following feedings if the client experiences nipple pain.

A woman who is about to be discharged after a vaginal birth notices a flea-like rash on her newborn's chest. The rash has tiny red lesions all across the nipple line. What does this rash indicate? A) It is a sign of a group beta streptococcus skin infection. B) It is a normal skin finding in a newborn. C) It is a self-limiting virus that does not require treatment. D) It is an indication that the woman has mistreated her newborn.

B) It is a normal skin finding in a newborn. This rash is most likely is erythema toxicum, also known as newborn rash.

During a postpartum exam on the day of birth, the woman reports that she is still so sore that she cannot sit comfortably. The nurse examines her perineum and find the edges of the episiotomy approximated without signs of a hematoma. Which intervention will be most beneficial at this point? A) Put on a witch hazel pad. B) Place an ice pack. C) Apply a warm washcloth. D) Notify a primary care provider.

B) Place an ice pack. The labia and perineum may be edematous after birth and bruised; the use of ice would assist in decreasing the pain and swelling. Applying a warm washcloth would bring more blood as well as fluid to the sore area, thereby increasing the edema and the soreness. Applying a witch hazel pad needs the order of the primary care provider. Notifying a care provider is not necessary at this time as this is considered a normal finding.

When caring for a postpartum client who has given birth vaginally, the nurse assesses the client's respiratory status, noting that it has quickly returned to normal. The nurse understands that which factor is responsible for this change? A) increased progesterone levels B) decreased intra-abdominal pressure C) use of anesthesia during birth D) decreased bladder pressure

B) decreased intra-abdominal pressure The nurse should identify decreased intra-abdominal pressure as the cause of the respiratory system functioning normally. Progesterone levels do not influence the respiratory system. Decreased bladder pressure does not affect breathing. Anesthesia used during birth causes the respiratory system to take a longer time to return to normal.

Elevation of a client's temperature is a crucial first sign of infection. However, when is elevated temperature not a warning sign of impending infection? A) when the elevated temperature exceeds 100.4º F (38º C) B) during the first 24 hours after birth owing to dehydration from exertion C) after any period of decreased intake D) when the white blood cell count is less than 10,000/mm³

B) during the first 24 hours after birth owing to dehydration from exertion Rapid breathing during labor and birth and limited oral intake can cause a self-limited period of dehydration that is resolved after birth by the diuresis that shortly follows. The option of "any period" is too broad and falsely encompasses all conditions. The other options are signs of infection.

Thirty minutes after receiving pain medication, a postpartum woman states that she still has severe pain in the perineal region. Upon assessing and palpating the site, what can the nurse expect to find that might be causing this severe pain? A) nothing—it is normal B) hematoma C) infection D) DVT

B) hematoma If a postpartum woman has severe perineal pain despite use of physical comfort measures and medication, the nurse should check for a hematoma by inspecting and palpating the area. If one is found, the nurse needs to notify the primary care provider immediately.

The nurse working on a postpartum client must check lochia in terms of amount, color, change with activity and time, and: A) consistency. B) odor. C) specific gravity. D) pH.

B) odor. The nurse when assessing lochia must do so in terms of amount, color, odor, and change with activity and time.

A nurse is caring for a client who is nursing her baby boy. The client reports afterpains. Secretion of which substance should the nurse identify as the cause of afterpains? A) prolactin B) oxytocin C) estrogen D) progesterone

B) oxytocin Secretion of oxytocin stimulates uterine contraction and causes the woman to experience afterpains. Decrease in progesterone and estrogen after placental delivery stimulates the anterior pituitary to secrete prolactin which causes lactation.

A mother points out to the nurse that following three meconium stools, her newborn has had a bright green stool. The nurse would explain to her that: A) her child may be developing an allergy to breast milk. B) this is a normal finding. C) this is most likely a symptom of impending diarrhea. D) her child will need to be isolated until the stool can be cultured.

B) this is a normal finding. Newborn stools typically pass through a pattern of meconium, green transitional, and then yellow.

When assessing the newborn's umbilical cord, what should the nurse expect to find? A) one smaller vein and two larger arteries B) two smaller arteries and one larger vein C) one smaller artery and two larger veins D) two smaller veins and one larger artery

B) two smaller arteries and one larger vein When inspecting the vessels in the umbilical cord, the nurse should expect to encounter one larger vein and two smaller arteries. In 0.5% of births (3.5% of twin births), there is only one umbilical artery, which can be linked to cardiac or chromosomal abnormalities.

A woman has just delivered a baby. Her prelabor vital signs were T - 98.8 B/P-P-R 120/70, 80, 20. Which combination of findings during the early postpartum period should be reported immediately to the RN? a) Shaking chills with a fever of 100.3 b) B/P-P-R 90/50, 120, 24 c) Bradycardia and excessive, soaking diaphoresis d) Blood loss of 250 mL and WBC 25,000 cells/mL

B/P-P-R 90/50, 120, 24 Explanation: The decrease in BP with an increase in HR and RR indicate a potential significant complication, and are out of the range of normals, from delivery and need to be reported to the RN and healthcare provider ASAP. Shaking chills with a temperature of 100.3ºF can occur due to stress on the body and is considered a normal finding. A fever of 100.4ºF should be reported to the RN. Options C and D are considered to be within normal limits after delivering a baby.

What is BUBBLE?

B: Breast U: Uterus B: Bladder B: Bowels L: Lochia E: Episiotomy E: Extremities E: Emotional status

A nurse tests a newborn's nervous functioning by stroking the sole of the baby's foot in an inverted "J" curve from the heel upward. The baby responds by fanning his toes. Which reflex has just been demonstrated? a) Rooting reflex b) Moro c) Extrusion d) Babinski reflex

Babinski reflex

A nurse tests a newborn's nervous functioning by stroking the sole of the baby's foot in an inverted "J" curve from the heel upward. The baby responds by fanning his toes. Which reflex has just been demonstrated?

Babinski reflex The Babinski reflex is demonstrated when the sole of a newborn's foot is stroked in an inverted "J" curve from the heel upward and the newborn fans the toes in response (positive Babinski sign). The rooting reflex is demonstrated when, in response to the newborn's cheek being stroked, the infant turns her head in that direction. This reflex serves to help a newborn find food: when a mother holds the child and allows her breast to brush the newborn's cheek, the reflex causes the baby to turn toward the breast. The extrusion reflex is demonstrated when a newborn extrudes any substance that is placed on the anterior portion of the tongue; this reflex prevents the swallowing of inedible substances. The Moro reflex is demonstrated when, in response to a sudden backward head movement, the newborn abducts and extends arms and legs, then swings the arms into an embrace position and pulls up the legs against the abdomen.

A nurse tests a newborn's nervous functioning by stroking the sole of the baby's foot in an inverted "J" curve from the heel upward. The baby responds by fanning his toes. Which reflex has just been demonstrated? a) Extrusion b) Moro c) Babinski reflex d) Rooting reflex

Babinski reflex Correct Explanation: The Babinski reflex is demonstrated when the sole of a newborn's foot is stroked in an inverted "J" curve from the heel upward and the newborn fans the toes in response (positive Babinski sign). The rooting reflex is demonstrated when, in response to the newborn's cheek being stroked, the infant turns her head in that direction. This reflex serves to help a newborn find food: when a mother holds the child and allows her breast to brush the newborn's cheek, the reflex causes the baby to turn toward the breast. The extrusion reflex is demonstrated when a newborn extrudes any substance that is placed on the anterior portion of the tongue; this reflex prevents the swallowing of inedible substances. The Moro reflex is demonstrated when, in response to a sudden backward head movement, the newborn abducts and extends arms and legs, then swings the arms into an embrace position and pulls up the legs against the abdomen.

Newborn sleep

Back to sleep No bottle propping Nothing in the crib, risk for suffocation no bumper guards

What are factors affecting attachment?

Background of parents Infant Care practices

The gestational age assessment is also called

Ballard score

Which intervention would be the best way for the nurse to prevent heat loss in a newborn while bathing? a) Limit the bathing time to 5 minutes. b) Bathe the baby under a radiant warmer. c) Bathe the baby in water between 90-93 degree water. d) Postpone breastfeeding until after the initial bath.

Bathe the baby under a radiant warmer.

The nurse is caring for a newborn of a mother with human immunodeficiency virus (HIV). What is the priority for the nurse to complete following delivery?

Bathe the newborn thoroughly The newborn should have a thorough bath immediately after birth to decrease the possibility of HIV transmission. It is recommended the newborn be tested for HIV at 14 to 21 days after birth, at 1-2 months and again at 4-6 months. Zidovudine should be administered within 6-12 hours post-delivery to help prevent transmission of HIV from the mother to the newborn.

Which is NOT a cause of jaundice in the newborn? a) Bilirubin overproduction b) Impaired bilirubin excretion c) Decreased bilirubin conjugation d) Bilirubin hyperexcretion

Bilirubin hyperexcretion

Which is NOT a cause of jaundice in the newborn? a) Bilirubin hyperexcretion b) Bilirubin overproduction c) Impaired bilirubin excretion d) Decreased bilirubin conjugation

Bilirubin hyperexcretion Correct Explanation: Overexcretion of bilirubin would not cause jaundice. Bilirubin overproduction, decreased bilirbuin conjugation, and impaired bilirubin excretion would cause hyperbilirubinemia, which leads to jaundice. (

Question: Place in the order in which they occur the following items regarding changes in fetal circulation at birth. 1. An increase in systemic blood pressure with continued increase in blood flow to the lungs 2. Pulmonary blood flow increases and pulmonary venous return to the left side of the heart increases 3. Birth 4. The foramen ovale closes 5. Pulmonary vascular resistance decreases 6. The ductus arteriosus closes

Birth Pulmonary vascular resistance decreases Pulmonary blood flow increases and pulmonary venous return to the left side of the heart increases The foramen ovale closes An increase in systemic blood pressure with continued increase in blood flow to the lungs The ductus arteriosus closes

A pregnant client mentions to the nurse that a friend has given her a variety of herbs to use during her upcoming labor to help manage pain. Specifically, she gave her chamomile tea, raspberry leaf tea, skullcap, catnip, jasmine, lavender, and black cohosh. Which of these should the nurse encourage the client not to take because of the risk of acute toxic effects such as cerebrovascular accident? a) Skullcap b) Catnip c) Jasmine d) Black cohosh

Black cohosh Explanation: Several herbal preparations have traditionally been used to reduce pain with dysmenorrhea or labor, although there is little evidence-based support for their effectiveness. Examples include chamomile tea for its relaxing properties; raspberry leaf tea (women freeze it into ice cubes to suck on), which is thought to strengthen uterine contractions; skullcap and catnip, which are thought to help with pain. Jasmine and lavender may both be mixed into oils and rubbed on the perineum before and during labor to soften the muscle and help prevent perineal tears. Black cohosh (squaw root), an herb that induces uterine contractions, is not recommended because of the risk of acute toxic effects such as cerebrovascular accident to the mother or fetus.

On assessment of a 2-day postpartum patient the nurses finds the fundus is boggy, at U and slightly to the right. What is the most likely cause of this assessment finding? a) Poor bladder tone b) Bladder distention c) Uteruine atony d) Full bowel

Bladder distention Explanation: The most often cause of a displaced uterus is a distended bladder. Ask the patient to void and then reassess the uterus. According to the scenario described, the most likely cause of the uterine findings would not be uterine atony. A full bowel or poor bladder tone would not cause a boggy and displaced fundus.

Which vital sign is not routinely assessed in a term, healthy newborn with 9/9 AGPARs? a) Pain b) Pulse c) Temperature d) Respirations e) Blood pressure

Blood pressure

Which vital sign is not routinely assessed in a term, healthy newborn with 9/9 AGPARs? a) Pain b) Pulse c) Respirations d) Temperature e) Blood pressure

Blood pressure Correct Explanation: Because the readings can be inaccurate, blood pressure is not routinely assessed in term, normal healthy newborns with normal AGPARs. It is assessed if there is a clinical indication such as suspected blood loss or low APGAR scores. Pain is assessed by objective signs of pain such as grimacing and crying in response to certain stimuli.

You are admitting a 10-pound newborn to the nursery. You know that it will be important to monitor what during the transition period? a) Temperature b) Heart rate c) Blood sugar d) Apgar score

Blood sugar

You are admitting a 10-pound newborn to the nursery. You know that it will be important to monitor what during the transition period? a) Temperature b) Heart rate c) Apgar score d) Blood sugar

Blood sugar Correct Explanation: Most facilities have protocols to guide nursing care in the treatment of hypoglycemia. Many pediatricians have preprinted orders that can be initiated if the glucose level falls below a predetermined level (usually 40-50 mg/dL).

Which occurrence is associated with cervical dilation and effacement?

Bloody show

1.A primigravida asks the nurse about signs she can look for that would indicate that the onset of labor is getting closer. The nurse should describe:

Bloody show Passage of the mucous plug (operculum) also termed pink/bloody show occurs as the cervix ripens

What are Mongolian spots?

Blue/purple splotches that appear on the lower back and buttocks of newborns. More prevalently found in African Americans, Asian, and Indian newborns but can occur in newborns of all races. They are not bruises.

Acrocyanosis What is it? Where is it located? Is it an expected finding?

Blueness of the hands and feet is normal in the newborn, will decrease over the first few days of life

A patient who delivered her infant 3 days ago and was discharged home calls her provider's office with a complaint of sweating all night. What is the cause of the increased perspiration?

Body secreting the excess fluids from pregnancy Copious diaphoresis occurs in the first few days after childbirth as the body rids itself of excess water and waste via the skin. The excessive diaphoresis is not caused by changes in hormones, nor because of the patient drinking too much fluid, nor because of the body trying to rid itself of the excess blood made during pregnancy.

Ophthalmia neonatorum is contracted when a mother has which sexually transmitted infection(s)? a) Chlamydia b) Gonorrhea c) Trichomonas d) Both B and C e) Both A and B

Both A and B

Ophthalmia neonatorum is contracted when a mother has which sexually transmitted infection(s)? a) Chlamydia b) Gonorrhea c) Trichomonas d) Both A and B e) Both B and C

Both A and B Correct Explanation: Colonization of chlamydia and gonorrhea in the vaginal tract can lead to ophthalmia neonatorum in the newborn.

BUBBLE-HE?

Breast size, shape, + engorgement Uterus-form or boggy Bladder-tender or distended BM Lochia-amt, color, odor, clots Episiotomy location-stitches, edema, redness Homan's sign-DVT Emotional status + bonding

A breastfeeding mother wants to know how to help her 2-week-old newborn gain the weight lost after birth. Which action should the nurse suggest as the best method to accomplish this goal?

Breastfeed the infant every 2 to 4 hours on demand. Breastfeeding the newborn every 2 to 4 hours on demand is the best way to help the infant gain weight the fastest. Normal weight gain for this age infant is .66 to 1 ounce (19 to 28 grams) per day, not 1.5 to 2 ounces (42.5 to 57 grams). Cereal is never given to infants this young. The mother does not need to pump her breast milk to measure it. As long as the newborn is feeding well and has 6+ wet diapers and 3+ stools, the infant is receiving adequate nutrition.

A breastfeeding mother wants to know how to help her 2-week-old newborn gain the weight lost after birth. Which action should the nurse suggest as the best method to accomplish this goal? a) Recommend that the mother pump her breast milk and measure it before feeding b) Weigh the infant daily to ensure that she is gaining 1 1/2 to 2 ounces per day c) Add cereal to the newborn's feedings twice a day d) Breastfeed the infant every 2-3 hours on demand

Breastfeed the infant every 2-3 hours on demand

What is the primary mechanism for temperature regulation in a newborn infant? a) Brown fat store usage b) Shivering and increased metabolic rate c) External with blankets by the nursing staff d) Skin to skin contact with mother

Brown fat store usage Explanation: Brown fat stores are the stores used by the newborn infant to maintain warmth until feeding begins and the infant is able to maintain temperature without assistance. The infant's thermoregulatory system is not fully functional at birth. Infants cannot shiver to warm themselves. The use of external blankets as well as skin to skin contact with the mother assist in keeping the baby's temperature within the normal range, but they are not the primary mechanism for temperature regulation in the newborn infant.

When educating patients in a maternal-newborn unit about prevention of infant abduction, what is essential in the effectiveness of prevention of abduction? a) Use of monitor attached to babies b) Cooperation by the parents with the hospital policies c) Staff awareness of infant abduction profiles d) Policy posted about security

Cooperation by the parents with the hospital policies

Which nursing assessment indicates that a woman who is in second-stage labor is almost ready to give birth? a. The fetal head is felt at 0 station during vaginal examination. b. Bloody mucus discharge increases. c. The vulva bulges and encircles the fetal head. d. The membranes rupture during a contraction.

C During the active pushing (descent) phase, the woman has strong urges to bear down as the presenting part of the fetus descends and presses on the stretch receptors of the pelvic floor. The vulva stretches and begins to bulge encircling the fetal head. Birth of the head occurs when the station is +4. A 0 station indicates engagement. Bloody show occurs throughout the labor process and is not an indication of an imminent birth. Rupture of membranes can occur at any time during the labor process and does not indicate an imminent birth.

While providing care to a patient in active labor, the nurse should instruct the woman that: a. The supine position commonly used in the United States increases blood flow. b. The "all fours" position, on her hands and knees, is hard on her back. c. Frequent changes in position will help relieve her fatigue and increase her comfort. d. In a sitting or squatting position, her abdominal muscles will have to work harder.

C Frequent position changes relieve fatigue, increase comfort, and improve circulation. Blood flow can be compromised in the supine position; any upright position benefits cardiac output. The "all fours" position is used to relieve backache in certain situations. In a sitting or squatting position, the abdominal muscles work in greater harmony with uterine contractions.

Newborn safety

Car seat upon discharge If underweight has to pass a car seat test via the nurse at the hospital, this is a monitoring of the infant including SAO2 rates while in the actual car seat that is going home with them must be able to maintain an adequate saturation rate. CPR training and first aid preferred for parents, but especially if NICU newborn going home Nothing in the crib Go back (on their back) to sleep Parents need to do a return demonstration of how to use the bulb syringe in case the baby starts gagging or trouble breathing.

What is the term for a small collection of blood that forms underneath the skull as a result of birth trauma? a) Caput succedaneum b) Erythema toxicum c) Cephalhematoma d) Vernix caseosa

Cephalhematoma

When assessing the fetus using Leopold maneuvers, the nurse feels a round, firm, movable fetal part in the fundal portion of the uterus and a long, smooth surface in the mother's right side close to midline. What is the likely position of the fetus? a. ROA c. RSA b. LSP d. LOA

C The fetus is positioned anteriorly in the right side of the maternal pelvis with the sacrum as the presenting part. RSA is the correct three-letter abbreviation to indicate this fetal position. The first letter indicates the presenting part in either the right or left side of the maternal pelvis. The second letter indicates the anatomic presenting part of the fetus. The third letter stands for the location of the presenting part in relation to the anterior, posterior, or transverse portion of the maternal pelvis. Palpation of a round, firm fetal part in the fundal portion of the uterus would be the fetal head, indicating that the fetus is in a breech position with the sacrum as the presenting part in the maternal pelvis. Palpation of the fetal spine along the mother's right side denotes the location of the presenting part in the mother's pelvis. The ability to palpate the fetal spine indicates that the fetus is anteriorly positioned in the maternal pelvis.

With regard to factors that affect how the fetus moves through the birth canal, nurses should be aware that: a. The fetal attitude describes the angle at which the fetus exits the uterus. b. Of the two primary fetal lies, the horizontal lie is that in which the long axis of the fetus is parallel to the long axis of the mother. c. The normal attitude of the fetus is called general flexion. d. The transverse lie is preferred for vaginal birth.

C The normal attitude of the fetus is general flexion. The fetal attitude is the relation of fetal body parts to one another. The horizontal lie is perpendicular to the mother; in the longitudinal (or vertical) lie the long axes of the fetus and the mother are parallel. Vaginal birth cannot occur if the fetus stays in a transverse lie.

With regard to the turns and other adjustments of the fetus during the birth process, known as the mechanism of labor, nurses should be aware that: a. The seven critical movements must progress in a more or less orderly sequence. b. Asynclitism sometimes is achieved by means of the Leopold maneuver. c. The effects of the forces determining descent are modified by the shape of the woman's pelvis and the size of the fetal head. d. At birth the baby is said to achieve "restitution" (i.e., a return to the C-shape of the womb).

C The size of the maternal pelvis and the ability of the fetal head to mold also affect the process. The seven identifiable movements of the mechanism of labor occur in combinations simultaneously, not in precise sequences. Asynclitism is the deflection of the baby's head; the Leopold maneuver is a means of judging descent by palpating the mother's abdomen. Restitution is the rotation of the baby's head after the infant is born.

What is nevus flammeus?

Called "port-wine" stain. commonly appears on the newborn's face or other body areas. It is a capillary angioma located directly under the skin.

What is nevus vasculosus?

Called "strawberry mark or strawberry angioma" is benign capillary hemagioma in the dermal and sudermallayers.

The maternity nurse understands that as the uterus contracts during labor, maternal-fetal exchange of oxygen and waste products: a. Continues except when placental functions are reduced. b. Increases as blood pressure decreases. c. Diminishes as the spiral arteries are compressed. d. Is not significantly affected.

C Uterine contractions during labor tend to decrease circulation through the spiral electrodes and subsequent perfusion through the intervillous space. The maternal blood supply to the placenta gradually stops with contractions. The exchange of oxygen and waste products decreases. The exchange of oxygen and waste products is affected by contractions.

The primapara tells the nurse, "My baby jumps every time I pick her up. Is she afraid that I will drop her?" Which response by the nurse would be best? A) "No, it is the blink reflex. It is meant to protect the eyes." B) "No, it is the tonic neck reflex. It signifies handedness." C) "No, it is the Moro reflex. This reflex stimulates the action of warding off an attacker." D) "Yes, she is afraid you will drop her."

C) "No, it is the Moro reflex. This reflex stimulates the action of warding off an attacker." The Moro reflex is known as the startle reflex. A startled newborn will extend the arms and legs away from the body and to the side. Then the arms come back toward each other with the fingers spread in a "C" shape. The arms look as if the newborn is trying to embrace something. The Moro reflex should be symmetrical.

The client is preparing to go home after a cesarean birth. The nurse giving discharge instructions stresses to the family that the client should be seen by her primary care provider within what time interval? A) 4 weeks B) 5 weeks C) 2 weeks D) 3 weeks

C) 2 weeks The general rule of thumb is for a woman who had a cesarean birth be seen within 2 weeks after hospital discharge, unless the primary care provider has indicated otherwise or if the client develops signs of infection or has other difficulties.

The nurse records a newborn's Apgar score at birth. A normal 1-minute Apgar score is: A) 1 to 2. B) 12 to 15. C) 7 to 10. D) 5 to 9.

C) 7 to 10. An Apgar score of 7 to 10 implies the infant is breathing well and cardiovascular adaptation is occurring.

The first stage of labor is often a time of introspection. In light of this, which information would guide the nurse's planning of nursing care? A) A woman should be left entirely alone during this period. B) A woman will rarely speak or laugh during this period. C) A woman may spend time thinking about what is happening to her. D) No nursing care is needed to be done during this time.

C) A woman may spend time thinking about what is happening to her. Women need a support person with them during all stages of labor.

A nurse is caring for an infant with an elevated bilirubin level who is under phototherapy. What evaluation data would best indicate that the newborn's jaundice is improving? A) Hematocrit is 38. B) Reticulocyte count is 6%. C) Bilirubin level went from 15 to 11. D) Skin looks less jaundiced.

C) Bilirubin level went from 15 to 11. The newborn has physiologic jaundice, which is related to decreased bilirubin conjugation. Newborns have relatively immature livers and cannot conjugate (break down) bilirubin as fast as needed. Bilirubin overproduction is responsible for causing jaundice. A serum bilirubin is the best way to determine whether the jaundice is improving. The other listed methods will not address the needed information.

While assessing the progress of the labor, the nurse explains that the fetal heart rate variability is moderate. Which explanation is best to use with the parents? A) FHR fluctuates less than 5 beats per minute. B) FHR fluctuates over 25 beats per minute. C) FHR fluctuates from 6 to 25 beats per minute. D) FHR fluctuation range is undetectable.

C) FHR fluctuates from 6 to 25 beats per minute.

At birth there are multiple changes in the cardiac and respiratory systems. What is one of the changes to occur at birth in the cardiovascular system? A) Oxygen is exchanged in the lungs. B) Fluid is removed from the alveoli and replaced with air. C) Pressure changes occur and result in closure of the ductus arteriosus. D) The oxygen in the blood decreases.

C) Pressure changes occur and result in closure of the ductus arteriosus. The ductus arteriosus is one of the openings through which there was fetal circulation. At birth, or within the first few days, this closes and the heart becomes the source of movement of blood to and from the lungs. The exchange of oxygen in the lungs is not a function of the cardiovascular system; it is a function of the respiratory system. Again, the removal of fluid from the alveoli is not a function of the cardiovascular system. The oxygen content of the blood increases; it does not decrease.

A nurse helps a postpartum woman out of bed for the first time postpartally and notices that she has a very heavy lochia flow. Which assessment finding would best help the nurse decide that the flow is within normal limits? A) The flow contains large clots. B) The flow is over 500 mL. C) The color of the flow is red. D) Her uterus is soft to your touch.

C) The color of the flow is red. A typical lochia flow on the first day postpartally is red; it contains no large clots; the uterus is firm, indicating that it is well contracted.

What is the most important thing a nurse can do during labor and birth to prevent maternal and fetal infection? A) Strictly follow universal precautions. B) Clean the woman's perineum with a Betadine scrub. C) Thoroughly wash the hands before and after client contact. D) Remove soiled drapes and linen; place an absorbent pad under the buttocks and two sterile perineal pads against the perineum.

C) Thoroughly wash the hands before and after client contact.

The nurse is determining how often contractions occur measuring from the beginning of the one contraction to the beginning of the next contraction. The nurse documents this finding as: A) duration. B) peak. C) frequency. D) intensity.

C) frequency. Frequency refers to how often the contractions occur and is measured from the beginning of one contraction to the beginning of the next contraction. Duration refers to how long a contraction lasts and is measured from the beginning of one contraction to the end of that same contraction. Intensity refers to the strength of the contraction determined by manual palpation or measured by an internal intrauterine pressure catheter. The peak or acme of a contraction is the highest intensity of a contraction.

A client appears to be resting comfortably 12 hours after giving birth to her first child. In contrast, she labored for more than 24 hours, the primary care provider had to use forceps to deliver the baby, and she had multiple vaginal examinations during labor. Based on this information what postpartum complication is the client at risk for developing? A) hemorrhage B) pulmonary emboli C) infection D) depression

C) infection There are many risk factors for developing a postpartum infection: operative procedures (e.g., forceps, cesarean section, vacuum extraction), history of diabetes, prolonged labor (longer than 24 hours), use of Foley catheter, anemia, multiple vaginal examinations during labor, prolonged rupture of membranes, manual extraction of placenta, and HIV.

What is a nursing intervention that helps prevent the most frequent side effect from epidural anesthesia in a pregnant client? A) administrating IV naloxone B) administrating IV ephedrine C) starting an IV and hanging IV fluids D) maintaining the client in a supine position

C) starting an IV and hanging IV fluids Prehydration with IV fluids helps to prevent the most common side effect of epidural anesthesia, which is hypotension (20%). If the client develops hypotension or respiratory depression, then IV ephedrine or IV naloxone, respectively, can be administered, but neither is preventative. Maintaining the client in a supine position is recommended for a spinal headache, which can be a side effect of epidural anesthesia but is not the most common side effect and is not preventative.

Which measurements were most likely obtained from a normal newborn born at 38 weeks to a healthy mother with no maternal complications? A) weight = 2000 g, length = 17 inches (43 cm), head circumference = 32 cm, and chest circumference = 30 cm B) weight = 2500 g, length = 18 inches (46 cm), head circumference = 32 cm, and chest circumference = 30 cm C) weight = 3500 g, length = 20 inches (51 cm), head circumference = 34 cm, and chest circumference = 32 cm D) weight = 4500 g, length = 22 inches (56 cm), head circumference = 36 cm, and chest circumference = 34 cm

C) weight = 3500 g, length = 20 inches (51 cm), head circumference = 34 cm, and chest circumference = 32 cm For a term infant, expected weight is 2500 to 4000 g; length is 19 to 21 inches (48 to 53 cm); head circumference is 33 to 35 cm; and chest circumference is 30.5 to 33 cm.

The nurse caring for a newborn has to perform assessment at various intervals. When should the nurse complete the second assessment for the newborn? A) within 30 minutes after birth, in the birthing area B) prior to the newborn being discharged C) within the first 2 to 4 hours, when the newborn reaches the nursery D) 24 hours after the newborn's birth

C) within the first 2 to 4 hours, when the newborn reaches the nursery The nurse should complete the second assessment for the newborn within the first 2 to 4 hours, when the newborn is in the nursery. The nurse should complete the initial newborn assessment in the birthing area and the third assessment before the newborn is discharged, whenever that may be.

What is erythema toxicum?

Called "newborn rash". is benign, idiopathic generalized, transient rash that occurs in up to 70% of all newborns during first week of life. Is usually on face, chest, and back. There is a lack of a pattern with the rash.

What information for a vagina exam serves as a basis for determining which stage of labor the women is ?

Cervical dilation and effacement Effacement: 0% = 2 cm long 50% = 1 cm long 100% = cervix is obliterated Dilation: 0 cm = cervix closed 5 cm= halfway dilated 10 cm= fully dilated and ready for birth passage

The laboring client is on continuous fetal monitoring when the nurse notes a decrease in the fetal heart rate with variable deceleration to 75 bpm. What is the initial nursing intervention?

Change the position of the client. Variable decelerations often indicate a type of cord compression. The initial response is to change the position and try to release the cord compression. If this does not work, apply oxygen while using the call light to alert others. If this continues, her fluid status needs to be assessed before increasing her IV rate.

A nurse is caring for a client postpartum who complains of sore nipples. The nurse observes that the client's newborn is unable to suck properly although latched well. What intervention should the nurse perform to assist the baby to suck properly? a) Prolong the gap between feedings b) Position baby to face the nipple c) Suggest bottle feeding d) Check the baby's frenulum

Check the baby's frenulum Explanation: The nurse should check the baby's frenulum to determine if it is shortened and notify the health care provider who may clip the frenulum to allow the movement necessary to feed. The nurse should suggest lifting the infant's head up to face the nipple when the infant pinches the nipple with his or her gums. Prolonging the gap between breastfeeding and bottle feeding are not applicable suggestion for improper sucking motions of a baby.

The nurse is assessing a woman in active labor. She notes a small mass above the symphysis pubis, rounded and distended, non-tender. What intervention should the nurse take next? a) Assume this is part of the uterus b) Check the chart for the last void c) Ask the patient if the mass has always been present d) Notify a physician about the mass

Check the chart for the last void Correct Explanation: The most probable explanation of the mass is the bladder, which is full. The nurse should determine the last void by the patient and offer to assist the patient to void or prepare to catheterize the patient to empty the bladder. This can be taken care of by the nurse. The patient would not likely know if the mass was always present or not, given its location. If it were the uterus, it would be tender to the touch.

The nurse is assessing a woman in active labor. She notes a small mass above the symphysis pubis, rounded, distended, and nontender. What intervention should the nurse take next?

Check the chart for the last void. The most probable explanation of the mass is the bladder, which is full. The nurse should determine the last void by the client and offer to assist the client to void or prepare to catheterize the client to empty the bladder. This can be taken care of by the nurse. The client would not likely know if the mass was always present or not, given its location. If it were the uterus, it would be tender to the touch.

What is the most important thing the nurse can teach the family of a newborn to prevent abduction while the baby is in the hospital?

Check the identification badge of any health care worker before he or she takes the baby from the room. Infant abduction is a concern, and all personnel should wear identification badges and introduce themselves to the parents before they enter the room or take the infant. If at any time the mother is suspicious, she has the right to not allow an individual to take the infant. Learning to recognize the baby's cry would be ineffective in the prevention of an infant abduction from the hospital because the baby may not be crying as it is carried out of the unit. Checking the name and number on the baby's identification bracelet would tell the family it is their baby, not if it is being abducted by someone who is not employed by the hospital.

What is the most important thing the nurse can teach the family of a newborn to prevent abduction while the baby is in the hospital? a) Check the identification badge of any health care worker before he or she takes the baby from the room. b) Check the number on the baby's identification bracelet. c) Check the name on the baby's identification bracelet. d) Learn to recognize the baby's cry.

Check the identification badge of any health care worker before he or she takes the baby from the room. Correct Explanation: Infant abduction is a concern, all personal should wear identification badges and introduce themselves to the parents before they enter the room or take the infant. If at any time the mother is suspicious, she has the right to not allow an individual to take the infant. Learning to recognize the baby's cry would be ineffective in the prevention of an infant abduction from the hospital, the baby may not be crying as it is carried out of the unit. Checking the name and number on the baby's identification bracelet would tell the family it is their baby, not if it is being abducted by someone who is not employed by the hospital.

The parent of a newborn asks what caused the bacterial infection ophthalmia neonatorum in the baby. The nurse correctly responds that ophthalmia neonatorum can be caused by which bacteria? Select all that apply.

Chlamydia Gonorrhea Colonization of chlamydia and gonorrhea in the vaginal tract can lead to ophthalmia neonatorum in the newborn.

Which of the following would be the best way the nurse can facilitate an effective birth plan for the patient to achieve adequate pain relief? a) The nurse prescribes alternative methods of pain relief b) The health care provider decides the best pain relief for the mother and family c) The client has the baby without any analgesic or anesthetic d) Client priorities and preferences are incorporated into the plan

Client priorities and preferences are incorporated into the plan Correct Explanation: The nurse and the patient would work together; the nurse needs to seek information on the desires of the patient and work to achieve the desired level of pain control for the labor and delivery experience.

Bonding

Close emotional attraction to a newborn by the parents that develops the first 30-60 minutes after birth

How do you promote comfort for an episiotomy?

Cold and heat applications, topical preparations, analgesics

A pregnant woman's pulse fluctuates throughout pregnancy and the early postpartum period. When assessing a 1-day postpartum woman's pulse, what is the first action a nurse should take in response to a rate of 56 bpm? a) Advise that the woman not get out of bed until the nurse returns with assistance. b) Compare the pulse rate of 56 bpm with her pulse rate on the first prenatal care visit. c) Ask the woman what she has had to eat today. d) Do nothing, this is normal.

Compare the pulse rate of 56 bpm with her pulse rate on the first prenatal care visit. Correct Explanation: During pregnancy, the distended uterus obstructs the amount of venous blood returning to the heart; after birth, to accommodate the increased blood volume returning to the heart, stroke volume increases. Increased stroke volume reduces the pulse rate to between 50 and 70 beats per minute. Be certain to compare a woman's pulse rate with the slower range expected in the postpartum period, not with the normal pulse rate in the general population. Pulse usually stabilizes to prepregnancy levels within 10 days.

A nurse, while examining a newborn, observes salmon patches on the nape of the neck and on the eyelids. Which is the most likely cause of these skin abnormalities? a) Bruising from the birth process b) An immature autoregulation of blood flow c) An allergic reaction to the soap used for the first bath d) Concentration of immature blood vessels

Concentration of immature blood vessels

A nurse is caring for a client administered general anesthesia for an emergency cesarean birth. The nurse notes the client's uterus is relaxed upon massage. What would the nurse do next?

Continue to massage the client's fundus. The nurse should monitor the client for uterine relaxation. If this is noted, the nurse would continually massage the client's fundus until it no longer felt boggy.

A woman gave birth vaginally approximately 12 hours ago and her temperature is now 100 degrees F. Which action would be most appropriate? a) Inspect the perineum for hematoma formation. b) Notify the health care provider about this elevation; this finding reflects infection. c) Obtain a urine culture; the woman most likely has a urinary tract infection. d) Continue to monitor the woman's temperature every 4 hours; this finding is normal.

Continue to monitor the woman's temperature every 4 hours; this finding is normal. Correct Explanation: A temperature of 100.4 degrees F or less during the first 24 hours postpartum is normal and may be the result of dehydration due to fluid loss during labor. There is no need to notify the physician, obtain a urine culture, or inspect the perineum (other than the routine assessment of the perineum), because this finding is normal.

What is the definition of Labor?

Contractions with cervical change.

A newborn is challenged to maintain an adequate body temperature. If a baby is placed too close to a cold air vent, the nurse can assume that the infant will lose heat by which mechanism? a) Conduction b) Convection c) Radiation d) None. This will not cause the infant to lose body heat.

Convection

Eliminating drafts in the delivery room and in the nursery will help to prevent heat loss in a newborn through which mechanism? a) Conduction b) Convection c) Evaporation d) Radiation

Convection Correct Explanation: Convection refers to loss of heat from the newborn's body to the cooler surrounding air.

A newborn is challenged to maintain an adequate body temperature. If a baby is placed too close to a cold air vent, the nurse can assume that the infant will lose heat by which mechanism? a) Conduction b) Convection c) Radiation d) None. This will not cause the infant to lose body heat.

Convection Correct Explanation: There are 4 main ways that a newborn loses heat, Convection is one of the four and occurs when cold air blows over the body of the infant resulting in a cooling to the infant. Conductive heat loss occurs when the newborn's skin touches a cold surface, causing body heat to transfer to the colder object. Heat loss occurs by radiation to a cold object that is close to, but not touching, the newborn. Evaporative heat loss happens when the newborn's skin is wet. As the moisture evaporates from the body surface, the newborn loses body heat along with the moisture. Option D is incorrect as the cold air blowing on the infant's skin will cause heat loss.

A newborn's axillary temperature is 97.6°F (36.4°C). He has a cap on his head. His T-shirt is damp with spit-up milk. His blanket is laid over him, and several children are in the room running around his bassinet. The room is comfortably warm, and the bassinet is beside the mother's bed away from the window and doors. What are the most likely mechanisms of heat loss for this newborn? a) Conduction and evaporation b) Convection and radiation c) Conduction and radiation d) Convection and evaporation

Convection and evaporation

A newborn's axillary temperature is 97.5 °F. He has a cap on his head. His T-shirt is damp with spit-up milk. His blanket is laid over him, and several children are in the room running around his bassinet. The room is comfortably warm, and the bassinet is beside the mother's bed away from the window and doors. What are the most likely mechanisms of heat loss for this newborn? a) Convection and evaporation b) Convection and radiation c) Conduction and evaporation d) Conduction and radiation

Convection and evaporation Correct Explanation: Conductive heat loss occurs when the newborn's skin touches a cold surface, causing body heat to transfer to the colder object. Heat loss by convection happens when air currents blow over the newborn's body. Evaporative heat loss happens when the newborn's skin is wet. As the moisture evaporates from the body surface, the newborn loses body heat along with the moisture. Heat loss also occurs by radiation to a cold object that is close to, but not touching, the newborn.

When educating patients in a maternal-newborn unit about prevention of infant abduction, what is essential in the effectiveness of prevention of abduction? a) Use of monitor attached to babies b) Policy posted about security c) Cooperation by the parents with the hospital policies d) Staff awareness of infant abduction profiles

Cooperation by the parents with the hospital policies Correct Explanation: The most essential piece to an effective infant abduction prevention plan is the cooperation of the parents. If the parents are not willing to participate in the unit policy, the unit is at risk. Posting security policies, placing monitors on the babies, and educating the staff about infant abduction profiles are not the most essential elements of an effective abduction prevention plan.

All of the following are signs of respiratory distress in the newborn EXCEPT a) Chest retractions b) Central cyanosis c) Nasal flaring d) Grunting e) Coughing and a respiratory rate above 50

Coughing and a respiratory rate above 50

All of the following are signs of respiratory distress in the newborn EXCEPT a) Central cyanosis b) Chest retractions c) Coughing and a respiratory rate above 50 d) Grunting e) Nasal flaring

Coughing and a respiratory rate above 50 Correct Explanation: Coughing and sneezing are normal reflexes present in newborns. The expected respiratory rate of newborn is 30 to 60 breaths per minute.

A woman is experiencing back labor and complains of intense pain in the lower back. Which is the most effective nursing intervention to relieve this type of pain? a) Effleurage of the abdomen during the contraction b) Counter pressure against the sacrum c) Conscious relaxation/guided imagery in low fowlers d) Pant-blow (breaths and puffs breathing techniques)

Counter pressure against the sacrum Correct Explanation: Counter pressure against the sacrum is a way to provide support and comfort for a women having intense back labor. Effleurage is ineffective for true back labor, as is conscious relaxation. Breathing will not diminish the pain of back labor.

A newborn male is circumcised. Which of the following instructions would you include in the discharge teaching plan for his parents? a) Cleanse the glans daily with alcohol. b) Notify her physician if it appears red and sore. c) Cover the glans generously with Vaseline. d) Soak the penis daily in warm water.

Cover the glans generously with Vaseline.

A newborn male is circumcised. Which of the following instructions would you include in the discharge teaching plan for his parents? a) Cleanse the glans daily with alcohol. b) Cover the glans generously with Vaseline. c) Notify her physician if it appears red and sore. d) Soak the penis daily in warm water.

Cover the glans generously with Vaseline. Correct Explanation: Covering the surgical site with an ointment such as petroleum jelly (Vaseline) prevents it from adhering to the diaper and being continually irritated. Normal appearance is red and raw. Soaking the penis daily in warm water is not recommended. Washing the penis with warm water, dribbled gently from a washcloth at each diaper change, is the recommended way of keeping the penis clean. You would not tell the parents to use alcohol on the glans.

What is descended tetes called ?

Cryptorchidism

In order to care for obstetric patients adequately, the nurse understands that labor contractions facilitate cervical dilation by: a. Contracting the lower uterine segment. b. Enlarging the internal size of the uterus. c. Promoting blood flow to the cervix. d. Pulling the cervix over the fetus and amniotic sac.

D Effective uterine contractions pull the cervix upward at the same time that the fetus and amniotic sac are pushed downward. The contractions are stronger at the fundus. The internal size becomes smaller with the contractions; this helps to push the fetus down. Blood flow decreases to the uterus during a contraction.

A postpartum client is having difficulty stopping her urine stream. Which should the nurse do next?

Educate the client on how to perform Kegel exercises. Clients should begin Kegel exercises on the first postpartum day to increase the strength of the perineal floor muscles. Priority for this client would be to educate her how to perform Kegel exercises as strengthening these muscles will allow her to stop her urine stream.

In order to evaluate the condition of the patient accurately during labor, the nurse should be aware that: a. The woman's blood pressure will increase during contractions and fall back to prelabor normal between contractions. b. Use of the Valsalva maneuver is encouraged during the second stage of labor to relieve fetal hypoxia. c. Having the woman point her toes will reduce leg cramps. d. The endogenous endorphins released during labor will raise the woman's pain threshold and produce sedation.

D The endogenous endorphins released during labor will raise the woman's pain threshold and produce sedation. In addition, physiologic anesthesia of the perineal tissues, caused by the pressure of the presenting part, decreases the mother's perception of pain. Blood pressure increases during contractions but remains somewhat elevated between them. Use of the Valsalva maneuver is discouraged during second stage labor because of a number of unhealthy outcomes, including fetal hypoxia. Pointing the toes can cause leg cramps, as can the process of labor itself.

The factors that affect the process of labor and birth, known commonly as the five Ps, include all except: a. Passenger. b. Powers. c. Passageway. d. Pressure.

D The five Ps are passenger (fetus and placenta), passageway (birth canal), powers (contractions), position of the mother, and psychologic response.

A new mother asks the nurse when the "soft spot" on her son's head will go away. The nurse's answer is based on the knowledge that the anterior fontanel closes after birth by _____ months. a. 2 b. 12 c. 8 d. 18

D The larger of the two fontanels, the anterior fontanel, closes by 18 months after birth.

A primigravida at 39 weeks of gestation is observed for 2 hours in the intrapartum unit. The fetal heart rate has been normal. Contractions are 5 to 9 minutes apart, 20 to 30 seconds in duration, and of mild intensity. Cervical dilation is 1 to 2 cm and uneffaced (unchanged from admission). Membranes are intact. The nurse should expect the woman to be: a. Admitted and prepared for a cesarean birth. b. Admitted for extended observation. c. Discharged home with a sedative. d. Discharged home to await the onset of true labor.

D This situation describes a woman with normal assessments who is probably in false labor and will probably not deliver rapidly once true labor begins. These are all indications of false labor without fetal distress. There is no indication that further assessment or cesarean birth is indicated. The patient will likely be discharged; however, there is no indication that a sedative is needed.

A new mother is concerned because it is 24 hours after birth and her breasts have still not become engorged with breast milk. How should the nurse respond to this concern? A) "I'm sorry to hear that. There are some excellent formulas on the market now, so you will still be able to provide for your infant's nutritional needs." B) "You may have developed mastitis. I'll ask the primary care provider to examine you." C) "You are experiencing lactational amenorrhea. It may be several weeks before your milk comes in." D) "It takes about 3 days after birth for milk to begin forming."

D) "It takes about 3 days after birth for milk to begin forming." The formation of breast milk (lactation) begins in a postpartal woman regardless of her plans for feeding. For the first 2 days after birth, an average woman notices little change in her breasts from the way they were during pregnancy as, since midway through pregnancy, she has been secreting colostrum, a thin, watery, prelactation secretion. On the third day post birth, her breasts become full and feel tense or tender as milk forms within breast ducts and replaces colostrum. There is no need to recommend formula feeding to the mother. Mastitis is inflammation of the lactiferous (milk-producing) glands of the breast; there is no indication that the client has this condition. Lactational amenorrhea is the absence of menstrual flow that occurs in many women during the lactation period.

When instructing a new mom on providing skin care to her newborn, which statement should not be included in the teaching? A) "Change diapers frequently." B) "Daily tub baths are not necessary." C) "Give the newborn sponge baths until the umbilical cord falls off." D) "Use talc powders to prevent diaper rash."

D) "Use talc powders to prevent diaper rash." Talc powders can be a respiratory hazard and should not be used with a newborn.

The nurse is teaching a postpartum woman and her spouse about postpartum blues. The nurse would instruct the couple to seek further care if the client's symptoms persist beyond which time frame? A) 1 week B) 4 weeks C) 3 weeks D) 2 weeks

D) 2 weeks Once postpartum blues are determined to be the likely cause of her mood symptoms, the nurse can offer anticipatory guidance that these mood swings are commonly experienced and usually resolve spontaneously within a week and offer reassurance. Women should also be counseled to seek further evaluation if these moods do not resolve within two weeks, as postpartum depression may be developing.

A pregnant woman, multipara, has been in labor for several hours. She cries out that her contractions are getting harder and that she cannot do this. The client is really irritable, nauseated, annoyed, and fearful of being left alone. Considering the client's behavior, the nurse would expect the cervix to be dilated how many centimeters? A) 3 to 5 B) 0 to 2 C) 5 to 6 D) 8 to 10

D) 8 to 10 The reaction of the client is indicative of entering or being in the transition phase of labor, stage 1. The dilation would be 8-10 cm. Before that, when dilation is 0 to 7 cm, the client has an easier time using positive coping skills.

A client in labor has requested the administration of narcotics to reduce pain. At 2 cm cervical dilatation, she says that she is managing the pain well at this point but does not want it to get ahead of her. What should the nurse do? A) Explain to the client that narcotics should only be administered an hour or less before birth. B) Agree with the client, and administer the drug immediately to keep the pain manageable. C) Refuse to administer narcotics because they can develop dependency in the client and the fetus. D) Advise the client to hold out a bit longer, if possible, before administration of the drug, to prevent slowing labor.

D) Advise the client to hold out a bit longer, if possible, before administration of the drug, to prevent slowing labor. The timing of administration of narcotics in labor is especially important as, if given too early (before 3 cm cervical dilatation), they tend to slow labor. If given close to birth, because the fetal liver takes 2 to 3 hours to activate a drug, the effect will not be registered in the fetus for 2 to 3 hours after maternal administration. For this reason, narcotics are preferably given when the mother is more than 3 hours away from birth. This allows the peak action of the drug in the fetus to have passed by the time of birth.

A nurse is caring for a client who gave birth about 10 hours earlier. The nurse observes perineal edema in the client. What intervention should the nurse perform to decrease the swelling caused due to perineal edema? A) Use ointments locally. B) Apply moist heat. C) Use a warm sitz bath or tub bath. D) Apply ice.

D) Apply ice. Ice is applied to perineal edema within 24 hours after birth. Use of ointments is not advised for perineal edema. Moist heat and a sitz or tub bath are encouraged if edema continues 24 hours after birth.

A client reports pain in the lower back, hips, and joints 10 days after the birth of her baby. What instruction should the nurse give the client after birth to prevent low back pain and injury to the joints? A) Try to avoid carrying the baby for a few days. B) Soak in a warm bath several times a day. C) Apply ice to the sore joints. D) Maintain correct posture and positioning.

D) Maintain correct posture and positioning. The nurse should recommend that clients maintain correct position and good body mechanics to prevent pain in the lower back, hips, and joints. Avoiding carrying her baby and soaking several times per day is unrealistic. Application of ice is suggested to help relieve breast engorgement in nonbreastfeeding clients.

A woman has just given birth vaginally to a newborn. Which action would the nurse do first? A) Administer vitamin K. B) Obtain footprints. C) Apply identification bracelet. D) Suction the mouth and nose.

D) Suction the mouth and nose. The first priority is to ensure a patent airway by suctioning the newborn's mouth and nose. Before leaving the birthing area, newborn identification procedures are completed, including applying the identification bracelet and possibly footprinting, depending on the agency's policy. Vitamin K is administered soon after birth, but it does not take priority over ensuring a patent airway.

Which intervention would be helpful to a bottle-feeding client who's experiencing hard or engorged breasts? A) applying warm compresses B) restricting fluids C) administering bromocriptine D) applying ice

D) applying ice Ice promotes comfort by decreasing blood flow (vasoconstriction), numbing the area, and discouraging further letdown of milk. Restricting fluids does not reduce engorgement and should not be encouraged. Warm compresses will promote blood flow and hence, milk production, worsening the problem of engorgement. Bromocriptine has been removed from the market for lactation suppression.

A nurse finds the uterus of a postpartum woman to be boggy and somewhat relaxed. This a sign of which condition? A) infection B) hemorrhage C) normal involution D) atony

D) atony The uterus in a postpartum client should be midline and firm. A boggy or relaxed uterus signifies uterine atony, which can predispose the woman to hemorrhage.

The nurse determines that the fetal heart rate averages approximately 140 beats per minute over a 10-minute period. The nurse identifies this as: A) short-term variability. B) fetal bradycardia. C) baseline variability. D) baseline FHR.

D) baseline FHR.

A nurse is describing the many changes that will occur during the early postpartum period with a group of young parents. The nurse reviews common reports experienced as the woman's body returns to her prepregnancy state. The nurse determines that the teaching was successful when the participants identify which report as being most common during the first week that will indicate their fluid volume is returning to normal? A) nocturia B) urinary urgency C) urinary frequency D) diaphoresis

D) diaphoresis The profuse diaphoresis is common during the early postpartum period. Many women will wake up drenched with perspiration. This diaphoresis is a mechanism to reduce the amount of fluids retained during pregnancy and restore prepregnant body fluid levels. It is common, especially at night during the first week after birth. Nocturia, urinary frequency, or urinary urgency are not associated with this fluid shift.

Discharge teaching is an important part of the labor and birth room nurse's position. New parents need to know the basics of baby care, like how to monitor fluid volume and when to call the primary care provider. What are the parameters for calling the care provider in regards to an infant's temperature? A) less than 96.7º F (35.9º C) or greater than 99.5º F (37.4º C) B) less than 96º F (35.6º C) or greater than 101º F (38.3º C) C) less than 97º F (36.1º C) or greater than 100.5º F (38.1º C) D) less than 97.7º F (36.5º C) or greater than 100º F (37.8º C)

D) less than 97.7º F (36.5º C) or greater than 100º F (37.8º C) Temperatures of less than 97.7 ° F (36.5° C) or greater than 100 ° F (37.8° C) should be reported to the primary care provider.

Two days ago, a woman gave birth to her third infant; she is now preparing for discharge home. After the birth of her second child, she developed an endometrial infection. Nursing goals for this discharge include all of the following except: A) the client will show no signs of infection. B) discuss methods that the woman will use to prevent infection. C) list signs of infection that she will report to her health care provider. D) maintain previous household routines to prevent infection.

D) maintain previous household routines to prevent infection. The nurse does not know whether previous routines were or were not the source of the infection. The other three options provide correct instructions to be given to this woman.

Assessment reveals that the fetus of a client in labor is in the vertex presentation. The nurse determines that which part is presenting? A) shoulders B) buttocks C) brow D) occiput

D) occiput With a vertex presentation, a type of cephalic presentation, the fetal presenting part is the occiput. The shoulders are the presenting part when the fetus is in a shoulder presentation. The brow or sinciput is the presenting part when a fetus is in a brow presentation. The buttocks are the presenting part when a fetus is in a breech presentation.

A nursing student is aware that fetal gas exchange takes place in which area? A) uterus B) bronchioles C) lungs D) placenta

D) placenta Many different changes occur for the newborn to survive outside the uterus. One such change is that gas exchange that once took place in the placenta now will take place in the lungs.

At what point should the nurse expect a healthy newborn to pass meconium? A) by 12 to 18 hours of life B) before birth C) within 1 to 2 hours of birth D) within 24 hours after birth

D) within 24 hours after birth The healthy newborn should pass meconium within 24 hours of life.

When caring for a postpartum client who has given birth vaginally, the nurse assesses the client's respiratory status, noting that it has quickly returned to normal. The nurse understands that which of the following is responsible for this change? a) Decreased intra-abdominal pressure b) Decreased bladder pressure c) Use of anesthesia during delivery d) Increased progesterone levels

Decreased intra-abdominal pressure Correct Explanation: The nurse should identify decreased intra-abdominal pressure as the cause of the respiratory system functioning normally. Progesterone levels do not influence the respiratory system. Decreased bladder pressure does not affect breathing. Anesthesia used during delivery causes the respiratory system to take a longer time to return to normal.

The nurse notices while holding him upright that a day-old newborn has a significantly indented anterior fontanelle. She immediately brings it to the attention of the physician. What does this finding most likely indicate? a) Dehydration b) Increased intracranial pressure c) Vernix caseosa d) Cyanosis

Dehydration

The nurse notices while holding him upright that a day-old newborn has a significantly indented anterior fontanelle. She immediately brings it to the attention of the physician. What does this finding most likely indicate? a) Vernix caseosa b) Cyanosis c) Dehydration d) Increased intracranial pressure

Dehydration

The nurse notices while holding him upright that a day-old newborn has a significantly indented anterior fontanelle. She immediately brings it to the attention of the physician. What does this finding most likely indicate? a) Cyanosis b) Vernix caseosa c) Dehydration d) Increased intracranial pressure

Dehydration Correct Explanation: The anterior fontanelle can be felt as a soft spot. It should not appear indented (a sign of dehydration) or bulging (a sign of increased intracranial pressure) when the infant is held upright. Vernix caseosa is the white, cream cheese-like substance that serves as a skin lubricant in utero. Some of it is invariably noticeable on a term newborn's skin, at least in the skin folds, at birth. Cyanosis is a condition of decreased oxygenation that results in the skin having a blue hue.

The nurse, assessing the lochia of a client, attempts to separate a clot and identifies the presence of tissue. Which of the following observations would indicate the presence of tissue? a) Yellowish white lochia b) Foul-smelling lochia c) Difficult to separate clots d) Easy to separate clots

Difficult to separate clots Explanation: If tissue is identified in the lochia, it is difficult to separate clots. Yellowish white lochia indicates increased leukocytes and decreased fluid content. Easily separable lochia indicates the presence of clots only. Foul-smelling lochia indicates endometritis.

When assessing a woman in the first stage of labor, the nurse recognizes that the most conclusive assessment that uterine contractions are effective would be: a) Bloody show b) Dilatation of cervix c) Engagement of fetus d) Rupture of amniotic membranes

Dilatation of cervix Correct Explanation: The best determination of effective contractions is dilation of the cervix. Engagement, membrane rupture, and bloody show may all occur before the cervix has dilated.

The nurse is monitoring a client in the first stage of labor. The nurse determines the client's uterine contractions are effective and progressing well based on which finding?

Dilation of cervix

The maternity nurse understands that as the uterus contracts during labor, maternal-fetal exchange of oxygen and waste products:

Diminishes as the spiral arteries are compressed.

A primigravida at 39 weeks of gestation is observed for 2 hours in the intrapartum unit. The fetal heart rate has been normal. Contractions are 5 to 9 minutes apart, 20 to 30 seconds in duration, and of mild intensity. Cervical dilation is 1 to 2 cm and uneffaced (unchanged from admission). Membranes are intact. The nurse should expect the woman to be:

Discharged home to await the onset of true labor

A client in the postpartum period complains of constipation. The nurse should inform the client of which of the following that contributes to postpartum constipation? a) Discomfort due to hemorrhoids b) Distention of abdominal muscles c) Relaxation of abdominal muscles d) Separation of rectus muscles

Discomfort due to hemorrhoids Explanation: The nurse should inform the client that hemorrhoids contribute to constipation postpartum. Distention of abdominal muscles, separation of rectus muscles, and relaxation of abdominal muscles are pregnancy-related developments and take time to heal.

Newborn nutrition

Do not prop if bottle feeding Feed 1-2 or 2-4 ounces per feeding Don't let baby fall asleep with bottle in mouth typical infant needs 110-120 calories/kg/day

You are the oncoming nursery nurse caring for a 3-hour-old newborn boy. You make your initial assessment and find the following: Respiratory rate 30 bpm, B/P 60/40 mm/Hg, heart rate 155, axillary temperature 98.2°F (36.8°C). You assess that the newborn is in a state of quiet alert. What would you do? a) Stimulate the newborn b) Document the data c) Call the physician d) Inform the charge nurse

Document the data

You are the oncoming nursery nurse caring for a 3-hour-old newborn boy. You make your initial assessment and find the following: Respiratory rate 30 bpm, B/P 60/40 mm/Hg, heart rate 155, temperature (Axillary) 36.8 °C. You assess that the newborn is in a state of quiet alert. What would you do? a) Stimulate the newborn b) Inform the charge nurse c) Call the physician d) Document the data

Document the data Correct Explanation: The normal respiratory rate is 30 to 60 breaths per minute and should be counted for a full minute when the infant is quiet. A newborn starts with a low blood pressure (60/40 mm/Hg) and a high pulse (120 to 160 bpm). Normal temperature range is between 97.7 °F (36.5 °C) and 99.5 °F (37.5 °C).

Mrs. Timms is now in the transition phase of labor. One of your concerns is the possibility of an ineffective breathing pattern. If one of your goals was for the woman's breathing pattern to be effective, what outcome would you expect? a) Uses accelerated breathing patterns continuously b) Refrains from using the pant-blow technique so she doesn't push c) Does not hyperventilate d) Pants through each contraction as she pushes

Does not hyperventilate Correct Explanation: Goal: The woman's breathing pattern is effective. Expected Outcomes: The woman uses accelerated breathing techniques during contractions. does not hyperventilate. uses pant-blow techniques to refrain from pushing despite pressure from the fetal head.

A nursing student is learning about intermittent fetal heart rate monitoring during labor. The student correctly chooses which of the following as used routinely for this procedure? (Select all that apply.)

Doppler fetoscope fetal monitor Intermittent fetal heart rate ascultation uses fetoscope, Doppler, or fetal monitor. An intrauterine pressure catheter is inserted into a pocket of amniotic fluid and is a continuous internal monitoring of contractions.

Elevation of a patient's temperature is a crucial first sign of infection. However, when is elevated temperature not a warning sign of impending infection? a) After any period of decreased intake b) When the white blood cell count is less than 10,000/mm³ c) When the elevated temperature exceeds 100.4° F d) During the first 24 hours after delivery owing to dehydration from exertion

During the first 24 hours after delivery owing to dehydration from exertion Correct Explanation: Rapid breathing during labor and delivery and limited oral intake can cause a self-limited period of dehydration that is resolved after delivery by the diuresis that shortly follows. The option of "any period" is too broad and falsely encompasses all conditions. The other options are signs of infection.

All the options are signs of respiratory distress in the newborn except: A) nasal flaring. B) chest retractions. C) grunting. D) coughing. E) respiratory rate >50 breaths/minute. F) central cyanosis

E) respiratory rate >50 breaths/minute. Coughing and sneezing are normal reflexes present in newborns. The expected respiratory rate of newborn is 30 to 60 breaths per minute.

A woman in early labor is using a variety of techniques to cope with her pain. When the nurse enters the room she notes that the woman is making light, circling movements with her fingertips across her abdomen. What technique is she using? a) Effleurage b) Abdominal imagery c) Massage d) Pain pathway blockage

Effleurage Correct Explanation: Effleurage, a form of touch that involves light circular fingertip movements on the abdomen, is a technique the woman can use in early labor. The theory is that light touch stimulates the nerve pathways to the brain and keeps them busy, thereby blocking the pain sensation.

What is molding? What causes it? What does it look like? Is there a problem?

Elongated shaping of the head from the accommodation (fitting through) the vaginal passage. IT occurs with a vaginal birth from a vertex (head) position in which elongation of the fetal head occurs with prominence of the occiput and overriding sagittal suture line. The head will have a cone shape. It will resolve in about a week after birth. Not considered a problem.

Bonding?

Emotional attraction in the 1st 30-60 min to hrs after birth Infant is quiet/alert and looks @ mom MOM

A breastfeeding client informs the nurse that she is unable to maintain her milk supply. What instruction should the nurse give to the client to improve milk supply?

Empty the breasts frequently. The nurse should tell the client to frequently empty the breasts to improve milk supply. Encouraging cold baths and applying ice on the breasts are recommended to relieve engorgement in nonbreastfeeding clients. Kegel exercises are encouraged to promote pelvic floor tone.

For several hours after birth a multigravida client who experienced a much more difficult labor this time than any time previously, wants to talk about why the birthing process was so hard for her. She is focusing on this aspect to the point that she seems relatively indifferent to her newborn. How should the nurse handle this situation?

Encourage her to discuss her experience of the birth and answer any questions or concerns she may have. The client needs to explore her birth experience and clarify her questions. The nurse should allow her to ask questions, be supportive, and encourage her to express her feelings. Redirecting her attention to the baby, asking her to describe how she plans to integrate the new baby into the family, or pointing out positive features of the new baby do not meet the needs of the client at this time.

A client reports she has not had a bowel moment since her infant was born 2 days ago. She asks the nurse what she can do to help her have a bowel movement. What intervention is appropriate to encourage having a bowel movement?

Encourage the client to eat more fiber rich foods. Encouraging fiber rich foods will help with prevention of constipation. The client needs plenty of water, to ambulate, and take stool softeners if ordered by the provider. Offering a stimulant laxative is not appropriate. Adding dairy products to the diet may be a good thing, but will not generally produce a bowel movement. Holding the feces until there is a strong urge to defecate will only increase the risk of constipation as well as possible resultant complications.

The patient under your care is complaining she has not had a bowel moment since her infant was born 2 days ago. She asks the nurse what she can do to help her have a bowel movement. What intervention is appropriate to encourage having a bowel movement? a) Add dairy products to the patient's diet b) Encourage the patient to eat more fiber rich foods c) Have her hold her feces until she really feels the need to defecate d) Offer the patient a stimulant laxative

Encourage the patient to eat more fiber rich foods Explanation: Encouraging fiber rich foods will help with prevention of constipation. The patient needs plenty of water, to ambulate, and take stool softeners if ordered by the provider. Offering a stimulant laxative is not appropriate. Adding dairy products to the diet may be a good thing, but will not generally produce a bowel movement. Holding the feces until there is a strong urge to defecate will only increase the risk of constipation as well as possible resultant complications.

How can the nurse be instrumental in preventing hypoglycemia in the newborn? Choose the best answer. a) Assessing the newborn's blood pressure within 1 hour of delivery b) Encouraging skin to skin for the first few minutes after birth c) Administering vitamin K within 1 hour of birth d) Encouraging early and frequent feedings

Encouraging early and frequent feedings Correct Explanation: The best way listed above to prevent hypoglycemia in the newborn is encouraging early and frequent feedings with the breast or a bottle. Skin to skin will aid in keeping the newborn warm and preventing hypothermia, which in time will also help to prevent hypoglycemia. However, a few minutes is not enough to prevent low glucose levels. It would need to be done as often as possible. Vitamin K is given to prevent hemorrhage. Blood pressure is not routinely checked in healthy, term newborns.

A client who gave birth by cesarean delivery 3 days ago is bottle-feeding her neonate. While collecting data the nurse notes that vital signs are stable, the fundus is four fingerbreadths below the umbilicus, lochia are small and red, and the client reports discomfort in her breasts, which are hard and warm to touch. The best nursing intervention based on this data would be: a) Using a breast pump to facilitate removal of stagnant breast milk. b) Having the client stand facing in a warm shower. c) Informing the physician that the client is showing early signs of breast infection. d) Encouraging the client to wear a supportive bra.

Encouraging the client to wear a supportive bra. Explanation: These assessment findings are normal for the third postpartum day. Hard, warm breasts indicate engorgement, which occurs approximately 3 days after birth. Vital signs are stable and don't indicate signs of infection. The client should be encouraged to wear a supportive bra, which will help minimize engorgement and decrease nipple stimulation. Ice packs can reduce vasocongestion and relieve discomfort. Warm water and a breast pump will stimulate milk production.

Which of the following is an appropriate nursing intervention for prevention of a urinary tract infection (UTI) in the postpartum woman? a) Screening for bacteriuria in the urine. b) Increasing oral fluid intake. c) Encouraging the woman to empty her bladder completely every 2 to 4 hours. d) Increasing intravenous fluids.

Encouraging the woman to empty her bladder completely every 2 to 4 hours. Explanation: The nurse should advise the woman to urinate every 2 to 4 hours while awake to prevent overdistention and trauma to the bladder. Maintaining a good fluid intake is also important, but it is not necessary to increase fluids if the woman is consuming enough. Screening for bacteria in the urine would require a physician's order and is not necessary as a prevention measure.

A client in the first stage of labor is admitted to a health care center. The nurse caring for the client instructs her to rock on a birth ball. The nurse informs her that this causes the release of certain natural substances, which reduces the pain. To which of the following substances is the nurse referring? a) Prostaglandins b) Endorphins c) Progesterone d) Relaxin

Endorphins Correct Explanation: The nurse is referring to the release of endorphins, which are natural analgesic substances released by the movement of the client on the birth ball. The nurse should encourage the client to rock or sit on the birth ball. This causes the release of endorphins. The client's movement on the birth ball does not produce prostaglandins, progesterone, or relaxin. Prostaglandins are local hormones that bring about smooth muscle contractions in the uterus. Progesterone is a hormone involved in maintaining pregnancy. Relaxin is a hormone that causes backache during pregnancy by acting on the pelvic joints.

Question: Place the cardinal movements of labor in the order in which they occur. Engagement, then descent Flexion Internal rotation Extension External rotation Expulsion

Engagement, then descent Flexion Internal rotation Extension External rotation Expulsion

The nurse is assessing a breastfeeding mom 72 hours after delivery. When assessing her breast, the patient complains of bilateral breast pain around the entire breast. What is the most likely cause of the pain? a) Mastitis b) Interductal yeast infection c) Engorgement d) Blocked milk duct

Engorgement Explanation: The patient is only 72 hours post delivery and is complaining of bilateral breast tenderness. Milk typically comes in at 72 hours after delivery, and with the production of the milk comes engorgement. The other problems do not typically develop until there is fully established breastfeeding. Therefore options A, B, and D are incorrect.

A nurse is applying ice packs to the perineal area of a client who has had a vaginal birth. Which intervention should the nurse perform to ensure that the client gets the optimum benefits of the procedure?

Ensure ice pack is changed frequently. The nurse should ensure that the ice pack is changed frequently to promote good hygiene and to allow for periodic assessments. Ice packs are wrapped in a disposable covering or clean washcloth and then applied to the perineal area, not directly. The nurse should apply the ice pack for 20 minutes, not 40 minutes. Ice packs should be used for the first 24 hours, not for a week after birth.

What are small unopened or plugged sebaceous glands that occur in a newborn's mouth and gums? a) Milia b) Epstein's pearls c) Stork bites d) Mongolian spots

Epstein's pearls

What are small unopened or plugged sebaceous glands that occur in a newborn's mouth and gums?

Epstein's pearls Unopened sebaceous glands are generally called milia. When they are in the mouth and gums, they are called Epstein's pearls.

What are small unopened or plugged sebaceous glands that occur in a newborn's mouth and gums? a) Epstein's pearls b) Milia c) Stork bites d) Mongolian spots

Epstein's pearls Correct Explanation: Unopened sebaceous glands are generally called milia. When they are in the mouth and gums, they are called Epstein's pearls.

The nurse is inspecting the mouth of a newborn and finds small, white cysts on the gums and hard palate. The nurse documents this finding as:

Epstein's pearls. Epstein's pearls are small, white epidermal cysts on the gums and hard palate that disappear in weeks. Thrush is white plaque inside the mouth caused by exposure to Candida albicans during birth, which cannot be wiped away with a cotton-tipped applicator. Milia are multiple pearly-white or pale yellow unopened sebaceous glands frequently found on a newborn's nose. Vernix caseosa is a thick white substance that protects the skin of the fetus. It is formed by secretions from the fetus's oil glands and is found during the first 2 or 3 days after birth in body creases and the hair.

A new mother asks the nurse why her baby's back and groin have a red and raised rash. Which of the following does the nurse correctly identify as the name of this condition? a) Mumps. b) Acrocyanosis. c) Erythema toxicum. d) Yeast infection.

Erythema toxicum

A new mother asks the nurse why her baby's back and groin have a red and raised rash. Which of the following does the nurse correctly identify as the name of this condition? a) Acrocyanosis. b) Erythema toxicum. c) Mumps. d) Yeast infection.

Erythema toxicum. Correct Explanation: Erythema toxicum is a rash of unknown cause, with pink papules and superimposed vesicles. It appears within 24 to 48 hours after birth and resolves spontaneously in a few days. Acrocyanosis is a blue color of the hands and feet appearing in most infants at birth. Acrocyanosis may persist for 7 to 10 days. Yeast is a fungal infection caused by Candida albicans; it usually manifests in the groin. The rash of C. albicans is excoriated and does not disappear without treatment. The presentation described in this scenario is not consistent with that of mumps.

Which of the following would the nurse expect to administer for eye prophylaxis in the newborn? a) Gentamicin ophthalmic ointment b) Erythromycin ophthalmic ointment c) Silver nitrate solution d) Vitamin K

Erythromycin ophthalmic ointment

Which of the following would the nurse expect to administer for eye prophylaxis in the newborn? a) Erythromycin ophthalmic ointment b) Vitamin K c) Silver nitrate solution d) Gentamicin ophthalmic ointment

Erythromycin ophthalmic ointment Correct Explanation: Erythromycin or tetracycline ophthalmic ointment is the agent of choice for newborn eye prophylaxis. Silver nitrate solution was once used for eye prophylaxis, but it is no longer used because it has little efficacy in preventing chlamydial eye disease. Vitamin K is used to promote blood clotting in the newborn. Gentamicin is not used for newborn eye prophylaxis.

The newborn has been placed in skin-to-skin contact with his mother. A blanket covers all of his body except his head. His hair is still wet with amniotic fluid, etc. What is the most likely type of heat loss this baby may experience? a) Convective b) Conductive c) Radiating d) Evaporative

Evaporative

The newborn has been placed in skin-to-skin contact with his mother. A blanket covers all of his body except his head. His hair is still wet with amniotic fluid, etc. What is the most likely type of heat loss this baby may experience? a) Radiating b) Conductive c) Convective d) Evaporative

Evaporative Correct Explanation: Evaporative heat loss occurs with the evaporation of fluid from the infant.

The mother of a newborn observes a diaper rash on her newborn's skin. Which intervention should the nurse instruct the parent to implement to treat the diaper rash? a) Use products such as talcum powder with each diaper change b) Expose the newborn's bottom to air several times a day c) Place the newborn's buttocks in warm water after each void or stool d) Use only baby wipes to cleanse the perianal area

Expose the newborn's bottom to air several times a day

The mother of a newborn observes a diaper rash on her newborn's skin. Which intervention should the nurse instruct the parent to implement to treat the diaper rash?

Expose the newborn's bottom to air several times a day. The nurse should instruct the parent to expose the newborn's bottom to air several times per day to treat and prevent diaper rashes. Use of baby wipes and products such as powder should be avoided. The parent should be instructed to place the newborn's buttocks in warm water after having had a diaper on all night but not with every diaper change.

Cultural influences?

Extended fam expected to care for infant so mom can rest Breastfeeding not indicated until milk comes in Food/drink dictated by beliefs Modesty Hygiene Emotional support Paternal presence @ delivery Gender of care providers

While assessing the progress of the labor, the nurse explains that the fetal heart rate variability is moderate. Which explanation is best to use with the parents?

FHR fluctuates from 6-25 beats per minute.

After pitocin has been administered, how often should the fetal monitoring be checked? How often should vitals be taken? How often should temperature be checked?

FHR/FT : Q15min. VS : Q30min Temp. :Q4hrs (if fever Q1hr.)

It is common for a newborn to have one or two erupted teeth (natal teeth) at birth. a) False b) True

False

Neonatal red blood cells have a life span of 120 days, while those of adults last 80 to 100 days. a) False b) True

False

A woman who has just given birth seems to be bonding with her newborn, despite the fact that earlier in labor she had expressed an intent to give the baby up for adoption. In this case, the nurse should encourage the mother to keep her baby.

False Do not attempt to change a woman's mind about keeping her child or placing the child for adoption during the postpartal period as she is extremely vulnerable to suggestion at this time, and such decisions are too long range and too important to be made at such an emotional time. Her earlier conclusion may be the sound one. Instead, offer nonjudgmental support. Be especially aware of your own feelings about this issue, to avoid influencing a woman's decision making unnecessarily.

It is common for a newborn to have one or two erupted teeth (natal teeth) at birth. a) False b) True

False Correct Explanation: It is highly unusual for a newborn to have erupted teeth (natal teeth) at birth.

Neonatal red blood cells have a life span of 120 days, while those of adults last 80 to 100 days. a) True b) False

False Correct Explanation: Neonatal red blood cells have a life span of 80 to 100 days. The adult red blood cell life span is 120 days.

The nursing instructor is teaching a group of nursing students about the uniqueness of pain involved with the birthing process. The instructor determines the session is successful when the students correctly choose which pain factor to be related to psychosocial influences?

Fear of pain during labor

A nurse is caring for a breastfeeding client who complains of engorgement. The nurse identifies that the client's condition is due to not fully emptying her breasts at each feeding. Which of the following should the nurse suggest to help her prevent engorgement? a) Dry the nipples following feedings b) Apply cold compresses to the breasts c) Provide the infant oral nystatin d) Feed the baby at least every two or three hours

Feed the baby at least every two or three hours Explanation: The nurse should suggest the client feed the baby every two or three hours to help her reduce and prevent further engorgement. Application of cold compresses to the breasts is suggested to reduce engorgement for non-breastfeeding clients. If the mother has developed a candidal infection on the nipples, the treatment involves application of an antifungal cream to the nipples following feedings and providing the infant with oral nystatin. The nurse can suggest drying the nipples following feedings if the client experiences nipple pain.

A nurse is caring for a breastfeeding client who reports engorgement. The nurse identifies that the client's condition is due to not fully emptying her breasts at each feeding. Which suggestion should the nurse make to help her prevent engorgement?

Feed the baby at least every two or three hours. The nurse should suggest the client feed the baby every two or three hours to help her reduce and prevent further engorgement. Application of cold compresses to the breasts is suggested to reduce engorgement for nonbreastfeeding clients. If the mother has developed a candidal infection on the nipples, the treatment involves application of an antifungal cream to the nipples following feedings and providing the infant with oral nystatin. The nurse can suggest drying the nipples following feedings if the client experiences nipple pain.

What would you teach to alleviate breast engorgement in a breast feeding mom?

Feeding at least every 2 hours,

As a woman enters the second stage of labor, which of the following would you expect to assess? a) Feelings of being frightened by the change in contractions b) Complaints of feeling hungry and unsatisfied c) Falling asleep from exhaustion d) Expressions of satisfaction with her labor progress

Feelings of being frightened by the change in contractions Correct Explanation: The nature of contractions changes so drastically to an urge to push that this can be frightening.

A pregnant client in her 32nd week of gestation has been admitted to a health care center with complaints of decreased fetal movement. Which of the following should the nurse determine first before placing the fetoscope on the woman's abdomen, so as to auscultate the fetal heart sounds? a) Fetal back b) Fetal shoulders c) Fetal buttocks d) Fetal head

Fetal back Correct Explanation: The nurse assessing the client should first determine the fetal back before placing the fetoscope on the client's abdomen. The fetal back is determined first because it is through the back that the heart signals are best transmitted. During labor, the fetal heart rate should be assessed to check for any variations indicating distress. Fetal heart rate is auscultated by placing a fetoscope on the client's abdomen in the area of the fetal back. Determining the fetal head, shoulders, and the buttocks would be of no help in localizing the heart sounds.

If a fetus were not receiving enough oxygen during labor because of uteroplacental insufficiency, which of the following patterns would you anticipate seeing on the monitor? a) Variable decelerations, too unpredictable to count b) Fetal baseline rate increasing at least 5 mm Hg with contractions c) Fetal heart rate declining late with contractions and remaining depressed d) A shallow deceleration occurring with the beginning of contractions

Fetal heart rate declining late with contractions and remaining depressed Correct Explanation: Lack of blood supply to the fetus because of poor placental filling prevents the fetal heart rate from recovering immediately following a contraction.

To care for a laboring woman adequately, the nurse understands that the __________ stage of labor varies the most in length.

First [The first stage of labor is considered to last from the onset of regular uterine contractions to full dilation of the cervix. The first stage is much longer than the second and third stages combined.]

On an Apgar evaluation, reflex irritability is tested by which of the following? a) Flicking the soles of the feet and observing the response b) Dorsiflexing a foot against pressure resistance c) Tightly flexing the infant's trunk and then releasing it d) Raising the infant's head and letting it fall back

Flicking the soles of the feet and observing the response

A 12-hour-old infant is receiving IV fluids for polycythemia. Which complications should a nurse monitor for in this client? a) Tachycardia b) Fluid overload c) Hypotension d) Decreased level of consciousness

Fluid overload

On examining a newborn's eyes, which of the following would you expect to assess? a) Has a white rather than a red reflex b) Produces tears when he cries c) Follows your finger a full 180 degrees d) Follows a light to the midline

Follows a light to the midline

On examining a newborn's eyes, which of the following would you expect to assess? a) Follows a light to the midline b) Has a white rather than a red reflex c) Follows your finger a full 180 degrees d) Produces tears when he cries

Follows a light to the midline Correct Explanation: Newborns do not usually follow past the midline until 3 months of age. They do not tear.

When doing a health assessment, at which of the following locations would you expect to palpate the fundus in a woman on the second postpartal day and how should it feel? a) Fundus two fingerbreadths below umbilicus and firm b) Fundus two fingerbreadths above symphysis pubis and hard c) Fundus height 4 cm below umbilicus and midline d) Fundus 4 cm above symphysis pubis and firm

Fundus two fingerbreadths below umbilicus and firm Correct Explanation: A uterine fundus typically regresses at a rate of one fingerbreadth a day, so on the second day postpartum it would be two fingerbreadths under the umbilicus and would feel firm.

What would you do preparing for discharge?

Give a subcutaneous injection of rubella vaccine and IM RhoGAM within 72 hrs

Reflexes: From the PowerPoints: know sucking, Moro, grasp, stepping, tonic neck, rooting, Babinski and gagging/sneezing/coughing reflexes

Go to page 588 in book

Beverly is being admitted to labor and delivery. When admitting an obstetric patient in early labor, the first intervention by the nurse is: a) Good rapport is established with the patient and significant other b) Perineal shave is done immediately to prepare for the examination c) Vital signs and FHR are assessed by internal electronic monitoring d) The personal belongings are properly checked and secured

Good rapport is established with the patient and significant other Correct Explanation: On admission the patient and her family need to establish a rapport with their caregiver. If the patient is stable and there is no immediate need, rapport should be established over actions that can be taken care of later.

What are contraindications to breastfeeding?

HIV Active drug use Cancer drugs Thyroid drugs Etc

What are some interventions for a newborn in respiratory distress? Use you critical thinking?

HOB, suction mouth then nose, SAO2, if needed oxygen, auscultate lungs

Describe 0 point characteristics

HR- absent Resp- Apneic Muscle tone- limp, flaccid Reflex irritability - no response Skin color- Cyanotic or pale

Describe 2 point characteristics

HR- greater than 100 Resp- regular resp. 30-60 good strong cry Muscle tone- tight flexion, good resistance to extension with quick return to flexed posture after extension Reflex irritability- sneeze, cough, vigorous cry Skin color-completely appropriate color (pink on both trunk and extremities)

Describe 1 point characteristics

HR- slow less than 100 Resp- slow irregular, shallow Muscle tone- some flexion, limited resistance to extension Reflex irritability - grimace or frown when irritated Skin color- appropriate body color,blue extremities acrocyanosis

A nurse is performing an assessment on a female client who gave birth 24 hours ago. On assessment, the nurse finds that the fundus is 2 cm above the umbilicus and boggy. Which intervention is a priority?

Have the client void, and then massage the fundus until it is firm. The fundus in a postpartum client should decrease 1 cm below the umbilicus each day. The fundus should also be firm to decrease the risk of postpartum hemorrhage. All of the listed interventions are appropriate, but a firm fundus is the priority.

Normal measurement of newborn? Head circumference Chest circumference Weight Length

Head circumference = 32-38 cm Chest circumference = 30-36 cm Weight = 2500-4000 g Length = 44-55 cm (17-22 inches)

The AGPAR score is based on which 5 parameters? a) Heart rate, respiratory effort, temperature, tone, and color b) Heart rate, breaths per minute, irritability, reflexes, and color c) Heart rate, muscle tone, reflex irritability, respiratory effort, and color d) Hear rate, breaths per minute, irritability, tone, and color

Heart rate, muscle tone, reflex irritability, respiratory effort, and color Correct Explanation: A newborn can receive an APGAR score ranging from 0 to 10. The score is based on 5 factors, each of which is assigned a 0, 1, or 2. Heart rate (should be above 100), muscle tone (should be able to maintain a flexion position), reflex irritability (newborn should cry or sneeze when stimulated), and respiratory effort are evaluted by the presence of a strong cry and by color. Color is evaluated by noting the color of the body and hands and feet.

During contractions, the electronic fetal monitor (EFM) shows variable V-shaped decelerations in the FHR lasting about 30 seconds with accelerations of about 5 bpm before and after each deceleration. Overshoot is absent and the baseline FHR is within normal limits. What should you do first? a) Help the woman change positions b) Discontinue supplemental oxygen c) Start an oxytocic infusion and decrease the rate of IV fluids d) Position the woman on her side with a pillow under her left hip

Help the woman change positions Correct Explanation: Changing positions is a first intervention to determine if this will improve the oxygen to the fetus. Supplemental oxygen should be maintained until the mother is stable. Placing the patient on her side may increase the work of breathing. Pharmacological interventions are premature.

Your client is in active labor. When you check the EFM tracing, you note variables that are nonreassuring. What would be your first nursing intervention? a) Obtain assistance to check for a compressed umbilical cord b) Prepare the woman for an emergency C-Section. c) Help the woman change positions d) Document the finding

Help the woman change positions Correct Explanation: First, assist the woman to change positions. Try to find a position that is comfortable for the woman that relieves the compression. If the variables stop after the position change, you will know that the compression has been relieved. However, if the variables continue, try a variety of position changes, including the knee-chest position.

During contractions, the electronic fetal monitor (EFM) shows variable V-shaped decelerations in the FHR lasting about 30 seconds with accelerations of about 5 bpm before and after each deceleration. Overshoot is absent, and the baseline FHR is within normal limits. What should the nurse do first?

Help the woman change positions. Changing positions is a first intervention to determine if this will improve the oxygen to the fetus. Supplemental oxygen should be maintained until the mother is stable. Placing the client on her side may increase the work of breathing. Pharmacological interventions are premature.

Infants receive vitamin K within the first hour after delivery. What is the rationale for administering the vitamin? a) Is a routine vitamin needed by the infant. b) Helps in formation of clotting factors, to prevent bleeding. c) Used to help infant fight infections. d) Administered to give the infant better eye sight.

Helps in formation of clotting factors, to prevent bleeding.

Infants receive vitamin K within the first hour after delivery. What is the rationale for administering the vitamin? a) Administered to give the infant better eye sight. b) Is a routine vitamin needed by the infant. c) Helps in formation of clotting factors, to prevent bleeding. d) Used to help infant fight infections.

Helps in formation of clotting factors, to prevent bleeding. Correct Explanation: Vitamin K is necessary in the formation of certain clotting factors. The newborn is lacking in vitamin K and the only method for the infant to receive it is to administer the vitamin IM. Vitamin K is manufactured by normal flora in the gut. Since the newborn has not yet eaten, there is no normal flora in the gut so the infant cannot manufacture vitamin K. Vitamin K is not administered to give the infant better eye sight nor is it to help fight infections.

What do we think may be the reason of persistent perineal pain despite meds?

Hematoma

A nursing student will pick which of the following as a correct laboratory value for a newborn? a) Hematocrit (HCT) 45% - 50% b) White blood cells (WBC)s 5-10,000mm³ c) Platelets 50,000-75,000/µL d) Hemoglobin (HBG) 17-20 g/dL

Hemoglobin (HBG) 17-20 g/dL The normal laboratory values for a newborn include HGB 17-20g/dL, HCT 52%-63%, platelets 100,000-300,000µL , RBCs 5.1-5.8, WBCs 10-30,000/mm³3

Which of the following laboratory test results would the nurse consider as a normal finding in a newborn soon after birth? a) Platelets: 400,000/uL b) Red blood cells: 3,500,000/uL c) Hemoglobin: 17.5 g/dL d) White blood cells: 5,000/mm3

Hemoglobin: 17.5 g/dL Correct Explanation: Hemoglobin typically ranges from 17 to 20 g/dL. White blood cells are initially elevated soon after birth as a result of birth trauma, typically ranging from 10,000 to 30,000/mm3. The newborn's platelet count is the same as that for an adult, ranging between 100,000 and 300,000/uL. After birth, the red blood cell count gradually increases as the cell size decreases. Normal count ranges from 5,100,000 to 5,800,000/uL.

The standard of care and recommendation by the Centers for Disease Control is to administer an immunization to all newborns. Which immunization is recommended to be administered prior to discharge? a) HiB b) DTaP c) Prevnar d) Hep B

Hep B

The standard of care and recommendation by the Centers for Disease Control is to administer an immunization to all newborns. Which immunization is recommended to be administered prior to discharge? a) Prevnar b) DTaP c) HiB d) Hep B

Hep B Correct Explanation: Hep B is the vaccination again Hepatitis B recommended by the CDC. All the other immunizations are recommended to be started at 2 months of age. Therefore options A, C, and D are incorrect.

At birth changes from fetal to newborn circulation must occur. What change causes the ductus arteriosus to close? a) Higher oxygen content of the circulating blood b) Increase in pressure in the left atrium of the heart c) Higher oxygen levels at the respiratory centers of the brain d) Drop in pressure in the neonate's chest

Higher oxygen content of the circulating blood

When auscultating the newborn's heart, the nurse would place the stethoscope at which area to auscultate the point of maximal impulse? a) At the midsternum, just below the suprasternal notch b) Lateral to the midclavicular line at the fourth intercostal space c) At the fifth intercostal space at the right midclavicular line d) At the third intercostal space adjacent to the midclavicular line

Lateral to the midclavicular line at the fourth intercostal space

A concerned client tells the nurse that her husband, who was very excited about the baby before its birth, is apparently happy but seems to be afraid of caring for the baby. What suggestion should the nurse give to the client's husband to resolve the issue?

Hold the baby frequently. The nurse should suggest that the father care for the newborn by holding and talking to the child. Reading up on parental care and speaking to his friends or the primary care provider will not help the father resolve his fears about caring for the child.

A nurse is educating the mother of a newborn about feeding and burping. Which strategy should the nurse offer to the mother regarding burping? a) Gently rub the newborn's abdomen while the newborn is in a sitting position b) Hold the newborn upright with the newborn's head on the mother's shoulder c) Lay the newborn on its abdomen in the mother's lap and gently pat the buttocks d) Lay the newborn on its back on its mother's lap

Hold the newborn upright with the newborn's head on the mother's shoulder

A client in her sixth week postpartum complains of general weakness. The client has stopped taking iron supplements that were prescribed to her during pregnancy. The nurse would assess the client for which of the following? a) Hyperglycemia b) Hypertension c) Hypovolemia d) Hypothyroidism

Hypovolemia Explanation: The nurse should assess the client for hypovolemia as the client must have had hemorrhage during birth and puerperium. Additionally, the client also has discontinued iron supplements. Hyperglycemia can be considered if the client has a history of diabetes. Hypertension and hyperthyroidism are not related to discontinuation of iron supplements.

What would you teach to alleviate breast engorgement in a bottle feeding mom?

Ice packs, supportive bra, and take mild analgesics such as acetaminophen. Avoid stimulation to the breasts that might foster milk production.

You are doing discharge teaching with the parents of a newborn baby girl. You know that it is important to teach them about diarrhea and dehydration. When should the parents notify the physician about diarrhea in the newborn.? a) If the infant has more than one episode of diarrhea in one day b) If the infant has more than four episodes of diarrhea in one day c) If the infant has more than two episodes of diarrhea in one day d) If the infant has more than three episodes of diarrhea in one day

If the infant has more than two episodes of diarrhea in one day

You are doing discharge teaching with the parents of a newborn baby girl. You know that it is important to teach them about diarrhea and dehydration. When should the parents notify the physician about diarrhea in the newborn.? a) If the infant has more than three episodes of diarrhea in one day b) If the infant has more than four episodes of diarrhea in one day c) If the infant has more than two episodes of diarrhea in one day d) If the infant has more than one episode of diarrhea in one day

If the infant has more than two episodes of diarrhea in one day Explanation: Diarrhea is defined as frequent stools with high water content. Because newborns dehydrate quickly, it is important for parents to notify the physician if the newborn has more than two episodes of diarrhea in one day.

What is the best rationale for trying to decrease the incidence of cold stress in the neonate? a) Evaporative heat loss happens when the neonate is not bundled and does not have a hat on. b) It takes energy to keep warm, so the neonate has to remain in an extended position. c) The neonate will stabilize its temperature by 8 hours after birth if kept warm and dry. d) If the neonate becomes cold stressed, it will eventually develop respiratory distress.

If the neonate becomes cold stressed, it will eventually develop respiratory distress.

While teaching a newborn nutrition class to a group of pregnant women, the nurse encourages breast-feeding because it is a major source of which immunoglobulin? a) IgG b) IgM c) IgE d) IgA

IgA

Physiological jaundice

Imbalance between the production and elimination of bilirubin that can lead to acute bilirubin encephalopathy

Taking-in?

Immediately after birth until 24-48 hrs Mother depends on others to help meet needs

A nurse is explaining the benefits of breastfeeding to a client who has just delivered. Which statement correctly explains the benefits of breastfeeding to this mother? a) Breastfed infants gain weight faster than formula fed infants after 6 month of age b) Breastfeeding provides more iron and calcium for the infant c) Immunoglobulin IgA in breast milk boosts a newborn's immune system d) Mothers who breastfeed have increased breast size following nursing

Immunoglobulin IgA in breast milk boosts a newborn's immune system

The nurse observes a newborn. He notes that the respiratory rate is 66, the newborn's nostrils flare out, and the newborn makes a grunting sound during respiration. What does the nurse conclude from these findings? The infant is: a) Burning brown fat b) Cold-stressed c) In respiratory distress d) Experiencing radiation heat loss

In respiratory distress

The nurse observes a newborn. He notes that the respiratory rate is 66, the nostrils flare out, and the newborn makes a grunting sound during respiration. What does the nurse conclude from these findings? The infant is: a) Burning brown fat b) Cold-stressed c) In respiratory distress d) Experiencing radiation heat loss

In respiratory distress Correct Explanation: The assessment findings discussed are signs of respiratory distress. An infant with a respiratory rate of greater than 60 with noise requires further assessment. All newborns burn brown fat to produce heat for their bodies. This is not something the nurse can assess. The scenario described does not indicate that the newborn is cold-stressed nor experiencing radiation heat loss. Therefore options A, B and D are incorrect.

Which of the following is FALSE regarding newborn behavioral patterns? a) An initial period of reactivity is followed by a longer period of decreased responsiveness. b) Newborns are usually predictable in the first several hours after birth. c) In the first few hours after birth, newborns do not typically demonstrate a response to visual stimuli. d) Newborns are usually awake in the first 30 following birth and will demonstrate a spontaneous Moro and rooting reflexes.

In the first few hours after birth, newborns do not typically demonstrate a response to visual stimuli.

A woman who is breastfeeding her newborn reports that her breasts seem quite full. Assessment reveals that her breasts are engorged. Which of the following would the nurse identify as the most likely factor for this development? a) Cracking of the nipple b) Inability of infant to empty breasts c) Inadequate secretion of prolactin d) Improper positioning of infant

Inability of infant to empty breasts Explanation: For the breastfeeding mother, engorgement is often the result of vascular congestion and milk stasis, primarily caused by the infant not fully emptying the mother's breasts at each feeding. Cracking of the nipple could lead to infection. Improper positioning may lead to nipple tenderness or pain. Inadequate secretion of prolactin causes a decrease in the production of milk.

The nurse is assisting Monica through labor, monitoring her closely, now that she has received an epidural. The nurse would report which finding to the anesthesiologist? a) Rapid progress of labor b) Urinary retention c) Dry, cracked lips d) Inability to push

Inability to push Correct Explanation: If she is not able to push, her epidural dose may be adjusted to decrease the impact on the sensory system. Dry lips indicate that she may need fluids, so you should give her some ice chips or a drink of water. Urinary retention and rapidly progressing labor should be directly reported to the obstetrician, not the anesthesiologist.

The nurse would expect which maternal cardiovascular finding during labor?

Increased cardiac output

A woman who delivered 10 hours ago is ambulating to the bathroom and calls for assistance with perineal care. When the nurse touches her skin, he notices that she is excessively warm. After reinforcing the woman's self-care, the nurse encourages increased oral intake. Why was this the appropriate instruction to give to this patient? a) Increased intake will rehydrate the patient and decrease her skin temperature. b) Increased intake will increase the patient's output and therefore will provide an opportunity for more frequent perineal self-care. c) The patient needs to walk to the bathroom more often. d) The patient will have to call for the nurse's help more often.

Increased intake will rehydrate the patient and decrease her skin temperature. Correct Explanation: The perception of increased skin temperature a short time post delivery is related to dehydration from the exertion of labor. Therefore rehydration should help to decrease skin temperature. Information is insufficient to suggest the presence of infection. Goals of more frequent perineal care and ambulation, as well as reinforcement of patient teaching, are not appropriate in this situation.

A nurse working on the postpartum floor is mentoring a new graduate and instructs the new nurse to make sure that patients empty their bladders. A full bladder can lead to which of the following complications? a) Increased lochia drainage b) Ruptured bladder c) Fluid volume overload d) Permanent urinary incontinence

Increased lochia drainage Correct Explanation: If the bladder is full in a postpartum mother, lochia drainage will be more than normal because the uterus cannot contract to suppress the bleeding. The other options do not happen if a woman has a distended bladder.

When describing the events that occur in a newborn when he or she experiences a cold environment, which of the following would the nurse identify as occurring first? a) Increased release of norepinephrine b) Breakdown of triglycerides c) Increased cardiac output d) Increased blood flow through brown fat

Increased release of norepinephrine

Which of the following nursing diagnosis would be highest in priority for a newborn? a) Ineffective thermoregulation related to heat loss to the environment. b) Altered nutrition less than body requirement related to limited formula intake. c) Altered urinary elimination related to post-circumcision status. d) Ineffective airway clearance related to mucous obstruction.

Ineffective airway clearance related to mucous obstruction.

Which nursing diagnosis would be highest in priority for a newborn?

Ineffective airway clearance related to mucous obstruction. Any airway clearance or obstruction issue is the highest priority for nursing interventions. The other options are valid nursing diagnoses for some newborns; however, they would not take precedence over an airway problem.

Which of the following nursing diagnosis would be highest in priority for a newborn? a) Ineffective thermoregulation related to heat loss to the environment. b) Altered nutrition less than body requirement related to limited formula intake. c) Altered urinary elimination related to post-circumcision status. d) Ineffective airway clearance related to mucous obstruction.

Ineffective airway clearance related to mucous obstruction. Correct Explanation: Any airway clearance or obstruction issue is the highest priority for nursing interventions. Options A, B, and C are valid nursing diagnoses for some newborns; however, they would not take precedence over an airway problem.

A nurse is teaching a couple about patterned breathing during their birth education. Which technique should the nurse suggest for slow-paced breathing?

Inhale slowly through nose and exhale through pursed lips. For slow-paced breathing, the nurse should instruct the woman to inhale slowly through her nose and exhale through pursed lips. In shallow or modified-pace breathing, the woman should inhale and exhale through her mouth at a rate of 4 breaths every 5 seconds. In pattern-paced breathing, the breathing is punctuated every few breaths by a forceful exhalation through pursed lips. Holding the breath for 5 seconds after every three breaths is not recommended in any of the three levels of patterned breathing.

A pregnant client has opted for hydrotherapy for pain management during labor. Which measure should the nurse consider when assisting the client during the birthing process?

Initiate the technique only when the client is in active labor. The recommendation for initiating hydrotherapy is that women be in active labor (>5 cm dilated) to prevent the slowing of labor contractions secondary to muscular relaxation. Women are encouraged to stay in the bath or shower as long as they feel they are comfortable. The water temperature should not exceed body temperature. The woman's membranes can be intact or ruptured.

How do you promote breastfeeding?

Initiate w/in 30-60 min of birth Breastfeed on demand-q2-3hrs Lactation consultant Rooming in Inc maternal caloric I by 500 and fluid by 2 qts Refrain from lotion/soaps on breast except lanolin

How is hypoglycemia prevented?

Initiated early feeding with breast feeding or milk

A multigravida is admitted to the hospital in active labor. The client's and the fetus' condition have been good since admission. The client calls out to the nurse, "the baby is coming!" What is the first action of the nurse? a) Contact the physician b) Auscultate the fetal heart tones c) Inspect the perineum d) Time the contractions

Inspect the perineum Correct Explanation: The nurse needs to determine if delivery is imminent and be prepared for delivery. Once the nurse assesses the coming labor, the heart sounds, contraction rate, and contacting the physician can all be done, if there is time.

A very healthy mother delivered a newborn with an immediate Apgar score of 10. The newborn was cradled in a kangaroo hold by both her mother and her father for 45 minutes. The parents feel ready to get cleaned up and let the newborn be taken care of by the health care personnel for a little while. What eye care action will the nurse now take? a) Instill 1 percent erythromycin eye drops b) Instill antibiotic 0.5 percent erythromycin c) Wait to see if the eyes show signs of irritation before any eye care treatment is completed d) Instill 0.5 percent silver nitrate eye drops

Instill antibiotic 0.5 percent erythromycin

A very healthy mother delivered a newborn with an immediate Apgar score of 10. The newborn was cradled in a kangaroo hold by both her mother and her father for 45 minutes. The parents feel ready to get cleaned up and let the newborn be taken care of by the health care personnel for a little while. What eye care action will the nurse now take? a) Instill 1 percent erythromycin eye drops b) Instill antibiotic 0.5 percent erythromycin c) Instill 0.5 percent silver nitrate eye drops d) Wait to see if the eyes show signs of irritation before any eye care treatment is completed

Instill antibiotic 0.5 percent erythromycin Correct Explanation: The standard eye care to prevent ophthalmia neonatorum is 0.5 percent erythromycin or 1 percent tetracycline eye drops. Although 1% silver nitrate drops where once used, it has not been shown to prevent chlamydial eye disease. One percent erythromycin and 0.5 percent silver nitrate are incorrect concentrations of these medications and should not be instilled into the eyes of the newborn. The nurse would not wait to see if the eyes show signs of irritation before completing eye care treatment on the newborn.

A very healthy mother gave birth to a newborn with an immediate Apgar score of 10. The newborn was cradled in a kangaroo hold by both her mother and her father for 45 minutes. The parents feel ready to get cleaned up and let the newborn be taken care of by the health care personnel for a little while. What eye care action will the nurse now take?

Instill antibiotic 0.5 percent erythromycin. The standard eye care to prevent ophthalmia neonatorum is 0.5 percent erythromycin or 1 percent tetracycline eye drops. Although 1% silver nitrate drops were once used, it has not been shown to prevent chlamydial eye disease. One percent erythromycin and 0.5 percent silver nitrate are incorrect concentrations of these medications and should not be instilled into the eyes of the newborn. The nurse would not wait to see if the eyes show signs of irritation before completing eye care treatment on the newborn.

A nurse is assessing a postpartum client. Which measure is appropriate?

Instruct the client to empty her bladder before the examination. An empty bladder facilitates examination of the fundus. The client should be supine with arms at her sides and her knees bent. The arms-overhead position is unnecessary. Clean gloves should be used when assessing the perineum; sterile gloves are not necessary. The postpartum examination should not be done quickly. The nurse can take this time to teach the client about the changes in her body after birth.

A nurse is assessing a postpartum client. Which of the following measures is appropriate? a) Wear sterile gloves when assessing the pad and perineum. b) Perform the examination as quickly as possible. c) Instruct the client to empty her bladder before the examination. d) Place the client in a supine position with her arms overhead for the examination of her breasts and fundus.

Instruct the client to empty her bladder before the examination. Correct Explanation: An empty bladder facilitates examination of the fundus. The client should be supine with arms at her sides and her knees bent. The arms-overhead position is unnecessary. Clean gloves should be used when assessing the perineum; sterile gloves are not necessary. The postpartum examination should not be done quickly. The nurse can take this time to teach the client about the changes in her body after delivery.

The process by which the reproductive organs return to the nonpregnant size and function is termed what? a) Evolution b) Involution c) Decrement d) Progression

Involution Explanation: Involution is the term used to describe the process of the return to nonpregnancy size and function of reproductive organs. Evolution is change in the genetic material of a population of organisms from one generation to the next. Decrement is the act or process of decreasing . Progression is defined as movement through stages such as the progression of labor. Options A, C, and D are distracters for this question.

A woman who is about to be discharged after a vaginal birth notices a flea-like rash on her newborn's chest. The rash has tiny red lesions all across the nipple line. This is likely an indication of what? a) It is a self-limiting virus that does not require treatment. b) It is an indication that the woman has mistreated her newborn. c) It is a sign of a group beta streptocoous skin infection. d) It is a normal skin finding in a newborn.

It is a normal skin finding in a newborn.

A woman who is about to be discharged after a vaginal birth notices a flea-like rash on her newborn's chest. The rash has tiny red lesions all across the nipple line. What does this rash indicate?

It is a normal skin finding in a newborn. This rash is most likely is erythema toxicum, also known as newborn rash.

The nurse is reviewing the uterine contraction pattern and identifies the peak intensity, documenting this as which phase of the contraction?

acme The acme is the peak intensity of a contraction. The increment refers to the building up of the contraction. The decrement refers to the letting down of the contraction. Diastole refers to the relaxation phase of a contraction.

While examining a 2-day old newborn, a nurse notices that the skin and sclera of the eyes appear yellow. The nurse recognizes this condition as which of the following? a) Jaundice b) Pallor c) Harlequin sign d) Cyanosis

Jaundice Correct Explanation: Hyperbilirubinemia is caused by the accumulation of excess bilirubin in blood serum. In the average newborn, the skin and sclera of the eyes begin to appear noticeably yellow on the second or third day of life as a result of a breakdown of fetal red blood cells (called physiologic jaundice), happening because, as the high red blood cell count built up in utero is being reduced, heme and globin are released. Cyanosis is a condition of decreased oxygenation that results in the skin having a blue hue. Pallor, or a pale appearance to skin, occurs as a result of anemia, or lack of red blood cells due to low iron stores, blood loss, poor circulation, or internal bleeding. The harlequin sign is when one side of the body appears red and the other pale, due to immature blood circulation.

At which time during a woman's labor might the nurse assist with a pudendal block? a) Before dilation only b) Just before delivery c) Just after delivery d) Early stage labor

Just before delivery Correct Explanation: Pudendal block is a local block in the perineal area and is used to numb for delivery. Application before labor begins or while it is in the early stages would be counterproductive, as the patient would not have proper feeling and would have a harder time pushing. After delivery it is pointless; the most painful part is over.

A client delivers a newborn in a local health care facility. What guidance should the nurse give to the client before discharge regarding thermoregulation of the newborn at home? a) Keep the newborn wrapped in a blanket, with a cap on its head b) Encourage the mother to keep the infant in her bed to ensure that the infant stays warm c) Ensure cool air is circulating over the newborn to prevent overheating d) Keep the infant's room temperature at least 80 degrees

Keep the newborn wrapped in a blanket, with a cap on its head

Infants have a substance in their lungs, surfactant. What is role of surfactant in the respiratory system? a) Allows oxygen to move in the lungs b) Keeps alveoli from collapsing with breaths c) Removes fluid from the lungs d) Expands the lungs with breaths

Keeps alveoli from collapsing with breaths

Infants have a substance in their lungs, surfactant. What is role of surfactant in the respiratory system? a) Removes fluid from the lungs b) Keeps alveoli from collapsing with breaths c) Allows oxygen to move in the lungs d) Expands the lungs with breaths

Keeps alveoli from collapsing with breaths Correct Explanation: The role of surfactant is to act on surface tension and assist to keep the alveoli open in the lungs so the lungs do not collapse with the respiratory effort of the newborn. Surfactant does not expand the lungs, remove fluid from the lungs, nor allow oxygen to move in the lungs.

Early in labor, a pregnant client asks why contractions hurt so much. Which of the following should the nurse mention? a) Distraction of the brain cortex by other stimuli b) Release of endorphins in response to contractions c) Lack of oxygen to the muscle fibers of the uterus due to compression of blood vessels d) Blocking of nerve transmission via mechanical irritation of nerve fibers

Lack of oxygen to the muscle fibers of the uterus due to compression of blood vessels Correct Explanation: During contractions, blood vessels constrict, reducing the blood supply to uterine and cervical cells, resulting in anoxia to muscle fibers. This anoxia can cause pain in the same way blockage of the cardiac arteries causes the pain of a heart attack. Endorphins are naturally occurring opiate-like substances that reduce pain, not cause it. Distraction and mechanical irritation of nerve fibers are also methods of reducing pain, not causes of pain.

The nurse measures a newborn's temperature immediately after birth and finds it to be 99°F (37.2°C). An hour later, it has dropped several degrees. The nurse understands that this heat loss can be explained in part by which of the following in the newborn? a) Continual kicking b) Lack of subcutaneous fat c) Continual crying d) Constriction of blood vessels

Lack of subcutaneous fat

The nurse measures a newborn's temperature immediately after birth and finds it to be 99°F. An hour later, it has dropped several degrees. The nurse understands that this heat loss can be explained in part by which of the following in the newborn? a) Continual crying b) Continual kicking c) Constriction of blood vessels d) Lack of subcutaneous fat

Lack of subcutaneous fat Correct Explanation: Insulation, an efficient means of conserving heat in adults, is not as effective in newborns because they have little subcutaneous fat to provide insulation. Newborns can conserve heat by constricting blood vessels and moving blood away from the skin. Brown fat, a special tissue found in mature newborns, apparently helps to conserve or produce body heat by increasing metabolism as well as regulating body temperature similar to that of a hibernating animal. Other ways newborns are able to increase their metabolic rate and produce more heat include kicking and crying.

When auscultating the newborn's heart, the nurse would place the stethoscope at which area to auscultate the point of maximal impulse? a) Lateral to the midclavicular line at the fourth intercostal space b) At the fifth intercostal space at the right midclavicular line c) At the third intercostal space adjacent to the midclavicular line d) At the midsternum, just below the suprasternal notch

Lateral to the midclavicular line at the fourth intercostal space Correct Explanation: The point of maximal impulse in a newborn is lateral to the midclavicular line at the fourth intercostal space. A displaced PMI may indicate a tension pneumothorax or cardiomegaly.

The student nurse is preparing to assess the fetal heart rate (FHR). She has determined that the fetal back is located toward the client's left side, the small parts toward the right side, and there is a vertex (occiput) presentation. The nurse should initially begin auscultation of the fetal heart rate in the mother's: a) Right upper quadrant b) Left lower quadrant c) Left upper quadrant d) Right lower quadrant

Left lower quadrant Correct Explanation: The best position to auscultate fetal heart tones in on the fetus back. In this position, the best place for the FHR monitor is on the left lower quadrant.

Discharge teaching is an important part of the labor and delivery room nurse's position. New parents need to know the basics of baby care, like how to monitor fluid volume and when to call the physician. What are the parameters for calling the physician in regards to an infants' temperature? a) less than 96.7 °F or greater than 99.5 °F. b) less than 96 °F or greater than 101 °F c) less than 97 °F or greater than 100.5 °F. d) Less than 97.7 °F or greater than 100 °F.

Less than 97.7 °F or greater than 100 °F. Correct Explanation: Temperatures of less than 97.7 °F or greater than 100 °F should be reported to the physician.

A 2-month-old infant is admitted to a local health care facility with an axillary temperature of 96.8°F (36°C). Which observed manifestation would confirm the occurrence of cold stress in this client? a) Increase in the body temperature b) Lethargy and hypotonia c) Hyperglycemia d) Increased appetite

Lethargy and hypotonia

What should the nurse consider when checking results of blood work done on a newborn? a) Site of the blood sample does not make a difference. b) The newborn's platelet count is higher than an adult's. c) The newborn's aggregation ability is lower than an adult's. d) Leukocytosis is usually present.

Leukocytosis is usually present.

A nurse places an external fetal monitor on a woman in labor. Which instruction would be best to give her?

Lie on her side so she is comfortable. The best position for all women during labor is on their side.

You place an external fetal monitor on a woman in labor. Which of the following instructions would be best to give her? a) Avoid flexing her knees so her abdomen is not tense. b) Lie supine so the tracing does not show a shadow. c) Avoid using her call bell to reduce interference. d) Lie on her side so she is comfortable.

Lie on her side so she is comfortable. Correct Explanation: The best position for all women during labor is on their side.

The nurse is preparing a teaching plan for new parents about why newborns experience heat loss. Which of the following would the nurse include? a) Expanded stores of glucose and glycogen b) Enhanced shivering ability c) Thick skin with deep lying blood vessels d) Limited voluntary muscle activity

Limited voluntary muscle activity

What is caput? What causes it? What does it look like? Is there a problem?

Localized edema on the scalp that occurs from pressure of the birth process. More commonly seen with a prolonged labor. Is a poorly demarcated soft tissue swelling that "crosses the suture line" (mnemonic "put a cap on it, this is not a treatment just a way to remember, as a cap would cross the suture line). No problem, will dissipate/resolve/go back to normal in about 3 days

What is cephalhematoma? What does it look like? Is there a problem?

Localized effusion of blood (hence hematoma in the word) beneath the periosteum of the skull. This condition is due to disruption of the vessels during birth. It occurs after prolonged labor and use of obstetric interventions (vaccum extraction or forceps delivery). Well-demarcated, often flcutant welling with no overlying skin discoloration. The swelling does not cross the suture lines and is firm to touch than an edematous area. Aspiration of the fluid is not required as to pull off with an needle aspiration puts at risk for infection so the risk of aspiration outweighs the benefit of evacuation of the blood Usually resolves 2-3 days after delivery Concern to watch for: hyperbilirubinemia occurs following the breakdown of the RBCs within the hematoma. In other words when the RBCs break down and are absorbed into the body, there is a chance for hyperbiirubinemia.

You assess a postpartum woman's perineum and notice that her lochial discharge is moderate in amount and red. You would record this as what type of lochia? a) Lochia serosa b) Lochia alba c) Lochia normalia d) Lochia rubra

Lochia rubra Explanation: Lochia rubra is red; it lasts for the first few days of the postpartal period.

A nurse is instructing a woman that it is important to lose pregnancy weight gain within 6 months of delivery, because studies show that keeping extra weight longer is a predictor of which of the following? a) diabetes b) long-term obesity c) feelings of increased self-esteem d) increased sex drive

Long-term obesity Correct Explanation: Women who have not returned to their prepregnant weight by 6 months postpartum are likely to retain extra weight. This inability to lose is a predictor of long-term obesity. It will not necessarily lead to diabetes, but it may decrease a woman's self-esteem and sex drive if she feels less attractive with the extra weight.

What should the nurse expect for a full-term newborn's weight during the first few days of life? a) Loss of 5% to 10% of the birth weight in the first few days in breastfed infants only b) Loss of 5% to 10% of birth weight in formula-fed and breastfed newborns c) A formula-fed newborn should gain 3% to 5% of the initial birth weight in the first 48 hours, but a breastfed newborn may lose up to 3%. d) An increase in 3% to 5% of birth weight by day 3 in formula-fed babies

Loss of 5% to 10% of birth weight in formula-fed and breastfed newborns

What should the nurse expect for a full-term newborn's weight during the first few days of life? a) Loss of 5% to 10% of the birth weight in the first few days in breastfed infants only b) A formula-fed newborn should gain 3% to 5% of the initial birth weight in the first 48 hours, but a breastfed newborn may lose up to 3%. c) Loss of 5% to 10% of birth weight in formula-fed and breastfed newborns d) An increase in 3% to 5% of birth weight by day 3 in formula-fed babies

Loss of 5% to 10% of birth weight in formula-fed and breastfed newborns Correct Explanation: The nurse should expect the newborn who is breastfed or formula-fed to lose 5% to 10% of birth weight in the first few days of life.

What are subtle signs of DVT?

Lower ext tightness/aching relieved by rest Edema in affected leg Warmth, tenderness and/or redness of calf Low grade fever

Two days ago, a woman delivered her third infant; she is now preparing for discharge home. After the delivery of her second child, she developed an endometrial infection. Nursing goals for this discharge include all of the following EXCEPT a) List signs of infection that she will report to her health care provider b) Maintain previous household routines to prevent infection c) The patient will show no signs of infection d) Discuss methods that the woman will use to prevent infection

Maintain previous household routines to prevent infection Correct Explanation: The nurse does not know whether previous routines were or were not the source of the infection. The other three options provide correct instructions to be given to this woman.

The nurse knows that the second stage of labor, the descent phase, has begun when

Many women may have an urge to bear down when the presenting part is below the level of the ischial spines

The following are nursing measures commonly offered to women in labor. Which nursing intervention would probably be most effective in applying the gate control theory for relief of labor pain? a) Give the prescribed medication b) Encourage the woman to rest between contractions c) Change the woman's position d) Massage for the woman's back

Massage for the woman's back Correct Explanation: Gate-control is based on the idea of distraction or redirection of the conduction of impulses up the neural pathways. Massage redirects the paths of sensation away from the pain to the other area. Encouragement is a form of psychological support. Position change will only distract the patient. Medication should be withheld until all non-pharmacological treatments have been exhausted.

A nurse assessing a postpartum patient notices excessive bleeding. What should be the nurse's first action? a) Massage the boggy fundus until it is firm. b) Nothing--excessive postpartum blood loss is normal. c) Document the findings. d) Call the physician.

Massage the boggy fundus until it is firm. Correct Explanation: The nurse needs to report any abnormal findings when assessing the lochia. If excessive bleeding occurs, the first step would be to massage the boggy fundus until it is firm to reduce the flow of blood. Then the nurse needs to document the findings.

Upon assessment, a nurse notes the client has a pulse of 90 bpm, moderate lochia, and a boggy uterus. What should the nurse do next?

Massage the client's fundus. Tachycardia and a boggy fundus in the postpartum woman indicate excessive blood loss. The nurse would massage the fundus to promote uterine involution. It is not priority to notify the healthcare provider, assess blood pressure, or change the peri-pad at this time.

Which statement is the best rationale for assessing maternal vital signs between contractions?

Maternal circulating blood volume increases temporarily during contractions.

A nurse is caring for a client who has just received an epidural. Which of the following is the MOST common side effect of epidural anesthesia? a) Maternal hypotension, which can lead to fetal tachycardia b) Maternal hypertension, which can lead to fetal bradycardia c) Maternal hypertension, which can lead to fetal tachycardia d) Maternal hypotension, which can lead to fetal bradycardia

Maternal hypotension, which can lead to fetal bradycardia Correct Explanation: Epidural anesthesia conveys the risk of hypotension, especially if the client has not received an adequate amount of fluid before the procedure is performed. A sudden drop in maternal blood pressure can cause uterine hypoperfusion, which may result in fetal bradycardia.

The amniotic fluid is green when the membranes rupture. What finding would the nurse document? a) Umbilical cord prolapse b) Meconium in the amniotic sac c) Amniotic fluid embolism d) Infection

Meconium in the amniotic sac Correct Explanation: Green tinted fluid with ROM is indicative of meconium in the amniotic sac, or the infant having a bowel movement in utero. Infection would be shown by puss or cloudy fluid. Umbilical cord prolapse occurs when pressure on the cord stops the flow of oxygen to the fetus. Amniotic embolism results when amniotic fluid enters circulation.

The nurse observes tiny white pinpoint papules on a newborn's nose. The nurse documents this finding as: a) Lanugo b) Milia c) Harlequin sign d) Vernix caseosa

Milia Correct Explanation: Milia are the tiny white pinpoint papules of unopened sebaceous glands frequently found on the newborn's nose. Lanugo is the fine downy hair that covers the newborn's shoulders, back, and upper arms. Vernix caseosa is the thick white substance that provides a protective covering of the skin of the fetus. The harlequin sign refers to a transient phenomenon in which a newborn appears red on the dependent side of the body and pale on the upper side when lying on his or her side

Inspection of a woman's perineal pad reveals a 5-inch stain. The nurse documents this amount as which of the following? a) Moderate b) Heavy c) Scant d) Light

Moderate Correct Explanation: Moderate lochia would describe a 4- to 6-inch stain, scant lochia a 1- to 2-inch stain, and light or small an approximately 4-inch stain. Heavy or large lochia would describe a pad that is saturated within 1 hour.

As the newborn descends through the vaginal canal there is a variation in head size and shape that can occur. Can you name these 3 variations?

Molding, caput succedaneum, cephlahematoma

The slight overlapping of cranial bones or shaping of the fetal head during labor is called:

Molding.

Shortly after the birth of a newborn, the mother notices a gray patch across the baby's buttocks. She is immediately concerned that the baby has been bruised during the birth and asks the nurse about this. The nurse recognizes patch as a birth mark and explains this to the mother. Which type of birth mark is this most likely to be? a) Cavernous hemangioma b) Mongolian spot c) Strawberry hemangioma d) Nevus flammeus

Mongolian spot

Shortly after the birth of a newborn, the mother notices a gray patch across the baby's buttocks. She is immediately concerned that the baby has been bruised during the birth and asks the nurse about this. The nurse recognizes patch as a birth mark and explains this to the mother. Which type of birth mark is this most likely to be? a) Cavernous hemangioma b) Nevus flammeus c) Strawberry hemangioma d) Mongolian spot

Mongolian spot Correct Explanation: Mongolian spots are collections of pigment cells (melanocytes) that appear as slate-gray patches across the sacrum or buttocks and possibly on the arms and legs of newborns. They tend to occur most often in children of Asian, Southern European, or African ethnicity and disappear by school age without treatment. Be sure to inform parents that although these marks look like bruises, they are not. Otherwise, they may worry their baby sustained a birth injury from improper handling. A nevus flammeus is a macular purple or dark-red lesion (sometimes called a port-wine stain because its color is the same as that of red wine). These lesions are present at birth and typically appear either on the face or a thigh. Strawberry hemangiomas are elevated areas formed by a combination of immature capillaries and endothelial cells. Cavernous hemangiomas are raised and irregular in shape ans so resemble a strawberry hemangioma in appearance but do not disappear with time.

An African American baby has discoloring which appears similar to bruising on his buttock after a normal vaginal delivery. This assessment should be documented as: a) Mongolian spots. b) Vascular nevi. c) Lanugo. d) Bruising.

Mongolian spots

A nurse is performing a detailed assessment of a female newborn. Which observations indicate normal findings? Select all that apply.

Mongolian spots short, creased neck swollen genitals Mongolian spots, swollen genitals in the female newborn, and a short, creased neck are normal findings in a newborn. Mongolian spots are blue or purple splotches that appear on the lower back and buttocks of newborns. Female babies may have swollen genitals as a result of maternal estrogen. The newborn's neck will appear almost nonexistent because it is so short. Creases are usually noted. Enlarged fontanelles are associated with hydrocephaly; congenital hypothyroidism; trisomies 13, 18, and 21; and various bone disorders such as osteogenesis imperfecta. Low-set ears are characteristic of many syndromes and genetic abnormalities such as trisomies 13 and 18 and internal organ abnormalities involving the renal system.

An African American baby has discoloring which appears similar to bruising on his buttock after a normal vaginal delivery. This assessment should be documented as: a) Lanugo. b) Bruising. c) Vascular nevi. d) Mongolian spots.

Mongolian spots. Correct Explanation: This is a Mongolian spot and occurs in ethnicities with darker colored skin. This is a normal finding, but does require documentation. Lanugo is the fine hair on the baby's body when it is born. Mongolian spots are not vascular nevi and they are not bruising.

When assessing infant reflexes the nurse documents a startled response and extension of the arms and legs as which reflex? a) Fencing b) Moro c) Rooting d) Tonic neck

Moro

When assessing infant reflexes, the nurse documents a startled response and extension of the arms and legs as which reflex?

Moro The Moro reflex is also known as the startle reflex. When the infant is startled they extend their arms and legs away from the body. The fencing reflex is also called the tonic neck reflex and is a total body assessment. The rooting reflex assesses the infant's ability to "look" for food.

When assessing infant reflexes the nurse documents a startled response and extension of the arms and legs as which reflex? a) Rooting b) Moro c) Tonic neck d) Fencing

Moro Correct Explanation: The moro reflex is also known as the startle reflex. When the infant is startled they extend their arms and legs away from the body. The fencing reflex is also called the tonic neck reflex and is a total body assessment. The rooting reflex assesses the infant's ability to "look" for food.

A nurse is aware that the newborn's neuromuscular maturity is typically completed within 24 hours after birth. Which assessment would the nurse be least likely to complete to determine the newborn's degree of maturity?

Moro reflex There are six activities or maneuvers that are evaluated to determine the newborn's degree of maturity: posture, square window, arm recoil, popliteal angle, scarf sign, heel-to-ear. The Moro reflex is an indication of the newborn's neurologic status.

Edinburgh postnatal depression scale?

Mother fills it out Done @ home If borderline high score, repeat during 2nd wk

A nurse is assessing a newborn during the first 24 hours after birth. Which findings would the nurse recognize as normal findings a) Enlarged labia with pseudomenstruation b) Rounded, symmetrical abdomen c) Mottled hands and feet d) Heart rate of 90-100 e) Positive Ortolani sign

Mottled hands and feet

Which medication is administered to reverse the depressant effects of opioids? a) Nalbuphine (Nubain) b) Butorphanol (Stadol) c) Naloxone (Narcan) d) Meperidine (Demerol)

Naloxone (Narcan) Correct Explanation: Naloxone (Narcan) is an opioid antagonist. Butorphanol (Stadol), nalbuphine (Nubain), and meperidine (Demerol) are opioids.

The nurse orienting a student to the nursery determines that teaching has been effective when the student states that the signs of neonate respiratory distress include which findings? Select all that apply.

Nasal flaring Respiratory rate of 64 breaths per minute Chest retractions Signs of respiratory distress in the newborn include tachypnea (respirations greater than 60 breaths/min, tachycardia (heart rate greater than 160/beats/min, nasal flaring, chest retractions, and generalized cyanosis. Blue hands and feet , referred to as acrocyanosis, is caused by poor peripheral circulation not respiratory distress.

General anesthesia is not used frequently in obstetrics because of the risks involved. There are physiologic changes that occur during pregnancy that make the risks of general anesthesia higher than it is in the general population. What is one of those risks? a) Fetal hypersensitivity to anesthetic b) Neonatal depression c) Woman is more sensitive to pre-anesthetic medications d) Woman is less sensitive to inhalation anesthetics

Neonatal depression Correct Explanation: General anesthesia is not used frequently in obstetrics because of the risks involved. The pregnant woman is at higher risk for aspiration. It requires more skill to intubate a pregnant woman because of physiologic changes in the trachea and thorax. In addition, general anesthetic agents cross the placenta and can result in the birth of a severely depressed neonate who requires full resuscitation.

General anesthesia is not used frequently in obstetrics because of the risks involved. There are physiologic changes that occur during pregnancy that make the risks of general anesthesia higher than it is in the general population. What is one of those risks?

Neonatal depression is possible.

What are stork bites?

New on the KFC & McDonald's menu, just kidding. (just wanted you to have some laughter for a moment) salmon patches that are superficial vascular areas found on the nap of the neck, eyelids, and between the eyes and upper lip. Myth of stork delivered the baby. They are caused by a concentration of immature blood vessels, shows as a red area on skin.

To prevent misidentification of a newborn identification bands are placed on the newborn and on the parents before the newborn is separated from the parents. What information is on all the bands? a) Mother's name and date and time of her birth b) Father's name and date and time of birth c) Hospital number, attending physician, and father's name d) Newborn's sex and date and time of birth

Newborn's sex and date and time of birth

To prevent misidentification of a newborn identification bands are placed on the newborn and on the parents before the newborn is separated from the parents. What information is on all the bands? a) Father's name and date and time of birth b) Hospital number, attending physician, and father's name c) Newborn's sex and date and time of birth d) Mother's name and date and time of her birth

Newborn's sex and date and time of birth Explanation: Information included on the bands is the mother's name, hospital number, and physician, newborn's sex, and date and time of birth.

A nursing student is preparing a class for new mothers about adaptations they can expect in their newborns. Which of the following about newborn vision whould the student accurately include in the presentation? a) Newborns have the ability to focus only on objects in close proximity. b) Newborns have the ability to focus on objects in midline. c) Newborns cannot focus on any objects. d) Newborns have the ability to focus only on objects far away.

Newborns have the ability to focus only on objects in close proximity.

A nursing student is preparing a class for new mothers about adaptations they can expect in their newborns. Which of the following about newborn vision whould the student accurately include in the presentation? a) Newborns cannot focus on any objects. b) Newborns have the ability to focus only on objects in close proximity. c) Newborns have the ability to focus only on objects far away. d) Newborns have the ability to focus on objects in midline.

Newborns have the ability to focus only on objects in close proximity. Correct Explanation: In regards to vision the newborn has the ability to focus on objects only in close proximity (7 to 12 inches away) and tracks objects in midline or beyond. Vision is the least mature sense at birth.

A client is worried that her newborn's stools are greenish, with an unpleasant odor. The newborn is being formula-fed. What instruction should the nurse give this client? a) No action is need; this is normal b) Switch to feeding breast milk c) Change to a soy-based formula d) Increase the newborn's fluid intake

No action is need; this is normal

The nurse is observing a client who gave birth yesterday. Where should the nurse expect to find the top of the client's fundus? a) One fingerbreadth above the umbilicus b) One fingerbreadth below the umbilicus c) At the level of the umbilicus d) Below the symphysis pubis

One fingerbreadth below the umbilicus Explanation: After a client gives birth, the height of her fundus should decrease by approximately one fingerbreadth (1 cm) each day. By the end of the first postpartum day, the fundus should be one fingerbreadth below the umbilicus. Immediately after birth, the fundus may be above the umbilicus; 6 to 12 hours after birth, it should be at the level of the umbilicus; 10 days after birth, it should be below the symphysis pubis.

-Fetal Heart Rate Pattern Acid-Base Status -Category 1

Normal Baseline 110-160 bpm &variability moderate, present/absent of accelerations/decelerations,no late/variable desceleration

How should a baby sleep?

Nothing in crib, sleeping on back

When caring for a newborn several hours after birth, the nurse assesses his respiratory rate and counts it at 42. What intervention should be implemented? a) Contact the pediatrician to report the newborn's tachypnea. b) Stimulate the newborn to cry. c) Nothing since this is a normal reading. d) Further assess the newborn for abnormal lung sounds.

Nothing since this is a normal reading.

What is the nurse responsible for in the management of labor and birth?

Nursing management for labor and birth involves involves assessment, comfort measures, emotional support, information and instruction, advocacy, and support for the partner. Nurses need to be respectful, available, encouraging, supportive, and professional in dealing with all women. Nurse should recognize it is a life changing experience for women and their families, giving care that that protects, promotes, and supports physiologic childbirth.

A nurse is observing respiratory effort in a newborn as part of Apgar scoring. Which method should the nurse use to do this?

Observe chest movement. Respirations are counted by observing chest movement. Reflex irritability may be evaluated by observing response to a suction catheter in the nostrils or response to having the soles of the feet slapped. Heart rate is typically determined by auscultation with a stethoscope but may also be obtained by observing and counting the pulsations of the umbilical cord at the abdomen, if the cord is still uncut. Muscle tone is evaluated by observing resistance to any effort to extend the newborn's extremities.

A nurse is observing respiratory effort in a newborn as part of Apgar scoring. Which of the following methods should he use to do this? a) Observing chest movement b) Observing response to a suction catheter in the nostrils c) Observing and counting the pulsations of the umbilical cord d) Observing resistance to any effort to extend the newborn's extremities

Observing chest movement

A nurse is observing respiratory effort in a newborn as part of Apgar scoring. Which of the following methods should he use to do this? a) Observing resistance to any effort to extend the newborn's extremities b) Observing and counting the pulsations of the umbilical cord c) Observing chest movement d) Observing response to a suction catheter in the nostrils

Observing chest movement Correct Explanation: Respirations are counted by observing chest movement. Reflex irritability may be evaluated by observing response to a suction catheter in the nostrils or response to having the soles of the feet slapped. Heart rate is typically determined by auscultation with a stethoscope but may also be obtained by observing and counting the pulsations of the umbilical cord at the abdomen, if the cord is still uncut. Muscle tone is evaluated by observing resistance to any effort to extend the newborn's extremities.

A client who has a breastfeeding newborn reports sore nipples. Which intervention can the nurse suggest to alleviate the client's condition?

Offer suggestions based on observation to correct positioning or latching. The nurse should observe positioning and latching-on technique while breastfeeding so that she may offer suggestions based on observation to correct positioning/latching. This will help minimize trauma to the breast. The client should use only water, not soap, to clean the nipples to prevent dryness. Breast pads with plastic liners should be avoided. Leaving the nursing bra flaps down after feeding allows nipples to air dry.

A nurse needs to monitor the blood glucose levels of a newborn under observation at a health care facility. When should the nurse check the newborn's initial glucose level? a) 24 hours after admission to the nursery b) On admission to the nursery c) 4 hours after admission to the nursery d) After the newborn has received the initial feeding

On admission to the nursery

The infant's temperature is 97.2°F (36.2°C) axillary an hour after birth. Which intervention is the appropriate for the nurse? a) Take the infant to the mother for bonding. b) Place a second stockinette on the baby's head c) Place the infant under a radiant warmer or in a heated isolette. d) Administer a warm bath with temperature slightly higher than usual

Place the infant under a radiant warmer or in a heated isolette.

A nurse is caring for a client who is nursing her baby boy. The client complains of afterpains. Secretion of which of the following should the nurse identify as the cause of afterpains? a) Oxytocin b) Estrogen c) Prolactin d) Progesterone

Oxytocin Explanation: Secretion of oxytocin stimulates uterine contraction and causes the woman to experience afterpains. Decrease in progesterone and estrogen after placental delivery stimulates the anterior pituitary to secrete prolactin which causes lactation

A father is asking questions about the circumcision of his son. He is asking the nurse if there are any disadvantages to the procedure. How should the nurse respond? a) Reduced risk of penile cancer b) Pain administration may not be effective during the procedure c) Lower rate of urinary tract infections d) Fewer complications than if done later in life

Pain administration may not be effective during the procedure

A father is asking questions about the circumcision of his son. He is asking the nurse if there are any disadvantages to the procedure. How should the nurse respond? a) Lower rate of urinary tract infections b) Fewer complications than if done later in life c) Reduced risk of penile cancer d) Pain administration may not be effective during the procedure

Pain administration may not be effective during the procedure Correct Explanation: The anesthetic block is not always effective. Not all providers use anesthetics prior to the procedure and the infant can feel the pain of the circumcision. A lower rate of urinary tract infections, a reduced risk of penile cancer, and fewer complications than if circumcised later in life are not disadvantages to the procedure; they are advantages.

You are the nurse preparing an educational event for pregnant women on the topic of labor pain and delivery. You know that you will need to include the origin of labor pain for each stage of labor. What information will you present for the first stage of labor? a) Diffuse abdominal pain signals a complication with progression of labor b) It is reported as the worst pain you will ever feel c) Pain is focal in nature d) Pain originates from the cervix and lower uterine segment

Pain originates from the cervix and lower uterine segment Correct Explanation: Pain sensations associated with labor originate from different places depending on the stage of labor. During the first stage of labor, the stretching required to efface and dilate the cervix stimulates pain receptors in the cervix and lower uterine segment.

The nurse is preparing an educational event for pregnant women on the topic of labor pain and birth. The nurse understands the need to include the origin of labor pain for each stage of labor. What information will the nurse present for the first stage of labor?

Pain originates from the cervix and lower uterine segment.

What should be done to the clavicle?

Palpate

Joanne has been in labor for 5 hours. Earlier there was a gradual increase in FHR baseline with variables, but Joanne has changed position several times and now the fetus shows no signs of hypoxia. Joanne's cervix is almost completely effaced and is dilated to 8 cm. However, the labor graph indicates that the fetus has stopped descending. What should you do first? a) Alert the team that internal fetal monitoring may be needed b) Institute effleurage and apply pressure to Joanne's lower back during contractions c) Encourage Joanne to push d) Palpate the area just above the symphysis pubis

Palpate the area just above the symphysis pubis Correct Explanation: Palpate to determine if the infant is engaged and what the presenting part of the infant is by the symphysis pubis, it is possible for infants to rotate and change position during labor. Assess the situation and act further if necessary, but until you have more information on fetal position, assume all is going well.

Postpartum bleeding must be assessed carefully during the first 24 hours after delivery. Prioritize the actions taken upon detection of increased vaginal bleeding in a patient who delivered within the last 24 hours. Palpate the fundus Massage the fundus if boggy Notify the physician or the nurse-midwife of excessive bleeding Increase IV pitocin or breastfeed the newborn Assess blood pressure Assist the patient to empty her bladder in the bathroom

Palpate the fundus Massage the fundus if boggy Notify the physician or the nurse-midwife of excessive bleeding Increase IV pitocin or breastfeed the newborn Assess blood pressure Assist the patient to empty her bladder in the bathroom Explanation: Determining the site of bleeding is the first assessment. Palpate the fundus. If the fundus is boggy, take steps to stimulate contractions by massaging. Notify the health care provider, and continue with the assessment. Stimulate contractions. Assess blood pressure and assess for safety to ambulate. A likely reason for a boggy uterus is dislocation by a full bladder.

Factors affecting attachment

Parents background, infant, care practices

What are the 5 P's affecting labor process

Passenger (fetus and placenta), Passageway (birth canal), Powers (contractions), Position of the mother, and Psychologic response.

Which finding would indicate to the nurse that a postpartum woman is experiencing bladder distention? a) Percussion reveals dullness b) Uterus is firm c) Bladder is nonpalpable d) Lochia is less than usual

Percussion reveals dullness Correct Explanation: A distended bladder is dull on percussion and can be palpated as a rounded mass. In addition, the uterus would be boggy and lochia would be more than usual.

A postpartum client reports stress incontinence. What information should the nurse suggest to the client to overcome stress incontinence?

Perform Kegel exercises. The nurse should ask the client to perform the Kegel exercises in which the client needs to alternately contract and relax the perineal muscles. Aerobic exercises will not help to strengthen perineal muscles. Reduced fluid intake and frequent emptying of the bladder will not help the client overcome stress incontinence.

A postpartum client complains of stress incontinence. What information should the nurse suggest to the client to overcome stress incontinence? a) Reduce fluid intake b) Frequently empty the bladder c) Perform Kegel's exercises d) Perform aerobic exercises

Perform Kegel's exercises Explanation: The nurse should ask the client to perform the Kegel's exercises in which the client needs to alternately contract and relax the perineal muscles. Aerobic exercises will not help to strengthen perineal muscles. Reduced fluid intake and frequent emptying of the bladder will not help the client overcome stress incontinence.

A client who gave birth about 12 hours ago informs the nurse that she has been voiding small amounts of urine frequently. The nurse examines the client and notes the displacement of the uterus from the midline to the right. What intervention would the nurse perform next?

Perform urinary catheterization. Displacement of the uterus from the midline to the right and frequent voiding of small amounts suggests urinary retention with overflow. Catheterization may be necessary to empty the bladder to restore tone. An IV and oxytocin are indicated if the client experiences hemorrhage due to uterine atony from being displaced. The healthcare provider would be notified if no other interventions help the client.

You are assisting with the circumcision of a 16-hour-old male infant. Immediately after the procedure, what kind of dressing would you apply to the surgical area? a) Petrolatum gauze dressing b) Small pressure dressing c) Sterile 2×2s and paper tape d) Steri strips

Petrolatum gauze dressing

You are assisting with the circumcision of a 16-hour-old male infant. Immediately after the procedure, what kind of dressing would you apply to the surgical area? a) Sterile 2×2s and paper tape b) Petrolatum gauze dressing c) Steri strips d) Small pressure dressing

Petrolatum gauze dressing Correct Explanation: Immediately after the procedure, place a petrolatum gauze dressing, as ordered by the physician.

What should be used every diaper change to a newborn with a circumcision ?

Petroleum jelly to the head of the penis

What kind of therapy should be used for hyperbilirubinemia?

Phototherapy..

The New Ballard scoring system evaluates newborns on which 2 factors? a) Body maturity and cranial nerve maturity b) Physical maturity and neuromuscular maturity c) Skin maturity and reflex maturity d) Tone maturity and extremities maturity

Physical maturity and neuromuscular maturity

The New Ballard scoring system evaluates newborns on which 2 factors? a) Physical maturity and neuromuscular maturity b) Body maturity and cranial nerve maturity c) Skin maturity and reflex maturity d) Tone maturity and extremities maturity

Physical maturity and neuromuscular maturity Correct Explanation: When determining a newborn's gestational age using the New Ballard scale, physical signs and neurologic characteristics are assessed.

What does the ballard score assess?

Physical signs and neurologic characteristics to determine newborn's gestational age

When a newborn is experiencing physiologic depression, the Apgar characteristics will disappear in a predictable manner. In which order, from first to last, will the nurse expect these characteristics disappear? All options must be used.

Pink coloration is lost. Respiratory effort decreases. Muscle tone decreases. Reflex irritability is noted. Heart rate decreases. The Apgar score is a method of evaluating a newborn's physical condition at 1 and 5 minutes after birth. Assessment is an indication of the newborn's overall central nervous system status. When the newborn experiences physiologic depression, the characteristics disappear in a predictable manner: first the pink coloration is lost, next the respiratory effort, then the tone, followed by reflex irritability, and finally the heart rate.

During a postpartum exam on the day of delivery, the woman complains that she is still so sore that she can't sit comfortably. You examine her perineum and find the edges of the episiotomy approximated without signs of a hematoma. Which intervention will be most beneficial at this point? a) Notify the RN b) Place an ice pack c) Put on a witch hazel pad. d) Apply a warm washcloth

Place an ice pack Explanation: The labia and perineum may be edematous after delivery and bruised, the use of ice would assist in decreasing the pain and swelling. Applying a warm washcloth would bring more blood as well as fluid to the sore area, thereby increasing the edema and the soreness. Applying a witch hazel pad needs the order of the physician. Notifying the RN is not necessary at this time as this is considered a normal finding.

During a postpartum exam on the day of birth, the woman reports that she is still so sore that she cannot sit comfortably. The nurse examines her perineum and find the edges of the episiotomy approximated without signs of a hematoma. Which intervention will be most beneficial at this point?

Place an ice pack. The labia and perineum may be edematous after birth and bruised; the use of ice would assist in decreasing the pain and swelling. Applying a warm washcloth would bring more blood as well as fluid to the sore area, thereby increasing the edema and the soreness. Applying a witch hazel pad needs the order of the primary care provider. Notifying a care provider is not necessary at this time as this is considered a normal finding.

A nurse is required to obtain the temperature of a healthy newborn who was placed in an open crib. Which is the most appropriate method for measuring a newborn's temperature? a) Obtain the temperature rectally b) Place electronic temperature probe in the midaxillary area c) Obtain the temperature orally d) Tape electronic thermistor probe to the abdominal skin

Place electronic temperature probe in the midaxillary area

A nurse is required to obtain the temperature of a healthy newborn who was placed in an open crib. Which is the most appropriate method for measuring a newborn's temperature?

Place electronic temperature probe in the midaxillary area. The nurse should obtain a newborn's temperature by placing an electronic temperature probe in the midaxillary area. The nurse should not tape an electronic thermistor probe to the abdominal skin, as this method is applied only when the newborn is placed under a radiant heat source. Rectal temperatures are no longer taken because of the risk of perforation. Oral temperature readings are not taken for newborns

A new mother is nervous about sudden infant death syndrome (SIDS) and asks the nurse how to prevent it when the newborn is ready to sleep. Beside placing the infant on a firm sleep surface, what should the nurse tell the mother to do? Select all that apply

Place the infant on his or her back. Not allow anyone to smoke around the infant The newborn or infant should be placed on her back to sleep on a firm sleep surface. Sleeping on the stomach (prone) may limit the newborn's ability to move her head, which increases the chance of suffocation. Parents should also prevent exposure to tobacco smoke.

The infant's temperature is 97.2° F (36.2° C) axillary an hour after birth. Which intervention is appropriate for the nurse?

Place the infant under a radiant warmer or in a heated isolette. If the infant has a low temperature of 97.2° F (36.2° C), the nurse should place the infant in a radiant warmer or in an isolette. Once the infant has a core temperature of greater than 97.7° F (36.5° C), the nurse will double bundle and recheck the temperature in 30 minutes. If an infant has a temperature that is considered low the nurse would not take the infant to its mother for bonding or administer a warm bath. The nurse would initiate interventions to stabilize the infant's temperature within normal range.

The infant's temperature is 97.2F axillary an hour after birth. Which intervention is the appropriate for the nurse? a) Administer a warm bath with temperature slightly higher than usual b) Place a second stockinette on the baby's head c) Place the infant under a radiant warmer or in a heated isolette. d) Take the infant to the mother for bonding.

Place the infant under a radiant warmer or in a heated isolette. Correct Explanation: If the infant has a low temperature of 97.2F, the nurse should place the infant in a radiant warmer or in an isolette. Once the infant has a core temperature of greater than 97.7F, the nurse will double bundle and re-check the temperature in 30 minutes. If an infant has a temperature that is considered low you would not take the infant to its mother for bonding nor administer a warm bath. You would initiate interventions to stabilize the infant's temperature within normal range

Which of the following factors in a postpartum woman's history would lead the nurse to watch the woman closely for an infection? a) Hemoglobin of 11.5 mg/dL b) Placenta removed via manual extraction c) Labor of 12 hours d) Multiparity

Placenta removed via manual extraction Correct Explanation: Manual removal of the placenta, a labor longer than 24 hours, a hemoglobin less than 10.5 mg/dL, and multiparity, such as more than three births closely spaced together, would place the woman at risk for postpartum hemorrhage.

When palpating for fundal height on a postpartal woman, which technique is preferable? a) Placing one hand on the fundus, one on the perineum b) Resting both hands on the fundus c) Placing one hand at the base of the uterus, one on the fundus d) Palpating the fundus with only fingertip pressure

Placing one hand at the base of the uterus, one on the fundus Correct Explanation: Supporting the base of the uterus before palpation prevents the possibility of uterine inversion with palpation.

Which basic type of pelvis includes the correct description and percentage of occurrence in women?

Platypelloid: flattened, wide, shallow; 3%

Newborn Ming has secretions in his mouth and nose. What are the first steps the nurse should take to clear his airways? a) Position Ming on his side with his head slightly below his body; use a bulb syringe to clear his nose. b) Position Ming on his side and guide his caregivers in suctioning his mouth with a bulb syringe. c) Position Ming on his side with his head slightly below his body; use a bulb syringe to clear his mouth. d) Position Ming on his side with his head slightly below his body; use a small suction catheter to clear his nose.

Position Ming on his side with his head slightly below his body; use a bulb syringe to clear his mouth.

Newborn Ming has secretions in his mouth and nose. What are the first steps the nurse should take to clear his airways? a) Position Ming on his side with his head slightly below his body; use a bulb syringe to clear his mouth. b) Position Ming on his side with his head slightly below his body; use a bulb syringe to clear his nose. c) Position Ming on his side and guide his caregivers in suctioning his mouth with a bulb syringe. d) Position Ming on his side with his head slightly below his body; use a small suction catheter to clear his nose.

Position Ming on his side with his head slightly below his body; use a bulb syringe to clear his mouth. Correct Explanation: The infant needs to have bulb suction used to remove the secretions, the head should be held slightly lower than the body to facilitate use of gravity. Right after birth is not the time for the parents of the newborn to be instructed in how to suction their infant. A bulb syringe, not a small suction catheter, is used to suction the mouth and nose of a newborn. Option D is incorrect as it does not clear the infant's mouth of secretions.

A newborn has secretions in his mouth and nose. What are the first steps the nurse should take to clear his airways?

Position the newborn on his side with his head slightly below his body; use a bulb syringe to clear his mouth. The infant needs to have bulb suction used to remove the secretions, the head should be held slightly lower than the body to facilitate use of gravity. Right after birth is not the time for the parents of the newborn to be instructed in how to suction their infant. A bulb syringe, not a small suction catheter, is used to suction the mouth and nose of a newborn. The remaining option is incorrect as it does not clear the infant's mouth of secretions.

What's the most common comp of preg?

Post partum depression

A nurse is caring for a client in the postpartum period. The client is emotionally sensitive, feels a sense of failure, and attempts to hurt herself and the baby. The nurse understands that the client is exhibiting symptoms of which of the following conditions? a) Anxiety disorders b) Postpartum depression c) Postpartum psychosis d) Postpartum blues

Postpartum depression Explanation: The client is showing signs of postpartum depression. Postpartum blues are due to lack of sleep and emotional labilities. Postpartum psychosis is symbolized by confusion, hallucinations, and delusions. Postpartum anxiety disorders involve shortness of breath, chest pain, and tightness.

A nurse is caring for a client on the second day postpartum. The client informs the nurse that she is voiding a large volume of urine frequently. Which of the following should the nurse identify as a potential cause for urinary frequency? a) Urinary tract infection b) Trauma to pelvic muscles c) Postpartum diuresis d) Urinary overflow

Postpartum diuresis Explanation: The nurse should identify postpartum diuresis as the potential cause for urinary frequency. Urinary overflow occurs if the bladder is not completely emptied. Urinary tract infection may be accompanied by fever and a burning sensation. Trauma to pelvic muscles does not affect urinary frequency.

Nancy has presented in the early phase of labor. She's experiencing abdominal pain and shows signs of growing anxiety about the pain. What is the best pain management technique the nurse can suggest at this stage? a) Administering an opioid such as meperidine (Demerol) or fentanyl (Sublimaze) b) Immersing Nancy in warm water in a pool or hot tub c) Practicing effleurage on the abdomen d) Administering a sedative such as secobarbital (Seconal) or pentobarbital (Nembutal)

Practicing effleurage on the abdomen Correct Explanation: In early labor, the less medication use the better, allow use of nonpharmacologic management and control the pain with gate theory. Sitting in a warm pool of water is relaxing and may lessen the pain, but it does not control the pain. Sedatives are not indicated as they may slow the birthing process. Opioids should be limited as they too may slow the progression of labor.

At birth there are multiple changes in the cardiac and respiratory systems. Which of the following is one of the changes to occur at birth in the cardiovascular system? a) Oxygen is exchanged in the lungs b) Fluid is removed from the alveoli and replaced with air c) Pressure changes occur and result in closure of the ductus arteriosus d) The oxygen in the blood decreases

Pressure changes occur and result in closure of the ductus arteriosus

At birth there are multiple changes in the cardiac and respiratory systems. Which of the following is one of the changes to occur at birth in the cardiovascular system? a) Oxygen is exchanged in the lungs b) Fluid is removed from the alveoli and replaced with air c) Pressure changes occur and result in closure of the ductus arteriosus d) The oxygen in the blood decreases

Pressure changes occur and result in closure of the ductus arteriosus Correct Explanation: The ductus arteriosus is one of the openings through which there was fetal circulation. At birth, or within the first few days, this closes and the heart becomes the source of movement of blood to and from the lungs. The exchange of oxygen in the lungs is not a function of the cardiovascular system; it is a function of the respiratory system. Again, the removal of fluid from the alveoli is not a function of the cardiovascular system. The oxygen content of the blood increases; it does not decrease. Therefore options A, B and D are incorrect.

Variable decellerations in fetal heart monitor indicates what?

Pressure on the cord causing circulation problems. • Prolapsed Cord • Fetus positioned on cord • Insufficient profusion to fetus

A father asks the nurse what medication is in the baby's eyes and why it is needed. Which of the following is the appropriate explanation? a) Destroy an infectious exudate of the vaginal canal. b) Prevent infection of the baby's eyes by bacteria which may have been in the vaginal canal. c) Prevent potentially harmful virus from invading the tear ducts. d) Prevent the baby's eyelids from sticking together to help see.

Prevent infection of the baby's eyes by bacteria which may have been in the vaginal canal.

A father asks the nurse what medication is in the baby's eyes and why it is needed. Which of the following is the appropriate explanation? a) Destroy an infectious exudate of the vaginal canal. b) Prevent infection of the baby's eyes by bacteria which may have been in the vaginal canal. c) Prevent potentially harmful virus from invading the tear ducts. d) Prevent the baby's eyelids from sticking together to help see.

Prevent infection of the baby's eyes by bacteria which may have been in the vaginal canal. Correct Explanation: Antibiotic ointment is used in the infant's eyes at delivery to prevent ophthalmia neonatorum, an infection which can lead to blindness. Option A is incorrect as it gives misinformation to the father. Antibiotic ointment is not used for a potentially harmful virus; therefore option C is incorrect. Option D is incorrect as it also gives misinformation to the father of the infant.

The nurse cares for a newborn with a congenital cardiac anomaly. What component of nursing care is the priority for the newborn? a) Maintain oxygen saturation at 95% or above b) Teach the parents to take pulse and blood pressure measurements c) Prevent pain as much as possible d) Accompany the newborn to all radiologic examinations

Prevent pain as much as possible

In relation to primary and secondary powers, the maternity nurse comprehends that:

Primary powers are responsible for effacement and dilation of the cervix.

Reciprocity

Process by which the infant's abilities and behaviors elect parental response

The nurse wants to maintain a neutral thermal environment for her assigned neonatal clients. Which intervention would best ensure that this goal is met? a) Promote early breastfeeding for the infants b) Avoid bathing the newborn until they are 24 hours old c) Avoid skin-to-skin contact with the mother until the infants are 8 hours old d) Keep the infant transporter temperature between 80 and 85 degrees F

Promote early breastfeeding for the infants

When a pregnant client arrives at the hospital and is taken to triage to determine what stage of labor she is in, the nurse gets a urine sample immediately, what is being tested for and what are the implications?

Protein : Hypertention Nitrates : Infection Ketones : Renal Problems and Glucose (Diabetic?)

A woman in labor at the hospital has just received an epidural block. Which intervention is priority before and during epidural placement? a) Increase oral fluids every hour to prevent dehydration b) Monitor temperature every four hours and give Tylenol if 100.4 c) Monitor the maternal apical pulse for Bradycardia d) Provide adequate IV fluids to maintain her blood pressure

Provide adequate IV fluids to maintain her blood pressure Correct Explanation: The patient will need to have a bolus of IV fluids prior to and then maintained during the epidural to be prepared in the event of the hypotensive episodes that may accompany epidural placement. The hypotensive event is transitory, and increasing oral hydration is unnecessary and may lead to nausea later. Monitor the mother's body temperature, but wait for instructions from the doctor as to when to administer medication. Bradycardia is not a common side effect of epidural medication

A nurse is caring for a newborn with transient tachypnea. What nursing interventions should the nurse perform while providing supportive care to the newborn? Select all that apply.

Provide oxygen supplementation. Ensure the newborn's warmth. Observe respiratory status frequently The nurse should give the newborn oxygen, ensure the newborn's warmth, and observe the newborn's respiratory status frequently. The nurse need not give the newborn warm water to drink or massage the newborn's back.

As you are examining the newborn female, you notice a small pinkish discharge from the vaginal area. What should you suspect? a) Impending hemorrhage from a congenital defect b) Pseudomenstruation, a normal finding c) Infection d) Evidence of birth trauma

Pseudomenstruation, a normal finding

Postpartum assessment?

Pt hx Preg, labor, + delivery events/interventions Vitals-temp (100.4=ok), bradycardia normal for 1 wk, BP can vary

A woman dilated to 10 centimeters and feeling the urge to "have a bowel movement" is refusing to push, she is screaming "it hurts down there too much to push." What is the option the nurse should suggest at this point for pain management to facilitate pushing? a) Paracervical block b) Pudendal block c) Parenteral medication d) Epidural anesthesia

Pudendal block Correct Explanation: The patient is too far dilated to have any parenteral medication or an epidural block. The best option is a local block or a pudendal block that will numb the vaginal wall to block the pain sensation to the pudendal nerve. The paracervical block is only used in the first stage of labor and this patient is in the second stage.

In order to care for obstetric patients adequately, the nurse understands that labor contractions facilitate cervical dilation by:

Pulling the cervix over the fetus and amniotic sac.

A patient delivered 2 days ago and is preparing for discharge. The nurse assesses respirations to be 26 rpm and labored, and the patient was short of breath ambulating from the bathroom this morning. Lung sounds are clear. The nurse alerts the physician and the nurse-midwife to her concern that the patient may be experiencing

Pulmonary embolism Correct Explanation: These symptoms suggest a pulmonary embolism. Mitral valve collapse and thrombophlebitis would not present with these symptoms; infection would have a febrile response with changes in lung sounds.

If there has been a SROM or AROM, how often should the nurse take vital signs?

Q 2hrs.

A woman arrives in the labor and birth department and is panting and screaming "the baby is coming". What is the priority intervention by the nurse?

Quickly move the woman to a labor bed, and check the perineum. The woman is showing signs of advanced labor, possibly in transition or stage two. She needs to be managed as an imminent birth and taken directly to a room for vaginal assessment. Vital signs, medical/obstetrical history, and her room assignment can be taken care of later in the process.

A woman arrives in labor and delivery, is panting and screaming "the baby is coming". What is the priority intervention by the nurse? a) Quickly move the woman to a labor bed, check the perineum b) Admit her to the unit and escort to a room c) Assess vital signs d) Ask medical and obstetrical history

Quickly move the woman to a labor bed, check the perineum Correct Explanation: The woman is showing signs of advanced labor, possibly in transition or stage two. She needs to be managed as an imminent birth and taken directly to a room for vaginal assessment. Vital signs, medical/obstetrical history, and her room assignment can be taken care of later in the process.

When assessing the fetus using Leopold maneuvers, the nurse feels a round, firm, movable fetal part in the fundal portion of the uterus and a long, smooth surface in the mother's right side close to midline. What is the likely position of the fetus?

RSA

The nurse uses a radiant warmer to transport a newborn to reduce heat loss via which mechanism? a) Evaporation b) Convection c) Conduction d) Radiation

Radiation

A nurse is teaching newborn care to students. The nurse correctly identifies which of the following as the predominant form of heat loss in the newborn? a) Nonshivering thermogenesis. b) Lack of brown adipose tissue. c) Sweating and peripheral vasoconstriction. d) Radiation, convection, and conduction.

Radiation, convection, and conduction.

A nurse is teaching newborn care to students. The nurse correctly identifies which of the following as the predominant form of heat loss in the newborn? a) Nonshivering thermogenesis. b) Sweating and peripheral vasoconstriction. c) Radiation, convection, and conduction. d) Lack of brown adipose tissue.

Radiation, convection, and conduction. Correct Explanation: Heat loss in the newborn occurs primarily through radiation, convection, and conduction because of the newborn's large ratio of body surface to weight, and because of the marked difference between core and skin temperatures. Nonshivering thermogenesis is a mechanism of heat production in the newborn. Lack of brown adipose tissue contributes to heat loss, particularly in premature infants, but it is not the predominant form of heat loss. Peripheral vasoconstriction is a method to increase heat production.

Baby Tarik has been circumcised, his temperature is stable, his breathing and heart rate are healthy, and he is ready to be discharged from the hospital. What can the nurse tell his parents to be on the lookout for that might indicate that Tarik needs medical attention? a) Straining when he is passing stools b) Redness at the base of the umbilical cord c) A yellowish crusty substance on the circumcision site d) Crying for 2 hours or more each day

Redness at the base of the umbilical cord Explanation: The cord should dry and fall off in the 7 to 10 days after delivery. If the cord base changes color or develops drainage the health care provider needs to be notified as these could be signs of infection. A yellowish crusty substance on the circumcision site indicates normal healing. Crying for two hours or more each day and straining at stools are normal in a newborn.

Not all mothers express joy at seeing their newborn upon delivery and during their hospitalization. A behavior that indicates impaired attachment of the mother to the newborn is a) Referring to a facial feature as "ugly" b) Dressing the child in old clothes c) Giving the child an uncommon name d) Bottle feeding

Referring to a facial feature as "ugly" Correct Explanation: Making negative comments about a newborn's features is a warning sign of impending attachment difficulties. The other options may be culturally rooted.

The nurse should tell a primigravida that the definitive sign indicating that labor has begun would be:

Regular, progressive uterine contractions that increase in intensity and frequency are the definitive sign of true labor along with cervical change.

A newborn in the nursery has a temperature of 97.4°F (36.3°C). What may happen first, if the infant continues to be cold stressed? a) Seizure b) Hypoglycemia c) Respiratory distress d) Cardiovascular distress

Respiratory distress

The nurse is preparing to apply a thermistor probe to a newborn to monitor the newborn's temperature. At which location would the nurse most likely apply the probe? a) Upper left arm b) Right upper abdominal quadrant c) Lower back d) Right great toe

Right upper abdominal quadrant Correct Explanation: A thermistor probe is taped to the newborn's abdomen, usually in the right upper quadrant. This allows for position changes without having to readjust the probe.

Charting on the nursing care plan patient care, which nursing diagnosis has the highest priority for a postpartum patient? a) Acute pain related to afterpains or episiotomy discomfort b) Risk for infection related to multiple portals of entry for pathogens, including the former site of the placenta, episiotomy, bladder and breasts c) Risk for injury: postpartum hemorrhage related to uterine atony d) Risk for injury: falls related to postural hypotension and fainting

Risk for injury: postpartum hemorrhage related to uterine atony Explanation: The highest priority is the risk for injury related to postpartum hemorrhage. The patient needs close observation and assessment for hemorrhage. All of the options presented are appropriate nursing diagnoses for a postpartum patient. However, options A, B, and D do not take precedence over the risk for postpartum hemorrhage.

A laboring client in transition phase is 8 cm dilated and feels the urge to push. What are the risks to the mom and what is the nursing intervention?

Risk of tearing perinium when <10cm dialted • Use open glottis, short breaths when feeling the urge to push.

When evaluating neurologic maturity to determine gestational age, which of the following is not part of the assessment? a) Popliteal angle b) Square window c) Rooting d) Posture

Rooting

When evaluating neurologic maturity to determine gestational age, which of the following is not part of the assessment? a) Rooting b) Popliteal angle c) Square window d) Posture

Rooting Correct Explanation: The six activities the newborn performs when being evaluated for gestational age based on neurologic maturity are as follows: posture, square window, arm recoil, popliteal angel, scar sign, and heel to ear

The nurse is documenting assessment of infant reflexes. She strokes the side of the infants face and the baby turns toward the stroke. What reflex has the nurse elicited? a) Moro b) Tonic neck c) Sucking d) Rooting

Rooting Correct Explanation: This is the rooting reflex and is used to encourage the infant to feed. This reflex and the sucking reflex work together to assist the infant with cues for feeding at the breast. The tonic neck (or fencing) reflex and the Moro (or startle reflex) are total body reflexes and assess neurologic function in the newborn.

A nurse is discussing breastfeeding with a new mother and demonstrates that when she strokes the baby's cheek, the baby turns his head in that direction. This reflex is known as which of the following? a) Moro reflex b) Babinski reflex c) Extrusion reflex d) Rooting reflex

Rooting reflex Correct Explanation: The rooting reflex is demonstrated when, in response to the newborn's cheek being stroked, the infant turns her head in that direction. This reflex serves to help a newborn find food: when a mother holds the child and allows her breast to brush the newborn's cheek, the reflex causes the baby to turn toward the breast. The extrusion reflex is demonstrated when a newborn extrudes any substance that is placed on the anterior portion of the tongue; this reflex prevents the swallowing of inedible substances. The Moro reflex is demonstrated when, in response to a sudden backward head movement, the newborn abducts and extends arms and legs, then swings the arms into an embrace position and pulls up the legs against the abdomen. The Babinski reflex is demonstrated when the sole of a newborn's foot is stroked in an inverted "J" curve from the heel upward and the newborn fans the toes in response (positive Babinski sign).

What are normal heart sounds? What does this sound come from? Where do you listen for heart sounds?

S1 (lub) S2 (dub) The "lub" is the first heart sound, commonly termed S1, and is caused by turbulence caused by the closure of mitral and tricuspid valves at the start of systole. The second heart sound, "dub" or S2, is caused by the closure of the aortic and pulmonic valves, marking the end of systole. PMI heard best, lateral of the midclavicular line 4th intercoastal space, which is slightly different that the placement for an adult

Sarah has just arrived at the hospital, in early labor, showing signs of extreme anxiety over the birth to come. Why is it so important that the nurse help Sarah relax? a) This is the time at which the nurse must establish that she is in control; she will be taking care of Sarah and Sarah needs to trust the nurse b) Sarah's anxiety can actually slow down the labor process and decrease the amount of oxygen reaching the uterus and the fetus c) Sarah needs to sleep now so that she can save her energy for the later stages of labor d) Sarah's anxiety will increase her blood pressure, increasing her risk with an epidural

Sarah's anxiety can actually slow down the labor process and decrease the amount of oxygen reaching the uterus and the fetus Correct Explanation: Anxiety out of control can decrease the oxygen of the mother by increasing her respiratory rate and increasing the demand on her body, and have a negative impact on the fetus. Encourage control of the anxiety. Anxiety will not negatively affect the action of the epidural. It is premature to be stern with the patient. While it is preferable that she save her energy, it is not damaging to her or to the fetus if she does not sleep.

When giving a postpartum client self-care instructions in preparation for discharge, the nurse instructs her to report heavy or excessive bleeding. How should the nurse describe "heavy bleeding?" a) Saturating 1 pad in 6 hours b) Saturating 1 pad in 1 hour c) Saturating 1 pad in 8 hours d) Saturating 1 pad in 3 hours

Saturating 1 pad in 1 hour Correct Explanation: Bleeding is considered heavy when a woman saturates a sanitary pad in 1 hour. Excessive bleeding occurs when a postpartum client saturates 1 pad in 15 minutes. Moderate bleeding occurs when the bleeding saturates less than 15 cm of a pad in 1 hour.

What is the primary function of uterine contractions after delivery of the infant and placenta? a) Return the uterus to normal size b) Seal off the blood vessels at the site of the placenta c) Stop the flow of blood d) Close the cervix

Seal off the blood vessels at the site of the placenta Explanation: The contractions of the uterus help to constrict the vessels where the placenta was located. This does decrease the flow of blood, but is secondary in occurrence to the constriction of the blood vessels. Uterine contraction also leads to uterine involution, which normally occurs at a predictable rate. Uterine involution assists in closing the cervix. Again, options A, C, and D are secondary to the constriction of blood vessels at the placental site.

Which of the following is NOT an opioid that is commonly used during labor and delivery? a) Nalbuphine (Nubain) b) Secobarbital (Seconal) c) Meperidine (Demerol) d) Butorphanol (Stadol)

Secobarbital (Seconal) Correct Explanation: Meperidine, butorphanol, and nalbuphine are opioids that are commonly used during labor and delivery. Secobarbital is a barbiturate.

Which action would lead the nurse to assess that a postpartal woman is entering the taking-hold phase of the postpartal period?

She did her perineal care independently. During the taking-in phase, women tend to be dependent; during the taking-hold phase, they begin independent actions.

Which maternal reaction is the most concerning?

She neglects to engage with or provide care for the baby and shows little interest in it. A mother not bonding with the infant or showing disinterest is a cause for concern and requires a referral or notification of the primary health care provider. Some mothers hesitate to take their newborn and express disappointment in the way the baby looks, especially if they want a child of one sex and have a child of the opposite sex. Expressing doubt about the ability to care for the baby is not unusual, and being tearful for several days with difficulty eating and sleeping is common with "postpartum blues".

Which information would the nurse emphasize in the teaching plan for a postpartal woman who is reluctant to begin taking warm sitz baths?

Sitz baths increase the blood supply to the perineal area. Sitz baths decrease pain and aid healing by increasing blood flow to the perineum.

Which of the following would you emphasize in the teaching plan for a postpartal woman who is reluctant to begin taking warm sitz baths? a) Sitz baths cause perineal vasoconstriction and decreased bleeding. b) The longer a sitz bath is continued, the more therapeutic it becomes. c) Sitz baths increase the blood supply to the perineal area. d) Sitz baths may lead to increased postpartal infection.

Sitz baths increase the blood supply to the perineal area. Correct Explanation: Sitz baths decrease pain and aid healing by increasing blood flow to the perineum.

Physical assessment normal finds? Skin Genitals Eyes Ears Nose Mouth

Skin: think, blood vessels lie close to the surface. sticky to transparent to sooth, varying degree of peeling/cracking, languo (soft downy hair) Gentials: males scrotum may have a darkened appearance, females may have peusdomenestration of a pink-tinged or bloody mucous type discharge Eyes, eyelids can be fused or open, newborn will have transition eyes (looks somewhat as a person would look if cross-eyed), the newborn can track midline, has normal blink reflex that persists into adulthood Ears open/patent, the greater the amount of ear cartilage the more mature the newborn) Nose: may have congestion until delivery secretions start clearing. If newborn is having trouble breathing mouth or nose wise such as gasping, gagging, clear the airway via the bulb syringe. As nurse after head clears in delivery such the M then N approach mouth then nose is preferred in case there is meconium sitting in the mouth to decrease startling and aspirating. Mouth, open, palate smooth/intact when nurse runs finger over upper roof of mouth, rooting reflex when check is stroked the infant turns its mouth toward the finger, sucking reflex, nurse places finger in mouth of newborn and the newborn will suck to feed, frenulum intact

On an Apgar evaluation, reflex irritability is tested by which of the following? a) Dorsiflexing a foot against pressure resistance b) Raising the infant's head and letting it fall back c) Tightly flexing the infant's trunk and then releasing it d) Slapping the soles of the feet and observing the response

Slapping the soles of the feet and observing the response Correct Explanation: Reflex irritability means the ability to respond to stimuli. It can be tested by slapping the foot or evaluating the response to a catheter passed into the nose.

On completion of a vaginal examination on a laboring woman, the nurse records: 50%, 6 cm, -1. What is a correct interpretation of the data

Station of -1 indicates that the fetal presenting part is above the ischial spines and has not yet passed through the pelvic inlet. Progress of effacement is referred to by percentages, with 100% indicating full effacement and dilation by centimeters, with 10 cm indicating full dilation

Newborn care: Diaper

Stool go back to stool transitions on chapter 17 can you name them in order, what they should look like? What is normal? Urine = 6-8 voids per day Cord care= do not pick/pull at the scab that forms it will dry and fall off, is a vascular area Circumcision = after procedure keep track that they have a void/urine as a surgical procedure has been performed on an area that has a function of urination/elimination, watch for REEDA (redness, ecchymosis, edema, drainage, (no need for the A), do not pick/pull the bell off, the gauze off, or a scab off as the penis is quite vascular

A nurse is teaching a new mother about what to expect for bowel elimination in her newborn. Because the mother is breast-feeding, the nurse should tell her which of the following? a) Stools should be brown and loose. b) Stools should be yellow-gold, loose, and stringy to pasty. c) Stools should be yellow-green and loose. d) Stools should be greenish and formed in consistency.

Stools should be yellow-gold, loose, and stringy to pasty.

A nurse is teaching a new mother about what to expect for bowel elimination in her newborn. Because the mother is breast-feeding, the nurse should tell her which of the following? a) Stools should be yellow-gold, loose, and stringy to pasty. b) Stools should be yellow-green and loose. c) Stools should be greenish and formed in consistency. d) Stools should be brown and loose.

Stools should be yellow-gold, loose, and stringy to pasty. Correct Explanation: The stools of a breast-fed newborn are yellow-gold, loose, and stringy to pasty in consistency. The stools of the formula-fed newborn vary depending on the type of formula ingested. They may be yellow, yellow-green, or greenish and loose, pasty, or formed in consistency, and they have an unpleasant odor.

Which factors influence cervical dilation?

Strong uterine contractions The force of the presenting fetal part against the cervix The pressure applied by the amniotic sac Scarring of the cervix

Opioids are the most frequently used medications to provide analgesia during labor. Which of the following drugs is an opioid that is used in obstetrics for relief of labor pain? a) Sublimaze b) Ultram c) Carbamazepine d) Toradol

Sublimaze Explanation: Opioids, medications with opium-like properties (also known as narcotic analgesics), are the most frequently administered medications to provide analgesia during labor. Opioids, such as meperidine (Demerol) and fentanyl (Sublimaze), frequently assist the woman to better tolerate labor contractions by causing relaxation and sleep between contractions (Grant, 2006b). Opioids are most frequently given by the intravenous (IV) route because this route provides fast onset and more consistent drug levels than do the subcutaneous or intramuscular routes.

Which of the following would the nurse do first after the birth of a newborn? a) Administer vitamin K. b) Apply identification bracelet. c) Obtain footprints. d) Suction the mouth and nose.

Suction the mouth and nose.

Which of the following would the nurse do first after the birth of a newborn? a) Obtain footprints. b) Administer vitamin K. c) Apply identification bracelet. d) Suction the mouth and nose.

Suction the mouth and nose. Correct Explanation: The first priority is to ensure a patent airway by suctioning the newborn's mouth and nose. Before leaving the birthing area, newborn identification procedures are completed, including applying the identification bracelet and possibly footprinting, depending on the agency's policy. Vitamin K is administered soon after birth but it does not take priority over ensuring a patent airway.

The nurse is providing discharge education on newborn care at home. The nurse provides instructions that infants need to be placed on their back to sleep. What is the nurse reducing the risk for with this education? a) Apnea episodes b) Sudden infant death syndrome c) Waking at night d) Gastroesophageal reflux

Sudden infant death syndrome

The nurse is providing discharge education on newborn care at home. The nurse provides instructions that infants need to be placed on their back to sleep. What is the nurse reducing the risk for with this education? a) Sudden infant death syndrome b) Gastroesophageal reflux c) Apnea episodes d) Waking at night

Sudden infant death syndrome Correct Explanation: The "back to sleep" campaign is a national campaign used to educate the public concerning the fact that the proper position for sleep of infants is on their back to help decrease the risk of SIDS. Placing the infant on their back to sleep does not reduce the risk for gastroesophageal reflux, apnea episodes, nor waking at night.

What's the func of the bottom hand during fundus assessment?

Support to not let the uterus be displaced

While waiting for the placenta to deliver during the third stage of labor you must assess the new mother's vital signs every 15 minutes. What signs would indicate impending shock? a) Bradypnea and hypertension b) Tachypnea and a widening pulse pressure c) Bradycardia and auscultation of fluid in the base of the lungs d) Tachycardia and a falling blood pressure

Tachycardia and a falling blood pressure Correct Explanation: Monitor the woman's vital signs at least every 15 minutes during the third stage of labor. Tachycardia and a falling blood pressure are signs of impending shock; immediately report these signs.

A newborn infant born by a cesarean birth is experiencing a common problem seen in these type of deliveries.What finding would the nurse anticipate in an infant following a cesarean birth? a) Cardiac murmur b) Hypoglycemia c) Tachypnea d) Hyperthermia

Tachypnea

The nurse is admitting a client who appears to be in advanced labor with imminent birth. Which action should the nurse prioritize?

Take blood pressure and determine if clonus or edema are present.

When caring for a newborn who is jittery and irritable 30 minutes after birth, what should the nurse do? a) Assess the baby's temperature with a thermal skin probe. b) Take blood, using a heel stick, to check for hypoglycemia. c) Rule out hypoglycemia by checking the mother's chart for diabetes or other risk factors. d) Place the child beneath a radiant warmer.

Take blood, using a heel stick, to check for hypoglycemia.

When caring for a newborn who is jittery and irritable 30 minutes after birth, what should the nurse do?

Take blood, using a heel stick, to check for hypoglycemia. One of the primary signs/symptoms of hypoglycemia in newborn infants is jitteryness and irritability. Anytime an infant is suspected of having hypoglycemia, the nurse needs to check the blood glucose level with a heel stick. The infant described in the scenario does not need to be placed under a radiant warmer or have its temperature assessed with a thermal skin probe. The nurse does not rule out hypoglycemia in an infant by checking the mother's chart to see if she is diabetic or has other risk factors.

When caring for a newborn who is jittery and irritable 30 minutes after birth, what should the nurse do? a) Take blood, using a heel stick, to check for hypoglycemia. b) Place the child beneath a radiant warmer. c) Assess the baby's temperature with a thermal skin probe. d) Rule out hypoglycemia by checking the mother's chart for diabetes or other risk factors.

Take blood, using a heel stick, to check for hypoglycemia. Correct Explanation: One of the primary signs/symptoms of hypoglycemia in newborn infants is jitteryness and irritability. Anytime an infant is suspected of having hypoglycemia, the nurse needs to check the blood glucose level with a heel stick. The infant described in the scenario does not need to be placed under a radiant warmer or have its temperature assessed with a thermal skin probe. You do not rule out hypoglycemia in an infant by checking the mother's chart to see if she is diabetic or has other risk factors.

A mother who is 4 days postpartum and is breastfeeding expresses to the nurse that her breast seems to be tender and engorged. Which suggestions should the nurse give to the mother to relieve breast engorgement? Select all that apply.

Take warm-to-hot showers to encourage milk release. Express some milk manually before breastfeeding. Apply warm compresses to the breasts prior to nursing. To relieve breast engorgement in the client, the nurse should educate the client to take warm-to-hot showers to encourage milk release, express some milk manually before breastfeeding, and apply warm compresses to the breasts before nursing. The mother should be asked to feed the newborn in a variety of positions—sitting up and then lying down. The breasts should be massaged from under the axillary area, down toward the nipple.

A patient who has just delivered a baby girl demonstrates behavior not indicative of bonding when she does which of the following? a) Strokes the infants' head b) Holds and smiles at the infant c) Talks to company and ignores the baby lying next to her d) Kisses the infant on her cheek

Talks to company and ignores the baby lying next to her Correct Explanation: Bonding is the close emotional attraction to a newborn by the parents that develops during the first 30 to 60 minutes after birth. The mother initiates bonding when she caresses her infant and exhibits certain behaviors typical of a mother tending to her child. Ignoring the infant while talking to visitors is not an example of proper bonding.

All of the following are characteristics of an infant abductor EXCEPT a) Married and/or lives with a male partner b) Lives near the hospital c) Targets a specific infant d) A woman of childbearing age

Targets a specific infant Correct Explanation: Most infant abductors do not target specific infants.

All of the following are ways the nurse can encourage bonding between the parents and the newborn EXCEPT a) Asking the parents' permission to pick up the newborn b) Encouraging parents to provide care while you are there to observe them c) Telling the mother that the best way to bond with her baby is to breastfeed d) Talking to the newborn in front of the parents

Telling the mother that the best way to bond with her baby is to breastfeed Correct Explanation: Modeling behavior such as talking to the newborn will aid in bonding. Being able to observe parents as they provide care to their newborn will give new parents confidence. Asking their permission to pick up the newborn will give them a sense of ownership. Although breastfeeding is an excellent way for a mother to bond with her baby, it is not the only way and it is not necessarily the best way.

An episiotomy or a cesarean incision requires assessment. Which assessment criterion for skin integrity is not initially noted? a) Temperature b) Edema c) Redness d) Drainage

Temperature Correct Explanation: The temperature of an incision would be determined only if the other parameters require this. A sterile glove would be used to assess skin temperature.

With an HBV-positive mother, what should the newborn receive? a) The HBV vaccination and 2 doses of hepatitis B immunoglobulin within 24 hours of birth b) The HBV vaccination and 1 dose of hepatitis B immunoglobulin within 24 hours of birth c) The HBV vaccination and 1 dose of hepatitis B immunoglobulin within 12 hours of birth d) Two doses of the hepatitis B immunoglobulin within 24 hours of birth

The HBV vaccination and 1 dose of hepatitis B immunoglobulin within 12 hours of birth Correct Explanation: If a mother has HBV or is suspected of having HBV, the newborn should be bathed and then should receive 1 dose of the hepatitis B vaccine and 1 dose of the hepatitis B immuoglobulin within 12 hours of birth.

Why should a nurse monitor a newborn after cesarean birth more closely than after a vaginal birth? a) Fetal lungs are uninflated and full of amniotic fluid that must be absorbed. b) Much of the fetal lung fluid is squeezed out as the fetus moves down the birth canal. c) The baby will have more fluid in its lungs, making respiratory adaptation more challenging. d) The baby's lifeline to oxygen is cut off when the umbilical cord is clamped, resulting in oxygen levels falling and carbon dioxide rising.

The baby will have more fluid in its lungs, making respiratory adaptation more challenging.

Why should a nurse monitor a newborn after cesarean birth more closely than after a vaginal birth? a) The baby will have more fluid in its lungs, making respiratory adaptation more challenging. b) Fetal lungs are uninflated and full of amniotic fluid that must be absorbed. c) Much of the fetal lung fluid is squeezed out as the fetus moves down the birth canal. d) The baby's lifeline to oxygen is cut off when the umbilical cord is clamped, resulting in oxygen levels falling and carbon dioxide rising.

The baby will have more fluid in its lungs, making respiratory adaptation more challenging. Correct Explanation: During a vaginal delivery the infant is squeezed by the uterine contractions. The infant who is born via c-section without labor first does not have the mechanical removal of the fluid from the lungs. This places the infant at increased risk for respiratory compromise. The need to more closely assess a newborn after delivery by cesarean section is not caused by the clamping of the umbilical cord. Amniotic fluid in the lungs of all newborns needs to be absorbed by the body. This is not just a need in an infant born by cesarean section.

While educating a class of postpartum patients before discharge home after delivery, one woman asks when "will I stop bleeding?" How should the nurse respond? a) You should stop bleeding and have no discharge in the next 1 to 2 weeks b) The bleeding may slowly decrease over the next 1 to 3 weeks, changing color to a white discharge, which may continue for up to 6 weeks c) Bleeding may occur on and off for the next 2 to 3 weeks d) The bleeding may continue for 6 weeks

The bleeding may slowly decrease over the next 1 to 3 weeks, changing color to a white discharge, which may continue for up to 6 weeks Explanation: The lochia changes color in the first few weeks postpartum; the active bleeding stops in the first week but a white discharge may continue for up to 6 weeks after delivery. Option A is incorrect because it is an incomplete answer. Option B is incorrect because bleeding does not occur "off and on"; the bleeding stops during the first week but a discharge continues to occur. Option C is incorrect because the discharge may continue for up to six weeks.

While educating a class of postpartum clients before discharge home after birth, one woman asks when "will I stop bleeding?" How should the nurse respond?

The bleeding may slowly decrease over the next 1 to 3 weeks, changing color to a white discharge, which may continue for up to 6 weeks. The lochia changes color in the first few weeks postpartum; the active bleeding stops in the first week, but a white discharge may continue for up to 6 weeks after birth. Bleeding does not occur "off and on"; the bleeding stops during the first week but a discharge continues to occur. The discharge may continue for up to six weeks, not just bleeding.

The nurse has received report regarding her patient in labor. The woman's last vaginal examination was recorded as 3 cm, 30%, and -2. The nurse's interpretation of this assessment is that:

The cervix is 3 cm dilated, it is effaced 30%, and the presenting part is 2 cm above the ischial spines.

A nurse helps a postpartum woman out of bed for the first time postpartally and notices that she has a very heavy lochia flow. Which assessment finding would best help the nurse decide that the flow is within normal limits?

The color of the flow is red. A typical lochia flow on the first day postpartally is red; it contains no large clots; the uterus is firm, indicating that it is well contracted.

You help a postpartum woman out of bed for the first time postpartally and notice that she has a very heavy lochia flow. Which of the following assessment findings would best help you decide that the flow is within normal limits? a) The flow is over 500 mL. b) The flow contains large clots. c) The color of the flow is red. d) Her uterus is soft to your touch.

The color of the flow is red. Correct Explanation: A typical lochia flow on the first day postpartally is red; it contains no large clots; the uterus is firm, indicating that it is well contracted.

Which is the best place to perform a heel stick on a newborn? a) The front of the heel (the outer arch) b) The fat pads on the lateral aspects of the foot c) The calcaneus d) The vascularized flat surface of the foot

The fat pads on the lateral aspects of the foot

Which is the best place to perform a heel stick on a newborn? a) The front of the heel (the outer arch) b) The vascularized flat surface of the foot c) The fat pads on the lateral aspects of the foot d) The calcaneus

The fat pads on the lateral aspects of the foot Correct Explanation: The calcaneus is the bone of the heel. A heel stick should not be done on the flat part of the foot or heel, but instead on the lateral aspect of the foot, where the fat pads are.

The American Academy of Pediatrics and the American Dietetic Association recommend breastfeeding exclusively for how long? a) The first 28 days b) The first 3 months c) The first 4 months d) The first 6 months

The first 6 months

The American Academy of Pediatrics and the American Dietetic Association recommend breastfeeding exclusively for how long? a) The first 28 days b) The first 3 months c) The first 4 months d) The first 6 months

The first 6 months Correct Explanation: Both the AAP and the ADA recommend breastfeeding exclusively for the first 6 months of life. After 6 months, breastfeeding does not need to be exclusive, but it should be continued until 12 months.

The maternity nurse should notify the health care provider about which assessment findings during labor? Positive urine drug screen Blood glucose level of 78 mg/dL Increased systolic blood pressure during first stage Elevated white blood cell count Oral temperature of 99.8° F Respiratory rate of 10 breaths/min

The health care provider should be alerted to a positive urine drug screen, because certain drugs will have an effect on pain medications that can be safely administered. The respiratory rate usually increases during labor. A rate of 10 is low and needs to be reported. Decreased blood glucose levels (due to exertion and glucose consumption for energy), and increased systolic blood pressure, elevated white blood cell count (due to stress response), and a slightly elevated temperature (up to 100.4° F) are expected findings during labor.

The nurse is performing a newborn assessment and the infant's lab work reveals a heelstick Hct of 66. What is the best response to this finding? a) The infant is suffering from polycythemia and needs a partial exchange transfusion to prevent complications. b) The hematocrit needs to be repeated as a venous stick to see what the central hematocrit level is. c) This is a normal lab value and no intervention is needed. d) A capillary hematocrit needs to be rechecked in 8 hours to see if is increases or decreases.

The hematocrit needs to be repeated as a venous stick to see what the central hematocrit level is.

With a hepatitis B (HbsAG) positive mother, what should the newborn receive? a) Two doses of the hepatitis B immunoglobulin within 24 hours of birth b) The hepatitis B vaccination and 2 doses of hepatitis B immunoglobulin within 24 hours of birth c) The hepatitis B vaccination and 1 dose of hepatitis B immunoglobulin within 12 hours of birth d) The hepatitis Bvaccination and 1 dose of hepatitis B immunoglobulin within 24 hours of birth

The hepatitis B vaccination and 1 dose of hepatitis B immunoglobulin within 12 hours of birth

A male baby is born at 5:15 AM on a Wednesday. At 1:15 PM on the same day, the nurse notes yellow staining of the skin on the head and face of this infant. What does this finding likely indicate? a) The infant has pathologic jaundice. b) The infant has physiologic jaundice. c) The unconjugated bilirubin levels in the infant are less than 4 mg/dL. d) The nurse should not expect the yellow staining to occur on the trunk or extremities.

The infant has pathologic jaundice.

A client delivers a baby at a local health care facility. The nurse observes that the infant is fussy and begins to move her hands to her mouth and suck on her hand and fingers. How should the nurse interpret these findings? a) The infant is entering the habituation state. b) The infant is in a state of hyperactivity. c) The infant is attempting self-consoling maneuvers. d) The infant is displaying a state of alertness.

The infant is entering the habituation state.

A nurse determines that a newborn has a 1-minute Apgar score of 5 points. What conclusion would the nurse make from this finding? a) The infant requires immediate and aggressive interventions for survival. b) The infant is experiencing moderate difficulty in adjusting to extrauterine life. c) The infant probably has either a congenital heart defect or an immature respiratory system. d) The infant is adjusting well to extrauterine life.

The infant is experiencing moderate difficulty in adjusting to extrauterine life.

A nurse determines that a newborn has a 1-minute Apgar score of 5 points. What conclusion would the nurse make from this finding?

The infant is experiencing moderate difficulty in adjusting to extrauterine life. The nurse should conclude that the newborn is facing moderate difficulty in adjusting to extrauterine life. The nurse should not conclude that the infant is in severe distress requiring immediate interventions for survival or has a congenital heart or respiratory disorder. If the Apgar score is 8 points or higher, it indicates that the condition of the newborn is better. An Apgar score of 0 to 3 points represents severe distress in adjusting to extrauterine life.

How do you know a female newborn is a term baby?

The labia majora covers the labia minora

It has been 8 hours since a woman gave birth vaginally to a healthy newborn. When assessing the woman's fundus, the nurse would expect to find it at: a) Between the umbilicus and symphysis pubis b) The level of the umbilicus c) 2 cm below the umbilicus d) 1 cm below the umbilicus

The level of the umbilicus Correct Explanation: Approximately 6 to 12 hours after birth, the fundus is usually at the level of the umbilicus. The fundus is between the umbilicus and symphysis pubis 1 to 2 hours after birth. The fundus typically is 1 cm below the umbilicus on the first postpartum day and 2 cm below the umbilicus on the second postpartum day.

Immediately after birth what is the first thing that is done to the newborn?

The newborn is suctioned to removed fluids and mucous from mouth and nose...prevents aspiration of fluid into the lungs

When performing Ortolani maneuver, which of the following should occur? Select all that apply. a) Attempt to abduct the hips 90 degrees while applying upward pressure. b) The newborn should be in a supine position. c) A click should be heard when the legs are abducted. d) Attempt to abduct the hips 180 degrees while applying upward pressure. e) The newborn should be in a prone position.

The newborn should be in a supine position. Attempt to abduct the hips 180 degrees while applying upward pressure.

When the nurse performs the Ortolani maneuver, what should occur? Select all that apply.

The newborn should be in a supine position. Attempt to abduct the hips 180 degrees while applying upward pressure. The newborn should be in the supine position. The nurse will flex the hips and knees to 90 degrees at the hip, then will attempt to abduct the hips 180 degrees while applying upward pressure. A "click" or a "cluck" should not be heard when the legs are abducted.

With regard to factors that affect how the fetus moves through the birth canal, nurses should be aware that:

The normal attitude of the fetus is called general flexion.

As the nurse in an obstetric clinic, you are conducting patient education with a group of expectant mothers. One young woman asks you to tell the group what labor pain is like. What would be your best response? a) It comes in waves. b) The pain of labor is unique and multidimensional. It originates from different places depending on what stage of labor you are in. c) It is best evaluated by talking with visitors in the labor room because they know you best. d) It has been described as the worst pain you will ever feel.

The pain of labor is unique and multidimensional. It originates from different places depending on what stage of labor you are in. Correct Explanation: Pain sensations associated with labor originate from different places, depending on the stage of labor.

Proximity

The physical and psychological experience of the parents being close to their infant 1. Contact 2. Emotional state 3. Individualization

Which of the following is true regarding fetal and newborn senses? a) A newborn cannot experience pain. b) A newborn does not have the ability to discriminate between tastes. c) The rooting reflex is an example that the newborn has a sense of touch. d) A fetus is unable to hear in utero. e) A newborn cannot see until several hours after birth.

The rooting reflex is an example that the newborn has a sense of touch.

Which of the following is true regarding fetal and newborn senses? a) A fetus is unable to hear in utero. b) A newborn cannot see until several hours after birth. c) The rooting reflex is an example that the newborn has a sense of touch. d) A newborn cannot experience pain. e) A newborn does not have the ability to discriminate between tastes.

The rooting reflex is an example that the newborn has a sense of touch. Correct Explanation: Newborns experience pain, have vision, and can discriminate between tastes. The rooting reflex is an example of a newborn's sense of touch. The fetus can hear in utero.

The nurse is assessing an infant's reflexes. While eliciting a rooting reflex, the infants strongly sucks on the nurse's finger. How does the nurse interpret this finding? a) The infant does not have a normal rooting reflex. b) The rooting reflex shows a strong sucking response. c) The rooting reflex was tested incorrectly. d) The infant displays a normal rooting reflex.

The rooting reflex was tested incorrectly.

The nurse is providing discharge education to a first time mother and father on their newborn female infant. The father notes the infant has a yellow skin color. How should the nurse explain what the father is noting? a) Yellow is the normal color for a newborn b) The tint is yellow from jaundice c) The infant needs to be in the sunlight to clear the skin d) This might be a sign of a bleeding problem

The tint is yellow from jaundice Correct Explanation: Newborns often have a yellow tint to the skin if the newborn develops jaundice. Any newborn developing jaundice needs to be assessed by the health care provider. Yellow is not the normal color for a newborn. Placing the infant in sunlight may help to clear the skin of the yellow color; however, unless instructed by the physician, this is not information provided in discharge teaching. Jaundice is not a sign of a bleeding problem.

Review of a woman's labor and birth record reveals a laceration that extends through the anal sphincter muscle. The nurse identifies this as which of the following? a) First-degree laceration b) Fourth-degree laceration c) Second-degree laceration d) Third-degree laceration

Third-degree laceration Explanation: A third-degree laceration extends through the anal sphincter muscle. A first-degree laceration involves only skin and superficial structures above the muscle. A second-degree laceration extends through the perineal muscles. A fourth-degree laceration continues through the anterior rectal wall.

Which of the following signs signify that the second stage of labor has begun? a) Emotions are calm and happy b) Frequency of contractions are 5-6 minutes c) The urge to push occurs d) Fetus is a -1 station

The urge to push occurs Correct Explanation: Second stage of labor is the pushing stage; this is typically identified by the woman's urge to push or a feeling of needing to have a bowel movement. The emotional state may be altered due to pain and pressure. Contraction frequency is variable and not clearly indicative of a particular stage. The fetus can be at stage -1 for any length of time.

Which signs signify that the second stage of labor has begun?

The urge to push occurs. Second stage of labor is the pushing stage; this is typically identified by the woman's urge to push or a feeling of needing to have a bowel movement. The emotional state may be altered due to pain and pressure. Contraction frequency is variable and not clearly indicative of a particular stage. The fetus can be at stage -1 for any length of time.

The nurse is aware that cord compression is not continuous when variable decelerations occur and that compression happens when which of the following takes place?

The uterus contracts and squeezes the cord against the fetus. Cord compression is not continuous when variable decelerations are occurring. The compression occurs when the uterus contracts and squeezes the cord against the fetus. It is relieved when the uterus relaxes between contractions. Prematurity and fetal sleep will cause decreased or absent variabilty.

What is vernix caseoa

Thick white substance that protects the skin of the fetus, some call this cheese as a slang term and is found during the 1st two-three days of birth and is usually in body creases and hair. It does not need to be removed because it will be absorbed into the skin. Most when temperature stable/homeostasis achieved will get firs bath and a lot comes off then but doesn't have to be removed is more of an appearance related to remove.

The nurse instructs the client about skin massage and the gate-control theory of pain. Which statement would be appropriate for the nurse to include for client understanding of the nonpharmacologic pain relief methods?

This is a technique to prevent the painful stimuli from entering the brain.

The nurse instructs the patient about skin massage and the gate-control theory of pain. Which of the following statements would be appropriate for the nurse to include for patient understanding of the nonpharmacological pain-relief methods? a) The gate control mechanism opens so all the stimuli pass through to the brain. b) This is a technique to prevent the painful stimuli from entering the brain. c) The gate control mechanism is located at the pain site. d) Pain perception is decreased if anxiety is present.

This is a technique to prevent the painful stimuli from entering the brain. Correct Explanation: Gate control diverts the pain stimuli from the pain site by replacing with a comfort stimuli in a new location.

The laboring patient who is at 3 cm dilation and 25% effaced is asking for analgesia. The nurse explains the analgesia usually is not administered prior to the establishment of the active phase. Identify the appropriate rationale for this practice. a) The effects would wear off before delivery. b) This may prolong labor and increase complications. c) This can lead to maternal hypertension. d) This would cause fetal depression in utero.

This may prolong labor and increase complications. Correct Explanation: Administration of pharmacologic agents too early in labor can stall the labor and lengthen the entire labor. The patient should be offered nonpharmacologic options at this point until she is in active labor.

What is the Babinski reflex? How do you assess this and what is a normal finding?

This reflex should be present at birth and disappear approx. 1 year of age, so is abnormal if not present at birth or if stays later in years of life. Stroke the lateral sole of the newborn's foot from the heel, across ball of foot, to toes, (in this exact order). Expected/normal reflex or "positive Babinski" = toes should fan out. A diminished response indicate a possible neurological problem and needs follow up/investigated further

What is the most important thing a nurse can do during labor and birth to prevent maternal and fetal infection?

Thoroughly wash the hands before and after client contact. The most important infection control technique in any health care setting is thoroughly washing hands on routine basis. Keeping the area clean is secondary, but is also important.

What is the most important thing that you can do during labor and delivery to prevent maternal and fetal infection? a) Clean the woman's perineum with a Betadine scrub b) Thoroughly wash your hands before and after patient contact c) Strictly follow universal precautions d) Remove soiled drapes and linen; place an absorbent pad under the buttocks and two sterile perineal pads against the perineum

Thoroughly wash your hands before and after patient contact Correct Explanation: The most important infection control technique in any health care setting is thoroughly washing hands on routine basis. Keeping the area clean is secondary, but is also important.

A nurse is preparing a patient for rhythm strip testing. She places the woman into a semi-Fowler's position. What is the appropriate rationale for this measure?

To prevent supine hypotension syndrome The term "rhythm strip testing" means assessment of the fetal heart rate for whether a good baseline rate and long- and short-term variability are present. For this, help a woman into a semi-Fowler's position (either in a comfortable lounge chair or on an examining table or bed with an elevated backrest) to prevent her uterus from compressing the vena cava and causing supine hypotension syndrome during the test. Placing her in this position does not decrease the heart rate of the fetus. It is not done to aid the woman as she pushes in labor, as she is not in labor yet. It is not done to prevent her from falling out of bed.

Which of the following is FALSE regarding bathing the newborn? a) Bathing should not be done until the newborn is thermally stable. b) While bathing the newborn, the nurse should wear gloves. c) To reduce the risk of heat loss, the bath should performed by the nurse, not the parents, within 2-4 hours of birth. d) Mild soap should be used on the body and hair, but not on the face.

To reduce the risk of heat loss, the bath should performed by the nurse, not the parents, within 2-4 hours of birth.

Which of the following is FALSE regarding bathing the newborn? a) While bathing the newborn, the nurse should wear gloves. b) To reduce the risk of heat loss, the bath should performed by the nurse, not the parents, within 2-4 hours of birth. c) Mild soap should be used on the body and hair, but not on the face. d) Bathing should not be done until the newborn is thermally stable.

To reduce the risk of heat loss, the bath should performed by the nurse, not the parents, within 2-4 hours of birth.

Which of the following is FALSE regarding bathing the newborn? a) Mild soap should be used on the body and hair, but not on the face. b) While bathing the newborn, the nurse should wear gloves. c) Bathing should not be done until the newborn is thermally stable. d) To reduce the risk of heat loss, the bath should performed by the nurse, not the parents, within 2-4 hours of birth.

To reduce the risk of heat loss, the bath should performed by the nurse, not the parents, within 2-4 hours of birth. Correct Explanation: Bathing the newborn is not necessary for thermal stability. It can be postponed until the parents are able to do it.

The primary difference between the labor of a nullipara and that of a multipara is the:

Total duration of labor.

Postpartum blues?

Transient emotional dist Anxiety, irritability, insomnia, sadness Begins @ 3-4 days, lasts 2 wks Resolves when mom sleeps No tx

Postpartum blues

Transient emotional disturbances beginning in the first week after childbirth and are characterized by anxiety, irritability, insomnia, crying, loss of appetite, and sadness. resolve by day 8

Babies of mothers with human immunodeficiency virus (HIV) infection should have a thorough bath immediately after birth to decrease the possibility of HIV transmission. a) False b) True

True

Babies of mothers with human immunodeficiency virus (HIV) infection should have a thorough bath immediately after birth to decrease the possibility of HIV transmission. a) True b) False

True

For the first hour after birth, the height of the fundus is at the umbilicus or even slightly above it.

True

The nurse can expect a patient who had a cesarean birth to have less lochia discharge than the patient who had a vaginal birth.

True Women who had a cesarean birth will have less lochia discharge than those who had a vaginal birth, but stages and color changes remain the same.

Following birth, a newborn is placed on the mother's abdomen for a period of skin-to-skin contact. a) False b) True

True Correct Explanation: Following birth, a newborn is placed on the mother's abdomen for a period of skin-to-skin contact, to allow the mother to begin breastfeeding if she wishes and to allow time for parents to enjoy and get acquainted with their newborn. It's important not to interrupt this time as newborns are alert (first period of activity) and respond well to the parents' first tentative touches or interactions with them during this time. (less)

The nurse can expect a patient who had a cesarean birth to have less lochia discharge than the patient who had a vaginal birth. a) True b) False

True Explanation: Women who had a cesarean birth will have less lochia discharge than those who had a vaginal birth, but stages and color changes remain the same.

If the monitor pattern of uteroplacental insufficiency were present, which action would the nurse do first?

Turn her or ask her to turn to her side. The most common cause of uteroplacental insufficiency is compression of the vena cava; turning the woman to her side removes the compression.

If the monitor pattern of uteroplacental insufficiency were present, which of the following would you do first? a) Turn her or ask her to turn to her side. b) Ask her to pant with the next contraction. c) Administer oxygen at 3 to 4 L by nasal cannula. d) Help the woman to sit up in a semi-Fowler's position.

Turn her or ask her to turn to her side. Correct Explanation: The most common cause of uteroplacental insufficiency is compression of the vena cava; turning the woman to her side removes the compression.

When assessing the umbilical cord of a newborn, which of the following would the nurse expect to find? a) Three arteries and no veins b) Two arteries and two veins c) One artery and two veins d) Two arteries and one vein

Two arteries and one vein Correct Explanation: The normal umbilical cord contains three vessels: two arteries and one vein.

When assessing the newborn's umbilical cord, what should the nurse expect to find? a) One smaller artery and two larger veins b) Two smaller arteries and one larger vein c) One smaller vein and two larger arteries d) Two smaller veins and one larger artery

Two smaller arteries and one larger vein

When assessing the newborn's umbilical cord, what should the nurse expect to find? a) One smaller artery and two larger veins b) Two smaller veins and one larger artery c) One smaller vein and two larger arteries d) Two smaller arteries and one larger vein

Two smaller arteries and one larger vein Explanation: When inspecting the vessels in the umbilical cord, the nurse should expect to encounter one larger vein and two smaller arteries. In 0.5% of births (3.5% of twin births), there is only one umbilical artery, which can be linked to cardiac or chromosomal abnormalities.

The nurse is caring for a client in the postpartum period. The client has difficulty in voiding and is catheterized. The nurse then would monitor the client for which of the following? a) Increased urine output b) Stress incontinence c) Loss of pelvic muscle tone d) Urinary tract infection

Urinary tract infection Explanation: The nurse would need to monitor the client for signs and symptoms of a urinary tract infection, a risk associated with catheterization. Stress incontinence is caused due to loss of pelvic muscle tone after birth. Increased urinary output is observed in diuresis. Catheterization does not cause loss of pelvic muscle tone, increased urine output, or stress incontinence.

A woman states that she still feels exhausted on her second postpartal day. Your best advice for her would be to do which of the following? a) Walk with you the length of her room. b) Avoid getting out of bed for another 2 days. c) Walk the length of the hallway to regain her strength. d) Avoid elevating her feet when she rests in a chair.

Walk with you the length of her room. Correct Explanation: Most women report feeling exhausted following childbirth. Ambulation is important, however, so a small amount, such as walking across a room, should be encouraged.

Which instructions should be given by the nurse to the patient in the second stage of labor to facilitate natural childbirth? a) Use a squatting position and use the squat bar for support b) Stay low on her back to ease the back pain c) Use the Valsalva maneuver for effective pushing d) Ask for privacy and have just the partner present

Use a squatting position and use the squat bar for support Correct Explanation: The position is very important during labor. The woman needs to be in a position of comfort. Allowing the woman to assume the most comfortable position will facilitate natural childbirth. The Valsalva maneuver may result in dangerous increases in blood pressure, so be sure to instruct the mother to breathe as she pushes. The nurse should not intervene with who comes in or what family members are present unless she is asked, or unless the visitation is upsetting the mother.

A first-time mother informs the nurse that she is unable to breastfeed her newborn through the day as she is usually away at work. She adds that she wants to express her breast milk and store it for her newborn to have later. What instruction would be correct to offer the mother to ensure the safety of the stored expressed breast milk? a) Use microwave ovens to warm the chilled milk b) Refreeze any unused milk for later use if it has not been out more that 2 hours c) Use the sealed and chilled milk within 24 hours d) Use any frozen milk within 6 months of obtaining it

Use the sealed and chilled milk within 24 hours

A first-time mother informs the nurse that she is unable to breastfeed her newborn through the day as she is usually away at work. She adds that she wants to express her breast milk and store it for her newborn to have later. What instruction would be correct to offer the mother to ensure the safety of the stored expressed breast milk?

Use the sealed and chilled milk within 24 hours. The nurse should instruct the woman to use the sealed and chilled milk within 24 hours. The nurse should not instruct the woman to use frozen milk within 6 months of obtaining it, to use microwave ovens to warm chilled milk, or to refreeze the used milk and reuse it. Instead, the nurse should instruct the woman to use frozen milk within 3 months of obtaining it, to avoid using microwave ovens to warm chilled milk, and to discard any used milk and never refreeze it.

Which factor puts a client on her first postpartum day at risk for hemorrhage? a) Uterine atony b) Hemoglobin level of 12 g/dl c) Moderate amount of lochia rubra d) Thrombophlebitis

Uterine atony Correct Explanation: Loss of uterine tone places a client at higher risk for hemorrhage. Thrombophlebitis doesn't increase the risk of hemorrhage during the postpartum period. The hemoglobin level and lochia flow are within acceptable limits.

While caring for woman in labor the fetal heart monitor demonstrates late decelerations. The most common cause for their occurrence is: a) Cord compression b) Uteroplacental insufficiency c) Maternal fatigue d) Maternal hypotension

Uteroplacental insufficiency Explanation: Late decelerations are associated with uteroplacental insufficiency. They typically indicate decreased blood flow to the uterus during the contractions. Maternal hypotension and fatigue would not be observed on the fetal heart monitor. Cord compression would be marked by fetal tachycardia

A postpartum client who had a cesarean birth reports right calf pain to the nurse. The nurse observes that the client has nonpitting edema from her right knee to her foot. The nurse knows to prepare the client for which test first? a) Transthoracic echocardiogram b) Venogram of the right leg c) Venous duplex ultrasound of the right leg d) Noninvasive arterial studies of the right leg

Venous duplex ultrasound of the right leg Correct Explanation: Right calf pain and nonpitting edema may indicate deep vein thrombosis (DVT). Postpartum clients and clients who have had abdominal surgery are at increased risk for DVT. Venous duplex ultrasound is a noninvasive test that visualizes the veins and assesses blood flow patterns. A venogram is an invasive test that utilizes dye and radiation to create images of the veins and wouldn't be the first choice. Transthoracic echocardiography looks at cardiac structures and isn't indicated at this time. Right calf pain and edema are symptoms of venous outflow obstruction, not arterial insufficiency.

Medications of Newborn? What are the two primary given usually within the 1 hour of delivery? Why are they given? Where are they given/route?

Vitamin K & Erythromycin or Gentamycin eye ointment Vitamin K: is a fat soluble vitamin that promotes clotting by increasing the synthesis of prothromin by the liver. A deficiency of the vitamin delays clotting and might (remember the newborn liver is not mature yet) , it takes about a week for the newborn to produce enough blood to prevent vitamin K deficiency bleeding. So this is why it is given right after birth. Given IM vastis lateralis 0.5-1mg ( I remember 0.5 mg because the eye ointment is 0.5% and that way I can remember them both) Erytrhomycin or gentamycin eye ointment: eye prophylaxis. all newborns born in the US must receive this medication is mandated in 50 states within the first hour or two of birth. Prevents ophthalmia neonatorum which can be associated with neonatal blindness,. Given usually 0.5% inner to out canthus in conjutivae sac thin line.

A mother asks the nurse why her newborn is getting an Vitamin K injection in the delivery room. The nurse explains that the injection is necessary because: a) the mother was febrile at the time of delivery and prophylactic Vitamin K is necessary. b) Vitamin K aids in protein metabolism. Newborns have defective protein metabolism until 24 hours of life. c) Vitamin K is needed for coagulation, and the newborn does not produce vitamin K in the few days following birth. d) Newborns are prone to hypoglycemia and Vitamin K helps maintain a steady blod glucose level.

Vitamin K is needed for coagulation, and the newborn does not produce vitamin K in the few days following birth.

What ques do you ask during postpartum assessment?

When did she last use the bathroom? How long has the pad been on? Any bleeding? What was done for the bleeding? Past preg hx? Infections of placenta? Intact placenta?

Which of the following interventions would a nurse implement to best prevent heat loss in a 1 day of age newborn? a) Bathe and wash the newborn when temperature is 36.4 C (97.5F) b) Warm all surfaces and objects that come in contact with the newborn. c) Keep the newborn under the radiant heater when not with mom. d) Cover the newborn with several blankets while under the warmer.

Warm all surfaces and objects that come in contact with the newborn. Explanation: The infant will have regulated the body temperature at this point in life. Interventions to prevent heat loss are the best way to prevent heat loss for this newborn. Keeping the newborn under a radiant heater and covering the newborn with several blankets while under the warmer could lead to hyperthermia which can be just as detrimental to the newborn as hypothermia. Infants are bathed when their temperatures are stable.

A postpartal woman asks you about perineal care. Which of the following recommendations would you give? a) Wash her perineum with her daily shower. b) Use an alcohol wipe to wash her suture line. c) Avoid using soap in her perineal care. d) Refrain from washing lochia from the suture line.

Wash her perineum with her daily shower. Explanation: A suture line should be kept free of lochia to discourage infection. Washing with soap and water at the time of a shower will help to do this.

The nurse is providing education to a mother who is going to bottle feed her infant. What information will the nurse provide to this mom regarding breast care? a) Wear a tight, supportive bra b) Massage the breast when they are painful c) Express small amounts of milk when they are too full d) Run warm water over the breast in the shower

Wear a tight, supportive bra Explanation: The patient trying to dry up her milk supply should do as little stimulation to the breast as possible. She needs to wear a tight, supportive bra and use ice. Running warm water over the breasts in the shower will only stimulate the secretion, and therefore the production, of milk. Massaging the breasts will stimulate them to expel the milk and therefore produce more milk, as will expressing small amounts of milk when the breasts are full.

The nurse is providing education to a mother who is going to bottle feed her infant. What information will the nurse provide to this mom regarding breast care?

Wear a tight, supportive bra. The client trying to dry up her milk supply should do as little stimulation to the breast as possible. She needs to wear a tight, supportive bra and use ice. Running warm water over the breasts in the shower will only stimulate the secretion, and therefore the production, of milk. Massaging the breasts will stimulate them to expel the milk and therefore produce more milk, as will expressing small amounts of milk when the breasts are full.

A nurse is caring for a non-breastfeeding client in the postpartum period. The client complains of engorgement. What suggestion should the nurse provide to alleviate breast discomfort? a) Apply warm compress b) Express milk frequently c) Apply hydrogel dressing d) Wear a well-fitting bra

Wear a well-fitting bra Correct Explanation: The nurse should suggest the client wear a well-fitting bra to provide support and help alleviate breast discomfort. Application of warm compress and expressing milk frequently is suggested to alleviate breast engorgement in breastfeeding clients. Hydrogel dressings are used prophylactically in treating nipple pain.

A nurse is caring for a nonbreastfeeding client in the postpartum period. The client reports engorgement. What suggestion should the nurse provide to alleviate breast discomfort?

Wear a well-fitting bra. The nurse should suggest the client wear a well-fitting bra to provide support and help alleviate breast discomfort. Application of warm compresses and expressing milk frequently is suggested to alleviate breast engorgement in breastfeeding clients. Hydrogel dressings are used prophylactically in treating nipple pain.

Newborn care: Bathing

Wear gloves, remember if f first bath has dried secretions such as blood on it. Plain water to face and eyelids, and mild soap to the other Prevent hypothermia, don't bath a cool baby, immediately wrap or skin-to-skin, or infant warmer after the bath if skin to skin can lay a blanket over baby as well.

A newborn has been taken to the nursery after birth. He has been cleaned in the labor and birth suite and is swaddled in a blanket. The nurse is going to check his pulse. What must the nurse do?

Wear gloves. Infection control is a priority nursing intervention. Gloves need to be worn when in contact with the infant who has not been bathed after birth. All options are valid options; however, a three-minute surgical scrub is generally only required at the beginning of a shift. The nurse should always wash the hands before putting on gloves to care for an infant and after taking gloves off. Standard precautions are used with every client.

Newborn Isaac has been taken to the nursery after delivery. He has been cleaned in the labor and delivery suite and swaddled in a blanket. The nurse is going to check his pulse. What must the nurse do? a) Perform a 3-minute surgical type scrub before touching him. b) Use infection transmission precautions. c) Clean his or her hands with a betadine scrub. d) Wear gloves.

Wear gloves. Correct Explanation: Infection control is a priority nursing intervention. Gloves need to be worn when in contact with the infant who has not been bathed after delivery. All options are valid options. However, a three-minute surgical scrub is generally only required at the beginning of a shift. You should always wash your hands before putting on gloves to care for an infant and after taking your gloves off. Standard precautions are used with every patient.

Which measurements were most likely obtained from a normal newborn delivered at 38 weeks to a healthy mother with no maternal complications? a) Weight = 2000 g, length = 17 inches, head circumference = 32 cm, and chest circumference = 30 b) Weight = 2500 g, length = 18 inches, head circumference = 32 cm, and chest circumference = 30 cm c) Weight = 3500 g, length = 20 inches, head circumference = 34 cm, and chest circumference = 32 cm d) Weight = 4500 g, length = 22 inches, head circumference = 36 cm, and chest circumference = 34 cm

Weight = 3500 g, length = 20 inches, head circumference = 34 cm, and chest circumference = 32 cm

The nurse caring for a newborn has to perform assessment at various intervals. When should the nurse complete the second assessment for the newborn? a) 24 hours after the newborn's birth b) Prior to the newborn being discharged c) Within 30 minutes after birth, in the birthing area d) Within the first 2 to 4 hours, when the newborn reaches the nursery

Within the first 2 to 4 hours, when the newborn reaches the nursery

The pain of labor is influenced by many factors. What is one of these factors? a) Woman has a high threshold for pain b) Woman has a high tolerance for pain c) Woman has lots of visitors during labor d) Woman's preparation for labor and delivery

Woman's preparation for labor and delivery Correct Explanation: The woman who enters labor with realistic expectations usually copes well and reports a more satisfying labor experience than does a woman who is not as well prepared.

There has been much research done on pain and the perception of pain. What is the result of research done on levels of satisfaction with the control of labor pain? a) Women report higher levels of satisfaction when regional anesthetics are used to control pain b) Women report higher levels of satisfaction when they felt they had a high degree of control over the pain experience c) Women report higher levels of satisfaction when different types of relaxation techniques are used to control pain d) Women report higher levels of satisfaction when the physician makes the decision on what type of pain control to use

Women report higher levels of satisfaction when they felt they had a high degree of control over the pain experience Correct Explanation: Research has shown that women report higher levels of satisfaction with their labor experience when they feel a high degree of control over the experience of pain (Stuebe & Barbieri, 2005).

What is the primary role of the nurse in pain management for a nurse working with labor patients? a) Provide any medication the patient request b) Monitor the patient for active labor and suggest she have an epidural to have increased satisfaction with her delivery experience c) Dictate the pain management during labor for the best outcome d) Work with the labor patient to plan pain management options

Work with the labor patient to plan pain management options Correct Explanation: The role of the nurse is to work with the patient and plan with the nurse the pain management technique desired by the patient to meet the level of expectation for the patient. Pain is subjective and each woman has a right to her own labor plan.

A newborn is born and the nurse realizes that the infant is at risk for evaporative heat loss. Which intervention would best prevent this from occurring? a) Wrap the infant in a warm, dry blanket b) Place the infant in the mother's abdomen after delivery c) Turn the delivery room temperature up. d) Bathe the infant immediately after birth

Wrap the infant in a warm, dry blanket

A nurse is assigned to care for a newborn with an elevated bilirubin level. Which symptom would the nurse expect to find during the infant's physical assessment? a) Yellow sclera b) Abdominal distension c) Heart rate of 130 bpm d) Respiratory rate of 24

Yellow sclera

A nurse is assigned to care for a newborn with an elevated bilirubin level. Which symptom would the nurse expect to find during the infant's physical assessment? a) Yellow sclera b) Heart rate of 130 bpm c) Respiratory rate of 24 d) Abdominal distension

Yellow sclera

The nurse describes the changes in stool that a new mother would see when feeding her newborn formula. Which of the following best indicates what the mother would observe after several days? a) Yellow-green, pasty, unpleasant-smelling stool b) Greenish, tarry, thick black stool c) Sour-smelling, yellowish-gold stool d) Thin, yellowish, seedy brown stool

Yellow-green, pasty, unpleasant-smelling stool

Bonding between a mother and her infant can be defined how?

a process of developing an attachment and becoming acquainted with each other Bonding in the maternal-newborn world is the attachment process that occurs between a mother and her newborn infant. This is how the mother and infant become engaged with each other and is the foundation for the relationship. Bonding is a process and not a single event. The process of bonding is not a year-long process, and the family growing closer together after the birth of a new baby is not bonding.

Within the first hour after birth, the nurse would expect to find the woman's fundus: a) At the level of the umbilicus b) One fingerbreadth below the umbilicus c) Between the umbilicus and symphysis pubis d) 2 cm above the umbilicus

a) At the level of the umbilicus Rationale: After birth, the fundus is located midline between the umbilicus and symphysis pubis but then slowly rises to the level of the umbilicus during the first hour after birth. Then the uterus contracts, approximately 1 cm (or fingerbreadth) each day after birth.

A fetus is in the LST position. The nurse interprets this as indicating which of the following as the presenting part? a) Buttocks b) Fetal head c) Shoulder d) Chin

a) Buttocks Rationale: The letter "S" indicates the sacrum or buttocks as the presenting part. The fetal head would be noted by the letter "O," indicating occiput. The fetal chin would be noted by the letter "M," indicating mentum. The fetal shoulder would be noted by the letter "A," indicating the acromion process.

A petite, 5-foot tall, 95-pound woman who is 28 years old is about to deliver her first child and would like to have a vaginal delivery. She has two sisters, both of whom have given birth vaginally. She has gained 25 pounds during a normal, uneventful pregnancy. What type of pelvis would a nurse expect this woman to have upon assessment of the patient? a) Cannot be determined b) Android c) Platypelloid d) Gynecoid

a) Cannot be determined Rationale: Pelvis shape cannot be determined by the information included in the statement. Early in the pregnancy, particularly if a woman has never delivered a baby vaginally, the practitioner may take pelvic measurements to estimate the size of the true pelvis. This helps to determine if the size is adequate for vaginal delivery. However, these measurements do not consistently predict which women will have difficulty delivering vaginally, so most practitioners allow the woman to labor and attempt a vaginal birth.

On examination, the hands and feet of a 12-hour-old infant are cyanotic without other signs of distress. The nurse should document this as:

acrocyanosis. Acrocyanosis is a blue tint to the hands and feet of newborns during the first few days of life. Acrocyanosis is a normal finding and is not indicative of a potential for respiratory distress, poor oxygenation, or cold stress.

A client in labor is agitated and nervous about the birth of her child. The nurse explains to the client that fear and anxiety cause the release of certain compounds which can prolong labor. Which of the following is the nurse referring to in the explanation? a) Catecholamines b) Relaxin c) Prostaglandins d) Oxytocin

a) Catecholamines Rationale: Fear and anxiety cause the release of catecholamines, such as norepinephrine and epinephrine which stimulate the adrenergic receptors of the myometrium. This in turn interferes with effective uterine contractions and results in prolonged labor. Estrogen promotes the release of prostaglandins and oxytocin. Relaxin is a hormone that is involved in producing backache by acting on the pelvic joints. Prostaglandins, oxytocin and relaxin are not produced due to fear or anxiety in clients during labor.

The nurse notes that the fetal head is at the vaginal opening and does not regress between contractions. The nurse interprets this finding as which of the following? a) Crowning b) Engagement c) Descent d) Restitution

a) Crowning Rationale: Crowning occurs when the top of the fetal head appears at the vaginal orifice and no longer regresses between contractions. Engagement occurs when the greatest transverse diameter of the head passes through the pelvic inlet. Descent is the downward movement of the fetal head until it is within the pelvic inlet. Restitution or external rotation occurs after the head is born and free of resistance. It untwists, causing the occiput to move about 45 degrees back to its original left or right position.

The initial descent of the fetus into the pelvis to zero station is which one of the cardinal movements of labor? a) Engagement b) Felxion c) Extension d) Expulsion

a) Engagement Rationale: The movement of the fetus into the pelvis from the upper uterus is engagement. This is the first cardinal movement of the fetus in preparation for the spontaneous vaginal delivery. Flexion occurs as the fetus encounters resistance from the soft tissues and muscles of the pelvic floor. Extension is the state in which the fetal head is well flexed with the chin on the chest as the fetus travels through the birth canal. Expulsion occurs after delivery of the anterior and posterior shoulders.

Fetal circulation can be affected by many factors during labor. Accurate assessment of the laboring woman and fetus requires knowledge of these expected adaptations. Which factor will not affect fetal circulation during labor? a) Fetal position b) Uterine contractions c) Blood pressure d) Umbilical cord blood flow

a) Fetal position

With regard to primary and secondary powers, the maternity nurse should understand that: a) Primary powers are responsible for effacement and dilation of the cervix b) Effacement generally is well ahead of dilation in women giving birth for the first time; they are more together in subsequent pregnancies c) Scarring of the cervix caused by a previous infection or surgery may make the delivery a bit more painful, but it should not slow or inhibit dilation d) Pushing in the second stage of labor is more effective if the woman can breathe deeply and control some of her involuntary needs to push, as the nurse directs

a) Primary powers are responsible for effacement and dilation of the cervix

When teaching a group of nursing students about uterine contractions, which of the following would the instructor include as a typical feature? a) Retracting of the lower segment b) Contracting stronger in the lower segment c) Thinning out of the upper segment d) Pulling down of the cervical portion

a) Retracting of the lower segment Rationale: The nurse should identify retraction of the lower segment of the uterus as a feature of typical uterine contractions. As labor progresses the uterine contractions become stronger. The upper segment of the uterus contracts more actively than the lower segment. The lower segment retracts, pulling up the cervix. The upper segment thickens with time and the lower segment thins.

When teaching a group of nursing students about uterine contractions, which of the following would the instructor include as a typical feature? a) Retracting of the lower segment b) Pulling down of the cervical portion c) Thinning out of the upper segment d) Contracting stronger in the lower segment

a) Retracting of the lower segment Rationale: The nurse should identify retraction of the lower segment of the uterus as a feature of typical uterine contractions. As labor progresses the uterine contractions become stronger. The upper segment of the uterus contracts more actively than the lower segment. The lower segment retracts, pulling up the cervix. The upper segment thickens with time and the lower segment thins.

A nurse is teaching a group of nursing students about the role of progesterone in labor. Which of the following should the nurse explain as the function of progesterone? a) Suppresses the uterine irritability throughout pregnancy b) Promotes oxytocin production from the posterior pituitary c) Sensitizes the uterus to effects of oxytocin on the myometrium d) Stimulates smooth muscle contraction in the uterus

a) Suppresses the uterine irritability throughout pregnancy Rationale: The function of progesterone is to suppress uterine irritability throughout pregnancy. The function of estrogen is to promote oxytocin production and to sensitize the uterus to the effects of oxytocin. Prostaglandin, and not progesterone, stimulates the smooth muscle contractions in the uterus.

The woman has just delivered a healthy baby boy, but the placenta has not yet delivered. What stage of labor does this scenario represent? a) Third b) Second c) Fourth d) First

a) Third Rationale: Stage 3 begins with the birth of the baby and ends with delivery of the placenta.

A nurse is assisting a client who is in the first stage of labor. Which of the following principles should the nurse keep in mind to help make this client's labor and birth as natural as possible? a) Women should be able to move about freely throughout labor b) The support person's access to the client should be limited to prevent the client from becoming overwhelmed c) Routine intravenous fluid should be implemented d) A woman should be allowed to assume a supine position

a) Women should be able to move about freely throughout labor Rationale: Six major concepts that make labor and birth as natural as possible are as follows: 1) labor should begin on its own, not be artificially induced; 2) women should be able to move about freely throughout labor, not be confined to bed; 3) women should receive continuous support from a caring other during labor; 4) no interventions such as intravenous fluid should be used routinely; 5) women should be allowed to assume a nonsupine position such as upright and side-lying for birth; and 6) mother and baby should be housed together after the birth, with unlimited opportunity for breast-feeding.

The nurse is assessing a laboring client and notes: 5 cm dilated, 80% effaced, zero station, contractions every 2 to 3 minutes, lasting 50 seconds, becoming increasingly uncomfortable, and apprehensive but appropriate and focused on breathing and relaxation. The nurse determines which nursing diagnosis is most appropriate for this client?

acute pain related to uterine contractions

The nurse caring for a client in preterm labor observes abnormal fetal heart rate (FHR) patterns. Which nursing intervention should the nurse perform next?

administration of O2 mask

A nurse who has worked in a nursery for 15 years informs the nursing student that feeding an infant early has advantages and describes the biggest advantage as which of the following? a) allows the mother to see if the baby can tolerate formula b) helps to ease the baby's hunger c) allows the baby to sleep longer d) allows the baby to pass stools, which helps to reduce bilirubin

allows the baby to pass stools, which helps to reduce bilirubin

A nurse who has worked in a nursery for 15 years informs the nursing student that feeding an infant early has advantages and describes the biggest advantage as which of the following? a) allows the baby to sleep longer b) allows the baby to pass stools, which helps to reduce bilirubin c) allows the mother to see if the baby can tolerate formula d) helps to ease the baby's hunger

allows the baby to pass stools, which helps to reduce bilirubin Correct Explanation: Newborns fed early pass stools sooner, which helps to reduce bilirubin. The other options might be helpful but are not the most important reason for feeding a newborn early.

A client is in the first stage of labor, latent phase. Her membranes are intact, and her contractions are mild. Considering the client's condition and phase of labor, the nurse knows that which aid will facilitate labor?

ambulation ad lib To facilitate the first stage of labor, ambulation and movement will allow better fetal descent and help to speed the labor process. Bed rest will slow or stop the labor process. The client may use the bathroom as needed, but this does not affect labor rate. The client should remain mobile.

Which newborn neuromuscular system adaptation would the nurse not expect to find?

an extrusion reflex at 9 months of age An extrusion reflex usually disappears around 4 months of age. A positive Babinski reflex can be seen until 3 months of age. The plantar grasp disappears around 8 to 9 months of age. The Moro reflex disappears around 4 to 5 months of age.

A primigravida client has just arrived in early labor and is showing signs of extreme anxiety over the birthing process. Why should the nurse prioritize helping the client relax?

anxiety can slow down labor and decrease oxygen to the fetus.

Which intervention would be helpful to a bottle-feeding client who's experiencing hard or engorged breasts?

applying ice Ice promotes comfort by decreasing blood flow (vasoconstriction), numbing the area, and discouraging further letdown of milk. Restricting fluids does not reduce engorgement and should not be encouraged. Warm compresses will promote blood flow and hence, milk production, worsening the problem of engorgement. Bromocriptine has been removed from the market for lactation suppression.

A nurse is assessing a newborn's gestational age, When determining neuromuscular maturity, which parameters would the nurse assess? Select all that apply.

arm recoil scarf sign Arm recoil and the scarf sign are used to evaluate neuromuscular maturity. Physical maturity indicators include skin, lanugo, plantar surface, breast, eye-ear, and genitals

During an admission assessment of a client in labor, the nurse observes that there is no vaginal bleeding yet. What nursing intervention is appropriate in the absence of vaginal bleeding when the client is in the early stage of labor?

assess amount of cervical dilation

When do you do the APGAR testing?

at 1 minute and 5 minute after birth

A newborn is discharged from the hospital before undergoing metabolic screening. A community health nurse scheduling a follow-up home visit knows that the most appropriate time to perform the heel stick is:

at least 24 hours after birth. This screening needs to be done on an infant 24 hours to 7 days after birth. The timing of the screening test is determined by the age of the infant, not the number of feedings.

A nurse finds the uterus of a postpartum woman to be boggy and somewhat relaxed. This a sign of which condition?

atony The uterus in a postpartum client should be midline and firm. A boggy or relaxed uterus signifies uterine atony, which can predispose the woman to hemorrhage.

A nurse finds the uterus of a postpartum woman to be boggy and somewhat relaxed. This a sign of which of the following? a) normal involution b) infection c) atony d) hemorrhage

atony Correct Explanation: The uterus in a postpartum patient should be midline and firm. A boggy or relaxed uterus signifies uterine atony, which can predispose the woman to hemorrhage.

A new mother, who is an adolescent, was cautious at first when holding and touching her newborn. She seemed almost afraid to make contact with baby and only touched it lightly and briefly. However, 48 hours after the birth, the nurse now notices that the new mother is pressing the newborn's cheek against her own and kissing her on the forehead. The nurse recognizes these actions as which behavior?

attachment When a woman has successfully linked with her newborn it is termed attachment or bonding. Although a woman carried the child inside her for 9 months, she often approaches her newborn not as someone she loves but more as she would approach a stranger. The first time she holds the infant, she may touch only the blanket. Gradually, as a woman holds her child more, she begins to express more warmth, touching the child with the palm of her hand rather than with her fingertips. She smoothes the baby's hair, brushes a cheek, plays with toes, and lets the baby's fingers clasp hers. Soon, she feels comfortable enough to press her cheek against the baby's or kiss the infant's nose; she has successfully bonded or become a mother tending to her child. Engrossment describes the action of new fathers when they stare at their newborn for long intervals. Involution is the process whereby the reproductive organs return to their nonpregnant state. Engorgement is the tension in the breasts as they begin to fill with milk.

A new mother tells the nurse at the baby's 3 month check-up, "When she cries, it seems like I am the only one who can calm her down." This is an example of which behavior?

attachment Attachment is the development of strong affection between an infant and a significant other. It does not occur overnight. It occurs through mutually satisfying experiences. Attachment behaviors include seeking, staying close to, and exchanging gratifying experiences with the infant. Bonding is the close emotional attraction to a newborn by the parents that develops in the first 30 to 60 minutes after birth. This is not an example of being spoiled.

A new mother tells the nurse at the baby's 3 month check-up, "When she cries, it seems like I am the only one who can calm her down." This is an example of which of the following? a) bonding b) attachment c) being spoiled d) none of the above

attachment Correct Explanation: Attachment is the development of strong affection between an infant and a significant other. It does not occur overnight. It occurs through mutually satisfying experiences. Attachment behaviors include seeking, staying close to, and exchanging gratifying experiences with the infant. Bonding is the close emotional attraction to a newborn by the parents that develops in the first 30 to 60 minutes after birth. This is not an example of being spoiled.

The client and her partner have prepared for a natural birth and bring a picture of a sunset over the ocean with them. The nurse predicts they will be using which technique during labor?

attention focusing

Assessment reveals that a woman's cervix is approximately 1 cm in length. The nurse would document this as: a) 100% effaced b) 50% effaced c) 0% effaced d) 75% effaced

b) 50% effaced Rationale: A cervix 1 cm in length is described as 50% effaced. A cervix that measures approximately 2 cm in length is described as 0% effaced. A cervix 1/2 cm in length would be described as 75% effaced. A cervix 0 cm in length would be described as 100% effaced.

The first stage of labor is often a time of introspection. In light of this, which of the following would guide your planning of nursing care? a) A woman should be left entirely alone during this period. b) A woman may spend time thinking about what is happening to her. c) No nursing care is needed to be done during this time. d) A woman will rarely speak or laugh during this period.

b) A woman may spend time thinking about what is happening to her. Rationale: Women need a support person with them during all stages of labor.

Nurses can help their clients by keeping them informed about the distinctive stages of labor. What description of the phases of the first stage of labor is accurate? a) Latent: milk, regular contractions; no dilation; blood show; duration of 2 to 4 hours b) Active: moderate, regular contractions; 4 to 7cm dilation; duration of 3 to 6 hours c) Lull: no contractions; dilation stable; duration of 20 to 60 minutes d) Transition: very strong but irregular contractions; 8 to 10cm dilation; duration of 1 to 2 hours

b) Active: moderate, regular contractions; 4 to 7cm dilation; duration of 3 to 6 hours

There are four essential components of labor. The first is the passageway. It is composed of the bony pelvis and soft tissues. What is one component of the passageway? a) Perineum b) Cervix c) False pelvis d) Uterus

b) Cervix Rationale: The cervix and vagina are soft tissues that form the part of the passageway known as the birth canal.

The nurse is measuring a contraction from the beginning of the increment to the end of the decrement for the same contraction, documenting this as which of the following? a) Peak b) Duration c) Frequency d) Intensity

b) Duration Rationale: Duration refers to how long a contraction lasts and is measured from the beginning of the increment to the end of the decrement for the same contraction. Intensity refers to the strength of the contraction determined by manual palpation or measured by an internal intrauterine catheter. Frequency refers to how often contractions occur and is measured from the increment of one contraction to the increment of the next contraction. The peak or acme of a contraction is the highest intensity of a contraction.

All statements about normal labor are true except: a) A single fetus presents by vertex b) It is completed within 8 hours c) A regular progression of contractions, effacement, dilation, and descent occurs d) No complications are involved

b) It is completed within 8 hours

The nurse assesses a client in labor and finds that the fetal long axis is longitudinal to the maternal long axis. The nurse documents this finding as which of the following? a) Attitude b) Lie c) Position d) Presentation

b) Lie Rationale: The nurse is assessing fetal lie, the relationship of the fetal long axis to the maternal long axis. When the fetal long axis is longitudinal to the maternal long axis, the lie is said to longitudinal. Presentation is the portion of the fetus that overlies the maternal pelvic inlet. Attitude is the relationship of the different fetal parts to one another. Position is the relationship of the fetal denominator to the different sides of the maternal pelvis.

A 32-year-old woman presents to the labor-and-delivery suite in active labor. She is multigravida, relaxed, and talking with her husband. When examined by the nurse, the fetus is found to be in a cephalic presentation. His occiput is facing toward the front and slightly to the right of the mother's pelvis, and he is exhibiting a flexed attitude. How does the nurse document the position of the fetus? a) LOA b) ROA c) LOP d) ROP

b) ROA Rationale: Document the fetal position in the clinical record using abbreviations (Box 8-1). The first letter describes the side of the maternal pelvis toward which the presenting part is facing ("R" for right and "L" for left). The second letter or abbreviation indicates the reference point ("O" for occiput, "Fr" for frontum, etc.). The last part of the designation specifies whether the presenting part is facing the anterior (A) or the posterior (P) portion of the pelvis, or whether it is in a transverse (T) position.

A nurse is performing a vaginal examination of a woman in the early stages of labor. The woman has been at 2 cm dilated for the past 2 hours, but effacement has progressed steadily. Which of the following should the nurse do to best encourage the client regarding her progress? a) Don't mention anything to the client yet; wait for further dilatation to occur. b) Say, "you are still 2 cm dilated, but the cervix is thinning out nicely." c) Say, "you haven't dilated any further, but hang in there; it will happen eventually." d) Say, "there has been no further dilatation; effacement is progressing."

b) Say, "you are still 2 cm dilated, but the cervix is thinning out nicely." Rationale: Women are anxious to have frequent reports during labor, to reassure them everything is progressing well. If giving a progress report, remember most women are aware of the word dilatation but not effacement. Just saying, "no further dilatation", therefore, is a depressing report. "You're not dilated a lot more, but a lot of thinning is happening and that's just as important" is the same report given in a positive manner.

Concerning the third stage of labor, nurses should be aware that: a) The placenta eventually detaches itself from a flaccid uterus b) The duration of the third stage may be as short as 3 to 5 minutes c) It is important that the dark, roughened maternal surface of the placenta appear before the shiny fetal surface d) The major risk for women during the third stage is a rapid heart rate

b) The duration of the third stage may be as short as 3 to 5 minutes

The nurse determines that the fetal heart rate averages approximately 140 beats per minute over a 10-minute period. The nurse identifies this as:

baseline FHR. The baseline FHR averages 110 to 160 beats per minute over a 10-minute period. Fetal bradycardia occurs when the FHR is less than 110 beats per minute for 10 minutes or longer. Short-term variability is the beat-to-beat change in FHR. Baseline variability refers to the normal physiologic variations in the time intervals that elapse between each fetal heartbeat observed along the baseline in the absence of contractions, decelerations, and accelerations.

On assessment of a 2-day postpartum client the nurse finds that the fundus is boggy, at the umbilicus, and slightly to the right. What is the most likely cause of this assessment finding?

bladder distention The most often cause of a displaced uterus is a distended bladder. Ask the client to void and then reassess the uterus. According to the scenario described, the most likely cause of the uterine findings would not be uterine atony. A full bowel or poor bladder tone would not cause a boggy and displaced fundus.

A nurse is monitoring the vital signs of a client 24 hours after birth. She notes that the client's blood pressure is 100/60 mm Hg. Which postpartum complication should the nurse most suspect in this client, based on this finding?

bleeding Blood pressure should also be monitored carefully during the postpartal period because a decrease in this can also indicate bleeding. In contrast, an elevation above 140 mm Hg systolic or 90 mm Hg diastolic may indicate the development of postpartal gestational hypertension, an unusual but serious complication of the puerperium. An infection would best be indicated by an elevated oral temperature. Diabetes would be indicated by an elevated blood glucose level.

Which vital sign is not routinely assessed in a term, healthy newborn with an Apgar score of 9?

blood pressure Because the readings can be inaccurate, blood pressure is not routinely assessed in term, normal healthy newborns with normal Apgar scores. It is assessed if there is a clinical indication such as suspected blood loss or low Apgar scores. Pain is assessed by objective signs of pain such as grimacing and crying in response to certain stimuli.

A woman has just given birth to a baby. Her prelabor vital signs were temperature: 98.8° F (37.1° C); blood pressure: 120/70 mm Hg; pulse; 80 beats/min. and respirations: 20 breaths/min. Which combination of findings during the early postpartum period are the most concerning?

blood pressure 90/50 mm Hg, pulse 120 beats/min, respirations 24 breaths/min. The decrease in BP with an increase in HR and RR indicate a potential significant complication and are out of the range of normals from birth and need to be reported immediately. Shaking chills with a temperature of 100.3º F (37.9º C) can occur due to stress on the body and is considered a normal finding. A fever of 100.4º F (38º C) should be reported. The other options are considered to be within normal limits after giving birth to a baby.

The nurse is admitting a 10-pound (4.5-kg) newborn to the nursery. What is important for the nurse to monitor during the transition period?

blood sugar Most facilities have protocols to guide nursing care in the treatment of hypoglycemia. Many pediatricians have preprinted orders that can be initiated if the glucose level falls below a predetermined level (usually 40-50 mg/dL).

A nurse teaches new parents that the best way to help prevent infections in the newborn is which of the following? a) keep them inside for the first month of life b) limit visitors c) breastfeed d) keep them warm at all times

breastfeed

A nurse teaches new parents that the best way to help prevent infections in the newborn is which of the following? a) breastfeed b) keep them inside for the first month of life c) keep them warm at all times d) limit visitors

breastfeed Correct Explanation: A major source of IgA, which helps in immunity, is human breast milk. Thus, breastfeeding is believed to have significant immunological advantages over formula. The other options such as keeping them in for a month and keeping them warm will not help prevent infections. Keeping the child away from people who have an infection might stop them from getting that infection. Doing so will not help build up the infant's immunity

As a part of the newborn assessment, the nurse examines the infant's skin. Which nursing observation would warrant further investigation?

bright red, raised bumpy area noted above the right eye A red bumpy area noted above the right eye is a hemangioma and needs further investigation to determine whether the hemangioma could interfere with the infant's vision. They may grow larger during the first year then fade and usually disappear by age 9. Stork bites or salmon patches and blue or purple splotches on buttocks (Mongolian spots) are common skin variations and are not concerning. Erythema toxicum, seen as a fine red rash over the chest and back, is also a normal skin variant that will disappear withn a few days.

A newborn's primary method of heat production is through nonshivering thermogenesis. This process oxidizes which of the following in response to cold exposure? a) brown fat b) muscles c) nerves d) white fat

brown fat

A woman is documented on the labor and delivery board to be 7cm dilated. Her family wants to know how long she will be in labor. The nurse should provide which information to the family? a) "She is doing well, in the second stage and it could be anytime now." b) "She is in the transition phase of labor and it will be with in 2 to 3 hours, might be sooner." c) "She is in active labor, she is progressing at this point and we will keep you posted." d) "She is still in early latent labor and has much too long to go to tell when she will deliver."

c) "She is in active labor, she is progressing at this point and we will keep you posted." Rationale: At 7cm dilated, she is considered in Active Phase of labor. There is no science that can predict the length of labor. She is progressing in labor and it is not best to give the family a specific time frame.

A fetus is assessed at 2 cm above the ischial spines. The nurse would document fetal station as: a) +2 b) 0 c) -2 d) +4

c) -2 Rationale: When the presenting part is above the ischial spines, it is noted as a negative station. Since the measurement is 2 cm, the station would be -2. A 0 station indicates that the fetal presenting part is at the level of the ischial spines. Positive stations indicate that the presenting part is below the level of the ischial spines.

The nurse should initially implement which intervention when a nulliparous woman telephones the hospital to report that she is in labor. a) Emphasize that food and fluid should stop or be light b) Tell the woman to stay home until her membranes rupture c) Ask the woman to describe why she believes that she is in labor d) Arrange for the woman to come to the hospital for labor evaluation

c) Ask the woman to describe why she believes that she is in labor Rationale: The nurse needs further information to assist in determining if the woman is in true or false labor. She will need to ask the patient questions to seek further assessment and triage information. Having her wait until membranes rupture may be dangerous, as she may give birth before reaching the hospital. She should continue fluid intake until it is determined whether or not she is in labor. She may be in false labor, and more information should be obtained before she is brought to the hospital.

Which of the following would alert the nurse that the client is in the transition phase of labor? a) Reduction of rectal pressure b) Decrease in the bloody show c) Beginning urge to bear down d) Enthusiasm in the client

c) Beginning urge to bear down Rationale: Starting of the urge to bear down is a feature associated with the transition phase of labor. The transition phase is the last phase of the first stage of labor. In this phase the process of cervical dilatation is completed. During this phase the client experiences an increase in rectal pressure, an increase in the bloody show and an urge to bear down. The contractions are stronger and hence the client feels irritable, restless and nauseous. The client feels enthusiastic during the latent phase and not the transition phase.

Many clients experience a slight fever after birth especially during the first 24 hours. To what should the nurse attribute this elevated temperature?

dehydration Many women experience a slight fever (100.4° F [38° C]) during the first 24 hours after birth. This results from dehydration because of fluid loss during labor. With the replacement of fluids the temperature should return to normal after 24 hours.

Braxton Hicks contractions are termed "practice contractions" and occur throughout pregnancy. When the woman's body is getting ready to go into labor, it begins to show anticipatory signs of impending labor. Among these signs are Braxton Hicks contractions that are more frequent and stronger in intensity. What differentiates Braxton Hicks contractions from true labor? a) Braxton Hicks contractions get closer together with activity b) Braxton Hicks contractions cause "ripening" of the cervix. c) Braxton Hicks contractions usually decrease in intensity with walking d) Braxton Hicks contractions do not last long enough to be true labor

c) Braxton Hicks contractions usually decrease in intensity with walking Rationale: Braxton Hicks contractions occur more frequently and are more noticeable as pregnancy approaches term. These irregular, practice contractions usually decrease in intensity with walking and position changes.

There are four essential components of labor. The first is the passageway. It is composed of the bony pelvis and soft tissues. What is one component of the passageway? a) Uterus b) False pelvis c) Cervix d) Perineum

c) Cervix Rationale: The cervix and vagina are soft tissues that form the part of the passageway known as the birth canal.

If a fetus is in an ROA position during labor, you would interpret this to mean the fetus is a) Presenting with the face as the presenting part b) In a common breech delivery position c) In a longitudinal lie facing the left posterior d) Facing the right anterior abdominal quadrant

c) In a longitudinal lie facing the left posterior Rationale: ROA (right occiput anterior) means the occiput of the fetal head points toward the mother's right anterior pelvis; the head is the presenting part.

The skull is the most important factor in relation to the labor and birth process. The fetal skull must be small enough to travel through the bony pelvis. What feature of the fetal skull helps to make this passage possible? a) Vertex presentation b) Caput succedaneum c) Molding d) Cephalohematoma

c) Molding Rationale: The cartilage between the bones allows the bones to overlap during labor, a process called molding that elongates the fetal skull thereby reducing the diameter of the head.

The four essential components of labor are known as the "four P's". Which of the four P's involves the pelvis? a) Psyche b) Powers c) Passageway d) Passenger

c) Passageway Rationale: The passageway is one of the 4 P's and involves the pelvis, both bony pelvis and the soft tissues, cervix, and vagina. The "passenger" refers to the fetus. The primary powers are the involuntary contractions of the uterus, whereas the secondary powers come from the maternal abdominal muscles. The psyche refers to the mother's mental state.

When educating a group of nursing students about the theories of onset of labor, the nurse identifies which of the following factors as the possible causes for onset of labor? Select all that apply. a) Increase in the production of progesterone b) Fall in the estrogen at 34-35 weeks of pregnancy c) Prostaglandin production in the myometrium d) Increase in the fetal cortisol levels e) Release of oxytocin by the pituitary

c) Prostaglandin production in the myometrium d) Increase in the fetal cortisol levels e) Release of oxytocin by the pituitary Rationale: The possible causes for the onset of labor include increase in the fetal cortisol levels, release of oxytocin by the posterior pituitary and the production of prostaglandins. Progesterone withdrawal, and not an increase, initiates labor. There is a rise in the estrogen levels at 34-35 weeks of pregnancy. Estrogen stimulates prostaglandin production and also promotes the release of oxytocin.

The nurse knows that the second stage of labor, the descent phase, has begun when: a) The amniotic membranes rupture b) The cervix cannot be felt during a vaginal examination c) The woman experiences a strong urge to bear down d) The presenting part is below the ischial spines

c) The woman experiences a strong urge to bear down

A post delivery CBC has noted an elevated WBC count of 22,000/mm3. Which rationale is accurate regarding the elevated WBC count? a) Abnormal finding and she needs antibiotics b) Might be a false result, recommend re-testing c) This is a normal variation due to stress of labor d) Occurs in patients who have cesarean birth, from the trauma of surgery

c) This is a normal variation due to stress of labor Rationale: An elevation of WBC up to 30,000mm/3 can be normal variation for any woman after delivery. This is related to the stress on her body from labor and delivery. Antibiotics are not indicated as this is a normal response to intense stress. The increase in WBC is not related to cesarean birth. Retesting would be wasteful as it is known that this is a normal response to any stress.

Forces of contractions, mild asphyxia, increased intracranial pressure, and cold stress all play a role in the newborn transition by releasing which of the following critical components? a) cortisol b) norepinephrine c) catecholamines d) epinephrine

catecholamines Correct Explanation: The physical forces of contractions at labor, mild asphyxia, increased intracranial pressure, and cold stress immediately experienced after birth lead to an increased release of catecholamines, which is critical for the changes involved in the transition to extrauterine life.

A nurse is caring for woman in labor. The woman's membranes just ruptured. The nurse assesses the characteristics of the fluid. Which finding would the nurse identify as normal?

clear

What is head circumference, chest circumference measured in?

cm (centimeters)

What is caput?

collection of serous fluid crosses the suture line because swelling is in sclap; normal , gone in days

While examining a newborn, a nurse observes salmon patches on the nape of the neck and on the eyelids. Which is the most likely cause of these skin abnormalities?

concentration of immature blood vessels A concentration of immature blood vessels causes salmon patches. Bruising does not look like salmon patches but would be more bluish-purple in appearance. Harlequin sign is a result of immature autoregulation of blood flow and is commonly seen in low birth weight newborns. An allergic reaction would be more generalized and would not be salmon colored.

A nurse is instructing a patient who is breastfeeding for the first time that before her milk comes in she should expect to see colostrum, which is best described as which of the following? a) milky white b) bluish white c) creamy yellow d) gray liquid

creamy yellow Correct Explanation: If a woman has any discharge from her nipples postpartum, it should be described and documented if it is not colostrum (creamy yellow) or foremilk (bluish white).

A mother who just given birth has difficulty sleeping despite her exhaustion from labor. What are the causes of this inability to rest? Select all that apply.

crying baby inability to get adequate pain relief frequent trips to the bathroom due to diuresis excess fatigue and overstimulation by visitors The period before labor and birth can be uncomfortable for the mother, thus preventing adequate rest and creating a sleep hunger. The early postpartum period involves many adjustments that can take a toll on the mother's sleep.

The patient is having a routine prenatal visit and asks the nurse what the childbirth education teacher meant when she used the term zero station. What is the best response by the nurse? a) "This is just a way of determining your progress in labor." b) This indicates that you start labor within the next 24 hours." c) "This means +1 and the baby is entering the true pelvis." d) "The presenting part is at the true pelvis and is engaged."

d) "The presenting part is at the true pelvis and is engaged." Rationale: Zero station is when the fetus is engaged in the pelvis, or has dropped. This is an encouraging sign for the patient. This sign is indicative that labor may be beginning, but there is no set time frame regarding when it will start. Labor has not started yet, and the fetus has not begun to move out of the uterus.

A fetus is assessed at 2 cm above the ischial spines. The nurse would document fetal station as: a) +4 b) +2 c) 0 d) -2

d) -2 Rationale: When the presenting part is above the ischial spines, it is noted as a negative station. Since the measurement is 2 cm, the station would be -2. A 0 station indicates that the fetal presenting part is at the level of the ischial spines. Positive stations indicate that the presenting part is below the level of the ischial spines.

A pregnant woman, multipara, has been in labor for several hours. She cries out that her contractions are getting harder and that she can't do this. The patient is really irritable, nauseated, annoyed, and fearful of being left alone. Considering the client's behavior, the nurse would expect the cervix to be dilated how many centimeters? a) 5-6 b) 0-2 c) 3-5 d) 8-10

d) 8-10 Rationale: The reaction of the patient is indicative of entering or being in the transition phase of labor, stage 1. The dilation would be 8-10 cm. Before that, when dilation is 0-7 cm, the patient has an easier time using positive coping skills.

Nurses can advise their clients that all are signs that precede labor except: a) A return of urinary frequency as a result of increased bladder pressure b) Persistent low backache from relaxed pelvic joints c) Stronger and more frequent uterine (Braxton Hicks) contractions d) A decline in energy, as the body stores up for labor

d) A decline in energy, as the body stores up for labor

Which of the following would be least effective in promoting a positive birth outcome for a woman in labor? a) Promoting the woman's feelings of control b) Providing clear information about procedures c) Encouraging the woman to use relaxation techniques d) Allowing the woman time to be alone

d) Allowing the woman time to be alone Rationale: Positive support, not being alone, promotes a positive birth experience. Being alone can increase anxiety and fear, decreasing the woman's ability to cope. Feelings of control promote self-confidence and self-esteem, which in turn help the woman to cope with the challenges of labor. Information about procedures reduces anxiety about the unknown and fosters cooperation and self-confidence in her abilities to deal with labor. Catecholamines are secreted in response to anxiety and fear and can inhibit uterine blood flow and placental perfusion. Relaxation techniques can help to reduce anxiety and fear, in turn decreasing the secretion of catecholamines and ultimately improving the woman's ability to cope with labor.

The third stage of labor is considered to be expulsion of the placenta. This stage can last anywhere from five to 20 minutes. What is a sign that the placenta is separating from the wall of the uterus? a) Expulsion of blood clots b) Firm fundus c) Shortening of the umbilical cord d) Globular shape to the fundus

d) Globular shape to the fundus Rationale: Signs that indicate the placenta is separating from the uterine wall include a gush of blood, lengthening of the umbilical cord, and a globular shape to the fundus.

A new OB/GYN physician has just finished evaluating her one hundredth patient. In reviewing the documentation from all patients thus far, which types of pelvis would you assume the physician has seen the most and the least? a) Android and platypelloid, respectively b) Gynecoid and android, respectively c) Anthropoid and gynecoid, respectively d) Gynecoid and platypelloid, respectively

d) Gynecoid and platypelloid, respectively Rationale: The gynecoid is the typical female pelvis shape; platypelloid pelvis is the least common type of pelvis in women. Women with anthropoid pelvic shapes are able to deliver normally one third of the time, and are somewhat rare. An android pelvis is similar to a male pelvis, and is seen in 16% of nonwhite women.

Assessment reveals that the fetus of a client in labor is in the vertex presentation. The nurse determines that the presenting part is which of the following? a) Brow b) Buttocks c) Shoulders d) Occiput

d) Occiput Rationale: With a vertex presentation, a type of cephalic presentation, the fetal presenting part is the occiput. The shoulders are the presenting part when the fetus is in a shoulder presentation. The brow or sinciput is the presenting part when a fetus is in a brow presentation. The buttocks are the presenting part when a fetus is in a breech presentation.

A client in labor has been admitted to the labor and birth suite. The nurse assessing her notes that the fetus is in a cephalic presentation. Which of the following should the nurse identify by the term presentation? a) Relation of the different fetal body parts to one another b) Relationship of the presenting part to the maternal pelvis c) Relation of the fetal presenting part to the maternal ischial spine d) Part of the fetal body entering the maternal pelvis first

d) Part of the fetal body entering the maternal pelvis first Rationale: The term presentation is the part of the fetal body that is entering the maternal pelvis first. Relationship of the presenting part to the sides of the maternal pelvis is called the position. Attitude is the term that describes the relation of the different fetal body parts to one another. Relation of the fetal presenting part to maternal ischial spine is termed the station.

When going through the transition phase of labor women often feel out of control. What do women in the transition phase of labor need? a) Their significant other beside them b) Intense nursing care c) Just to be left alone d) Positive reinforcement

d) Positive reinforcement Rationale: Any women, even ones who have had natural childbirth classes, have a difficult time maintaining positive coping strategies during this phase of labor. Many women describe feeling out of control during this phase of labor. A woman in transition needs support, encouragement, and positive reinforcement.

A 32-year-old woman presents to the labor-and-delivery suite in active labor. She is multigravida, relaxed, and talking with her husband. When examined by the nurse, the fetus is found to be in a cephalic presentation. His occiput is facing toward the front and slightly to the right of the mother's pelvis, and he is exhibiting a flexed attitude. How does the nurse document the position of the fetus? a) ROP b) LOP c) LOA d) ROA

d) ROA Rationale: Document the fetal position in the clinical record using abbreviations (Box 8-1). The first letter describes the side of the maternal pelvis toward which the presenting part is facing ("R" for right and "L" for left). The second letter or abbreviation indicates the reference point ("O" for occiput, "Fr" for frontum, etc.). The last part of the designation specifies whether the presenting part is facing the anterior (A) or the posterior (P) portion of the pelvis, or whether it is in a transverse (T) position.

In order to accurately assess the health of the mother accurately during labor, the nurse should be aware that: a) The woman's blood pressure increases during contractions and falls back to prelabor normal between contractions b) Use of the Valsalva maneuver is encouraged during the second stage of labor to relieve fetal hypoxia c) Having the woman point her toes reduces leg cramps d) The endogenous endorphins released during labor raise the woman's pain threshold and produce sedation

d) The endogenous endorphins released during labor raise the woman's pain threshold and produce sedation

During the second stage of labor, a woman is generally a) very aware of activities immediately around her. b) anxious to have people around her. c) no longer in need of a support person. d) turning inward to concentrate on body sensations.

d) turning inward to concentrate on body sensations. Rationale: Second-stage contractions are so unusual that most women are unable to think of things other than what is happening inside their body.

When caring for a postpartum client who has given birth vaginally, the nurse assesses the client's respiratory status, noting that it has quickly returned to normal. The nurse understands that which factor is responsible for this change?

decreased intra-abdominal pressure The nurse should identify decreased intra-abdominal pressure as the cause of the respiratory system functioning normally. Progesterone levels do not influence the respiratory system. Decreased bladder pressure does not affect breathing. Anesthesia used during birth causes the respiratory system to take a longer time to return to normal.

Many patients experience a slight fever after delivery especially during the first 24 hours. To what should the nurse attribute this elevated temperature? a) fluid volume overload b) dehydration c) infection d) change in the temperature from the delivery room

dehydration Explanation: Many women experience a slight fever (100.4 degrees F) during the first 24 hours after delivery. This results from dehydration because of fluid loss during labor. With the replacement of fluids the temperature should return to normal after 24 hours.

A 39-week-gestation client presents to the labor and birth unit reporting abdominal pain. What should the nurse do first?

determine if the client is in true or false labor.

The nurse explains Leopold's maneuvers to a pregnant client. For which purposes are these maneuvers performed? Select all that apply.

determining the presentation of the fetus determining the position of the fetus determining the lie of the fetus Leopold maneuvers help the nurse to determine the presentation, position, and lie of the fetus. The approximate weight and size of the fetus can be determined with ultrasound sonography or abdominal palpation.

The nurse, assessing the lochia of a client, attempts to separate a clot and identifies the presence of tissue. Which observation would indicate the presence of tissue?

difficult to separate clots If tissue is identified in the lochia, it is difficult to separate clots. Yellowish-white lochia indicates increased leukocytes and decreased fluid content. Easily separable lochia indicates the presence of clots only. Foul-smelling lochia indicates endometritis.

When assessing a woman in the first stage of labor, the nurse recognizes that the most conclusive assessment that uterine contractions are effective would be:

dilatation of cervix. The best determination of effective contractions is dilation of the cervix. Engagement, membrane rupture, and bloody show may all occur before the cervix has dilated.

While making a follow-up home visit to a client in her first week postpartum, the nurse notes that she has lost 5 pounds. Which reason for this loss would be the most likely?

diuresis Diuresis is the most likely reason for the weight loss during the first postpartum week. Lactation accelerating postpartum weight loss is a popular notion, but it is not statistically significant. Blood loss or nausea in postpartum week does not cause major weight loss.

What is a sucking reflex and how do you check it, what is normal findings?

elicited by gently stimulating the newborn's lips by touching them. The newborn will typically open the mouth and make sucking noises or feeding motions. Expected/normal reflex = will suck on finger

Which nursing intervention is appropriate for prevention of a urinary tract infection (UTI) in the postpartum woman?

encouraging the woman to empty her bladder completely every 2 to 4 hours The nurse should advise the woman to urinate every 2 to 4 hours while awake to prevent overdistention and trauma to the bladder. Maintaining a good fluid intake is also important, but it is not necessary to increase fluids if the woman is consuming enough. Screening for bacteria in the urine would require a primary care provider's order and is not necessary as a prevention measure.

A client in the first stage of labor is admitted to a health care center. The nurse caring for the client instructs her to rock on a birth ball. The nurse informs her that this causes the release of certain natural substances, which reduces the pain. To which substance is the nurse referring?

endorphins The nurse is referring to the release of endorphins, which are natural analgesic substances released by the movement of the client on the birth ball. The nurse should encourage the client to rock or sit on the birth ball. This causes the release of endorphins. The client's movement on the birth ball does not produce prostaglandins, progesterone, or relaxin. Prostaglandins are local hormones that bring about smooth muscle contractions in the uterus. Progesterone is a hormone involved in maintaining pregnancy. Relaxin is a hormone that causes backache during pregnancy by acting on the pelvic joints.

The nurse is assessing a breastfeeding mom 72 hours after birth. When assessing her breasts, the client reports bilateral breast pain around the entire breast. What is the most likely cause of the pain?

engorgement The client is only 72 hours postbirth and is reporting bilateral breast tenderness. Milk typically comes in at 72 hours after birth, and with the production of the milk comes engorgement. The other problems do not typically develop until there is fully established breastfeeding

A new mother asks the nurse why her baby's back and groin have a red and raised rash. The nurses uses which term to correctly identify this condition?

erythema toxicum Erythema toxicum is a rash of unknown cause, with pink papules and superimposed vesicles. It appears within 24 to 48 hours after birth and resolves spontaneously in a few days. Acrocyanosis is a blue color of the hands and feet appearing in most infants at birth. Acrocyanosis may persist for 7 to 10 days. Yeast is a fungal infection caused by Candida albicans; it usually manifests in the groin. The rash of C. albicans is excoriated and does not disappear without treatment. The presentation described in this scenario is not consistent with that of mumps.

A woman is in the fourth stage of labor. During the first hour of this stage, the nurse would assess the woman's fundus at which frequency?

every 15 minutes During the first hour of the fourth stage of labor, the nurse would assess the woman's fundus every 15 minutes and then every 30 minutes for the next hour.

When planning the care of a woman in the active phase of labor, the nurse would anticipate assessing the fetal heart rate at which interval?

every 15 to 30 minutes During the active phase of labor, FHR is monitored every 15 to 30 minutes. FHR is assessed every 30 to 60 minutes during the latent phase of labor. The woman's temperature is typically assessed every 4 hours during the first stage of labor and every 2 hours after ruptured membranes. Blood pressure, pulse, and respirations are assessed every hour during the latent phase and every 30 minutes during the active and transition phases. Contractions are assessed every 30 to 60 minutes during the latent phase, every 15 to 30 minutes during the active phase, and every 15 minutes during transition.

A woman is in the fourth stage of labor. During the first hour of this stage, the nurse would assess the woman's fundus at which frequency?

every 15min

A pregnant client in her 32nd week of gestation has been admitted to a health care center reporting decreased fetal movement. What should the nurse determine first before placing the fetoscope on the woman's abdomen, so as to auscultate the fetal heart sounds?

fetal back The nurse assessing the client should first determine the fetal back before placing the fetoscope on the client's abdomen. The fetal back is determined first because it is through the back that the heart signals are best transmitted. During labor, the fetal heart rate should be assessed to check for any variations indicating distress. Fetal heart rate is auscultated by placing a fetoscope on the client's abdomen in the area of the fetal back. Determining the fetal head, shoulders, and the buttocks would be of no help in localizing the heart sounds.

The nurse is admitting a client who is in early labor. After determining that the birth is not imminent, which assessment should the nurse perform next?

fetal status

On an Apgar evaluation, how is reflex irritability tested?

flicking the soles of the feet and observing the response Reflex irritability means the ability to respond to stimuli. It can be tested by flicking the foot or evaluating the response to a catheter passed into the nose.

When examining a newborn's eyes, the nurse would expect which assessment?

follows a light to the midline Newborns do not usually follow past the midline until 3 months of age. They do not tear.

A nurse is caring for a client who has just received an episiotomy. The nurse observes that the laceration extends through the perineal area and continues through the anterior rectal wall. How does the nurse classify the laceration?

fourth degree The nurse should classify the laceration as fourth degree because it continues through the anterior rectal wall. First-degree laceration involves only skin and superficial structures above muscle; second-degree laceration extends through perineal muscles; and third-degree laceration extends through the anal sphincter muscle but not through the anterior rectal wall.

A client who gave birth to twins 6 hours ago becomes restless and nervous. Her blood pressure falls from 130/80 mm Hg to 96/50 mm Hg. Her pulse drops from 80 to 56 bpm. She was induced earlier in the day and experienced abruptio placentae. Based on this information, what postpartum complication would the nurse expect is happening?

hemorrhage Some risk factors for developing hemorrhage after birth include precipitous labor, uterine atony, placenta previa and abruptio placentae, labor induction, operative procedures, retained placenta fragments, prolonged third stage of labor, multiparity, and uterine overdistention.

When doing a health assessment, at which location would the nurse expect to palpate the fundus in a woman on the second postpartal day and how should it feel?

fundus two fingerbreadths below umbilicus and firm A uterine fundus typically regresses at a rate of one fingerbreadth a day, so on the second day postpartum it would be two fingerbreadths under the umbilicus and would feel firm.

A pregnant client with a history of spinal injury is being prepared for a cesarean birth. Which method of anesthesia is to be administered to the client?

general anesthesia General anesthesia is administered in emergency cesarean births. Local anesthetic is injected into the superficial perineal nerves to numb the perineal area generally before an episiotomy. Although an epidural block is used in cesarean births, it is contraindicated in clients with spinal injury. Regional anesthesia is contraindicated in cesarean births.

Hypoglycemia in newborn is

glucose less than 30 mg/dl or plasma less than 40 mg/dl

A client is being admitted to labor and birth. When admitting an obstetric client in early labor, the first intervention by the nurse is:

good rapport is established with the client and significant other. On admission the client and her family need to establish a rapport with their caregiver. If the client is stable and there is no immediate need, rapport should be established over actions that can be taken care of later.

The AGPAR score is based on which 5 parameters?

heart rate, muscle tone, reflex irritability, respiratory effort, and color A newborn can receive an APGAR score ranging from 0 to 10. The score is based on 5 factors, each of which is assigned a 0, 1, or 2. Heart rate (should be above 100), muscle tone (should be able to maintain a flexion position), reflex irritability (newborn should cry or sneeze when stimulated), and respiratory effort are evaluted by the presence of a strong cry and by color. Color is evaluated by noting the color of the body and hands and feet

Infants receive vitamin K within the first hour after birth. What is the rationale for administering the vitamin?

helps in formation of clotting factors, to prevent bleeding Vitamin K is necessary in the formation of certain clotting factors. The newborn is lacking in vitamin K, and the only method for the infant to receive it is to administer the vitamin IM. Vitamin K is manufactured by normal flora in the gut. Since the newborn has not yet eaten, there is no normal flora in the gut, so the infant cannot manufacture vitamin K. Vitamin K is not administered to give the infant better eyesight, nor is it used to help fight infections.

Thirty minutes after receiving pain medication, a postpartum woman states that she still has severe pain in the perineal region. Upon assessing and palpating the site, what can the nurse expect to find that might be causing this severe pain?

hematoma If a postpartum woman has severe perineal pain despite use of physical comfort measures and medication, the nurse should check for a hematoma by inspecting and palpating the area. If one is found, the nurse needs to notify the primary care provider immediately.

Thirty minutes after receiving pain medication, a postpartum woman states that she sill has severe pain in the perineal region. Upon assessing and palpating the site, what can the nurse expect to find that might be causing this severe pain? a) DVT b) hematoma c) nothing--it is normal d) infection

hematoma Explanation: If a postpartum woman has severe perineal pain despite use of physical comfort measures and medication, the nurse should check for a hematoma by inspecting and palpating the area. If one is found, the nurse needs to notify the physician immediately.

When caring for postpartum clients, the nurse would expect the birth attendant to prescribe what laboratory study the morning after the birth of the baby?

hemoglobin and hematocrit H&H Monitor the H&H and note the H&H before birth. Most practitioners prescribe a postpartum H&H on the morning after birth. If the values drop significantly, the woman may have experienced postpartum hemorrhage. Note the blood type and Rh. If the woman is Rh-, she will need a Rho(D) immune globulin workup. Determine the woman's rubella status. If she is nonimmune, she will need a rubella immunization before she is discharged home.

A patient who delivered twins 6 hours ago becomes restless and nervous. Her blood pressure falls from 130/80 to 96/50. Her pulse drops from 80 to 56. She was induced earlier in the day and experienced abruptio placentae. Based on this information, what postpartum complication would the nurse expect is happening? a) fluid volume overload b) pulmonary emboli c) infection d) hemorrhage

hemorrhage Correct Explanation: Some risk factors for developing hemorrhage after delivery include precipitous labor, uterine atony, placenta previa and abruptio placentae, labor induction, operative procedures, retained placenta fragments, prolonged third stage of labor, multiparity, and uterine overdistention.

A client in her sixth week postpartum reports general weakness. The client has stopped taking iron supplements that were prescribed to her during pregnancy. The nurse would assess the client for which condition?

hypovolemia The nurse should assess the client for hypovolemia as the client must have had hemorrhage during birth and puerperium. Additionally, the client also has discontinued iron supplements. Hyperglycemia can be considered if the client has a history of diabetes. Hypertension and hyperthyroidism are not related to discontinuation of iron supplements.

The nurse is discharge teaching the parents of a newborn baby girl. The nurse knows that it is important to teach them about diarrhea and dehydration. When should the parents notify the primary care provider about diarrhea in the newborn?

if the infant has more than two episodes of diarrhea in one day Diarrhea is defined as frequent stools with high water content. Because newborns dehydrate quickly, it is important for parents to notify the care provider if the newborn has more than two episodes of diarrhea in one day.

While trying to decide whether to bottle feed or breastfeed her newborn infant, a new mother questions the lactation specialist concerning the greatest benefit of breastfeeding her infant. What would be the best response? a) decreased expense for feedings b) immunity against many different bacteria c) Ease of digestion of breast milk d) convenience of breastfeeding

immunity against many different bacteria

The nurse would identify for a new mother which of the following as the greatest benefit of breastfeeding? a) decreased expense b) convenience c) immunity against many different bacteria d) better taste

immunity against many different bacteria Correct Explanation: Human breast milk provides a passive mechanism to protect the newborn against the dangers of a deficient intestinal defense system. It contains antibodies, leukocytes, and many other substances that can interfere with bacterial colonization and prevent harmful penetration. Convenience and being less expensive are also benefits, but not the most important one. Tasting better is an individual preference: some babies will take only breast milk, while others prefer formula.

A woman who is breastfeeding her newborn reports that her breasts seem quite full. Assessment reveals that her breasts are engorged. Which factor would the nurse identify as the most likely cause for this development?

inability of infant to empty breasts For the breastfeeding mother, engorgement is often the result of vascular congestion and milk stasis, primarily caused by the infant not fully emptying the mother's breasts at each feeding. Cracking of the nipple could lead to infection. Improper positioning may lead to nipple tenderness or pain. Inadequate secretion of prolactin causes a decrease in the production of milk.

A newborn is passing greenish-black stool of tarry consistency. The nursing student correctly identifies this type of stool as which of the following? a) stool of a formula-fed newborn b) transitional stool c) stool of a breast-fed newborn d) meconium stool

meconium stool

A client appears to be resting comfortably 12 hours after giving birth to her first child. In contrast, she labored for more than 24 hours, the primary care provider had to use forceps to deliver the baby, and she had multiple vaginal examinations during labor. Based on this information what postpartum complication is the client at risk for developing?

infection There are many risk factors for developing a postpartum infection: operative procedures (e.g., forceps, cesarean section, vacuum extraction), history of diabetes, prolonged labor (longer than 24 hours), use of Foley catheter, anemia, multiple vaginal examinations during labor, prolonged rupture of membranes, manual extraction of placenta, and HIV.

A patient appears to be resting comfortably 12 hours after delivering her first child. In contrast, she labored for more than 24 hours, the physician had to use forceps to deliver the baby, and she had multiple vaginal examinations during labor. Based on this information what postpartum complication is the patient at risk for developing? a) hemorrhage b) depression c) pulmonary emboli d) infection

infection Correct Explanation: There are many risk factors for developing a postpartum infection: operative procedures(eg, forceps, cesarean section, vacuum extraction), history of diabetes, prolonged labor (longer than 24 hours), use of Foley catheter, anemia, multiple vaginal examinations during labor, prolonged rupture of membranes, manual extraction of placenta, and HIV

A patient appears to be resting comfortably 12 hours after delivering her first child. In contrast, she labored for more than 24 hours, the physician had to use forceps to deliver the baby, and she had multiple vaginal examinations during labor. Based on this information what postpartum complication is the patient at risk for developing? a) pulmonary emboli b) depression c) infection d) hemorrhage

infection Correct Explanation: There are many risk factors for developing a postpartum infection: operative procedures(eg, forceps, cesarean section, vacuum extraction), history of diabetes, prolonged labor (longer than 24 hours), use of Foley catheter, anemia, multiple vaginal examinations during labor, prolonged rupture of membranes, manual extraction of placenta, and HIV.

Risk to newborn ...circumcision

infection, hemmorrhage, skin dehiscence, adhesions, urethral fistula, pain

A nurse is preparing to administer vitamin K to a newborn. The nurse would administer the drug by which route?

intramuscularly in the thigh The American Academy of Pediatrics recommends that vitamin K be administered to all newborns soon after birth in a single intramuscular dose of 0.5 to 1 mg. An oral vitamin K preparation is also being given to newborns outside the United States, but at least three doses are needed over a one month period. It is not given intravenously or topically. Erythromycin or tetracycline is used for eye prophylaxis.

A nurse is preparing to administer Vitamin K to a newborn. The nurse would adminsiter the drug:

intramuscularly. The American Academy of Pediatrics recommends that vitamin K be administered to all newborns soon after birth in a single intramuscular dose of 0.5 to 1 mg. An oral vitamin K preparation is also being given to newborns outside the United States, but at least three doses are needed over a one month period. It is not given subcutaneously or intravenously.

The process by which the reproductive organs return to the nonpregnant size and function is termed what?

involution Involution is the term used to describe the process of the return to nonpregnancy size and function of reproductive organs. Evolution is change in the genetic material of a population of organisms from one generation to the next. Decrement is the act or process of decreasing. Progression is defined as movement through stages such as the progression of labor.

The nurse explains to a client who recently gave birth that she will undergo both retrogressive and progressive changes in the postpartal period. Which changes are retrogressive? Select all that apply.

involution of the uterus contraction of the cervix decrease of pregnancy hormones return of blood volume to prepregnancy level Retrogressive changes represent a return to prepregnancy conditions and include involution of the uterus, contraction of the cervix, decrease of pregnancy hormones, and return of the blood volume to prepregnancy level. Progressive changes involve changes to new processes or roles, such as the formation of breast milk (lactation) and the beginning of a parental role.

A nurse recommends to a client in labor to try concentrating intently on a photo of her family as a means of managing pain. The woman looks skeptical and asks, "How would that stop my pain?" Which explanation should the nurse give?

it distracts you from the sensations of pain.

The nurse measures a newborn's temperature immediately after birth and finds it to be 99° F (37.2° C). An hour later, it has dropped several degrees. The nurse understands that this heat loss can be explained in part by which factor in the newborn?

lack of subcutaneous fat Insulation, an efficient means of conserving heat in adults, is not as effective in newborns because they have little subcutaneous fat to provide insulation. Newborns can conserve heat by constricting blood vessels and moving blood away from the skin. Brown fat, a special tissue found in mature newborns, apparently helps to conserve or produce body heat by increasing metabolism as well as regulating body temperature similar to that of a hibernating animal. Other ways newborns are able to increase their metabolic rate and produce more heat include kicking and crying.

A patient is admitted to the labor and delivery unit. Upon examination, she is found to be dilated 3 cm. The nurse notes that the woman is having contractions that last about 45 seconds and are about 5 minutes apart. Based on this information, in which phase of labor is this patient?

latent phase Contractions during the latent phase of labor are typically 5 to 10 minutes apart and last 30 to 45 seconds. The cervix is dilated 1 to 3 cm, and effacement begins.

The nurse is completing an assessment of a newborn. When auscultating the newborn's heart, the nurse would place the stethoscope at which area to auscultate the point of maximal impulse?

lateral to the midclavicular line at the fourth intercostal space The point of maximal impulse in a newborn is lateral to the midclavicular line at the fourth intercostal space. A displaced PMI may indicate a tension pneumothorax or cardiomegaly.

The student nurse is preparing to assess the fetal heart rate (FHR). She has determined that the fetal back is located toward the client's left side, the small parts toward the right side, and there is a vertex (occiput) presentation. The nurse should initially begin auscultation of the fetal heart rate in the mother's:

left lower quadrant. The best position to auscultate fetal heart tones in on the fetus back. In this position, the best place for the FHR monitor is on the left lower quadrant.

A nurse is conducting a physical examination of a newborn. The nurse documents which finding as within normal parameters? Select all that apply.

length of 54 cm weight of 3,300 grams temperature of 98.6° F (37° C) Typical newborn findings include length of 45 to 55 cm, weight of 2,700 to 4,000 grams, head circumference of 33 to 35 cm, chest circumference of 30 to 33 cm, temperature of 97.7° F to 99.5° F (36.5° C to 37.5° C), and apical pulse rate of 120 to 160 beats/minute.

Extremely low birth weight

less than 1000 g <2.5 lb

Very low birth weight

less than 1500 g ..<3.5 lb

Low birth weight

less than 2500 g ..<5.5 lb

Discharge teaching is an important part of the labor and birth room nurse's position. New parents need to know the basics of baby care, like how to monitor fluid volume and when to call the primary care provider. What are the parameters for calling the care provider in regards to an infant's temperature?

less than 97.7° F (36.5° C) or greater than 100° F (37.8° C) Temperatures of less than 97.7 ° F (36.5° C) or greater than 100 ° F (37.8° C) should be reported to the primary care provider.

Discharge teaching is an important part of the labor and delivery room nurse's position. New parents need to know the basics of baby care, like how to monitor fluid volume and when to call the physician. What are the parameters for calling the physician in regards to an infants' temperature? a) less than 96°F (35.6°C) or greater than 101°F (38.3°C) b) less than 96.7°F (35.9°C) or greater than 99.5°F (37.4°C). c) less than 97°F (36.1°C) or greater than 100.5°F (38.1°C). d) Less than 97.7°F (36.5°C) or greater than 100°F (37.8°C).

less than 97°F (36.1°C) or greater than 100.5°F (38.1°C).

The nurse assesses a postpartum woman's perineum and notices that her lochial discharge is moderate in amount and red. The nurse would record this as what type of lochia?

lochia rubra Lochia rubra is red; it lasts for the first few days of the postpartal period.

A nurse is required to obtain the fetal heart rate (FHR) for a pregnant client. If the presentation is cephalic, which maternal site should the nurse monitor to hear the FHR clearly?

lower quadrant of the maternal abdomen In a cephalic presentation, the FHR is best heard in the lower quadrant of the maternal abdomen. In a breech presentation, it is heard at or above the level of the maternal umbilicus.

A new mother who is breastfeeding reports that her right breast is very hard, tender, and painful. Upon examination the nurse notices several nodules and the breast feels very warm to the touch. What do these findings indicate to the nurse?

mastitis Engorged breasts are hard, tender, and taut. If the breasts have nodules, masses, or areas of warmth, they may have plugged ducts, which can lead to mastitis if not treated promptly.

A new mother who is breastfeeding reports that her right breast is very hard, tender, and painful. Upon examination the nurse notices several nodules and the breast feels very warm to the touch. What do these findings indicate to the nurse? a) an improperly positioned baby during feedings b) mastitis c) normal findings in breastfeeding mothers d) too much milk being retained

mastitis Correct Explanation: Engorged breasts are hard, tender, and taut. If the breasts have nodules, masses, or areas of warmth, they may have plugged ducts, which can lead to mastitis if not treated promptly.

The nurse notes that a client's amniotic fluid is green when the membranes rupture. What finding would the nurse document?

meconium in the amniotic sac Green tinted fluid with ROM is indicative of meconium in the amniotic sac, or the infant having a bowel movement in utero. Infection would be shown by pus or cloudy fluid. Umbilical cord prolapse occurs when pressure on the cord stops the flow of oxygen to the fetus. Amniotic embolism results when amniotic fluid enters circulation.

A newborn is passing greenish-black stool of tarry consistency. The nursing student correctly identifies this type of stool as which of the following? a) stool of a formula-fed newborn b) meconium stool c) stool of a breast-fed newborn d) transitional stool

meconium stool Correct Explanation: Meconium is a newborn's first stool. It is composed of amniotic fluid, shed mucosal cells, intestinal secretions, and blood. Breast-fed newborns will pass stools that are yellow-gold, loose, and stringy to pasty in consistency. A formula-fed newborn will have stools that are yellow, yellow-greeen, or greenish and loose, pasty, or formed in consistency based upon the type of formula.

Assessment of a newborn reveals tiny white pinpoint papules on a newborn's nose. The nurse documents this finding as:

milia. Milia are the tiny white pinpoint papules of unopened sebaceous glands frequently found on the newborn's nose. Lanugo is the fine downy hair that covers the newborn's shoulders, back, and upper arms. Vernix caseosa is the thick white substance that provides a protective covering of the skin of the fetus. The harlequin sign refers to a transient phenomenon in which a newborn appears red on the dependent side of the body and pale on the upper side when lying on his or her side.

Inspection of a woman's perineal pad reveals a 5-inch stain. How should the nurse document this amount?

moderate Moderate lochia would describe a 4- to 6-inch stain, scant lochia a 1- to 2-inch stain, and light or small an approximately 4-inch stain. Heavy or large lochia would describe a pad that is saturated within 1 hour.

A nurse is assessing a client's lochia every 15 minutes for the first hour during the fourth stage of labor. Which finding would the nurse expect to assess?

moderate lochia rubra with no clots During the first hour following birth, the nurse should find moderate lochia rubra with no clots. Lochia rubra with few clots or saturation of two or more pads within this first hour are not abnormal findings that require further investigation. Lochia alba appears around the 10th day postpartum.

Which lochia pattern should be reported immediately?

moderate lochia serosa on day 4 postpartum, increasing in volume and changing to rubra on day 5 Lochia by day 4 should be decreasing in amount, and the color should be changing to pink tinge. Red rubra on day 4 may indicate bleeding, and the healthcare provider should be notified. A moderate flow of lochia rubra on day 3 postpartum, changing to serosa on day 5, is a normal finding; as is lochia progressing from rubra to serosa to alba within 10 days of delivery; and so is moderate lochia rubra on day 3, mixed serosa and rubra on day 4, and light serosa on day 5.

S/s of problem with newborn What are some physical signs of a newborn in distress/problem? Can you name some?

nasal flaring, chest/sternal retractions/grunting on exhalation, labored breathing, generalized cyanosis, flaccid body, abnormal breath sounds, abnormal resp. rates (chapter 17), abnormal heart rate (chapter 17), abnormal newborn size

A new mother talking to a friend states, "I wish my baby was more like yours. You are so lucky. My baby has not slept straight through the night even once. It seems like all she wants to do is breastfeed. I am so tired of her." This is an example of which of the following? a) negative bonding b) negative attachment c) positive attachment d) positive bonding

negative attachment Correct Explanation: Expressing disappointment or displeasure in the infant, failing to explore the infant visually or physically, and failing to claim the infant as part of the family are just a few examples of negative attachment behaviors.

The majority of skin variations are transient and fade or disappear with time. The nurse assesses a permanent skin variation in a newborn and counsels the parents to monitor it because of its link to potential childhood cancer. The nurse implements this counseling based on which finding?

nevus flammeus Nevus flammeus, also called a port wine stain, may be associated with structural malformations, bony or muscular overgrowth, and certain childhood cancers and should be monitored with periodic examinations. Erythema toxicum is a benign rash that resembles flea bites. Mongolian spots are blue or purple splotches that appear on the lower back and buttocks. Nevus vasculosus is also called strawberry mark and is a benign capillary hemangioma that tends to resolve by age 3 without treatment.

The nurse is used to working on the postpartum floor taking care of women who have had normal vaginal births. Today, however, the nurse has been assigned to help care for women who are less than 24 hours post cesarean birth. The nurse realizes that some areas will not be assessed. What would the nurse leave out of the client assessments?

perineum Usually a woman who experiences cesarean birth does not have an episiotomy, although rarely this may be the case.

The nurse is assisting with the circumcision of a 16-hour-old male infant. Immediately after the procedure, what kind of dressing would the nurse apply to the surgical area?

petrolatum gauze dressing Immediately after the procedure, place a petrolatum gauze dressing, as prescribed by the primary care provider.

The nurse working on a postpartum must check lochia in terms of amount, color, change with activity and time, and: a) odor b) pH c) consistency d) specific gravity

odor Correct Explanation: The nurse when assessing lochia must do so in terms of amount, color, odor, and change with activity and time.

Where do milia come from?

oil glands and they disappear within 2-4 weeks

The nurse is observing a client who gave birth yesterday. Where should the nurse expect to find the top of the client's fundus?

one fingerbreadth below the umbilicus After a client gives birth, the height of her fundus should decrease by approximately one fingerbreadth (1 cm) each day. By the end of the first postpartum day, the fundus should be one fingerbreadth below the umbilicus. Immediately after birth, the fundus may be above the umbilicus; 6 to 12 hours after birth, it should be at the level of the umbilicus; 10 days after birth, it should be below the symphysis pubis.

A nurse is caring for a client who is nursing her baby boy. The client reports afterpains. Secretion of which substance should the nurse identify as the cause of afterpains?

oxytocin Secretion of oxytocin stimulates uterine contraction and causes the woman to experience afterpains. Decrease in progesterone and estrogen after placental delivery stimulates the anterior pituitary to secrete prolactin which causes lactation.

A father is asking questions about the circumcision of his son. He is asking the nurse if there are any disadvantages to the procedure. How should the nurse respond?

pain administration may not be effective during the procedure The anesthetic block is not always effective. Not all providers use anesthetics prior to the procedure, and the infant can feel the pain of the circumcision. A lower rate of urinary tract infections, a reduced risk of penile cancer, and fewer complications than if circumcised later in life are not disadvantages to the procedure; they are advantages.

A nurse is auscultating the fetal heart rate of a woman in labor. To ensure that the nurse is assessing the FHR and not the mother's heart rate, which action would be most appropriate for the nurse to do?

palpate the mothers radial pulse at the same time

The Ballard scoring system evaluates newborns on which two factors?

physical maturity and neuromuscular maturity When determining a newborn's gestational age using the Ballard scale, the nurse assesses physical signs and neurologic characteristics

What color should mucous membranes be?

pink

A nursing student is aware that fetal gas exchange takes place in which of the following? a) lungs b) uterus c) bronchioles d) placenta

placenta

A nursing student is aware that fetal gas exchange takes place in which of the following? a) placenta b) lungs c) bronchioles d) uterus

placenta Correct Explanation: Many different changes occur for the newborn to survive outside the uterus. One such change is that gas exchange that once took place in the placenta now will take place in the lungs.

A patient who is in her 9th month of pregnancy comes to the emergency department and reports that bright red blood is coming from her vagina. She denies having any pain. What needs to be ruled out before a vaginal examination can be performed?

placenta previa Vaginal examinations should never be done if the woman presents with bright red painless bleeding until placenta previa is ruled out. The other options would not be concerns at this time based on the findings.

A nurse is reviewing a postpartum woman's history and labor and birth record. The nurse determines the need to closely monitor this client for infection based on which factor?

placenta removed via manual extraction Manual removal of the placenta places a woman at risk for postpartum infection, as does a hemoglobin level less than 10.5 mg/dL. Precipitous labor, less than 3 hours, and multiparity, more than three births closely spaced, place a woman at risk for postpartum hemorrhage.

When assessing fetal heart rate patterns, which finding would alert the nurse to a possible problem?

prolonged decelerations Prolonged decelerations are associated with prolonged cord compression, abruptio placentae, cord prolapse, supine maternal position, maternal seizures, regional anesthesia, or uterine rupture. Variable decelerations are the most common deceleration pattern found. They are usually transient and correctable. Early decelerations are thought to be the result of fetal head compression. They are not indicative of fetal distress and do not require intervention. Fetal accelerations are transitory increases in FHR and provide evidence of fetal well-being.

When examining a newborn female, the nurse notices a small pinkish discharge from the vaginal area. What should the nurse suspect?

pseudomenstruation, a normal finding Pseudomenstruation is seen when a newborn female has a small amount of pinkish discharge. It comes from the withdrawal of maternal hormones and is a normal finding.

A nurse is auscultating the lungs of a postpartum patient and notices crackles and some dyspnea. The patient's respiratory rate is 12 breaths/min; she appears in some distress. What complication should the nurse suspect based on these data? a) fluid volume deficit b) infection c) hemorrhage d) pulmonary edema

pulmonary edema Correct Explanation: Any change in the respiratory rate of a postpartum woman might indicate pulmonary edema, atelectasis, or pulmonary embolism and must be reported. Lungs should be clear upon auscultation

A nursing student learns that a certain condition in 1 in every 2,000 pregnancies is a major cause of death. What is this condition? a) hemorrhage b) infection c) hypertension d) pulmonary embolism

pulmonary embolism Explanation: Pulmonary embolism occurs in 1 in 2,000 pregnancies and is a major cause of maternal mortality.

When an infant smiles at the mother and the mother in turn smiles and kisses her baby, this would be which phase of attachment? a) proximity b) reciprocity c) commitment d) all of the above

reciprocity Correct Explanation: Proximity refers to the physical and psychological experience of the parents being close to their infant. Reciprocity is the process by which the infant's abilities and behaviors elicit parental responses (ie, the smile by the infant gets a smile and kiss in return). Commitment refers to the enduring nature of the relationship.

What tonic neck reflex? How do you assess this and what is a normal finding?

resembles the stance of a fencer and is often called "fencing reflex". Lie newborn on back, turn head to one side. Expected/normal reflex = the side the baby head is turned toward the arm toward which the baby is facing should extend straight away from the body with the hand partially open. The arm on the opposite side of the way head faces will be flexed and fist clenched. When you turn the baby's head the other way the opposite should happen on both sides.

A nurse is caring for a client who has been administered an epidural block. Which should the nurse assess next?

respiratory rate

A nurse is caring for a client who has been administered an epidural block. Which should the nurse assess next?

respiratory rate The nurse must monitor for respiratory depression. Monitoring the client's respiratory rate will be the best indicator of respiratory depression.

The nurse is preparing to apply a thermistor probe to a newborn to monitor the newborn's temperature. At which location would the nurse apply the probe?

right upper abdominal quadrant A thermistor probe is taped to the newborn's abdomen, usually in the right upper quadrant. This allows for position changes without having to readjust the probe.

When evaluating neurologic maturity to determine gestational age, the nurse understands that which activity is not part of the assessment?

rooting The six activities the newborn performs when being evaluated for gestational age based on neurologic maturity are as follows: posture, square window, arm recoil, popliteal angel, scar sign, and heel to ear.

The nurse is documenting assessment of infant reflexes. She strokes the side of the infant's face, and the baby turns toward the stroke. What reflex has the nurse elicited?

rooting This is the rooting reflex and is used to encourage the infant to feed. This reflex and the sucking reflex work together to assist the infant with cues for feeding at the breast. The tonic neck (or fencing) reflex and the Moro (or startle reflex) are total body reflexes and assess neurologic function in the newborn

A nurse is discussing breastfeeding with a new mother and demonstrates that when she strokes the baby's cheek, the baby turns his head in that direction. This movement is known as which reflex?

rooting reflex The rooting reflex is demonstrated when, in response to the newborn's cheek being stroked, the infant turns her head in that direction. This reflex serves to help a newborn find food: when a mother holds the child and allows her breast to brush the newborn's cheek, the reflex causes the baby to turn toward the breast. The extrusion reflex is demonstrated when a newborn extrudes any substance that is placed on the anterior portion of the tongue; this reflex prevents the swallowing of inedible substances. The Moro reflex is demonstrated when, in response to a sudden backward head movement, the newborn abducts and extends arms and legs, then swings the arms into an embrace position and pulls up the legs against the abdomen. The Babinski reflex is demonstrated when the sole of a newborn's foot is stroked in an inverted "J" curve from the heel upward and the newborn fans the toes in response (positive Babinski sign).

A pregnant client requires administration of an epidural block for management of pain during labor. For which conditions should the nurse check the client before administering the epidural block? Select all that apply.

spinal abnormality hypovolemia coagulation defects The nurse should check for any abnormality of the spine, hypovolemia, or coagulation defects in the client. An epidural is contraindicated in women with these conditions. Varicose veins and skin rashes or bruises are not contraindications for an epidural block. They are contraindications for massage used for pain relief during labor.

A nurse is providing care to a woman during the third stage of labor. Which finding would alert the nurse that the placenta is separating?

sudden gush of dark blood from the vagina Signs that the placenta is separating including a firmly contracting uterus, a change in uterine shape from discoid to globular ovoid, a sudden gush of dark blood from the vaginal opening, and lengthening of the umbilical cord protruding from the vagina.

Which factor might result in a decreased supply of breast milk in a postpartum client?

supplemental feedings with formula Routine formula supplementation may interfere with establishing an adequate milk volume because decreased stimulation to the client's nipples affects hormonal levels and milk production. Vitamin C levels have not been shown to influence milk volume. One drink containing alcohol generally tends to relax the client, facilitating letdown. Excessive consumption of alcohol may block letdown of milk to the infant, though supply is not necessarily affected. Frequent feedings are likely to increase milk production.

A nurse working in the neonatal nursery anticipates the physician to order which of the following for a premature newborn having difficulty breathing? a) epinephrine b) norepinephrine c) surfactant d) albuteral

surfactant

A nurse working in the neonatal nursery anticipates the physician to order which of the following for a premature newborn having difficulty breathing? a) albuteral b) epinephrine c) surfactant d) norepinephrine

surfactant Correct Explanation: Surfactant is a protein that keeps small air sacs in the lungs from collapsing. Its use was introduced in 1990 and continues today, especially for premature babies and those who have respiratory distress syndrome. The other medications are not given to help premature babies breathe.

While waiting for the placenta to deliver during the third stage of labor the nurse must assess the new mother's vital signs every 15 minutes. What sign would indicate impending shock?

tachycardia and a falling blood pressure Monitor the woman's vital signs at least every 15 minutes during the third stage of labor. Tachycardia and a falling blood pressure are signs of impending shock; the nurse should immediately report these signs.

A woman presents in advanced labor, and birth appears imminent. What is the most important and appropriate aspect of admission for this woman?

taking her blood pressure and determining whether clonus or edema is present In advanced labor the most important assessments must be completed first. The assessment for signs or symptoms of preeclampsia must be assessed first. The history can be obtained after the birth of the baby or if labor slows down. Plans for the newborn can be figured out later. Blood tests can be run as soon as a sample can be taken from the mother.

A new mother gave birth to her baby 24 hours ago and today has been content to rest in her hospital bed, hold her baby, allow the nurse to care for her, and to discuss her labor and birth experience with visitors. Which phase of the puerperium is this client in?

taking-in phase The taking-in phase is largely a time of reflection. During this 1- to 3-day period, a woman is largely passive. She prefers having a nurse attend to her needs and make decisions for her, rather than do these things herself. As a part of thinking and pondering about her new role, the woman usually wants to talk about her pregnancy, especially about her labor and birth. After a time of passive dependence, a woman enters the taking-hold phase and begins to initiate action. She prefers to get her own washcloth or to make her own decisions. In the letting-go phase, a woman finally redefines her new role. She gives up the fantasized image of her child and accepts the real one; she gives up her old role of being childless or the mother of only one or two (or however many children she had before this birth). Rooming-in is a feature offered by hospitals in which the infant is allowed to stay in the same hospital room as the mother following birth; it is not a phase of the puerperium.

Rubin identified a series of changes that a new mother makes during the postpartum period. The correct sequence of these changes is:

taking-in, taking-hold, letting-go. The new mother makes progressive changes to know her infant, review the pregnancy and labor, validate her safe passage through these phases, learn the initial tasks of mothering, and let go of her former life to incorporate this new child.

All of the following are ways the nurse can encourage bonding between the parents and the newborn except:

telling the mother that the best way to bond with her baby is to breastfeed. Modeling behavior such as talking to the newborn will aid in bonding. Being able to observe parents as they provide care to their newborn will give new parents confidence. Asking their permission to pick up the newborn will give them a sense of ownership. Although breastfeeding is an excellent way for a mother to bond with her baby, it is not the only way and it is not necessarily the best way

A nurse is teaching new parents about keeping follow-up appointments and calling their health care provider if they notice signs of illness in their newborn. The nurse determines that the teaching was successful when the parents identify which signs as needing to be reported? Select all that apply.

temperature of 38.3° C (101° F) or higher refuse feeding abdominal distention Parents should call their health care provider if they note any of the following warning signs: temperature of 38.3° C (101° F) or higher; forceful, persistent vomiting; refusal to take feedings; two or more green, watery diarrheal stools; infrequent wet diapers and change in bowel movements from normal pattern; lethargy or excessive sleepiness; inconsolable crying and extreme fussiness; abdominal distention; or difficult or labored breathing.

To assess the frequency of a woman's labor contractions, you would time a) how many contractions occur in 5 minutes. b) the interval between the acme of two consecutive contractions. c) the end of one contraction to the beginning of the next. d) the beginning of one contraction to the beginning of the next.

the beginning of one contraction to the beginning of the next. Correct Explanation: Measuring from the beginning of one contraction to the next marks the time between contractions.

In order to accurately assess the health of the mother accurately during labor, the nurse should be aware that

the endogenous endorphins released during labor raise the woman's pain threshold and produce sedation. In addition, physiologic anesthesia of the perineal tissues, caused by the pressure of the presenting part, decreases the mother's perception of pain.

Which is the best place to perform a heel stick on a newborn?

the fat pads on the lateral aspects of the foot The calcaneus is the bone of the heel. A heel stick should not be done on the flat part of the foot or heel, but instead on the lateral aspect of the foot, where the fat pads are.

A new student asks what "neonatal period" means. The instructor defines it as which of the following? a) the first week of life b) the first 28 days of life c) the first 72 hours of life d) the first 3 days of life

the first 28 days of life Correct Explanation: The term neonatal period is defined as the first 28 days of life.

The American Academy of Pediatrics and the American Dietetic Association recommend breastfeeding exclusively for how long?

the first 6 months Both the AAP and the ADA recommend breastfeeding exclusively for the first 6 months of life. After 6 months, breastfeeding does not need to be exclusive, but it should be continued until 12 months

With a hepatitis B (HbsAG) positive mother, what should the newborn receive?

the hepatitis B vaccination and 1 dose of hepatitis B immunoglobulin within 12 hours of birth If a mother has hepatitis B or is suspected of having hepatitis B, the newborn should be bathed and then should receive 1 dose of the hepatitis B vaccine and 1 dose of the hepatitis B immuoglobulin within 12 hours of birth.

During the second day postpartum, a nurse notices that a client is initiating breastfeeding with her infant and changing her infant's diapers with some assistance from her partner. Which phase does the nurse recognize that the woman is experiencing?

the taking-hold phase The taking-in phase is characterized by the woman's dependency on and passivity with others. Maternal needs are dominant, and talking about the birth is an important task. The new mother follows suggestions, is hesitant about making decisions, and is still preoccupied with her needs. The taking-hold phase is characterized by the woman becoming more independent and interested in learning how to care for her infant. Learning how to be a competent parent is an important task. The letting-go phase is an interdependent phase after birth in which the mother and family move forward as a family system, interacting together. The binding-in phase is a distractor for this question.

A mother points out to you that following three meconium stools, her newborn has had a bright green stool. THe nurse would explain to her that a) her child will need to be isolated until the stool can be cultured. b) this is a normal finding. c) this is most likely a symptom of impending diarrhea. d) her child may be developing an allergy to breast milk.

this is a normal finding.

A primipara client gave birth vaginally to a healthy newborn girl 48 hours ago. The nurse palpates the client's fundus and documents which finding as normal?

two fingerbreadths below the umbilicus During the first few days after birth, the uterus typically descends downward from the level of the umbilicus at a rate of 1 cm (1 fingerbreadth) per day so that by day 2, it is about 2 fingerbreadths below the umbilicus.

After completing a class for new parents, the nurse notes the session is successful when the class recognizes the newborn should be bathed how often?

two or three times per week Bathing two or three times weekly is sufficient for the first year; more frequent bathing may dry the skin.

What is Milia?

unopened sebaceous glands frequently found on newborn nose, may appear on chin and forehead.

The nurse is caring for a client in the postpartum period. The client has difficulty in voiding and is catheterized. The nurse then would monitor the client for which condition?

urinary tract infection The nurse would need to monitor the client for signs and symptoms of a urinary tract infection, a risk associated with catheterization. Stress incontinence is caused due to loss of pelvic muscle tone after birth. Increased urinary output is observed in diuresis. Catheterization does not cause loss of pelvic muscle tone, increased urine output, or stress incontinence.

A nurse is caring for a client with postpartum hemorrhage. What should the nurse identify as the significant cause of postpartum hemorrhage?

uterine atony Uterine atony is the significant cause of postpartum hemorrhage. Discomfort from hemorrhoids increases risk for constipation during postpartum, diuresis causes weight loss during the first postpartal week, whereas iron deficiency causes anemia in the puerperium.

A postpartum woman is experiencing subinvolution. When reviewing the client's history for factors that might contribute to this condition, which factors would the nurse identify? Select all that apply.

uterine infection hydramnios prolonged labor Factors that inhibit involution include prolonged labor and difficult birth, uterine infection, overdistention of the uterine muscles such as from hydramnios, a full bladder, close childbirth spacing, and incomplete expulsion of amniotic membranes and placenta. Breastfeeding, early ambulation, and an empty bladder would facilitate uterine involution.

While caring for a woman in labor, the nurse notes that the fetal heart monitor demonstrates late decelerations. The most common cause for their occurrence is:

uteroplacental insufficiency. Late decelerations are associated with uteroplacental insufficiency. They typically indicate decreased blood flow to the uterus during the contractions. Maternal hypotension and fatigue would not be observed on the fetal heart monitor. Cord compression would be marked by fetal tachycardia.

The nurse is making a home visit to a woman who is 4 days postpartum. Which finding would indicate to the nurse that the woman is experiencing a problem?

uterus 1 cm below umbilicus By the fourth postpartum day, the uterus should be approximately 4 cm below the umbilicus. Being only at 1 cm indicates that the uterus is not contracting as it should. Lochia serosa is normal from days 3 to 10 postpartum. After birth the vagina is edematous and thin with few rugae. It eventually thickens and rugae return in approximately 3 weeks. Diaphoresis is common during the early postpartum period, especially in the first week. It is a mechanism to reduce fluids retained during pregnancy and restore prepregnant body fluid levels.

A postpartum client who had a cesarean birth reports right calf pain to the nurse. The nurse observes that the client has nonpitting edema from her right knee to her foot. The nurse knows to prepare the client for which test first?

venous duplex ultrasound of the right leg Right calf pain and nonpitting edema may indicate deep vein thrombosis (DVT). Postpartum clients and clients who have had abdominal surgery are at increased risk for DVT. Venous duplex ultrasound is a noninvasive test that visualizes the veins and assesses blood flow patterns. A venogram is an invasive test that utilizes dye and radiation to create images of the veins and would not be the first choice. Transthoracic echocardiography looks at cardiac structures and is not indicated at this time. Right calf pain and edema are symptoms of venous outflow obstruction, not arterial insufficiency.

The nurse assesses a postpartum woman for thromboembolism based on the understanding that her risk is increased because of which factor?

vessel damage during birth A woman's risk for thromboembolism increases due to her hypercoagulable state, vessel damage during birth, and immobility. The increase in white blood cell count is unrelated to her risk for thromboembolism. Coagulation factors remain elevated for 2 to 3 weeks postpartum. An episiotomy is not a risk factor for thromboembolism.

Which measurements were most likely obtained from a normal newborn born at 38 weeks to a healthy mother with no maternal complications?

weight = 3500 g, length = 20 inches (51 cm), head circumference = 34 cm, and chest circumference = 32 cm For a term infant, expected weight is 2500 to 4000 g; length is 19 to 21 inches (48 to 53 cm); head circumference is 33 to 35 cm; and chest circumference is 30.5 to 33 cm.

The nurse caring for a newborn has to perform assessment at various intervals. When should the nurse complete the second assessment for the newborn?

within the first 2 to 4 hours, when the newborn reaches the nursery The nurse should complete the second assessment for the newborn within the first 2 to 4 hours, when the newborn is in the nursery. The nurse should complete the initial newborn assessment in the birthing area and the third assessment before the newborn is discharged, whenever that may be.

While visiting a client at home on the 10th day postpartum, the nurse assesses the client's lochia. Which color would the nurse expect the lochia to be?

yellowish white The normal color of lochia on the tenth day of postpartum is yellowish white. The color of lochia changes from red to pink by approximately four or five days postpartum. The color of lochia is never yellowish pink.

When performing Ortolani's maneuver, which of the following should occur? Select all that apply. a) Attempt to abduct the hips 180 degrees while applying upward pressure. b) The newborn should be in a supine position. c) A click should be heard when the legs are abducted. d) Attempt to abduct the hips 90 degrees while applying upward pressure. e) The newborn should be in a prone position.

• Attempt to abduct the hips 180 degrees while applying upward pressure. • The newborn should be in a supine position. Correct Explanation: The newborn should be in the supine position. The nurse will flex the hips and knees to 90 degrees at the hip, then will attempt to abduct the hips 180 degrees while applying upward pressure. A "click" or a "cluck" should not be heard when the legs are abducted.

A 25-year-old P3023 spontaneously ruptured clear fluid at home and has had a normal labor progression. The nurse and the midwife do not anticipate any complications. What should the nurse do to prepare for the birth? Select all that apply. a) Move the newborn warmer to the delivery area and turn it on. b) Document events as they are happening. c) Open the newborn crash cart or box to ensure easy access to all supplies. d) Check the functionality of the oxygen source and equipment. e) Connect the meconium aspirator to the wall suction and turn it on.

• Document events as they are happening. • Check the functionality of the oxygen source and equipment. • Move the newborn warmer to the delivery area and turn it on.

A newly delivered mother has difficulty sleeping despite her exhaustion from labor. This inability to rest is due to Select all that apply. a) Bottle feeding b) Excess fatigue and overstimulation by visitors c) Frequent trips to the bathroom due to diuresis d) The baby's crying e) Inability to get adequate pain relief

• The baby's crying • Inability to get adequate pain relief • Frequent trips to the bathroom due to diuresis • Excess fatigue and overstimulation by visitors Explanation: The period before labor and delivery can be uncomfortable for the mother, thus preventing adequate rest and creating a sleep hunger. The early postpartum period involves many adjustments that can take a toll on the mother's sleep.

A 25-year-old P3023 spontaneously ruptured clear fluid at home and has had a normal labor progression. The nurse and the midwife do not anticipate any complications. What should the nurse do to prepare for the birth? Select all that apply. a) Document events as they are happening. b) Check the functionality of the oxygen source and equipment. c) Open the newborn crash cart or box to ensure easy access to all supplies. d) Move the newborn warmer to the delivery area and turn it on. e) Connect the meconium aspirator to the wall suction and turn it on.

• Document events as they are happening. • Check the functionality of the oxygen source and equipment. • Move the newborn warmer to the delivery area and turn it on. Correct Explanation: To prepare for the birth of a normal newborn in an uncomplicated labor, the nurse should ensure that there is an adequately warm area to receive the newborn. She needs to ensure that oxygen and suction equipment is readily available and functional and must ensure that all equipment for resuscitation is available (not necessarily opened). The nurse should also make a record of the development of the labor as it progresses. A meconium aspirator is necessary only when meconium is present.

A nurse is caring for a newborn with transient tachypnea. What nursing interventions should the nurse perform while providing supportive care to the newborn? Select all that apply. a) Ensure the newborn's warmth b) Massage the newborn's back c) Provide oxygen supplementation d) Provide warm water to drink e) Observe respiratory status frequently

• Ensure the newborn's warmth • Provide oxygen supplementation • Observe respiratory status frequently

Given that the first 24 hours after delivery is a time for return to homeostasis, which postpartum findings are considered acceptable during this time? Select all that apply. a) Fundus one fingerbreadth above umbilicus b) Hypotonic bowel sounds c) Urination of 50 mL every hour d) Moderate saturation of peripad every 3 hours e) Inverted nipples following breastfeeding

• Fundus one fingerbreadth above umbilicus • Moderate saturation of peripad every 3 hours Correct Explanation: A fundus can rise to slightly above or below the umbilicus in the first 24 hours, and moderate saturation of 2/3 of the pad is appropriate. Inverted nipples always require intervention if breastfeeding. Hypotonic bowel sounds also require assessment more frequently than routinely ordered, and 50 mL urine is inadequate given the occurrence of diuresis.

While teaching a student, the nurse should include which of the following signs and symptoms to recognize hypoglycemia in the neonate? (Select all that apply.) a) Tachypnea b) Jitteriness c) Poor feeding d) Bradypnea

• Jitteriness • Poor feeding • Tachypnea

When assessing a newborn's gestational age, the nurse evaluates which of the following parameters to indicate physical maturity? Select all that apply. a) Genitals b) Scarf sign c) Arm recoil d) Posture e) Lanugo

• Lanugo • Genitals

When assessing a newborn's gestational age, the nurse evaluates which of the following parameters to indicate physical maturity? Select all that apply. a) Lanugo b) Posture c) Scarf sign d) Genitals e) Arm recoil

• Lanugo • Genitals

When assessing a newborn's gestational age, the nurse evaluates which of the following parameters to indicate physical maturity? Select all that apply. a) Posture b) Lanugo c) Arm recoil d) Scarf sign e) Genitals

• Lanugo • Genitals Correct Explanation: Physical maturity indicators include skin, lanugo, plantar surface, breast, eye-ear, and genitals. Arm recoil, posture, and the scarf sign are used to evaluate neuromuscular maturity.

Which of the following findings would the nurse identify as normal when assessing a newborn? Select all that apply. a) Chest circumference of 35 cm b) Length of 54 cm c) Temperature of 37 degrees C d) Weight of 3,300 grams e) Head circumference of 30 cm f) Apical pulse rate of 100 beats/minute

• Length of 54 cm • Temperature of 37 degrees C • Weight of 3,300 grams Correct Explanation: Typical newborn findings include length of 45 to 55 cm, weight of 2,700 to 4,000 grams, head circumference of 33 to 35 cm, chest circumference of 30 to 33 cm, temperature of 36.5 to 37.5 degrees C, and apical pulse rate of 120 to 160 beats/minute.

Which of the following findings would the nurse identify as normal when assessing a newborn? Select all that apply. a) Chest circumference of 35 cm b) Weight of 3,300 grams c) Apical pulse rate of 100 beats/minute d) Head circumference of 30 cm e) Temperature of 98.6°F (37°C) f) Length of 54 cm

• Length of 54 cm • Weight of 3,300 grams • Temperature of 98.6°F (37°C)

A nurse is performing a detailed newborn assessment of a female newborn. Which observations indicate a normal finding? Select all that apply. a) Low-set ears b) Swollen genitals c) Enlarged fontanelles d) Mongolian spots e) Short, creased neck

• Mongolian spots • Swollen genitals • Short, creased neck

Patient teaching is conducted throughout a patient's hospitalization and is reinforced before discharge. Which self-care items are to be reinforced before discharge? a) Infant formula selection b) Signs and symptoms of infection c) Activity d) Resumption of prepregnancy diet e) Resumption of intercourse

• Signs and symptoms of infection • Activity • Resumption of intercourse Correct Explanation: The correct answers give information on managing changes in her new role as a mother. The assumption cannot be made that her prepregnancy diet is still appropriate, and the formula choice should be discussed with her pediatrician.

A mother who is 4 days postpartum, and is breastfeeding, expresses to the nurse that her breast seems to be tender and engorged. What education should the nurse give to the mother to relieve breast engorgement? Select all that apply. a) Massage the breasts from the nipple toward the axillary area b) Express some milk manually before breastfeeding c) Take warm-to-hot showers to encourage milk release d) Feed the newborn in the sitting position only e) Apply warm compresses to the breasts prior to nursing

• Take warm-to-hot showers to encourage milk release • Express some milk manually before breastfeeding • Apply warm compresses to the breasts prior to nursing

When performing Ortolani maneuver, which of the following should occur? Select all that apply. a) Attempt to abduct the hips 180 degrees while applying upward pressure. b) Attempt to abduct the hips 90 degrees while applying upward pressure. c) A click should be heard when the legs are abducted. d) The newborn should be in a prone position. e) The newborn should be in a supine position.

• The newborn should be in a supine position. • Attempt to abduct the hips 180 degrees while applying upward pressure.

Postpartum infection is one event that is known to impede the recovery process of a new mother. Which characteristics after delivery make a woman more susceptible to infection? Select all that apply. a) Urinary stasis b) White blood cell count 25,000/mm³ c) Episiotomy d) Denuded endometrial arteries

• Urinary stasis • Denuded endometrial arteries • Episiotomy Explanation: The urinary system after delivery is prone to infection, prompting a focus on cleanliness and frequent urination. The open uterine arteries are at risk for infection, as is any break in skin integrity. An elevated white blood cell count (from 10,000/mm³ to 30,000/mm³) is the body's defense against infection. A count greater than 30,000/mm³ or less than 10,000/mm³ prompts further investigation.

The nurse is providing teaching to a new mother who is breastfeeding. The mother demonstrates understanding of teaching when she identifies which characteristics as being true of the stool of breast-fed newborns? Select all that apply. a) Stringy to pasty consistency b) Completely odorless c) Yellowish gold color d) Formed in consistency e) Firm in shape

• Yellowish gold color • Stringy to pasty consistency

A nursing student is studying postpartal complications. Thromboembolic conditions have which of the following risk factors? (Select all that apply.) a) cigarette smoking b) diabetes c) multiparity d) irritable bowel e) anemia f) obesity

• anemia • diabetes • cigarette smoking • obesity • multiparity Explanation: Risk factors for developing thromboembolic conditions include anemia, diabetes, cigarette smoking, obesity, preeclampsia, hypertension, varicose veins, pregnancy, cesarean section, multiparity, inactivity, and advanced maternal age.

A nursing instructor explains to students that, regardless of their gestational age, all newborns experience the same pattern that includes which of the following periods? (Select all that apply.) a) period of decreased responsiveness b) period of increased responsiveness c) third period of reactivity d) second period of reactivity e) first period of reactivity

• first period of reactivity • period of decreased responsiveness • second period of reactivity

The nurse who works on a post-partum floor is mentoring a new graduate. She informs the new nurse that a post-partum assessment of the mother includes which of the following? (check all that apply) a) head-to-toe assessment b) pain level c) head-to-toe assessment of newborn d) newborn's vital signs e) vital signs of mother

• head-to-toe assessment • pain level • vital signs of mother Correct Explanation: Post-partum assessment of the mother usually includes vital signs, pain level and a systematic head-to-toe assessment of the mother. The others are care of the newborn and done by the nurse in the nursery.

A nursing instructor informs the student that the following are stimuli for initiating respirations in the newborn? (Select all that apply.) a) elevated CO2 b) hypoxia c) hypercapnia d) alkalosis e) acidosis

• hypercapnia • hypoxia • acidosis

A nursing instructor informs students that recent research has shown advantages of delayed cord clamping include which of the following? (Select all that apply.) a) improving the newborn's cardiopulmonary adaptation b) improving oxygen transport c) increasing blood pressure d) preventing childhood anemia e) preventing childhood obesity f) increasing red blood cell flow

• improving the newborn's cardiopulmonary adaptation • preventing childhood anemia • increasing blood pressure • improving oxygen transport • increasing red blood cell flow

A new mother is nervous about sudden infant death syndrome (SIDS) and asks the nurse how to prevent it when the newborn is ready to sleep. Beside placing the infant on a firm sleep surface, the nurse tells the mother to: (Select all that apply) a) keep the infant dressed warmly at night. b) provide a pacifier when putting the infant to sleep. c) not allow anyone to smoke around the infant. d) let the newborn sleep in the same bed as the parents. e) place the infant on his or her back.

• place the infant on his or her back. • not allow anyone to smoke around the infant.


Conjuntos de estudio relacionados

mccall psyc380 chapter 14 - self-confidence

View Set

Chapter 7 Critical thinking questions

View Set

CHAPTER 64 - Management of Patients with Neurologic Infections, Autoimmune Disorders, and Neuropathies

View Set

Personal Finance: Savings by Nation

View Set

7.4 - Divide powers with same base

View Set